Test Bank - Ifa Part 3 (2015 Edition)

You might also like

Download as docx, pdf, or txt
Download as docx, pdf, or txt
You are on page 1of 284

TEST BANK

Intermediate

Financial
Accounting
Part 3

ZEUS VERNON B. MILLAN


ALL RIGHTS RESERVED
2015

No part of this work covered by the


copyright hereon may be reproduced
or used in any form or by any means -
electronic or mechanical, including
photocopying – without the written
permission of the author.

ISBN 978-621-95096-3-3

Published by:
BANDOLIN ENTERPRISE
No. 100 Montebello Village, Bakakeng Sur, Baguio City 2600, Philippines
TABLE OF CONTENTS
CHAPTER 3

PRESENTATION OF FINANCIAL STATEMENTS. ………………1


CHAPTER 37: MULTIPLE CHOICE – COMPUTATIONAL (SET B) – (FOR CLASSROOM INSTRUCTION PURPOSES)..............1
CHAPTER 37: THEORY OF ACCOUNTS REVIEWER.......................................................................................................8
CHAPTER 37 - SUGGESTED ANSWERS TO THEORY OF ACCOUNTS QUESTIONS ..............................................................54
CHAPTER 38
REVENUE...................................................................................... 55
CHAPTER 38: MULTIPLE CHOICE – COMPUTATIONAL (SET B) – (FOR CLASSROOM INSTRUCTION PURPOSES)...........55
CHAPTER 38: THEORY OF ACCOUNTS REVIEWER.....................................................................................................59
CHAPTER 38 - SUGGESTED ANSWERS TO THEORY OF ACCOUNTS QUESTIONS ..............................................................73
CHAPTER 39
NON-CURRENT ASSETS HELD FOR SALE AND DISCONTINUED OPERATIONS 74
CHAPTER 39: MULTIPLE CHOICE – COMPUTATIONAL (SET B) – (FOR CLASSROOM INSTRUCTION PURPOSES)...........74
CHAPTER 39: THEORY OF ACCOUNTS REVIEWER.....................................................................................................79
CHAPTER 39 - SUGGESTED ANSWERS TO THEORY OF ACCOUNTS QUESTIONS ............................................................100
CHAPTER 40
ACCOUNTING POLICIES, CHANGES IN ESTIMATES AND ERRORS 101
CHAPTER 40: MULTIPLE CHOICE – COMPUTATIONAL (SET B) – (FOR CLASSROOM INSTRUCTION PURPOSES)..........101
CHAPTER 40: THEORY OF ACCOUNTS REVIEWER...................................................................................................104
CHAPTER 40 - SUGGESTED ANSWERS TO REVIEW THEORY QUESTIONS ....................................................................123
CHAPTER 41
STATEMENT OF CASH FLOWS..............................................124
CHAPTER 41: MULTIPLE CHOICE – COMPUTATIONAL (SET B) – (FOR CLASSROOM INSTRUCTION PURPOSES).........124
CHAPTER 41: THEORY OF ACCOUNTS REVIEWER...................................................................................................127
CHAPTER 41 - SUGGESTED ANSWERS TO REVIEW THEORY QUESTIONS ....................................................................148
CHAPTER 42
EVENTS AFTER THE REPORTING PERIOD.......................149
CHAPTER 42: MULTIPLE CHOICE – COMPUTATIONAL (FOR CLASSROOM INSTRUCTION PURPOSES)...........................149
CHAPTER 42: THEORY OF ACCOUNTS REVIEWER...................................................................................................150
CHAPTER 42 - SUGGESTED ANSWERS TO THEORY OF ACCOUNTS QUESTIONS ............................................................156
CHAPTER 43
RELATED PARTY DISCLOSURES..........................................157
CHAPTER 43: MULTIPLE CHOICE – COMPUTATIONAL (FOR CLASSROOM INSTRUCTION PURPOSES)...........................157
CHAPTER 43: THEORY OF ACCOUNTS REVIEWER...................................................................................................157
CHAPTER 43 - SUGGESTED ANSWERS TO THEORY OF ACCOUNTS QUESTIONS ............................................................163
CHAPTER 44
OPERATING SEGMENTS.........................................................164
CHAPTER 44: MULTIPLE CHOICE – COMPUTATIONAL (SET B) – (FOR CLASSROOM INSTRUCTION PURPOSES).........164
CHAPTER 44: THEORY OF ACCOUNTS REVIEWER...................................................................................................165
CHAPTER 44 - SUGGESTED ANSWERS TO THEORY OF ACCOUNTS QUESTIONS ............................................................177
CHAPTER 45
INTERIM FINANCIAL REPORTING......................................178
CHAPTER 45: MULTIPLE CHOICE – COMPUTATIONAL (SET B) – (FOR CLASSROOM INSTRUCTION PURPOSES).........178
CHAPTER 45: THEORY OF ACCOUNTS REVIEWER...................................................................................................182
CHAPTER 45 - SUGGESTED ANSWERS TO REVIEW THEORY QUESTIONS ....................................................................194
CHAPTER 46
DEVELOPMENT STAGE ENTITIES.......................................195
CHAPTER 46: THEORY OF ACCOUNTS REVIEWER.....................................................................................................195
CHAPTER 46 - SUGGESTED ANSWERS TO REVIEW THEORY QUESTIONS ....................................................................197
CHAPTER 47
CASH BASIS TO ACCRUAL BASIS OF ACCOUNTING........198
CHAPTER 47: MULTIPLE CHOICE – COMPUTATIONAL (SET B) – (FOR CLASSROOM INSTRUCTION PURPOSES).........198
CHAPTER 47: THEORY OF ACCOUNTS REVIEWER.........................................................ERROR! BOOKMARK NOT DEFINED.
CHAPTER 47 - SUGGESTED ANSWERS TO THEORY OF ACCOUNTS QUESTIONS ..................ERROR! BOOKMARK NOT DEFINED.
CHAPTER 48
FINANCIAL REPORTING IN HYPERINFLATIONARY ECONOMIES 202
CHAPTER 48: MULTIPLE CHOICE – COMPUTATIONAL (SET B) – (FOR CLASSROOM INSTRUCTION PURPOSES).........202
CHAPTER 48: THEORY OF ACCOUNTS REVIEWER...................................................................................................210
CHAPTER 48 - SUGGESTED ANSWERS TO THEORY OF ACCOUNTS QUESTIONS ............................................................222
CHAPTER 49
PFRS FOR SMALL AND MEDIUM-SIZED ENTITIES (SMES)223
CHAPTER 49: MULTIPLE CHOICE – COMPUTATIONAL (FOR CLASSROOM INSTRUCTION PURPOSES)...........................223
CHAPTER 49: THEORY OF ACCOUNTS REVIEWER (W/ PROBLEMS INCLUDED)..........................................................227
CHAPTER 49 - SUGGESTED ANSWERS TO THEORY OF ACCOUNTS QUESTIONS ............................................................278
Chapter 37
Presentation of Financial Statements

Chapter 37: Multiple choice – Computational (SET B) – (For classroom instruction


purposes)
Current assets
1. The ledger of RELISH TASTE Co. as of December 31, 20x1 includes the following:
Assets  
Cash 20,000
Trade accounts receivable (net of ₱20,000 credit balance in accounts) 80,000
Held for trading securities 160,000
Financial assets designated at FVPL 60,000
Investment in equity securities at FVOCI 140,000
Investment in bonds measured at amortized cost (due in 3 years) 120,000
Prepaid assets 20,000
Deferred tax asset (expected to reverse in 20x2) 24,000
Investment in Associate 72,000
Investment property 92,000
Sinking fund 76,000
Property, plant, and equipment 200,000
Goodwill 56,000
Totals 1,120,000

How much is the total current assets?


a. 380,000 b. 500,000 c. 360,000 d. 384,000

Current liabilities
2. The ledger of CONGRUENT HARMONIOUS Co. as of December 31, 20x1 includes the
following:
Liabilities  
Bank overdraft 20,000
Trade accounts payable (net of ₱20,000 debit balance in accounts) 80,000
Notes payable (due in 20 semi-annual payments of ₱8,000) 160,000
Interest payable 60,000
Bonds payable (due on March 31, 20x2) 140,000
Discount on bonds payable (60,000)
Dividends payable 20,000
Share dividends payable 24,000
Deferred tax liability (expected to reverse in 20x2) 72,000
Income tax payable 88,000
Contingent liability 200,000
Reserve for contingencies 56,000
Totals 860,000

How much is the total current liabilities?


a. 384,000 b. 456,000 c. 584,000 d. 364,000

Current and noncurrent liabilities


3. The ledger of COURIER MESSENGER Co. as of December 31, 20x1 includes the
following:
10% Note payable 160,000
12% Note payable 240,000
14% Mortgage note
payable 120,000

1
Interest payable -

Additional information:
 COURIER Co.’s financial statements were authorized for issue on April 15, 20x2.
 The 10% note payable is due on July 1, 20x2 and pays semi-annual interest every July 1
and December 31. On January 28, 20x2, COURIER Co. entered into a refinancing
agreement with a bank to refinance the entire note by issuing a long-term obligation.
 The 12% note payable is due on March 31, 20x2 and pays annual interest every March
31. On January 31, 20x2, COURIER Co. extended the maturity of the note to March 31,
20x3 under the existing loan agreement. The extension of maturity date is at the option
of COURIER.
 The 14% mortgage note is due on December 31, 20x9. Per agreement with the creditor,
COURIER is to pay quarterly interests on the note, failure to do so will render the note
payable on demand. COURIER failed to pay the 3 rd and 4th quarterly interests on the
note during 20x1.

How much is the total current liabilities?


a. 280,000 b. 310,000 c. 316,000 d. 288,400

Current and noncurrent liabilities


4. The ledger of SQUAMOUS SCALY Co. as of December 31, 20x1 includes the following:
15% Note payable 100,000
16% Bonds payable 200,000
18% Serial bonds
payable 400,000
Interest payable -

Additional information:
 SQUAMOUS Co.’s financial statements were authorized for issue on April 15, 20x2.
 The 15% note payable was issued on January 1, 20x1 and is due on January 1, 20x5. The
note pays annual interest every year-end. The agreement with the lender provides that
SQUAMOUS Co. shall maintain an average current ratio of 2:1. If at any time the current
ratio falls below the agreement, the note payable will become due on demand. As of the
3rd quarter in 20x1, SQUAMOUS’s average current ratio is 0.50:1. Immediately,
SQUAMOUS informed the lender of the breach of the agreement. On December 31, 20x1,
the lender gave SQUAMOUS a grace period ending on December 31, 20x2 to rectify the
deficiency in the current ratio. SQUAMOUS promised the creditor to liquidate some of
its long-term investments in 20x2 to increase its current ratio.
 The 16% bonds are 10-year bonds issued on December 31, 1992. The bonds pay annual
interest every year-end.
 The 18% serial bonds are issued at face amount and are due in semi-annual
installments of ₱40,000 every April 1 and September 30. Interests on the bonds are also
due semi-annually. The last installment on the bonds is due on September 30, 20x7.

How much is the total current liabilities?


a. 218,000 b. 200,000 c. 280,000 d. 298,000

Working capital
5. Below are the account balances prepared by the bookkeeper for REEDY SLENDER
Company as of December 31, 20x1:
Assets   Liabilities  
Cash 60,000 Accounts payable 80,000
Accounts receivable, net 176,000 Notes payable 400,000
Inventory 160,000
Prepaid income tax 32,000
Prepaid assets 20,000
Investment in subsidiary 40,000

2
Land held for sale 112,000
Property, plant, and
200,000
equipment
Totals 800,000   480,000

Additional information:
 Cash consists of the following:
Petty cash fund (unreplenished petty cash expenses, ₱6,000) 8,000
Cash in bank (40,000)
Payroll fund 56,000
Tax fund 28,000
Cash to be contributed to a sinking fund set up for the
retirement of bonds maturing on December 31, 20x3 8,000
Total Cash 60,000

 Checks amounting to ₱122,000 were written to suppliers and recorded on December


30, 20x1, resulting to a bank overdraft of ₱40,000. The checks were mailed on January
5, 20x2.

 Accounts receivable consists of the following:


Accounts receivable 160,000
Allowance for uncollectibility ( 20,000)
Credit balance in customers’ accounts ( 12,000)
Selling price of unsold goods sent on consignment
to FRAIL, Inc. at 120% of cost and excluded from
REEDY’s inventory 48,000
Accounts receivable, net 176,000

 The inventory includes cost of goods amounting to ₱40,000 that are expected to be sold
beyond 12 months but within the ordinary course of business. Also, the inventory
includes cost of consigned goods received on consignment from WEAK Co. amounting
to ₱20,000.

 Prepaid income tax represents excess of payments for quarterly corporate income taxes
during 20x1 over the actual annual corporate income tax as of December 31, 20x1.

 Prepaid assets includes a ₱8,000 security deposit on an operating lease which is


expected to expire on March 31, 20x3. The security deposit will be received on lease
expiration.

 The land qualified for classification as “asset held for sale” under PFRS 5 Non-current
Assets Held for Sale and Discontinued Operations as of December 31, 20x1.

 Accounts payable is net of ₱24,000 debit balance in suppliers’ accounts. Accounts


payable includes the cost of goods held on consignment from WEAK Co. which were
included in inventory.

 The notes payable are dated July 1, 20x1 and are due on July 1, 20x4. The notes payable
bears an annual interest rate of 10%. Interest is payable annually.

How much is the adjusted working capital?


a. 430,000 b. 406,000 c. 442,000 d. 426,000

Working capital
6. The ledger of NEOPHYTE BEGINNER Co. as of December 31, 20x1 includes the
following:
Assets  

3
Petty cash fund 28,000
Cash in bank – Banco De Oro 60,000
Cash in bank – Metrobank 20,000
Accounts receivable (including ₱60,000 pledged accounts) 140,000
Accounts receivable – assigned 100,000
Equity in assigned receivables 40,000
Notes receivable (including ₱80,000 notes receivable discounted) 180,000
Notes receivable discounted 80,000
Advances to subsidiary 128,000
Held for trading securities 80,000
Inventory 248,000
Deferred charges 72,000
Cash surrender value 24,000
Bond sinking fund 400,000
Total assets 1,600,000

Liabilities  
Accounts payable 160,000
Estimated warranty liability 56,000
Loans payable related to assigned receivables (due in 12
60,000
months)
Accrued expenses 52,000
Bonds payable (due on December 31, 20x2) 400,000
Premium on bonds payable 32,000
Total liabilities 760,000

Additional information:
 Petty cash fund includes IOU’s from employees amounting to ₱8,000. The remaining
balance of ₱20,000 represents bills and coins.
 Cash in bank – Banco de Oro represents the balance per bank statement. As of
December 31, 20x1, deposits in transit amounted to ₱40,000 while outstanding checks
amounted to ₱12,000. Included in the bank statement as of December 31, 20x1 is an
NSF check amounting to ₱32,000.
 Cash in bank – Metrobank represents the balance per ledger. As of December 31, 20x1,
deposits in transit amounted to ₱8,000 while outstanding checks amounted to ₱4,000.
 Accounts receivable (unassigned) includes uncollectible past due accounts of ₱16,000
which need to be written-off.
 Also included in accounts receivable (unassigned) is a ₱20,000 receivable from a
customer which was given a special credit term. Under the special credit term, the
customer shall pay the ₱20,000 receivable in equal quarterly installments of ₱2,500.
The last payment is due on December 31, 20x3.
 The held for trading securities include the reacquisition cost of NEOPHYTE Co.’s shares
amounting to ₱16,000.
 Inventory includes ₱120,000 goods in transit purchased FOB Destination but excludes
₱48,000 goods in transit purchased FOB Shipping point.

How much is the working capital?


a. 394,000 b. 420,000 c. 349,000 d. 402,000

Reconstruction of financial statement


Use the following information for the next three questions:
The ledger of NAÏVE UNAFFECTEDLY SIMPLE Co. in 20x1 includes the following:
Jan. 1, Dec. 31,
20x1 20x1
Current assets 2,400,000 ?
Noncurrent assets 8,000,000 ?
Current liabilities 1,800,000 2,000,000

4
Noncurrent
liabilities ? 6,000,000

Additional information:
 NAÏVE’s working capital as of December 31, 20x1 is twice as much as the working
capital as of January 1, 20x1.
 Total equity as of January 1, 20x1 is ₱3,400,000. Profit for the year is ₱4,800,000 while
dividends declared amounted to ₱2,000,000. There were no other changes in equity
during the year.

7. How much is the noncurrent liabilities as of January 1, 20x1?


a. 5,000,000 b. 5,200,000 c. 5,300,000 d. 5,400,000

8. How much is the current assets as of December 31, 20x1?


a. 3,200,000 b. 3,400,000 c. 3,600,000 d. 4,200,000

9. How much is the noncurrent assets as of December 31, 20x1?


a. 9,000,000 b. 11,000,000 c. 8,000,000 d. 12,000,000

Reconstruction of financial statements


10. The ledger of LOQUACIOUS TALKATIVE Co. in 20x1 includes the following:
Cash 400,000
Accounts receivable 800,000
2,000,00
Inventory 0
Accounts payable 600,000
Note payable 200,000

During the audit of LOQUACIOUS’s 20x1 financial statements, the following were noted by
the auditor:
 Cash sales in 20x2 amounting to ₱40,000 were inadvertently included as sales in 20x1.
LOQUACIOUS recognized gross profit of ₱12,000 on the sales.
 A collection of an ₱80,000 accounts receivable in 20x2 was recorded as collection in
20x1. A cash discount of ₱4,000 was given to the customer.
 During January 20x2, a short-term bank loan of ₱100,000 obtained in 20x1 was paid
together with ₱10,000 interest accruing in January 20x2. The payment transaction in
20x2 was inadvertently included as a 20x1 transaction.

How much is the adjusted working capital as of December 31, 20x1?


a. 2,420,000 b. 2,482,000 c. 2,342,000 d. 2,402,000

Reclassification adjustment
Use the following information for the next two questions:
In 20x1, LUSTROUS BRIGHT Co. disposed of a foreign operation for which a cumulative
translation gain of ₱400,000 is recognized in equity. LUSTROUS Co. is subject to a 30% tax
rate.

11. How much is the net of tax reclassification adjustment to other comprehensive income
in 20x1?
a. 280,000 b. (280,000) c. 120,000 d. (120,000)

12. How much is the gross of tax effect of the reclassification adjustment to profit or loss in
20x1?
a. 280,000 b. (280,000) c. 400,000 d. (400,000)

Comprehensive income
Use the following information for the next two questions:

5
The following items were presented for the purpose of determining comprehensive
income.
Profit for the year 4,000
Increase in revaluation surplus 2,000
Remeasurements of the net defined benefit liability (asset) - loss (400)
Net change in translation of foreign operation (800)
Dividends declared (200)
Stock rights 600

13. How much is the other comprehensive income?


a. 1,600 b. 800 c. 1,200 d. 4,800

14. How much is the total comprehensive income?


a. 4,800 b. 5,200 c. 5,400 d. 4,600

Function of expense
Use the following information for the next two questions:
The following are among the expenses incurred by GYRATE REVOLVE Co. during the year.
in ‘000s
Interest expense ₱ 48
Cost of inventories sold 1,200
Insurance expense 200
Advertising expense 40
Freight-out 20
Freight-in 8
Loss on sale of equipment 4
Legal and other professional fees 24
Rent expense (one-half occupied by sales department) 16
Sales commission expense 28
Doubtful accounts expense 32
15. How much are the distribution costs or selling expenses?
a. 96 b. 128 c. 232 d. 316

16. How much are the administrative expenses?


a. 316 b. 232 c. 264 d. 361

Gross profit
17. The records of MARAUD PLUNDER Co. showed the following information:
Increase in accounts receivable 200,000
Collections on accounts 1,600,000
Cash sales 240,000
Increase in inventory 80,000
Freight-in 28,000
Freight-out 26,000
Decrease in accounts payable 120,000
Disbursements for purchases 960,000
Purchase discounts 8,000

How much is the gross profit for the year?


a. 1,252,000 b. 1,244,000 c. 1,226,000 d. 1,225,000

Gross profit
18. The records of DEADLOCK STANDSTILL Co. showed the following information:
Accounts receivable, net, Jan. 1, 20x1 80,000
Accounts receivable, net, Dec. 31, 20x1 320,000
Accounts receivable turnover 4:1
Inventory, Jan. 1, 20x1 240,000

6
Inventory, Dec. 31, 20x1 120,000
Inventory turnover 3:1
How much is the gross profit for the year?
a. 240,000 b. 260,000 c. 280,000 d. 300,000

Cost of sales
19. The records of CANDOR FAIRNESS Co. showed the following information:
Decrease in accounts payable 120,000
Disbursements for purchases 880,000
Increase in raw materials 200,000
Direct labor is 50% of raw materials used in production
Manufacturing overhead is 20% of prime costs
Increase in work-in-process inventory 80,000
Decrease in finished goods inventory 100,000

How much is the cost of goods sold?


a. 1,082,000 b. 1,032,000 c. 1,048,000 d. 1,028,000

Reconstruction of financial statement


20. WLETER TURMOIL Co. reported profit after tax of ₱420,000. WELTER’s income tax rate
is 30%. Operating expenses for the year were 15% of sales and 25% of cost of sales.
Other expenses were 10% of sales. How much is the sales?
a. 4,000,100 b. 3,900,000 c. 4,100,000 d. 4,000,000

Total comprehensive income


Use the following information for the next two questions:
The records of RESTIVE UNEASY Co. on December 31, 20x1 showed the following
information:
Sales 4,000,000
Sales discounts 40,000
Cost of sales 1,600,000
Distribution costs 192,000
Administrative costs 480,000
Casualty loss on typhoon 80,000
Dividends received from investments in FVPL 48,000
Dividends received from investment in associate 96,000
Share in the profit of an associate 144,000
Dividends declared and paid 56,000
Interest expense 88,000
Unrealized gain on investments in FVPL 60,000
Unrealized gain on investments in FVOCI 76,000
Income tax expense 600,000
Loss on revaluation 52,000
Remeasurements of the net defined benefit liability (asset)
44,000
- gain
Correction of understatement in depreciation in prior year 64,000
Translation adjustment of foreign operation – loss 16,000

21. How much is the other comprehensive income?


a. (24,000) b. 152,000 c. 52,000 d. 127,000

22. How much is the total comprehensive income?


a. 1,224,000 b. 1,242,00 c. 1,448,000 d. 1,424,000

Reconstruction of financial statement


23. PRECLUDE PREVENT Co. has the following information on December 31, 20x1:
 Cost of sales is ₱520,000.

7
 Operating expenses are 13% of sales and 20% of cost of sales.
 Interest expense is 5% of sales.
 Income tax rate is 30%. There were no temporary differences during the year.

How much is the profit for the year?


a. 98,200 b. 104,200 c. 105,200 d. 95,200

Shareholders’ equity
24. The ledger of INDENTATION CUT Co. in 20x1 includes the following:
Share capital 400,000
Share premium 80,000
Retained earnings, appropriated 72,000
Retained earnings, unappropriated 168,000
Revaluation surplus 120,000
Remeasurements of the net defined benefit liability (asset) – gain 60,000
Cumulative net unrealized gain on fair value
changes of investment in FVOCI 92,000
Effective portion of losses on hedging instruments in a
cash flow hedge 40,000
Cumulative translation loss on foreign operation 20,000
Treasury shares, at cost 52,000

How much is the total shareholders’ equity?


a. 880,000 b. 932,000 c. 960,000 d. 696,000

Reconstruction of financial statements


Use the following information for the next two questions:
INFRINGE VIOLATE Co. was incorporated on January 1, 20x1. The following were the
transactions during the year:
 Total consideration from share issuances amounted to ₱4,000,000.
 A land and building were acquired through a lump sum payment of ₱800,000. A
mortgage amounting to ₱200,000 was assumed on the land and building.
 Total payments of ₱160,000 were made during the year on the mortgage assumed on
the land and building. The payments are inclusive of interest amounting to ₱20,000.
 Additional capital of ₱400,000 was obtained through bank loans. None of the bank loans
were paid during the year. Half of the bank loans required a secondary mortgage on the
land and building.
 There is no accrued interest as of year-end.
 Dividends declared during the year but remained unpaid amounted to ₱120,000.
 No other transactions during the year affected liabilities.
 Retained earnings as of December 31, 20x1 is ₱240,000.

25. How much is the profit for the year?


a. 420,000 b. 360,000 c. 280,000 d. 320,000

26. How much is the total assets as of December 31, 20x1?


a. 4,802,000 b. 4,940,000 c. 4,780,000 d. 4,820,000
Reconstruction of financial statements
27. GENTEEL POLITE Co. had the following information for 20x1:
Accounts receivable turnover 10:1
Total assets turnover 2:1
Average receivables during the year ₱800,000
Total assets, January 1, 20x1 1,600,000

How much is the total assets as of December 31, 20x1?


a. 6,480,000 b. 6,380,000 c. 6,240,000 d. 6,400,000

8
The answers and solutions to the computational problems above
(Multiple choice – Computational (SET B) can be found in the
accompanying Teacher’s Manual.
Chapter 37: Theory of Accounts Reviewer
Objective and scope of PAS 1
1. This refers to financial statements that are intended to meet the needs of users who are
not in a position to require an entity to prepare reports tailored to their particular
information needs.
a. All-purpose financial statements c. Managerial reports
b. General purpose financial statements d. Unisex financial statements

2. Regarding the presentation of financial statements, which of the following statements is


correct?
a. PAS 1 Presentation of Financial Statements prescribes the basis for presentation of
general and specific purpose financial statements to ensure comparability both with
the entity’s financial statements of previous periods and with the financial
statements of other entities.
b. PAS 1 does not apply to the structure and content of condensed interim financial
statements.
c. PAS 1 applies only to entities that present separate financial statements;
consolidated financial statements are dealt with under PAS 27.
d. PAS 1 uses terminology that is suitable for both profit and non-profit-oriented
entities, including public sector business entities.

3. Regarding the presentation of financial statements, which of the following statements is


correct?
a. PAS 1 Presentation of Financial Statements applies only to businesses in the private
sector
b. PAS 1 applies to corporations only; PAS 1 does not apply to entities that do not have
share capital.
c. PAS 1 applies equally to all entities, including those that present consolidated
financial statements and those that present separate financial statements.
d. All the recognition, measurement and disclosure requirements for specific
transactions and other events are included in PAS 1.

4. Philippine Financial Reporting Standards (PFRSs) are Standards and Interpretations


adopted by the Financial Reporting Standards Council (FRSC). They comprise all of the
following, except
a. Philippine Financial Reporting Standards (PFRS)
b. Philippine Accounting Standards (PAS)
c. Interpretations originated by the International Financial Reporting Interpretations
Committee (IFRIC) or the former Standing Interpretations Committee (SIC) adopted
by the FRSC and interpretations originated by Philippine Interpretations Committee
(PIC)
d. Philippine Accounting Practice Statements (PAPS)
e. All of the following comprise the Reporting Standards

5. Financial statements are a structured representation of the financial position and


financial performance of an entity. The main objective of financial statements is
a. to provide information about the financial position, financial performance and cash
flows of an entity that is useful to a wide range of users in making economic
decisions.
b. to show the results of the management’s stewardship of the resources entrusted to
it.
c. to provide information about the products of the entity, its achievements during the
year, and its plans for the following year(s).

9
d. to provide information essential in making buy or sell decisions

6. Financial statements are a structured representation of the financial position and


financial performance of an entity. The objective of general purpose financial
statements is to provide information about an entity’s (choose the incorrect statement)
a. financial position c. cash flows
b. financial performance d. valuation

7. In addition to financial statements, an entity also provided a report quantifying the


benefits from ISO certifications obtained during the year on quality management and
environmental compliance. Management regards the employees as an important user
group and expects that these reports may encourage employees to adhere to company
policies on quality and environmental matters. Which of the following statements is
correct?
a. Entities shall prepare additional reports showing quantitative information in
accordance with relevant PFRSs.
b. Entities are prohibited from issuing additional quantitative reports.
c. Reports and statements presented outside financial statements are outside the
scope of PFRSs.
d. Entities should prepare all additional reports showing either qualitative or
quantitative information in accordance with relevant PFRSs.

8. All of the following statements correctly refer to PAS 1 Presentation of Financial


Statements, except
a. The objective of this Standard is to prescribe the basis for presentation of general
purpose financial statements, to ensure comparability both with the entity’s
financial statements of previous periods and with the financial statements of other
entities.
b. PAS 1 shall be applied to all-purpose financial statements prepared and presented in
accordance with Philippine Financial Reporting Standards (PFRSs).
c. The application of PFRSs, with additional disclosure when necessary, is presumed to
result in financial statements that achieve a fair presentation.
d. Inappropriate accounting policies are not rectified either by disclosure of the
accounting policies used or by notes or explanatory material.

9. The following statements relate to PAS1 Presentation of Financial Statements. Choose


the correct statement.
a. Many entities also present, outside the financial statements, reports and statements
such as environmental reports and value added statements, particularly in
industries in which environmental factors are significant and when employees are
regarded as an important user group. Reports and statements presented outside
financial statements should be accounted for using applicable PFRSs.
b. Applying a requirement is impracticable when the entity cannot apply it after
making every reasonable effort to do so.
c. An entity whose financial statements do not comply with PFRSs shall make an
explicit and unreserved statement of such noncompliance in the notes. If the entity’s
financial statements do comply with PFRSs, there is no need to make an explicit and
unreserved statement of such compliance in the notes.
d. Financial statements shall not be described as complying with PFRSs unless they
comply with most of the requirements of PFRSs.

10.Which of the following statements is/are correct?


I. External financial information is generally more highly summarized than the
information reported internally.
II. The Securities and Exchange Commission recommends, but does not require, that all
nationally registered companies have an annual independent audit as protection for
the shareholders.

10
III. PFRSs have, in most cases, eliminated the need for accountants to exercise
professional judgment in interpreting and applying accounting standards.
a. I, II, III b. I, II c. II, III d. I

11.Which of the following statements is (are) correct?


I. General purpose financial statements are those intended to meet the needs of users
who are in a position to demand reports tailored to meet their particular
information needs
II. Managerial reports are a structured representation of the financial position and
financial performance of an entity.
a. both are true b. both are false c. I is true d. II is true

12.Which of the following is a report presented outside the financial statements and hence,
not covered by PFRSs?
(Item #1) Environmental Reports; (Item #2) Explanatory Notes; (Item #3)Value added
statements
a. Yes, Yes, Yes c. No, Yes, Yes
b. Yes, No, Yes d. Yes, Yes, No
(AICPA)

13.The accounting terminology for earned surplus under current PFRSs is


a. retained earnings reservation
b. additional paid-in capital
c. capital contribution in excess of par value
d. retained earnings
(RPCPA)

14.Which of the following is not an implied objective of financial reporting?


a. to help allocate limited resources
b. to influence the market price of shares traded in the stock exchange
c. to reduce the risk of making economic decisions
d. to report on the stewardship of enterprise resources
(Adapted)

General features
15.The general features of financial statement presentation as prescribed under PAS 1
Presentation of Financial Statements includes
I. Fair presentation and compliance with PFRS
II. Going concern
III. Accrual basis of accounting
IV. Materiality and aggregation
V. Offsetting
VI. Frequency of reporting
VII. Comparative information
VIII. Consistency of presentation
a. I, II, III, IV b. I, II, III, IV, V c. I, II, III, IV, V, VIII d. all of these

16.Which of the following is not included among the general features of financial statement
presentation?
a. Growing concern c. Frequency of reporting
b. Accrual basis d. Comparative information

17.A company is issuing its comparative financial statements for years 20x1 and 20x2. If
the company is required to issue an additional statement of financial position, such
statement should be dated
a. as of Jan. 1, 20x1 c. as of Dec. 31, 20x2
b. as of Jan. 1, 20x2 d. as of Dec. 31, 20x1

11
18.During the year, an accountant omitted centavos in the amounts recognized in the
journals. Such omissions were considered individually immaterial and were treated as a
normal company practice. However, it was found out as of year-end that the sum of the
centavos omitted, when totaled, is material. The omission is
a. considered immaterial, hence, requires no adjustment
b. considered material, hence, requires no adjustment
c. considered immaterial but no adjustment is necessary
d. considered material, hence, requires an adjustment

19.Omissions or misstatements of items are material if they could, individually or


collectively; influence the economic decisions of users taken on the basis of the financial
statements. Materiality depends on
a. the peso amount of financial consequence of the omission or misstatement judged in
the surrounding circumstances
b. the size and peso amount of the omission or misstatement judged in the
surrounding circumstances
c. the peso amount and nature of the omission but not the misstatement judged in the
surrounding circumstances
d. the size and nature of the omission or misstatement judged in the surrounding
circumstances

20.The presentation of comparative financial statements is


a. encouraged by PFRSs
b. required by PFRSs
c. not required by PFRSs but permitted due to industry standards
d. a violation of PFRSs

21.According to PAS 1, in virtually all circumstances, a fair presentation is achieved by


compliance with applicable PFRSs. A fair presentation also requires an entity (choose
the incorrect statement).
a. to select and apply accounting policies in accordance with PAS 8 Accounting Policies,
Changes in Accounting Estimates and Errors.
b. to present information, including accounting policies, in a manner that provides
relevant, reliable, comparable and understandable information.
c. to have its financial statements examined by an external party
d. to provide additional disclosures when compliance with the specific requirements
in PFRSs is insufficient to enable users to understand the impact of particular
transactions, other events and conditions on the entity’s financial position and
financial performance.

22.In virtually all circumstances, a fair presentation is achieved by compliance with


applicable PFRSs. A fair presentation also requires an entity: (choose the incorrect
statement)
a. to select and apply accounting policies in accordance with PAS 8
b. to provide additional disclosures when compliance with the specific requirements
in PFRSs is insufficient to enable users to understand the impact of particular
transactions
c. to establish a system of internal control, the responsibility for which is the entity’s
auditor.
d. to present information, including accounting policies, in a manner that provides
relevant, reliable, comparable and understandable information.

23.In the extremely rare circumstances in which management concludes that compliance
with a requirement in a PFRS would be so misleading that it would conflict with the
objective of financial statements set out in the Conceptual Framework and the relevant

12
regulatory framework requires, or otherwise does not prohibit, such a departure, the
entity need not disclose which of the following?
a. that management has concluded that the financial statements present fairly the
entity’s financial position, financial performance and cash flows
b. that it has complied with applicable PFRSs, except that it has departed from a
particular requirement to achieve a fair presentation
c. that it has complied with other applicable standards other than those issued by the
IASB and adopted by the FRSC and the description of those accounting standards
which the entity has complied to.
d. the title of the PFRS from which the entity has departed, the nature of the departure,
including the treatment that the PFRS would require, the reason why that treatment
would be so misleading in the circumstances that it would conflict with the objective
of financial statements set out in the Conceptual Framework, and the treatment
adopted
e. for each period presented, the financial effect of the departure on each item in the
financial statements that would have been reported in complying with the
requirement.

24.Identify the incorrect statement.


a. When an entity has departed from a requirement of a Standard or an Interpretation
in a prior period, and that departure affects the amounts recognized in the financial
statements for the current period, it shall disclose the (a) title of the Standard or
Interpretation from which the entity has departed and the (b) impact of such
departure.
b. In the extremely rare circumstances in which management concludes that
compliance with a requirement in a Standard or an Interpretation would be so
misleading that it would conflict with the objective of financial statements set out in
the Conceptual Framework, but the relevant regulatory framework prohibits
departure from the requirement, the entity shall, to the maximum extent possible,
reduce the perceived misleading aspects of compliance by disclosing:(a) the title of
the Standard or Interpretation in question and (b) for each period presented, the
adjustments to each item in the financial statements that management has
concluded would be necessary to achieve a fair presentation.
c. Financial statements shall be prepared on a going concern basis unless management
either intends to liquidate the entity or to cease trading, or has no realistic
alternative but to do so.
d. PAS 1 requires an entity preparing financial statements, to make an assessment of
the entity’s ability to continue as a going concern. In assessing whether the going
concern assumption is appropriate, management takes into account all available
information about the future, which is at least, but is not limited to, five years from
the balance sheet date.

25.Identify the incorrect statement.


a. An entity shall prepare its financial statements, including cash flow information,
using the accrual basis of accounting.
b. The final stage in the process of aggregation and classification is the presentation of
condensed and classified data, which form line items on the face of the balance
sheet, income statement, statement of changes in equity and cash flow statement, or
in the notes.
c. Applying the concept of materiality means that a specific disclosure requirement in
a Standard or an Interpretation need not be satisfied if the information is not
material.
d. PAS 1 requires that an entity presenting its current year financial statements to also
present its financial statements for the previous year.

13
26.When preparing financial statements, management shall make an assessment of an
entity’s ability to continue as a going concern. An entity shall prepare financial
statements on a going concern basis unless
I. management intends to liquidate the entity
II. management intends to cease trading
III. management has no realistic alternative but to cease trading
IV. there are material uncertainties
a. I, II b. I, II, III c. I, II, III, IV d. II, III, IV

27.When management is aware, in making its assessment of material uncertainties related


to events or conditions that may cast significant doubt upon the entity’s ability to
continue as a going concern, the entity shall
a. not prepare the financial statements on a going concern basis and shall disclose that
fact, together with the basis on which it prepared the financial statements and the
reason why the entity is not regarded as a going concern.
b. not prepare the financial statements on a going concern basis but no disclosure is
necessary
c. prepare the financial statements on a going concern basis but shall disclose the
uncertainties.
d. prepare the financial statements on a going concern basis but no disclosure is
necessary

28.In assessing whether the going concern assumption is appropriate, management takes
into account all available information about the future, which is at least, but is not
limited to,
a. 12 months from the end of the reporting period.
b. 12 months to 3 years from the end of the reporting period.
c. 3 to 5 years from the end of the reporting period.
d. at least 5 years from the end of the reporting period.

29.When the accrual basis of accounting is used, an entity recognizes items as assets,
liabilities, equity, income and expenses (the elements of financial statements)
a. as cash is received and as cash is paid
b. as cash is earned and as cash is incurred
c. when they provide relevant information to expected users
d. when they satisfy the definitions and recognition criteria for those elements in the
Conceptual Framework and in the PFRSs.

30.An entity shall present separately each material class of similar items. An entity shall
present separately items of a dissimilar nature or function unless they are immaterial.
According to PAS 1, the final stage in the process of aggregation and classification is
a. the presentation of a comprehensive financial report understandable by all expected
users
b. the presentation of condensed and classified data, which form line items in the
financial statements.
c. the presentation of a concise information, comprehensive enough to permit
informed judgment to external and internal users.
d. the presentation of a balanced trial balance.

31.When preparing financial statements, a management of an entity assessed that a


specific disclosure required by a PFRS is immaterial and, thus, such disclosure was
omitted from the financial statements. The financial statements prepared
a. are misstated
b. are not misstated
c. are misstated and a rectification should be made in the notes
d. are not misstated but additional disclosures should be made in the notes

14
32.Which of the following statements about financial statements are in accordance with
PAS 1?
I. Extraordinary items must be disclosed on the face of the income statement as
additions to or deductions from profit before tax.
II. The authorized share capital of the company must be disclosed by note or on the
face of the balance sheet.
III. The total of staff costs for the period must be disclosed by note or on the face of the
income statement.
IV. The accounting policies adopted by the company must be disclosed but only if they
do not comply with accounting standards.
V. Proposed ordinary dividends should not be recognized as liabilities unless they have
been proposed or declared before the balance sheet date.
a. 1, 2, 3 and 4 b. 1, 2, 3 and 5 c. 2, 3 and 5 d. 1, 4 and 5
(ACCA)

33.All of the following are examples of offsetting, except


a. presenting Accounts receivable net of allowance for doubtful accounts.
b. gains and losses on the disposal of non-current assets, including investments and
operating assets, are reported by deducting from the proceeds on disposal the
carrying amount of the asset and related selling expenses
c. expenditure related to a provision that is recognized in accordance with PAS 37 and
reimbursed under a contractual arrangement with a third party are netted against
the related reimbursement in the income statement.
d. foreign exchange gains and losses or gains and losses arising on financial
instruments held for trading are netted and presented as net gains or net losses in
the income statement

34.Under PAS 1, an entity shall present a complete set of financial statements


a. including comparative information at least annually.
b. at least annually, with or without comparative information
c. on as-needed basis, with or without comparative information
d. at least every three years when there are limited users.

35.When an entity changes the end of its reporting period and presents financial
statements for a period longer or shorter than one year, the entity shall
I. disclose the period covered by the financial statements
II. restate comparative financial information
III. disclose the reason for using a longer or shorter period
IV. disclose the fact that amounts presented in the financial statements are not entirely
comparable
a. I, II, III b. I, III, IV c. I, II, III, IV d. I, III

36.When an entity’s balance sheet date changes and the annual financial statements are
presented for a period longer or shorter than one year, an entity shall disclose, in
addition to the period covered by the financial statements:
I. the reason for using a longer or shorter period
II. the fact that comparative amounts for the income statement, statement of changes
in equity, cash flow statement and related notes are not entirely comparable
III. the amounts charged to the beginning balance of the retained earnings, net of tax
IV. pro-forma financial statements, as a supplemental information in the notes
a. I, II b. I, III c. I, III, IV d. I, II, III, IV

37.An entity disclosing comparative information shall present, as a minimum,


a. two statements of financial position, two of each of the other statements, and related
notes.
b. two statements of financial position, two of each of the other statements, and two
related notes.

15
c. two statements of financial position, one of each of the other statements, and related
notes.
d. three statements of financial position, two of each of the other statements, and
related notes.

38.When the entity changes the presentation or classification of items in its financial
statements, the entity shall reclassify comparative amounts unless reclassification is
impracticable. When the entity reclassifies comparative amounts, the entity shall
disclose:
I. the nature of the reclassification
II. the amount of each item or class of items that is reclassified
III. the reason for the reclassification
IV. the nature of the adjustments that would have been made if the amounts had
been reclassified
a. I, II b. I, II, III, IV c. I, III, IV d. I, II, III

39.An entity shall retain the presentation and classification of items in the financial
statements from one period to the next unless:
I. another presentation or classification would provide a more reliable and
relevant information to users.
II. a PFRS requires a change in presentation.
III. in no circumstance that an entity may change the presentation and classification
of items in the financial statements
a. I, II b. I c. II d. III

40.SOP SOAK, Inc. decided to extend its reporting period from a year (12-month period) to
a 15-month period. Which of the following is not required under PAS 1 in case of change
in reporting period?
a. SOP Inc. should disclose the reason for using a longer period than a period of 12
months.
b. SOP Inc. should change the reporting period only if other similar entities in the
geographical area in which it generally operates have done so in the current year;
otherwise its financial statements would not be comparable to others.
c. SOP Inc. should disclose that comparative amounts used in the financial statements
are not entirely comparable.
d. SOP Inc. should disclose the period covered by the financial statements.
(Adapted)

41.Which principle/guideline justifies a company violating an accounting principle


because the amounts are immaterial?
a. Conservatism b. Full disclosure c. Immateriality d. Materiality
42.Which of the following statements is incorrect?
a. working capital usually is viewed a one measure of liquidity.
b. current liabilities are short-term liabilities whose liquidation is reasonably expected
to require the use of current assets or the creation of other current liabilities.
c. all assets reported on the balance sheet are reported to acquisition cost in
conformity with the historical cost principle.
d. in financial reporting, it is improper to offset current assets with current liabilities
unless there is a legal right of setoff.
(AICPA)

43.The life of a business is


a. a series of balance sheets c. a and b
b. a series of income statements d. perpetual

44.You should request the following financial information before you invest in a company:

16
a. statement of financial position, statement of profit or loss and other comprehensive
income, statement of changes in equity, statement of cash flows and notes.
b. statement of earnings, statement of retained earnings, cash flow statement, and the
balance sheet.
c. statement of earnings, balance sheet, and the accompanying notes.
d. journals and ledgers
(Adapted)

45.Fair presentation requires an entity to


I. Select and apply accounting policies in accordance with FRSC standards.
II. Present information, including accounting policies, in a manner that provides
relevant, reliable, comparable and understandable information.
III. Provide additional disclosures when compliance with the specific requirements of
FRSC standards is insufficient to enable users to understand the impact of particular
transactions and other events on the entity’s financial position and financial
performance.
a. I only b. I and II only c. I and III only d. I, II and III

46.Philippine Financial Reporting Standards comprise the following, except:


a. Philippine Accounting Standards c. Adopted SIC Interpretations
b. Adopted IFRIC Interpretations d. BOA Pronouncements

47.Which of the following statements is (are) correct?


I. Assessing whether an omission or misstatement could influence economic decisions
of users, and so be material, requires consideration of the characteristics of those
users
II. Users are assumed to have a reasonable knowledge of business and economic
activities and accounting and willingness to study the information with reasonable
diligence.
a. Both are true b. Both are false c. I is true d. II is true

48.The inability to apply the requirement of a specific standard after making every
reasonable effort to do so refers to the condition known as:
a. impossibility c. impracticability
b. infeasibility d. inapplicability

49.Which is not a general feature of financial statement presentation?


a. fair presentation and compliance with PFRS
b. structure and content
c. offsetting
d. materiality and aggregation

50.Which is not correct?


a. the application of PFRSs, with additional disclosure when necessary, is presumed to
result in financial statement that achieve fair presentation
b. an entity whose financial statement comply with PFRSs shall make an implicit and
reserved statement of such compliance
c. inappropriate accounting policies are not rectified by disclosures of the accounting
policies used or by notes or explanatory materials
d. omission or misstatement of an item is material if they could individually or
collectively, influence the decision of users taken on the basis of the financial
statements

51.Fair presentation of financial statements requires an entity to: (Select the incorrect
one.)
a. select and apply accounting policies in accordance with PAS 8.

17
b. present information, including accounting policies, in a manner that provides
relevant, reliable, comparable and understandable information
c. provide additional disclosures when compliance with specific requirement in PFRS
is insufficient to enable users to understand the impact of a particular transaction
d. have its accounting policies unchanged when a more appropriate alternative exists

52.When the reporting period is changed, which is not a required disclosure?


a. the new period covered by the financial statement
b. the reason for using a longer or shorter period
c. the fact that comparative amounts for the income statement, statement of changes
in equity, cash flow statement and related notes are not entirely comparable
d. the reason why other similar entities should also change their reporting period

53.Which of the following is correct regarding the preparation of financial statements?


I. When an entity has a history of profitable operations and ready access to financial
resources, a conclusion that the going concern basis of accounting is appropriate
may be reached without detailed analysis.
II. An entity shall prepare all of its financial statements, including the cash flow
statement, using the accrual basis of accounting
a. I is true c. Both I and II are true
b. II is true d. Both I and II are false

54.When preparing financial statements, an entity needs to assess whether going concern
assumption is appropriate. In assessing the appropriateness of going concern, which of
the following need not be considered
a. maturities of obligations
b. potential sources of financing
c. current and expected profitability
d. availability of unqualified audit opinion

55.You are a CPA practicing public accounting. You were engaged by Lugi Bank
Corporation to audit its financial statements for the year ended December 31, 20x1. You
did not perform any audit procedure on the appropriateness of management’s use of
going concern assumption because of a tight deadline. Three (3) months after you
issued an unqualified opinion (‘clean’ opinion), Lugi Bank Corporation has liquidated.
Which of the following is most likely to be incorrect?
a. You may decide to take up another course in college. Goodbye accounting career.
b. There is a possibility that you will end up in jail.
c. You may acquire anemia from sleepless nights.
d. You can never be held responsible for your audit opinion because a “tight” deadline
is a valid reason to omit audit procedures. Further, PAS 1 does not require
assessment of going concern.

56.A newly acquired plant asset is to be depreciated over its useful life. The rationale for
this process is the
a. Economic entity assumption. c. Materiality assumption.
b. Monetary unit assumption. d. Going concern assumption.
(Adapted)

57.The standard of fair presentation in conformity with PFRSs does not require that:
a. changes in accounting policies from period to period should be disclosed
b. there should be a proper balance between disclosure and summarization of financial
accounting information
c. information in the underlying records should be properly reflected and described in
the financial statements in conformity with PFRSs
d. the financial statements should provide all available information about the entity
(Adapted)

18
58.Which of the following statements is incorrect?
a. An entity shall present separately each material class of similar items.
b. An entity shall present separately items of a dissimilar nature or function unless
they are immaterial.
c. PAS 1 affects only the presentation of owner changes in equity and of
comprehensive income. It does not change the recognition, measurement or
disclosure of specific transactions and other events required by other PFRSs.
d. When an entity presents notes, it shall not present it with equal prominence with
the other financial statements so as not to imply inaccuracies in the information
presented on the face of the financial statements.

59.Which of the following statements is incorrect?


a. PAS 1 requires an entity to present all owner changes in equity in a statement of
changes in equity.
b. All non-owner changes in equity are required to be presented in one statement of
profit or loss and other comprehensive income or in two statements
c. Components of comprehensive income are permitted to be presented in the
statement of changes in equity in some rare cases.
d. An entity may do away with a requirement of a standard if such requirement is
impracticable.

60.PFRSs apply only to material items. It is therefore


a. appropriate to leave immaterial errors uncorrected
b. appropriate to make intentional errors which are immaterial
c. permitted to deviate from provisions of the PFRSs that do not materially affect the
usefulness of the financial statements.
d. appropriate to deviate from provisions of the PFRSs if the auditor says so.

61.Which of the following statements is incorrect?


a. An entity is permitted to deviate from some requirements of PFRSs in certain cases
if compliance with the provisions of the PFRSs would be so misleading.
b. An entity is permitted to deviate from some requirements of PFRSs if compliance
with the requirement is impracticable.
c. A requirement of a standard is said to be impracticable when an entity cannot apply
it after making every reasonable effort to do so
d. A requirement of a standard is said to be impracticable when the entity’s accountant
is a non-CPA.

62.Which of the following statements is incorrect?


a. An entity may present the components of profit or loss either as part of a single
statement of profit or loss and other comprehensive income or in a separate income
statement or in the notes.
b. PAS 1 requires that total comprehensive income should be presented in the financial
statements.
c. PAS 1 requires an entity to disclose income tax relating to each component of other
comprehensive income.
d. The presentation of disclosures on dividends in the statement of comprehensive
income is not permitted.

63.Which of the following statements is incorrect?


a. When an income statement is presented it is part of a complete set of financial
statements and shall be displayed immediately before the statement of
comprehensive income.
b. An entity shall offset financial assets and financial liabilities or income and expenses
if the entity has legal right of set-off.

19
c. Many entities also present, outside the financial statements, reports and statements
such as environmental reports and value added statements, particularly in
industries in which environmental factors are significant and when employees are
regarded as an important user group. Reports and statements presented outside
financial statements are outside the scope of PFRSs.
d. Information about expected dates of realization of assets and liabilities is useful in
assessing the liquidity and solvency of an entity.

64.Which of the following statements is incorrect regarding offsetting of assets and


liabilities in the statement of financial position?
a. Assets and liabilities in the statement of financial position may be offset in general.
b. Offsetting does not give rise to gain or loss recognition, which distinguishes it from
the derecognition of an instrument
c. An entity shall not offset assets and liabilities or income and expenses, unless
required or permitted by a PFRS.
d. Financial assets and financial liabilities shall be offset if the entity has both the legal
right of offset and the intention of settling the asset and liabilities at a net basis.

65.Which of the following statements is incorrect?


a. If a line item is not individually material, it is aggregated with other items either in
those statements or in the notes. An item that is not sufficiently material to warrant
separate presentation in those statements may warrant separate presentation in the
notes.
b. Changes in the general purchasing power of the peso are not reflected in the basic
financial statements.
c. An important characteristic of financial statements is that the information they
contain describes the future and decision making is therefore based on the past.
d. Assets and liabilities in the statement of financial position should not be offset even
if a legal right of offset exists.

66.The elements of financial statements are measured using a mixture of costs and values.
The values used pertain to the end of reporting period.
a. Therefore, financial statement elements do not describe the future but rather
reflects costs as of transaction dates and values as of end of reporting period.
b. Therefore, financial statement elements describe the future rather than reflecting
costs as of transaction dates and values as of end of reporting period.
c. Therefore, financial statement elements are automatically adjusted for changes in
the general purchasing power of the presentation currency.
d. Therefore, financial statements are useful for predicting both the future and the
past.

67.Financial statements describe the results of past events and transactions.


a. Therefore, financial statements are not useful in predicting the future.
b. Therefore, financial statements can be used to make predictions about outcomes of
future transactions.
c. Therefore, financial statements are useful in predicting the past but not the future.
d. Therefore, most accountants are historical.

68.Which of the following statements is incorrect?


a. Financial statements are normally prepared using the “stable monetary concept.”
b. The stable monetary concept assumes that the general purchasing power of money
used in financial statement elements measurement remains relatively constant.
c. Financial statements present the same levels of purchasing power of the peso.
d. Financial statements generally ignore changes in the general purchasing power of
the peso.

20
69.Since financial statement elements are normally presented in their respective historical
costs or costs at transaction dates or values at the end of reporting period,
a. they do not reflect the same levels of purchasing power because the levels of
purchasing power vary between transaction dates. Therefore, the financial
statements must be adjusted to the purchasing power current as of end of reporting
period.
b. they do not reflect the same levels of purchasing power because the levels of
purchasing power vary between transaction dates. Nonetheless, the changes in
purchasing power are disregarded because of the concept of “stable monetary
assumption.”
c. they do reflect the same levels of purchasing power because the levels of purchasing
power do not vary between transaction dates.
d. financial statement elements are normally measured at fair value at each reporting
date. Therefore, there is no need to make adjustments for inflation.

Structure and content of financial statements in general


70.PAS 1 requires entities to present an additional statement of financial position in
certain instances. The purpose of this provision is
a. to provide information that is useful in analyzing an entity’s financial statements
b. to promote convergence with US GAAP (FASB)
c. to present information that otherwise would be concealed in the notes
d. to comply with the principle of consistency

71.PAS 1 requires an entity to present a statement of financial position as at the beginning


of the preceding period in a complete set of financial statements in all of the following
instances, except
a. when the entity applies an accounting policy retrospectively
b. when the entity makes a retrospective restatement
c. when the entity reclassifies items in the financial statements
d. when the entity makes a change in accounting estimate

72.This information, along with other information in the notes, assists users of financial
statements in predicting the entity’s future cash flows and, in particular, their timing
and certainty.
a. assets, liabilities, and equity
b. income and expenses, including gains and losses
c. contributions by and distributions to owners in their capacity as owners
d. cash flows
e. all of the above

73.A complete set of financial statements comprises:


I. a statement of financial position as at the end of the period
II. a statement of profit or loss and other comprehensive income for the period
III. an income statement for the period without a statement of comprehensive
income
IV. a statement of changes in equity for the period
V. a statement of cash flows for the period
VI. notes, comprising a summary of significant accounting policies and other
explanatory information
VII. comparative information in respect of the preceding period
VIII. a statement of financial position as at the beginning of the preceding period
when an entity applies an accounting policy retrospectively or makes a
retrospective restatement of items in its financial statements, or when it
reclassifies items in its financial statements.
a. I, III, IV, V, VI c. I, II, IV, V, VI, VII, VIII
b. I, III, IV, V, VI, VII d. all of these

21
74.Which of the following statements is incorrect concerning definition of terms provided
under PAS 1 Presentation of Financial Statements?
a. Owners are holders of instruments classified as equity.
b. Profit or loss is the total of income less expenses, including the components of other
comprehensive income.
c. Reclassification adjustments are amounts reclassified to profit or loss in the current
period that were recognized in other comprehensive income in the current or
previous periods.
d. Total comprehensive income is the change in equity during a period resulting from
transactions and other events, other than those changes resulting from transactions
with owners in their capacity as owners.
e. Total comprehensive income comprises all components of ‘profit or loss’ and of
‘other comprehensive income’.

75.Which of the following is correct regarding the use of terminology under PAS 1
Presentation of Financial Statements?
a. Entities are required by PAS 1 to use the terms “other comprehensive income,”
“profit or loss” and “total comprehensive income.” The use of the term “net income”
is prohibited.
b. Entities are required by PAS 1 to use the term “statement of financial position” in
presenting its assets, liabilities and equity as of a given point of time. The use of the
term “balance sheet” is prohibited.
c. Entities are required by PAS 1 to use the term “statement of cash flows” in
presenting the sources and uses of cash for a period. The use of the term “cash flow
statement” is prohibited.
d. Entities may use the terms “balance sheet,” “net income,” “income statement,” and
“cash flow statement” to describe their financial statements and other terms
provided they are not misleading.

76.When an entity applies an accounting policy retrospectively or makes a retrospective


restatement of items in its financial statements or when it reclassifies items in its
financial statements, it shall present, as a minimum,
a. three statements of financial position, three of each of the other statements, and two
related notes.
b. two statements of financial position, two of each of the other statements, and related
notes.
c. three statements of financial position, two of each of the other statements, and
related notes.
d. three statements of financial position, two of each of the other statements, and two
related notes.

77.FORRAY RAID PILLAGE Co. made a correction of prior period error during the current
year. When FORRAY prepares financial statements for the current year, it shall present
statement of financial position as at
I. the end of the current period
II. the end of the preceding period
III. the beginning of the preceding period.
a. I, II b. II, III c. I d. I, II, III

78.Which of the following statements is incorrect?


a. PFRSs apply to financial statements and to other information presented in an annual
report or other document.
b. PAS 1 requires an entity to present assets and liabilities in order of liquidity only
when a liquidity presentation provides information that is reliable and is more
relevant than a current/non-current presentation.
c. Financial statements are often made more understandable by presenting
information in thousands or millions of units of the presentation currency. This is

22
acceptable as long as the level of rounding in presentation is disclosed and material
information is not omitted.
d. Financial statements shall be presented at least annually.

79.Each component of the financial statements shall be identified clearly. In addition, the
following information shall be displayed prominently, and repeated when it is
necessary for a proper understanding of the information presented:
I. the name of the reporting entity or other means of identification, and any change
in that information from the preceding reporting date
II. whether the financial statements cover the individual entity or a group of
entities
III. the balance sheet date or the period covered by the financial statements,
whichever is appropriate to that component of the financial statements
IV. the presentation currency, as defined in PAS 21 The Effects of Changes in Foreign
Exchange Rates
V. the level of rounding used in presenting amounts in the financial statements.
a. I, II, III b. I, II, III, IV c. I, II, IV, V d. I, II, III, IV, V

80.The information provided by financial reporting pertains to:


a. individual business enterprises and the economy as a whole, rather than to
industries or to members of society as consumers
b. individual business enterprises, industries and the economy as a whole, rather than
to members of society as consumers
c. individual enterprises, rather than to industries of the economy as a whole or to
members of society as consumers
d. individual business enterprises and industries rather than to the economy as a
whole or to members of society as consumers
(AICPA)

81.Which of the following reports is not a component of the financial statements according
to PAS 1?
a. Statement of financial position or balance sheet. c. Director’s report.
b. Statement of changes in equity. d. Notes
(Adapted)

82.The financial statements most frequently provided include all of the following except
the
a. balance sheet c. statement of cash flows
b. income statement d. statement of retained earnings.

83.According to PAS1 Presentation of Financial Statements, the notes within the financial
statements contain information in addition to that presented in which of the following?
I. Report on sustainability
II. Chairman's statement
III. Statement of financial position
IV. Statement of financial performance
a. I and II b. II and III c. III and IV d. I, III, and IV
(ACCA)

84.Which of the following are included in a complete set of financial statements, according
to PAS1 Presentation of Financial Statements?
I. A statement by the board of directors of compliance with local legislation
II. A statement of changes in equity
III. Summarized statements of financial position for the last five years
IV. A statement of cash flows
a. I, II, and III b. II and IV c. I and IV d. all of these
(ACCA)

23
85.Which of the following financial statements is concerned with the entity at a point in
time?
a. Statement of changes in financial position c. Income statement
b. Cash flow statement d. Balance sheet

86.In which of the following instances an entity is not required to present a statement of
financial position as at the beginning of the preceding period?
a. the entity applies an accounting policy retrospectively
b. the entity makes a retrospective restatement of items in its financial statements
c. the entity changes its financial reporting period
d. the entity makes reclassification adjustments as defined in PAS 1

87.Which of the following statements is correct regarding PAS 1 Presentation of Financial


Statements?
a. Fair presentation requires the faithful representation of the effects of transactions,
other events and conditions in accordance with the definitions and recognition
criteria for assets, liabilities, income and expenses set out in PAS 1.
b. The application of PFRSs, with additional disclosure when necessary, is presumed to
result in accurate financial statements.
c. An entity whose financial statements comply with PFRSs shall make an explicit and
unreserved statement of such compliance in the notes. An entity shall not describe
its financial statements as complying with the PFRSs unless they comply with all the
requirements of PFRSs.
d. When presenting the statement of financial position, the shareholders’ equity
should always be presented after assets and liabilities.

88.Which of the following statements is incorrect regarding PAS 1 Presentation of Financial


Statements?
a. Entities are encouraged to present the analysis of expenses as to their nature or
function in the statement of profit or loss and other comprehensive income or in the
separate income statement (if presented).
b. The “function of expense” method of presenting expenses is also called the “cost of
sales” method.
c. An entity classifying expenses by function shall disclose additional information on
the nature of expenses, including depredation and amortization expense and
employee benefits expense.
d. According to PAS 1, a statement of retained earnings can be prepared in lieu of
statement of changes in equity.

89.Which of the following statements is incorrect regarding PAS 1 Presentation of Financial


Statements?
a. An entity shall disclose, either in the statement of financial position or in the notes,
further subclassifications of the line items presented, classified in a manner
appropriate to the entity’s operations.
b. An entity need not provide a specific disclosure required by a PFRS if the
information is not material.
c. The detail provided in subclassifications depends on the requirements of PFRSs and
on the size, nature and function of the amounts involved.
d. Disclosures are presented only in the notes.

90.Which of the following correctly relate to the provisions of PAS 1?


I. PAS 1 provides that a description of the accounting policies adopted by the
reporting entity is not required to be presented in the financial statements.
II. The basic time period for which financial statements are presented is less than a
year, preferably semi-annually.

24
III. When funds are simply segregated by the entity for the purpose of discharging a
liability, it is proper to offset the liability against the segregated funds.
IV. Financial statements for prior periods included for comparative purposes should be
presented as previously presented, except when a changed presentation is
warranted.
V. A statement of cash flows will be omitted in some circumstances, for example from
financial statements restricted for internal use and financial statements prepared
for special purposes only.
a. I, IV, V b. III, V c. IV, V d. III, IV, V

91.The following statements accurately describe the general features prescribed in PAS 1:
I. an entity is viewed as continuing in operation in the absence of evidence to the
contrary
II. financial reporting is primarily concerned with reporting information on economic
resources and obligations and changes in them
III. the financial reporting process provides information about the economic activities
of an entity for specified time periods that are shorter than the life of the entity
IV. financial reporting measurements are primarily based on prices at which economic
resources and obligations are exchanged
V. financial reporting necessarily involves informed judgment
a. all of these c. II, III, IV and V only
b. I, II, III, and IV only d. III, IV and V only

92.In cases of any departure from conformity with the PFRSs the CPA must indicate:
(Item #1) Nature of departure; (Item #2) Approximate effects thereof
a. No, Yes b. Yes, No c. No, No d. Yes, Yes
(Adapted)

Statement of financial position


93.The statement of financial position may be presented
I. based on current and noncurrent classification
II. based on liquidity
III. mixture of current and noncurrent and liquidity
a. I b. I, II c. I, II, III d. II

94.Regarding the presentation of the statement of financial position, which of the following
statements is correct?
a. PAS 1 requires that the line item “Property, plant and equipment” be the first line
item to be presented in the financial statements.
b. The use of different measurement bases for different classes of assets suggests that
their nature or function differs and, therefore, that an entity presents them as
separate line items.
c. When the statement of financial position is presented using the current and
noncurrent classification, the line item “Cash and cash equivalents” should always
be presented first under the current assets section.
d. When an entity opts not to present its statement of financial position using the
current and noncurrent classification, no disclosure in the notes is necessary for
assets and liabilities expected to be realized or settled within 12 months and beyond
12 months after the reporting date.

95.As of year-end, an entity had unsettled income taxes to the government. Such liability is
charged to the “Income taxes payable” account and is expected to be settled within
twelve months after the reporting date. In the statement of financial position prepared
as of year-end, the liability for the taxes is normally shown as
a. a separate line item in the current liabilities section
b. included in “Trade and other payables” in the current liability section
c. a separate line item in the noncurrent liabilities section

25
d. a separate line item in the current assets section

96.When the entity’s normal operating cycle is not clearly identifiable, it is assumed to be
a. 12 months b. 3 months c. 6 months d. no assumption

97.This refers to presenting separately on the face of financial statements items which are
material and combining immaterial items with similar items.
a. Offsetting c. Fair presentation
b. Materiality and aggregation d. Frequency of reporting

98.An asset shall be classified as current when it satisfies any of the following criteria,
except
a. it is expected to be realized in, or is intended for sale or consumption in, the entity’s
normal operating cycle
b. it is held primarily for the purpose of being traded
c. it is expected to be realized within twelve months after the balance sheet date
d. it is cash or a cash equivalent that is restricted

99.All of the following statements are correct, except


a. The operating cycle of an entity is the time between the acquisition of assets for
processing and their realization in cash or cash equivalents.
b. When the entity’s normal operating cycle is not clearly identifiable, its duration is
assumed to be twelve months.
c. Current assets include assets (such as inventories and trade receivables) that are
sold, consumed or realized as part of the normal operating cycle even when they are
not expected to be realized within twelve months after the balance sheet date.
d. Some liabilities are part of the working capital used in the entity’s normal operating
cycle. Such operating items are classified as current liabilities even if they are due to
be settled more than twelve months after the balance sheet date.
e. All of these are correct.

100. A liability shall be classified as current when it satisfies any of the following criteria,
except
a. it is expected to be settled in the entity’s normal operating cycle
b. it is held primarily for the purpose of being traded
c. it is due to be settled within twelve months after the balance sheet date
d. the entity has an unconditional right to defer settlement of the liability for at least
twelve months after the balance sheet date.

101. If an entity expects, and has the discretion, to refinance or roll over an obligation for
at least twelve months after the reporting period under an existing loan facility, it
classifies the obligation as non-current,
a. even if it would otherwise be due within a shorter period.
b. only if the remaining period to maturity of the original obligation exceeds 12
months from the end of reporting period.
c. only if the original maturity of the obligation is longer than 12 months.
d. choices b and c

102. When an entity breaches an undertaking under a long-term loan agreement on or


before the balance sheet date with the effect that the liability becomes payable on
demand, (choose the incorrect statement)
a. the liability is classified as current, even if the lender has agreed, after the balance
sheet date and before the authorization of the financial statements for issue, not to
demand payment as a consequence of the breach
b. the liability is classified as non-current, even if the lender has agreed, after the
balance sheet date and before the authorization of the financial statements for issue,
not to demand payment as a consequence of the breach

26
c. The liability is classified as current because, at the balance sheet date, the entity
does not have an unconditional right to defer its settlement for at least twelve
months after that date.
d. The liability is normally classified as current, however, the liability is classified as
non-current if the lender agreed by the balance sheet date to provide a period of
grace ending at least twelve months after the balance sheet date, within which the
entity can rectify the breach and during which the lender cannot demand immediate
repayment.

103. The judgment on whether additional items are presented separately on the
statement of financial position is based on an assessment of:
I. the nature and liquidity of assets
II. the function of assets within the entity
III. the amounts, nature and timing of liabilities
IV. the need for external financing
a. I, III b. I, II c. I, II, III d. II, III, IV

104. Banks and other financial institutions present their statement of financial position
based on
a. current and noncurrent classification c. nature of expense
b. liquidity d. function of expense

105. Which of the following statements is correct?


a. Normally, all items of income and expense recognized in a period are included in
profit or loss. This includes the effects of changes in accounting policies and
correction of prior period errors.
b. Other Standards deal with items that may meet the Conceptual Framework
definitions of income or expense but are usually excluded from profit or loss.
c. The use of different measurement bases for different classes of assets suggests that
their nature or function differs and, therefore, that they should be presented as one
line item.
d. An entity shall not present any items of income and expense as extraordinary items
on the face of the income statement but it may do so in the notes.
e. Entities classifying expenses by nature shall disclose additional information on the
function of expenses, including depreciation and amortization expense and
employee benefits expense.

106. In respect of loans classified as current liabilities, if the following events occur
between the balance sheet date and the date the financial statements are authorized for
issue, these events qualify for disclosure as non-adjusting events in accordance with
PAS 10 Events After the Reporting Period.
I. refinancing on a long-term basis
II. rectification of a breach of a long-term loan agreement
III. the receipt from the lender of a period of grace to rectify a breach of a long-term loan
agreement ending at least twelve months after the balance sheet date
a. I, II b. II c. II, III d. I, II, III

107. The main purpose of the statement of financial position is to reflect


a. the fair value of the entity's assets at some point in time.
b. the status of the entity 's assets in case of forced liquidation of the firm.
c. items of value, debts and net worth.
d. the firm's potential for growth in stock values in the stock market.
(Adapted)

108. As a minimum, the face of the statement of financial position shall include all of the
following line items, except
a. Biological assets c. Deferred tax assets and liabilities

27
b. Investment property d. Goodwill
(Adapted)

109. Which one of the following is not required to be presented as minimum information
on the face of the statement of financial position, according to PAS 1?
a. Investment property.
b. Investments accounted under the equity method.
c. Biological assets.
d. Contingent liability.
(Adapted)

110. Which of the following item is not an element of working capital?


a. temporary investments c. good-in process
b. treasury stock d. cash in bank
(RPCPA)

111. The ratio of total cash, trade receivables and marketable securities to current
liabilities is
a. current ratio c. working capital
b. acid test ratio d. receivable turnover
(RPCPA)

112. The following statements relate to the concept of asset. Which is false?
a. The primary characteristic of an asset is its capacity to provide the entity with
probable economic benefits.
b. There is an expiration of economic benefits when an asset is used up in the
production of another asset.
c. A business entity may recognize an asset even if it does not possess legal title.
d. The assets of an entity result from past transactions or other past events.
(Adapted)

113. According to PAS1 Presentation of Financial Statements, which of the following must
be included in an entity's statement of financial position?
I. Investment property
II. Number of shares authorized
III. Provisions
IV. Shares in an entity owned by that entity
a. I, II, and III b. I and III c. III and IV d. all of these
(ACCA)

114. According to PAS1 Presentation of Financial Statements, which of the following must
be included in an entity's statement of financial position?
I. Cash and cash equivalents
II. Property, plant and equipment analyzed by class
III. Share capital and reserves analyzed by class
IV. Deferred tax
a. I, II, and III b. I and III c. I and IV d. all of these
(ACCA)

115. Are the following statements true or false, according to PAS1 Presentation of
Financial Statements?
I. Biological assets should be shown in the statement of financial position.
II. The number of shares authorized for issue should be shown in the statement of
financial position or the statement of changes in equity or in the notes.
a. False, False b. False, True c. True, False d. True, True
(ACCA)

28
116. In which section of the statement of financial position should cash that is restricted
to the settlement of a liability due 18 months after the reporting period be presented,
according to PAS1 Presentation of Financial Statements?
a. Current assets c. Non-current liabilities
b. Equity d. Non-current assets
(ACCA)

117. In which section of the statement of financial position should employment taxes that
are due for settlement in 15 months' time be presented, according to PAS1 Presentation
of Financial Statements?
a. Current liabilities c. Non-current liabilities
b. Current assets d. Non-current assets
(ACCA)

118. DECRY TO BELITTLE Company has a loan due for repayment in six months' time,
but DECRY has the option to refinance for repayment two years later. DECRY plans to
refinance this loan. In which section of its statement of financial position should this
loan be presented, according to PAS1 Presentation of Financial Statements?
a. Current liabilities c. Non-current liabilities
b. Current assets d. Non-current assets
(ACCA)

119. A liability shall be classified as current in all of the following instances, except
a. It is a non-trade payable due to be settled within twelve months after balance sheet
date or within the normal operating cycle, whichever is longer.
b. It is expected to be settled in the entity’s normal operating cycle.
c. It is held primarily for the purpose of being traded.
d. The entity does not have an unconditional right to defer settlement of the liability
for at least twelve months after the balance sheet date.

120. In case of a breach of a loan covenant with the effect that the liability becomes
payable on demand, the liability is classified as noncurrent when
a. It is not probable that further breaches or violations will occur within twelve
months of the balance sheet date.
b. The lender has agreed, prior to the approval of the financial statements, not to
demand payment as a consequence of the breach.
c. The lender has agreed after the balance sheet date and before the statements are
authorized for issue to provide a grace period ending at least twelve months after
the balance sheet date.
d. The lender has agreed on or before the balance sheet date to provide a grace period
ending at least twelve months after the balance sheet date for the entity to rectify
the breach.

121. A currently maturing long-term debt is classified as noncurrent when


a. The borrower has the discretion to refinance or roll over the liability for at least
twelve months after the reporting period under the existing loan facility.
b. The lender has the discretion to refinance or roll over the liability for at least twelve
months after the balance sheet date under the existing loan facility.
c. An agreement to reschedule payment on a long-term basis is completed after the
reporting period but before the financial statements are authorized for issue.
d. Equity security has in fact been issued after the reporting period and before the
statements are authorized for issue the proceeds from which are used to settle the
liability at maturity date.

122. The basis for classifying assets as current or noncurrent is the period of time
normally elapsed from the time the accounting entity expends cash to the time it
converts

29
a. Inventory back into cash, or 12 months, whichever is shorter.
b. Receivables back into cash, or 12 months, whichever is longer.
c. Tangible fixed assets back into cash, or 12 months. whichever is longer.
d. Inventory back into cash, or 12 months, whichever is longer.
(Adapted)

123. Which of the following is/are a limitation(s) of a Balance Sheet?


I. It does not contain certain assets and liabilities despite its claim to be the statement
of all assets and liabilities
II. Some factors, which have a vital bearing on the earnings of the entity, are not
disclosed
III. Personal judgment plays a great part in determining the figures on the balance
sheet.
a. I b. II and III c. III d. all of these
(Adapted)

124. Which of the following accounts would not be classified under current assets on the
balance sheet?
a. Supplies c. 90-day Note Receivable
b. Prepaid Insurance d. 2-year Note Receivable
(Adapted)

125. In Philippine settings, current assets and current liabilities are most commonly
presented in the balance sheet in the order of
a. materially c. chronologically
b. liquidity d. alphabetically

126. When classifying assets as current and non-current for reporting purposes,
a. The amount at which current assets are carried and reported must reflect realizable
cash values.
b. Prepayments for items such as insurance or rent are included in an “other assets”
group rather than as current assets as they will ultimately be expensed.
c. The time period by which current assets are distinguished from non-current assets
is determined by the seasonal nature of the business.
d. Assets are classified as current if they are reasonably expected to be realized in cash,
or consumed during the normal operating cycle.
(Adapted)

127. The balance sheet allows investors to assess all of the following except
a. How efficient the company’s assets are used.
b. The liquidity and financial flexibility of the company.
c. The capital structure of the company
d. The net realizable value of the company.
(Adapted)

128. Most components of the balance sheet are reported at


a. historical cost plus allowance for inflation. c. historical cost.
b. fair value. d. replacement value.
(Adapted)

129. Which statement is correct concerning presentation of information on the face of


the statement of financial position?
I. Additional line items, headings and subtotals shall be presented on the face of the
balance sheet when such presentation is relevant to an understanding of the entity’s
financial position.
II. PAS 1 does not prescribe the order or format in which items are to be presented.
a. I only b. II only c. Both I and II d. Neither I nor II

30
130. For accounting purposes, the “operating cycle concept”
a. Causes the distinction between current and noncurrent items to depend on whether
they will affect cash within one year.
b. Permits some assets to be classified as current even though they are expected to be
realized beyond one year from the end of the reporting period.
c. Has become obsolete.
d. Affects the income statement but not the balance sheet.
(AICPA)

131. The operating cycle of a business is that span of time which


a. Coincides with economy’s business cycle which runs from one trough of the
company’s business activity to the next.
b. Corresponds with its natural business year which runs from one trough of the
particular firm’s business activity to the next.
c. Is set by the industry’s trade association usually on an average length of time for all
firms which are members of the association.
d. Runs from cash disbursement for items of inventory through their sale to the
realization of cash from sale.
(AICPA)

132. The current asset section of a balance sheet most likely will include:
a. all deferred income taxes resulting from interperiod income tax allocation
b. goodwill arising in a business combination accounted for as acquisition
c. rent receivable for a security deposit on a lease
d. a receivable from a customer not collectible for over one year
(Adapted)

133. Which one of the following assets is similar to certain current assets, but is not one?
a. Accounts receivable c. long term payment of expenses
b. Prepaid insurance d. short-term investment in equity security
(AICPA)

134. A corporation paid a six year insurance premium on January 1, Year 1 ,for P12,000.
It recorded the prepayment in two asset accounts –one with a P2,000 debit balance and
one with a P10,000 debit balance. Under which of the following captions should the
account with the P10,000 balance be classified on a balance sheet dated January, Year
1?
a. Operational assets c. Deferred charges
b. Other assets d. Current assets
(AICPA)
135. Which of the following statements is true?
a. deferred charges are distinguished from prepaid expenses on the basis of the time
over which their benefits will be realized.
b. working capital is a very useful measure because it reveals how much would be left
if all the assets were to be sold and the proceeds were used to pay all the current
liabilities.
c. the normal operating cycle of a business is the average length of the time from cash
expenditure, to inventory, to sale and back to accounts receivable.
d. Retained earnings often is restricted (or appropriated) to ensure that cash will be
available for plant expansion earnings are restricted the cash cannot be spent.

136. A public utility reports noncurrent assets as the first item on its balance sheet. This
is an example of
a. Improper statement presentation c. Industry practice
b. Conservatism d. Substance over form
(AICPA)

31
137. Deferred tax assets and liabilities shall be classified on the balance sheet as
a. Current c. Partly current and partly noncurrent
b. Noncurrent d. Part of equity

138. A liability shall be classified as a current liability when it satisfies any of the
following criteria, except
a. It is expected to be settled in the entity’s normal operating cycle.
b. It is primarily held for the purpose of being traded.
c. It is expected to be realized within twelve months after the balance sheet date.
d. It is cash or a cash equivalent that is restricted from being exchanged or used to
settle a liability for at least twelve months after the balance sheet date.
(Adapted)

139. Which obligations are classified as current liabilities even if they are due to be
settled after more than twelve months from the end of the reporting period?
I. Trade payables and accruals for employee and other operating cost that are part of
the entity’s working capital
II. A portion of long-term interest-bearing liabilities
III. Bank overdrafts arising from settlements of purchases of inventory
IV. Dividends payable
a. I and III b. I, III, and IV c. III only d. all of these

140. When an entity breaches a covenant under a long-term loan agreement on or before
the balance sheet date with the effect that the liability becomes payable on demand, the
liability is classified as noncurrent when
I. The lender has agreed after the balance sheet date and before the financial
statements are authorized for issue not to demand payment as a consequence of the
breach.
II. The lender has agreed on or before the balance sheet date to provide a grace period
ending at least twelve months after the balance sheet date for the entity to rectify
the breach.
a. Both I and II b. Neither I nor II c. I only d. II only

141. A corporation owed the following notes payable, which will mature during the
coming year. The corporation plans to settle the notes as follows:
Note payable A: Refinance by issuing a new 10 year bond
Note payable B: Give the holder merchandise inventory
Note payable C: Give the creditor a long term investment in equity instruments of
another entity

Which note is properly classified as a current liability?


a. Note payable A c. Note payable C
b. Note payable B d. All are current liabilities
(Adapted)

142. Which of the following statements is (are) correct?


I. Presentation of assets or liabilities by order of liquidity can be chosen anytime
should management so desires it.
II. A liability held primarily for the purpose of being traded is to be classified as current
a. I is true b. I and II are true c. II is true d. I and II are not true

143. The operating cycle of an enterprise


a. is the time between the acquisition of materials entering into a process and their
realization in cash or an instrument that is readily convertible into cash.
b. causes the distinction between current and noncurrent items to depend on whether
they will affect cash within one year.

32
c. is the period of time normally elapsed from the time the enterprise expends cash to
the time it converts trade receivables back into cash.
d. Is a period of one year.
(Adapted)

144. An operating cycle


a. is twelve months or less in length
b. is the average time required for a company to collect its receivable
c. is used to determine current assets when it is longer than one year
d. starts with inventory and ends with cash
(Adapted)

145. Working capital is


a. The group assets which enables the business to operate profitably
b. Capital which has been reinvested in the business.
c. Unappropriated retained earnings.
d. Current assets less current liabilities.
(Adapted)

146. How is working capital defined?


a. Current assets minus current liabilities
b. Total current assets
c. Capital contributed by shareholders
d. Capital contributed by shareholders plus retained earnings
(Adapted)

147. Of the following items, the one which should be classified as a current asset is
a. Trade installment receivables normally collectible in 18 months.
b. Cash designated for the redemption of callable preferred stock.
c. Cash surrender value of a life insurance policy of which the company is beneficiary.
d. A deposit on machinery ordered, delivery of which will be made within sixteen
months.
(Adapted)

148. According to PAS 1, a liability shall be classified as current when (choose the
incorrect one)
a. It is expected to be settled in the entity’s normal operating cycle.
b. It is held primarily for the purpose of being traded.
c. It is due to settled within twelve months after balance sheet date or within the
normal operating cycle, whichever is longer.
d. The entity does not have an unconditional right to defer settlement of the liability
for at least twelve months after the balance sheet date.

149. A currently maturing long-term debt is classified as noncurrent when


a. an agreement to reschedule payment on a long-term basis is completed after the
end of reporting period but before the statements are authorized for issue.
b. equity security has in fact been issued after the end of reporting period but before
the statements are authorized for issue, the proceeds from which are used to settle
the liability on the date of maturity.
c. the lender has the discretion to refinance or roll over the liability for at least twelve
months after the end of reporting period under an existing loan facility.
d. the borrower has the discretion to refinance or roll over the liability for at least
twelve months after the end of reporting period under an existing loan facility.

150. Some borrowing agreements incorporate covenants which have the effect that the
liability becomes payable on demand if certain conditions related to the covenants are
breached. In these circumstances, the liability is classified as noncurrent when

33
a. The lender has agreed, prior to the approval of the financial statements, not to
demand payment as a consequence of the breach.
b. It is not probable that further breaches or violations will occur within twelve
months of the balance sheet date.
c. The lender has agreed after the balance sheet date and before the statements are
authorized for issue to provide a grace period ending at least twelve months after
the balance sheet date.
d. The lender has given the lender, on or before the balance sheet date, a grace period
to rectify the breach ending at least twelve months after the balance sheet date.

151. The current assets section of a balance sheet should never include
a. a receivable from a customer not collectible for over one year.
b. the premium paid on short-term bond investment.
c. goodwill arising from the purchase of a going business not expected to be disposed
of within 12 months from end of reporting period.
d. customers' accounts with credit balances.
(Adapted)

152. PAS 1 requires an entity to include in a complete set of financial statements a


statement of financial position as at the beginning of the preceding period whenever the
entity retrospectively applies an accounting policy or makes a retrospective
restatement of items in its financial statements, or when it reclassifies items in its
financial statements. The purpose of this requirement is
a. to discourage auditors from subsuming in retained earnings unaccounted
differences in accounts and required reconciliations
b. to promote vigilance on entities over errors and to discourage frequent changes in
accounting policies
c. to provide information that is useful in analyzing an entity’s financial statements
d. all of these

Statement of profit or loss and other comprehensive income


153. Which of the following statements is incorrect?
a. Comprehensive income includes all revenues, expenses, gains, losses, and prior
period adjustments.
b. PAS 1 requires an entity to disclose income tax relating to each component of other
comprehensive income.
c. The presentation of disclosures on dividends in the statement of profit or loss and
other comprehensive income is not permitted.
d. An entity may present components of other comprehensive income gross of tax or
net of tax on the face of the statement of profit or loss and other comprehensive
income.

154. PAS 1 requires an entity to disclose reclassification adjustments and income tax
relating to each component of other comprehensive income. Reclassification
adjustments are
a. the amounts reclassified to profit or loss in the current period that were currently or
previously recognized in other comprehensive income.
b. the amounts reclassified to total comprehensive income that were previously
recognized in equity.
c. the amounts that previously caused the statement elements to be misstated.
d. the amounts that previously recognized using an inappropriate accounting policy.

155. Which of the following statements is correct regarding the provisions of PAS 1?
a. PAS 1 requires the presentation of an income statement that includes items of
income and expense recognized in profit or loss. Items of income and expense not
recognized in profit or loss should be presented in the statement of changes in
equity, together with owner changes in equity.

34
b. PAS 1 labels the statement of changes in equity comprising profit or loss, other
items of income and expense and the effects of changes in accounting policies and
correction of errors as ‘statement of recognized income and expense.
c. PAS 1 requires an entity to disclose income tax relating to each component of other
comprehensive income.
d. PAS 1 permits non-owner changes in equity to be presented together with owner
changes in equity in the statement of changes in equity.

156. Identify the correct statement.


a. PAS 1 does precludes presenting financial statements based on a 53-week period or
longer, because the resulting financial statements are likely to be materially
different from those that would be presented for one year.
b. When the method of presentation adopted by an entity is the classification based on
liquidity, the entity need not disclose amounts of assets or liabilities expected to be
recovered or settled after more than twelve months.
c. For financial institutions, such as banks, a presentation of assets and liabilities based
on the current/noncurrent presentation is more appropriate.
d. An entity is permitted to present some of its assets and liabilities using a
current/non-current classification and others in order of liquidity.
e. The Function of Expense Method should be used in income statement presentation
and the Nature of Expense should be used in the notes. Entities are prohibited from
using the Nature of Expense Method in presenting of income statements.

157. As a minimum, the face of the income statement shall include line items that present
the following amounts for the period:
I. revenue
II. finance costs
III. share of the profit or loss of associates and joint ventures accounted for using the
equity method
IV. tax expense
V. a single amount comprising the total of (i) the post-tax profit or loss of
discontinued operations and (ii) the post-tax gain or loss recognized on the
measurement to fair value less costs to sell or on the disposal of the assets or
disposal group(s) constituting the discontinued operation
VI. a single amount comprising the total of the post-tax profit or loss on early
extinguishment of long-term financial debts
VII. profit or loss
a. I, II, V, VII c. I, II, III, IV, V, VII
b. I, II, III, IV, V, VII d. all of these

158. When an entity opts to present the income statement classifying expenses by
function, which of the following is not required to be disclosed as “additional
information”?
a. Depreciation expense. c. Director’s remuneration.
b. Employee benefits expense. d. Amortization expense.
(Adapted)

159. Which of the following items is not classified as “other comprehensive income”?
a. Extraordinary gains from extinguishment of debt
b. Foreign currency translation adjustments
c. Minimum pension liability equity adjustment for a defined-benefit pension plan
d. Unrealized gains for the year on FVOCI investments
(AICPA)

160. Which of the following statements is correct regarding reporting comprehensive


income?

35
a. Accumulated other comprehensive income is reported in the shareholders’ equity
section of the statement of financial position.
b. A separate income statement is required.
c. Comprehensive income must include all changes in shareholders’ equity for the
period.
d. Comprehensive income is reported in the year-end statements but not in the interim
statements.

161. Which of the following statements is incorrect?


a. The choice of method of presenting expenses is not irrevocable. If the other method
is expected to present more relevant information, a change should made. However,
changes between permitted accounting policies should not be made so often so as
not to violate the principle of consistency.
b. In a single-statement presentation, all items of income and expense are presented
together in one statement.
c. In a two-statement presentation, the first statement (‘income statement’) presents
income and expenses recognized in profit or loss and the second statement
(‘statement of comprehensive income’) begins with profit or loss and presents, in
addition, items of income and expense that PFRSs require or permit to be
recognized outside profit or loss.
d. Dividends received from investments in associates accounted for using the equity
method are not recognized in profit or loss but may be recognized in other
comprehensive income.
162. Which of the following statements is incorrect?
a. An investor in an associate may present in its other comprehensive income its share
in the associate’s other comprehensive income.
b. An investor in an associate shall present in profit or loss its share in the associate’s
profit or loss.
c. An investor in an associate shall not recognize dividends received from the associate
in its profit or loss but may recognize the dividends received in its other
comprehensive income.
d. Dividends received by an investor from its associate are accounted for as reduction
in the investment in associate account.

163. Which of the following is not included in comprehensive income?


a. translation differences related to foreign operations
b. fair value gains or losses on FVOCI securities.
c. fair value gains or losses on FVPL securities.
d. gains on reissuance of treasury shares

164. Which of the following is not included in comprehensive income?


a. Remeasurements of the net defined benefit liability (asset)
b. revaluation gains on property, plant, and equipment
c. fair value gains or losses on investment properties.
d. gains on retirement of ordinary shares

165. Income and expenses for the period is presented in


a. a statement of profit or loss and other comprehensive income
b. a separate income statement and a statement of comprehensive income
c. income statement only
d. a or b

166. Components of other comprehensive income are presented in the


a. statement of profit or loss and other comprehensive income
b. separate income statement
c. notes
d. statement of changes in equity

36
167. It comprises items of income and expense including reclassification adjustments
that are not recognized in profit or loss as required or permitted by other PFRSs.
a. Comprehensive income c. Other comprehensive income
b. Profit or loss elements d. Nominal accounts

168. The components of other comprehensive income exclude


a. Changes in revaluation surplus
b. Remeasurements of the net defined benefit liability (asset)
c. Fair value gains and losses on FVPL securities
d. Effective portion of a cash flow hedge

169. Identify the incorrect statement.


a. An entity shall not present any items of income or expense as extraordinary items,
in the statement of profit or loss and other comprehensive income or the separate
income statement (if presented), but such items may be disclosed in the notes.
b. An entity shall disclose the amount of income tax relating to each component of
other comprehensive income, including reclassification adjustments, either in the
statement of profit or loss and other comprehensive income or in the notes.
c. An entity may present components of other comprehensive income either net of
related tax effects, or before related tax effects with one amount shown for the
aggregate amount of income tax relating to those components.
d. An entity may present reclassification adjustments in the statement of profit or loss
and other comprehensive income or in the notes.

170. An entity sold FVOCI securities during the year. In preparing the statement of profit
or loss and other comprehensive income, the entity should
a. compute the gain by deducting the historical cost of the FVOCI from the proceeds
b. should not present the gain in the statement of profit or loss and other
comprehensive income but in equity
c. make a reclassification adjustment for the cumulative unrealized gains or losses
previously recognized in equity.
d. recognize directly in equity any cumulative unrealized gains or losses on the FVOCI
sold

171. Reclassification adjustments may arise on which of the following?


a. on settlements of employee pension benefits under a defined benefit plan
b. changes in revaluation surplus
c. on derecognition of FVOCI securities
d. when a hedged forecast transaction affects profit or loss

172. Reclassification adjustments will not arise on all of the following, except
a. derecognition of foreign operation
b. changes in remeasurements of the net defined benefit liability (asset)
c. on derecognition of FVOCI
d. changes in revaluation surplus

173. An entity shall present an analysis of expenses recognized in profit or loss using a
classification based on
a. Function b. Nature c. Liquidity d. a or b

174. Increases in revaluation surplus are presented in the statement of profit or loss and
other comprehensive income as
a. income c. revenue
b. item of other comprehensive income d. not presented

37
175. Are the following statements true or false, according to PAS1 Presentation of
Financial Statements?
I. Provisions should be recognized in the statement of financial position.
II. A revaluation surplus on non-current assets should be recognized in profit or loss.
a. False, False b. False, True c. True, False d. True, True
(ACCA)

176. Are the following statements true or false, according to PAS1 Presentation of
Financial Statements?
I. An entity presenting a single statement of profit or loss and other comprehensive
income should present a statement of changes in equity
II. An entity presenting a separate income statement and a statement of
comprehensive income should present a statement of changes in equity
a. False, False b. False, True c. True, False d. True, True
(ACCA)

177. What is the purpose of reporting comprehensive income?


a. To report changes in equity due to transactions with owners.
b. To report a measure of overall enterprise performance.
c. To replace net income with a better measure.
d. To combine income from continuing operations with income from discontinued
operations and extraordinary items.
(AICPA)

178. All of the following are not acceptable methods of reporting other comprehensive
income and its components, except
a. In a statement of comprehensive income. c. In the notes only.
b. In a statement of income d. In a statement of changes in equity.

179. Accounting income is a concept in which:


a. income is measured as the amount of "real wealth" that an entity could consume
during a period and be as well off at the end of that period as it was at the beginning
b. the transactions approach is used to record income, expenses, gains and losses
throughout the reporting period
c. market values adjusted for the effects of inflation or deflation are used to calculate
real wealth
d. income equals the change in market value of the firm's outstanding common stock
for the period.
(AICPA)

180. Which of the following items would cause earnings to differ from comprehensive
income for an enterprise in an industry not having specialized accounting principles?
a. Unrealized loss on investments classified as FVOCI securities.
b. Unrealized loss on investments classified as held for trading securities.
c. Loss on exchange of similar assets.
d. Loss on exchange of dissimilar assets.
(AICPA)

181. Comprehensive income excludes changes in equity resulting from which of the
following?
a. Loss from discontinued operations.
b. Effect of changes in accounting estimate to current operations
c. Dividends paid to stockholders.
d. Unrealized loss on securities classified as FVOCI.

182. Which of the following options for displaying comprehensive income is(are)
preferred under PAS 1?

38
I. A continuation of profit or loss at the bottom of the statement of profit or loss and
other comprehensive income.
II. A separate statement that begins with profit or loss.
III. In the statement of changes in equity.
a. I. b. II. c. II and III. d. I and II.
(Adapted)

183. Which of the following is not classified as other comprehensive income?


a. Remeasurements of the net defined benefit liability (asset).
b. Subsequent decreases of the fair value of FVOCI securities that have been previously
written down as impaired.
c. Decreases in the fair value of securities measured at amortized cost.
d. None of the above.
(Adapted)

184. When a full set of general-purpose financial statements are presented,


comprehensive income and its components should
a. Appear as a part of discontinued operations, extraordinary items, and cumulative
effect of a change in accounting principle.
b. Be reported net of related income tax effect, in total and individually.
c. Appear in a supplemental schedule in the notes to the financial statements.
d. Be displayed in a financial statement that has the same prominence as the other
components of a complete set of financial statements.
(AICPA)

185. Which of the following is deducted from goods available for sale to determine cost of
goods sold?
a. Purchases c. Beginning inventory
b. Freight in d. Ending inventory

186. The account Freight-out is shown on the income statement as a


a. component of the cost of goods sold. c. selling expense.
b. deduction from sales. d. general and administrative expense.

187. Which of the following names is not associated with the income statement?
a. Profit or loss c. Statement of Operations
b. Statement of financial position d. a and c
(Adapted)

188. The income statement heading will specify which of the following?
a. a point in time b. a period of time c. a or c d. a and c
(Adapted)
189. HIATUS BREAK Co. engages in a buy-and-sell business. During the year, HIATUS
prepared two income statements covering the same period. One statement is prepared
using the nature of expense method while the other one is prepared using the function
of expense method. Which of the following statements is correct regarding these
income statements?
a. The nature of expense method income statement will show higher profit than the
function of expense method.
b. The sum of the amounts in the line items “net change in inventories” and “net
purchases” in the nature of expense method income statement equals the amount of
“cost of sales” in the function of expense method income statement.
c. The same disclosure requirements apply whether HIATUS uses the nature of
expense method or the function of expense method.
d. A “gross profit” line item will appear in both income statements.

190. Sales revenue less cost of goods sold is called

39
a. gross profit. c. net earnings.
b. cost of sales. d. earnings before income taxes.
(Adapted)

191. The “bottom line” in a statement of profit or loss and other comprehensive income is
a. profit or loss c. gross profit
b. other comprehensive income d. total comprehensive income

192. Amounts earned by an entity from its main operating activities are
a. income b. revenues c. gains d. b or d

193. Is a retailer's Interest Expense an operating expense or a non-operating expense?


a. operating expense c. a or b
b. non-operating expense d. neither a nor b
(Adapted)

194. The income statement line item gross profit will appear on which income statement
format?
a. single-step b. multiple-step c. a or b d. neither a nor b
(Adapted)

195. The income statement format that segregates the operating income and expenses
from the non-operating income and expenses is the
a. single-step b. multiple-step c. a or b d. neither a nor b
(Adapted)

196. Interest earned on investments would appear in which section of a multiple-step


income statement?
a. non-operating c. would not appear
b. operating d. as part of gross income
(Adapted)

197. When alternative acceptable accounting methods exist, a better quality of earnings
generally is produced from selecting an accounting method that has the effect of
reporting the
a. greatest amount of retained earnings currently.
b. greatest amount of assets currently.
c. lowest amount of future earnings.
d. lowest amount of current earnings.
(Adapted)

198. Which of the following items would not be reported on a statement of profit or loss
and other comprehensive income?
a. Revaluation losses
b. Prior period adjustments
c. Share in associate’s revaluation gain
d. Discontinued operations

199. Gains or losses from extraordinary items should be shown on the income statement
a. immediately following income from continuing operations.
b. after discontinued operations.
c. as an item in other revenues and expenses.
d. not specifically identified as extraordinary items

200. Which of the following best describes an income statement?


a. It reports income and expenses for a specific accounting period.

40
b. It reports the amount and composition of assets and liabilities for a specific
accounting period.
c. It reports investment activities for a specified accounting period.
d. It reports cash receipts and cash disbursements for a specific accounting period.
(Adapted)

201. The Income Statement:


I. reflects the current operating performance of the entity.
II. indicates whether the entity is healthy and growing or not.
III. explains the changes in assets, liabilities and equity of the entity.
IV. is a snapshot of a entity's operations at a given time.
a. I, II & IV b. II & III c. I, II, III & IV d. I only
(Adapted)

202. Gross profit is the difference between net sales and cost of goods sold. Which of the
following most likely will not affect gross profit?
a. write-down of inventories
b. freight incurred by the consignor in delivering consigned goods to the consignee
c. allowance for sales returns
d. freight-out incurred by the seller

203. Which of the following statements is (are) correct?


I. Under the accrual basis of accounting, income is recognized in the period in which
cash is received.
II. Net sales minus Cost of goods sold equals income from operations.
III. The combination of Selling expenses and Administrative expenses is referred to as
total expenses.
IV. Cash basis of accounting best measures profitability during a short time interval.
V. Gross profit minus all other expenses recognized in profit or loss except cost of sales
is best defined as the profit or loss for the year.
a. III and V b. V only c. III, IV and V d. all of these

204. Depreciation is a process of allocating the cost of a building over its useful life in
a(n)
a. equal and equitable manner.
b. accelerated and accurate manner.
c. rational and systematic manner.
d. conservative market based manner.
(Adapted)

205. The cost of a depreciable long-lived asset is expensed


a. when it is paid for.
b. as the asset benefits the company.
c. in the period in which it is acquired.
d. in the period in which it is disposed of.
(Adapted)

206. Amortization is the process of


a. valuing an asset at its fair value.
b. increasing the value of an asset over its useful life in a rational and systematic
manner.
c. allocating the cost of an asset to expense over its useful life in a rational and
systematic manner.
d. writing down an asset to its fair value each reporting period.
(Adapted)

207. Total comprehensive income is (choose the incorrect statement)

41
a. the change in equity during a period resulting from transactions and other events,
other than those changes resulting from transactions with owners in their capacity
as owners.
b. includes increases or decreases in revaluation surplus during the period.
c. includes both unrealized gains or losses on FVPL securities and FVOCI.
d. includes only changes in assets that are not recognized in profit or loss but rather
credited directly in equity (e.g. revaluation surplus and changes in fair values of
FVOCI)

208. The major distinction between the multiple-step and single-step income statement
formats is the separation of
a. Operating and nonoperating data
b. income tax expense and administrative expenses
c. cost of goods sold expense and administrative expenses.
d. The effect on income taxes of extraordinary items and the effect on income taxes of
profit or loss from ordinary activities
(Adapted)

209. Which of the following should be included in general and administrative expenses?
(Item #1) Interest; (Item #2) Advertising
a. Yes, Yes b. Yes, No c. No, Yes d. No, No
(AICPA)
210. Accumulated other comprehensive income should be reported on the balance sheet
as a component of (Item #1) Retained earnings; (Item #2) Additional paid-in capital
a. Yes, Yes b. Yes, No c. No, Yes d. No, No
(AICPA)

211. Which of the following changes during a period is not a component of other
comprehensive income?
a. Unrealized gains or losses on FVOCI
b. Stock dividends issued to shareholders.
c. Foreign currency translation adjustments.
d. Minimum pension liability adjustments.
(AICPA)

212. Corrections of errors are reported in


a. Other comprehensive income. c. Retained earnings.
b. Other income/(expense). d. Stockholders’ equity
(AICPA)

213. Which of the following changes during a period is not a component of other
comprehensive income?
a. Minimum pension liability.
b. Treasury share, at cost.
c. Foreign currency translation adjustment on foreign operation.
d. Reclassification adjustments
(AICPA)

214. All of the following components are shown in the statement of profit or loss and
other comprehensive income net of applicable income taxes except
a. Gain or loss on valuation adjustments of FVOCI
b. Cumulative effect of a change in accounting principle.
c. Discontinued operations.
d. Remeasurements of the net defined benefit liability (asset)
(Adapted)

42
215. PAS 1 requires an entity to include in a complete set of financial statements a
statement of financial position as at the beginning of the preceding period whenever the
entity retrospectively applies an accounting policy or makes a retrospective
restatement of items in its financial statements, or when it reclassifies items in its
financial statements. The purpose of this requirement is
a. to discourage auditors from subsuming in retained earnings unaccounted
differences in accounts and required reconciliations
b. to promote vigilance on entities over errors and to discourage frequent changes in
accounting policies
c. to provide information that is useful in analyzing an entity’s financial statements
d. any of these

216. Which of the following statements is incorrect regarding financial statement


presentation?
a. PAS 1 affects only the presentation of owner changes in equity and of
comprehensive income. It does not change the recognition, measurement or
disclosure of specific transactions and other events required by other PFRSs.
b. PAS 1 requires an entity to present all owner changes in equity in a statement of
changes in equity.
c. All non-owner changes in equity are required to be presented in one statement of
profit or loss and other comprehensive income or in two statements
d. When an income statement is presented it is part of a complete set of financial
statements and shall be displayed immediately after the statement of
comprehensive income.

217. Which of the statements is false?


a. A loss caused by impairment in the value of an intangible asset should be classified
as an extraordinary item.
b. If a franchise becomes worthless prior to the end of its estimated useful life, the
unamortized balance in the franchise account should be immediately written off as
an impairment loss.
c. A lease bonus payment made in advance should be debited to a leasehold account,
which is an asset account.
d. Leasehold improvements should be amortized over the shorter of the term of the
lease or the useful life of the improvements.
(AICPA)

218. Which of the following statements is(are) correctly stated?


I. The write-off of intangible assets generally should be reported as part of continuing
operations but disclosed in the notes as an extraordinary item.
II. For an item to be disclosed in the notes as extraordinary but presented as part of
continuing operations in the profit or loss, the event or transaction which gave rise
to it should either be unusual in nature or infrequency of occurrence.
III. An income statement is usually not sufficient to describe total change in equity
during a period.
IV. The income statement of a period should include and properly describe all items of
income and expenses that do not result from transactions with owners.
V. An income statement is sufficient to describe the total change in owners’ equity
during a period because changes arise from sources other than profit oriented
activities.
a. I, III, IV b. I, III, V c. III, IV d. III only

219. Which of the following items belong to the classes of expenses?


I. expenditures to acquire assets
II. distribution to owners
III. costs of assets used to produce revenue
IV. costs of assets ceasing to provide future economic benefits

43
V. costs of assets that have expired during the period
a. all of these c. III, IV and V only
b. I, III, IV and V only d. III and IV only
(RPCPA)
220. The method of income determination which measures the results of enterprise
transactions and involves the determination of the amount of revenue earned by an
entity during a given period and the amount of expenses applicable to that revenue is
known as the:
(Item #1) Transaction approach; (Item #2) Economic approach
a. no, yes b. yes, no c. no, no d. yes, yes
(AICPA)

221. Conventionally, accountants measure income


a. as a change in the value of owners’ equity
b. by applying a value-added concept
c. by using a transaction approach
d. by equity method
(AICPA)

222. According to current standards, profit or loss for the period


a. Is the same as comprehensive income.
b. Excludes certain income and expenses that are included in comprehensive income.
c. Include certain income and expenses that are excluded from comprehensive income.
d. Include certain losses that are excluded from comprehensive income.
(AICPA)

223. Comprehensive income includes which of the following?


(Item #1) Operating income; (Item #2) Investments by owners
a. Yes, No b. Yes, Yes c. No, Yes d. No, No
(AICPA)

224. Comprehensive income includes which of the following?


(Item #1)Gross margin; (Item #2) Operating income
a. Yes, No b. Yes, Yes c. No, Yes d. No, No
(AICPA)

225. Comprehensive income includes which of the following?


(Item #1) Fair value gains; (Item #2) Gross Margin
a. Yes, No b. Yes, Yes c. No, Yes d. No, No
(AICPA)

226. Comprehensive income includes which of the following?


(Item #1) Loss on Discontinued Operations; (Item #2) Investment by Owners
a. Yes, No b. Yes, Yes c. No, Yes d. No, No
(AICPA)

227. Periodic net earnings are conventionally measured by a


a. Transactions approach.
b. Transactions approach including recognition of unrealized gains and losses in other
comprehensive income.
c. Capital maintenance approach.
d. Market value approach including recognition of all realized gains and some
unrealized losses.
(AICPA)

228. Which of the following statements is incorrect?

44
a. Reports prepared at the request of an entity’s management are not general purpose
financial statements if they are prepared specifically to meet the needs of
management only.
b. When preparing financial statements, the accountant shall never assume that the
business will continue to operate indefinitely.
c. Applying accrual accounting results in more accurate measurement of profit or loss
for the period than cash basis accounting.
d. One objective of financial reporting is to help financial statement users evaluate the
cash flows of the reporting entity.

229. Which of the following statements is incorrect?


a. Management motivations can influence the accounting policy choices made.
b. Performance evaluation is an objective of financial reporting.
c. Profit or loss for the period provides a good measure of a business's debt-paying
ability.
d. PAS 1 requires that the components of total comprehensive income should be
presented prominently in the financial statements rather than in the notes.

230. Which of the following statements is incorrect?


a. Total comprehensive income for the period provides a good measure of a business's
debt-paying ability.
b. PAS 1 requires an entity to disclose reclassification adjustments and income tax
relating to each component of other comprehensive income.
c. PAS 1 requires the presentation of dividends recognized as distributions to owners
and related amounts per share in the statement of changes in equity or in the notes.
Dividends are distributions to owners in their capacity as owners and the statement
of changes in equity presents all owner changes in equity.
d. PAS 1 requires an entity to disclose comparative information in respect of the
previous period, i.e., to disclose as a minimum two of each of the statements and
related notes.

231. Which of the following statements is incorrect?


a. The relationship of current assets and current liabilities provides a good measure of
a business's debt-paying ability.
b. The single-step and multistep income statements result in different profit or loss
figures.
c. The difference between gross sales and net sales is equal to the sum of sales
discounts and returns and allowances.
d. Components of comprehensive income are not permitted to be presented in the
statement of changes in equity.

232. Which of the following statements is incorrect regarding comprehensive income?


a. Comprehensive income is a broad measure of the changes in equity over a period
except for contributions from, or distributions to, owners. Comprehensive income
includes all income items which ultimately increase equity from transactions related
to non-owner sources.
b. Comprehensive income is broader than profit or loss and includes certain items of
income and expenses not included in profit or loss.
c. Comprehensive income includes any type of inflow which culminates to an earning
process, other than from an owner acting in his capacity as owner.
d. An unrealized loss on investments in FVOCI is not recognized in current earnings
and is not a factor in measuring comprehensive income.

Statement of changes in equity


233. PAS 1 requires the presentation of dividends recognized as distributions to owners
and related amounts per share in the
a. in the notes

45
b. statement of financial position or in the notes
c. statement of income, statement of changes in equity, or in the notes
d. statement of changes in equity or in the notes

234. Which of the following statements is incorrect?


a. Items of Other Comprehensive Income may be presented in the statement of profit
or loss and other comprehensive income gross of tax or net of tax.
b. The statement of profit or loss and other comprehensive income does not include
transactions with owners in their capacity as owners. Such transactions are
presented in the statement of changes in equity.
c. All non-owner changes in equity should be presented in a single statement or in two
statements.
d. The choice of one of the methods of presenting expenses is irrevocable, hence, once
chosen it must not be changed unless the going concern assumption becomes
inappropriate.

235. Are the following statements true or false, according to PAS1 Presentation of
Financial Statements?
I. Dividends paid should be recognized in the statement of profit or loss and other
comprehensive income.
II. A loss on disposal of assets should be recognized in the statement of changes in
equity.
a. False, False b. False, True c. True, False d. True, True
(ACCA)

236. To prepare a statement of changes in owner`s equity you need to know


a. the owners’ names
b. the date the company started
c. the beginning balance in the capital account
d. the address of the company

237. The first row in a statement of changes in equity is most likely the
a. profit c. distribution to owners
b. owners’ investments d. beginning capital

238. The statement of changes in equity may prominently display all of the following,
except
a. Effect of changes in accounting policies
b. Correction of prior period errors
c. Dividends to owners
d. Components of comprehensive income for the period

239. The preferred method of presenting statement of changes in equity in current PFRSs
is
a. horizontal presentation where each component is presented in columns and
reconciled downwards.
b. vertical presentation where there are at least two columns representing information
for the current period and the comparative period
c. dramatic presentation
d. high definition and 3D

240. The heading for the statement of changes in equity contains


a. name of the business, name of the statement, and period covered
b. name of the business, name of the statement, and current date
c. name of the business, current date, and period covered
d. name of the business, name of the owner, and period covered
(Adapted)

46
241. Elements in the equity section is normally reported in order of:
a. Classes of share capital c. Permanency
b. Time to maturity d. Liquidity
(AICPA)

242. A complete set of financial statement does not include:


a. a statement of retained earnings
b. a cash flow statement
c. notes to financial statements
d. statement of profit or loss and other comprehensive income

243. Regarding the preparation of a statement of changes in equity, which of the


following statements is incorrect?
a. PAS 1 Presentation of Financial Statements requires an entity to present, in a
statement of changes in equity, all owner changes in equity.
b. All non-owner changes in equity (i.e. comprehensive income) are required to be
presented in one statement of profit or loss and other comprehensive income or in
two statements (a separate income statement and a statement of comprehensive
income).
c. Components of comprehensive income not presented in the statement of profit or
loss and other comprehensive income are presented in the statement of changes in
equity.
d. Components of comprehensive income are not permitted to be presented in the
statement of changes in equity.

244. An entity shall present a statement of changes in equity showing in the statement all
of the following, except
a. components of total comprehensive income for the period
b. total comprehensive income for the period
c. the effects of retrospective application or retrospective restatement
d. reconciliation of each component of equity

245. An entity shall disclose the amount of dividends recognized as distributions to


equity holders during the period and the related amount per share on
a. the balance sheet d. the notes
b. the face of the income statement e. either c or d
c. the statement of changes in equity

246. The statement of changes in equity is prepared  


a. as needed
b. as an integral part of the financial statements and as a supporting document for the
income statement
c. as an integral part of the financial statements and as a supporting document for the
statement of financial position
d. as a supporting document for the financial statements but not an integral part
thereof

247. All changes in equity arising from transactions with owners in their capacity as
owners are required to be presented separately from non-owner changes in equity. An
entity is not permitted to present components of comprehensive income in the
statement of changes in equity. The purpose of this requirement is
a. to segregate taxable from non-taxable items
b. to provide more relevant and reliable information that is useful in making day-to-
day decisions
c. to provide better information by aggregating items with shared characteristics and
separating items with different characteristics

47
d. to make accounting for changes in equity and the preparation of financial
statements simpler thereby decreasing the salaries of accountants

248. PAS 1 requires income and expenses to be presented separately from owner
changes in equity
I. in one statement (a statement of profit or loss and other comprehensive income)
II. in two statements (a separate income statement and a statement of comprehensive
income)
a. I only b. II only c. I or II d. none of these

249. The components of other comprehensive income is to be displayed in the


a. income statement
b. statement of profit or loss and other comprehensive income
c. statement of changes in equity
d. any of these

250. PAS 1 requires an entity to disclose income tax relating to each component of other
comprehensive income. The purpose of this requirement is
a. to provide users with tax information relating to these components because the
components often have tax rates different from those applied to profit or loss
b. to encourage taxing authorities to also read financial statements
c. to provide users with information that other comprehensive income may have tax
consequences and that items of other comprehensive income are only temporary
income
d. to make accounting for other comprehensive income a complex matter in order to
continually challenge the competence of CPA’s

251. PAS 1 requires an entity to disclose reclassification adjustments relating to


components of other comprehensive income. Reclassification adjustments are amounts
reclassified to profit or loss in the current period that were recognized in other
comprehensive income in previous periods. The purpose of this requirement is
a. to promote consistency and to discourage frequent reclassifications and frequent
changes in presentation
b. to provide users with information to assess the effect of such reclassifications on
profit or loss
c. to provide more relevant and reliable information
d. any of these

252. PAS 1 requires dividends recognized as distributions to owners and related


amounts per share to be presented in the statement of changes in equity or in the notes.
The presentation of such disclosures in the statement of profit or loss and other
comprehensive income is not permitted. The purpose of this requirement is
a. to simplify the preparation of financial statements, including disclosures in the
notes, for the benefit of the society.
b. to ensure that owner changes in equity are presented separately from non-owner
changes in equity
c. to improve comparability of financial statements generated from accounting
softwares
d. to ensure that non-owner changes in equity are not identified separately from
owner changes in equity

253. Non-owner changes in equity should be presented in


a. Statement of profit or loss and other comprehensive Income
b. Statement of changes in equity
c. Statement of cash flows
d. Not presented

48
Notes
254. An entity normally presents notes in the following order
I. Summary of significant accounting policies applied
II. Other disclosures
III. Statement of compliance with PFRSs
IV. Supporting information for items presented in the other financial statements
a. I, III, IV, II b. III, IV, I, II c. III, I, IV, II d. I, II, III, IV

255. According to PAS 1 Presentation of Financial Statements, this provides narrative


descriptions or disaggregations of items disclosed in those statements and information
about items that do not qualify for recognition in those statements.
a. Notes to financial statements c. Disclaimer of opinion
b. Supplementary schedules and reports d. Notes

256. All of the following correctly relate to the notes, except


a. present the breakdown of aggregated items on the face of the statement and to
rectify any inappropriate accounting policies.
b. present information about the basis of preparation of the financial statements and
the specific accounting policies used
c. disclose the information required by PFRSs that is not presented elsewhere in the
financial statements
d. provide information that is not presented elsewhere in the financial statements, but
is relevant to an understanding of any of them.

257. Which of the following statements is correct regarding the preparation of notes?
a. An entity should cross-reference items in the statement of financial position and
statement of profit or loss and other comprehensive income, but not the statement
of cash flows and statement of changes in equity, to the notes.
b. An entity may present notes providing information about the basis of preparation of
the financial statements and specific accounting policies as a separate section of the
financial statements.
c. The notes is an optional statement. An entity may decide not to present it.
d. The notes does not occupy a bulk portion of a complete set of financial statements.

258. Which of the following information is not specifically a required disclosure of PAS 1?
a. Name of the reporting entity or other means of identification, and any change in that
information from the previous year.
b. Names of major/significant shareholders of the entity.
c. Level of rounding used in presenting the financial statements.
d. Whether the financial statements cover the individual entity or a group of entities.
(Adapted)

259. Which of the following best states the purpose of the notes?
a. to provide additional disclosures regarding off-balance sheet items
b. to provide information regarding accounting policies adopted by the issuer
c. to provide necessary disclosures required by PFRSs
d. to provide necessary disclosures required by PSAs

260. Choose the incorrect statement


a. Disclosure notes facilitate the evaluation of enterprise position and performance
because they include information which helps to explain the quality of earnings.
b. Disclosure notes are an integral part of the financial statements.
c. Companies often look for opportunities to smooth earnings.
d. Accounting concepts, principles and standards are just as broad and general today
as they were sixty years ago.
(Adapted)

49
261. Which of the following statements is correct?
a. Certified Public Accountants are not independent for the benefit of the users of the
financial statements, because they are paid by the client.
b. Accounting concepts, principles and standards are just as broad and general today
as they were sixty years ago.
c. Due to the excellent work of the FRSC, there are very few choices among alternative
accounting policies today.
d. Disclosures are an integral part of the financial statements
(Adapted)

262. Which of the following statements is incorrect?


a. The principles of accounting, as used in reporting standards, refer not only to
accounting principles but also the methods of applying them.
b. Aspects of financial position presented in the balance sheet are related to changes in
financial position presented in the income statement, statement of changes in
equity, and statement of cash flows.
c. Asset valuation accounts are neither assets nor liabilities.
d. The cash basis of accounting is acceptable primarily in enterprises that do not have
substantial credit transactions or inventories.
(RPCPA)

263. Are the following statements in relation to materiality true or false, according to
PAS1 Presentation of Financial Statements?
I. Materiality of items depends on their individual or collective influence on the
economic decisions of users.
II. Materiality of an item depends on its absolute size and nature.
a. False, False b. False, True c. True, False d. True, True
(ACCA)

264. To properly prepare notes to financial statements you need to know


a. the reporting entity’s name c. the registered address of the company
b. the date the company started d. all of these

265. Notes to financial statements are beneficial in meeting the disclosure requirements
of financial reporting. The note should not be used to
a. Describe significant accounting policies
b. Describe depreciation methods employed by the company
c. Describe principles and methods peculiar to the industry in which the company
operates, when these principles and methods are predominantly followed in that
industry.
d. Correct an improper presentation in the financial statements.
(Adapted)

266. You are preparing the “general information” section of a notes to financial
statements. Which of the following information sources is most relevant in addition to
direct inquiry with management?
a. board of directors minutes of meetings
b. lease contract
c. general ledger
d. latest authorized articles of incorporation

267. Which is not a required disclosure?


a. the key assumption concerning the future, and other key sources of estimation
uncertainty at the balance sheet date
b. the judgment management has made in the process of applying the accounting
policies

50
c. a and b
d. the number of the entity’s employees

268. The following statements relate to various financial accounting topics:


I. reporting deferred income taxes in the balance sheet is an example of intraperiod
tax allocation
II. a lease that is in substance a purchase is termed a direct financing or a sales-type
lease by the lessor and a finance lease by the lessee
III. earnings per share information are required on the income statements of publicly-
held corporations for each of the following (a) income from continuing operations,
(b) results of discontinued operations, and (c) net income
IV. accounting is responsible for providing standards that insure accurate financial
information that cannot be manipulated or improperly reported.

State whether the foregoing statements are false:


a. all of the statements are false c. only two statements are false
b. only one statement is false d. three statements are false
(RPCPA)

269. An entity shall disclose all of the following in the notes, except
a. the amount of dividends proposed or declared before the financial statements were
authorized for issue but not recognized as a distribution to owners during the
period, and the related amount per share
b. the amount of any cumulative preference dividends not recognized.
c. the domicile and legal form of the entity and a list of its incorporators.
d. the name of the parent and the ultimate parent of the group.

270. An entity shall disclose all of the following in the notes, except
a. the domicile and legal form of the entity, its country of incorporation and the
address of its registered office (or principal place of business, if different from the
registered office)
b. a description of the nature of the entity’s operations and its principal activities; and
c. the name of the parent and the ultimate parent of the group.
d. number of employees of the entity
271. An entity shall disclose all of the following in the notes, except
a. Summary of accounting policies adopted
b. Date the financial statements were authorized for issue and who gave that
authorization
c. Components of comprehensive income if not presented in a separate financial
statement with equal prominence as the other financial statements
d. The date the entity started its operations.

272. An entity shall disclose all of the following in the notes, except
a. Schedules supporting the line items presented in the other financial statements.
b. Narrative descriptions of items cross referenced from the other financial
statements.
c. The date the entity received a secondary license from a regulatory agency.
d. The registered addresses and nationalities of the entity’s incorporators.

273. Which of the following should be disclosed in a summary of significant accounting


policies?
a. Basis of profit recognition on long-term construction contracts.
b. Future minimum lease payments in the aggregate and for each of the five succeeding
fiscal years.
c. Depreciation expense.
d. Composition of sales by segment.
(AICPA)

51
274. The notes to financial statements should never be used to
a. disclose a summary of the accounting policies adopted by the reporting entity
b. disclose information required by PFRSs
c. disclose information not required by PFRSs but are relevant in the understanding of
information presented in the other components of a complete set of financial
statements
d. rectify inappropriate accounting policies

275. Which of the following statements is incorrect?


a. Extraordinary gains and losses, distinguished by their unusual nature and by the
infrequency of their occurrence, should be presented together with other items of
income and expenses in the income statement and need not be distinguished
separately either on the face of the financial statements or in the notes.
b. An income statement that separates income and expenses into operating and
nonoperating items is called a classified, or multiple step income statement.
c. Description of the accounting policies adopted by the reporting entity is not
required as an integral part of the financial statements.
d. Disclosure of accounting policies should identify and describe the accounting
principles followed by the reporting entity and the methods of applying those
principles that materially affect the determination of financial position, changes in
financial position or results of operation

276. Which of the following should be disclosed in the summary of significant accounting
policies?
(Item #1) Maturity dates of long-term debt; (Item #2) Composition of inventories
a. Yes, Yes b. Yes, No c. No, No d. No, Yes
(AICPA)

277. Which of the following should be disclosed in the summary of significant accounting
policies?
(Item #1) Composition of plant assets; (Item #2) Inventory pricing
a. Yes, Yes b. No, Yes c. No, No d. Yes, No
(AICPA)

278. Which of the following information should be disclosed in the summary of


significant accounting policies?
a. Refinancing of debt subsequent to end of reporting period.
b. Guarantees of indebtedness of others.
c. Criteria for determining which investments are treated as cash equivalents.
d. Adequacy of pension plan assets relative to vested benefits.
(AICPA)

279. Which of the following facts concerning fixed assets should be included in the
summary of significant accounting policies? (Item #1) Depreciation method (Item #2)
Composition
a. No, Yes b. Yes, Yes c. Yes, No d. No, No
(AICPA)

280. The summary of significant accounting policies should disclose the


a. Pro forma effect of retroactive application of an accounting change.
b. Basis of profit recognition on long-term construction contracts.
c. Adequacy of pension plan assets in relation to vested benefits.
d. Future minimum lease payments in the aggregate and for each of the five succeeding
fiscal years.
(AICPA)

52
281. Which of the following information should be included in FRACTIOUS
TROUBLESOME, Inc.’s 20x1 summary of significant accounting policies?
a. Property, plant, and equipment is recorded at cost with depreciation computed
principally by the straight-line method.
b. During 20x1, the UNRULY component was sold.
c. Business component 20x1 sales are QUARRELSOME ₱1M, IRRITABLE ₱2M, and
RECALCITRANT ₱3M.
d. Future ordinary share dividends are expected to approximate 60% of earnings.
(AICPA)

282. Which of the following is an acceptable method of reporting other comprehensive


income and its components?
a. In a statement of profit or loss and other comprehensive income.
b. In a statement of changes in equity
c. In the notes only.
d. All of these

283. Which of the following statements is (are) correct?


I. The preparation of a worksheet is optional.
II. The preparation of a trial balance is optional.
III. The presentation of an income statement is optional.
IV. The presentation of a statement of profit or loss and other comprehensive income is
optional.
V. The presentation of notes is optional.
a. I, II, and III b. I and II c. III, IV and V d. all of these

53
Chapter 37 - Suggested answers to theory of accounts questions
1. B 41. D 81. C 121. A 161. D 201. D 241. C
2. B 42. D 82. D 122. D 162. C 202. D 242. A
3. C 43. B 83. C 123. D 163. D 203. B 243. C
4. D 44. A 84. B 124. D 164. D 204. C 244. A
5. A 45. D 85. D 125. B 165. D 205. B 245. E
6. D 46. D 86. C 126. D 166. A 206. C 246. C
7. C 47. A 87. C 127. C 167. C 207. D 247. C
8. B 48. C 88. D 128. C 168. C 208. A 248. C
9. B 49. B 89. D 129. C 169. A 209. D 249. B
10. D 50. B 90. C 130. B 170. D 210. D 250. A
11. B 51. D 91. A 131. D 171. D 211. B 251. D
12. B 52. D 92. D 132. D 172. A 212. C 252. B
13. D 53. A 93. C 133. C 173. D 213. B 253. A
14. B 54. D 94. B 134. C 174. B 214. B 254. C
15. D 55. D 95. A 135. A 175. C 215. C 255. D
16. A 56. D 96. A 136. C 176. D 216. D 256. A
17. A 57. D 97. B 137. B 177. B 217. A 257. B
18. D 58. D 98. D 138. D 178. A 218. D 258. B
19. D 59. C 99. E 139. A 179. B 219. C 259. C
20. B 60. C 100. D 140. D 180. A 220. B 260. D
21. C 61. D 101. A 141. D 181. C 221. C 261. D
22. C 62. A 102. B 142. C 182. D 222. B 262. D
23. C 63. B 103. C 143. A 183. C 223. A 263. C
24. D 64. A 104. B 144. C 184. D 224. B 264. D
25. A 65. C 105. B 145. D 185. D 225. B 265. D
26. B 66. A 106. D 146. A 186. C 226. A 266. D
27. C 67. B 107. C 147. A 187. B 227. A 267. D
28. A 68. C 108. D 148. C 188. B 228. B 268. C
29. D 69. B 109. D 149. D 189. B 229. C 269. C
30. B 70. A 110. B 150. D 190. A 230. A 270. D
31. B 71. D 111. B 151. C 191. D 231. B 271. C
32. C 72. E 112. B 152. C 192. B 232. D 272. D
33. A 73. C 113. B 153. A 193. B 233. D 273. A
34. A 74. B 114. C 154. A 194. B 234. D 274. D
35. B 75. D 115. D 155. C 195. B 235. A 275. C
36. A 76. C 116. D 156. D 196. A 236. C 276. C
37. A 77. D 117. A 157. C 197. D 237. D 277. B
38. D 78. A 118. C 158. C 198. B 238. D 278. C
39. A 79. D 119. A 159. A 199. D 239. A 279. C
40. B 80. C 120. D 160. A 200. A 240. A 280. B
281. A
282. A
283. A

54
Chapter 38
Revenue

Chapter 38: Multiple choice – Computational (SET B) – (For classroom instruction


purposes)
Discounts and rebates
1. Among the transactions of TREPIDANT TREMBLING during the year were the
following:
 On December 29, 20x1, TREPIDANT sold goods with a list price of ₱400,000 to a
customer who was given a credit term of “10%, 2/10, n/20.” As of December 31,
20x1, the receivable from the sale is not yet collected. Past experience shows that
customers always avail of prompt discounts.
 TREPIDANT launched a new product in April 20x1. As part of the promotion,
TREPIDANT made arrangements with its customers that if they accumulate total
purchases amounting to ₱4,800,000 of the new product within one year ending on
March 31, 20x2, they will receive a retrospective discount of 3%. As of December 31,
20x1, the total purchases of a certain customer amounted to ₱4,400,000.
TREPIDANT expects that this customer will make additional purchases in the
coming months and, therefore, will be entitled to the retrospective discount.
TREPIDANT uses a calendar year reporting period.

How much is the total net revenue to be recognized from the transactions described above?
a. 352,800 b. 4,268,000 c. 4,620,800 d. 4,628,000

Sales taxes
Use the following information for the next two questions:
EMBELLISH DECORATE Company is required to file quarterly sales tax returns with a tax
bureau by the 20th day following the end of the calendar quarter. However, if sales taxes
collected in a month exceed ₱4,000, such taxes should be remitted immediately by the 20 th
day of the month following the month of such collection. Sales taxes are computed as 3% of
sales. EMBELLISH records sales taxes collected as sales revenue and sales taxes paid as
charges against sales revenue. EMBELLISH’s sales revenue ledger shows the following:

Debit Credit
January 164,800
February 4,800 123,600
March 115,360
4,800 403,760

2. How much is the sales revenue in EMBELLISH’s first quarter interim financial
statements?
a. 392,000 b. 387,340 c. 396,000 d. 398,960

3. How much is the sales taxes payable in EMBELLISH’s first quarter interim financial
statements?
a. 6,906 b. 6,820 c. 7,680 d. 6,960

Deferred payment
4. POIGNANT CUTTING Co. had the following transactions during the year:
 On December 29, 20x1, POIGNANT sold goods costing ₱800,000 for ₱1,000,000 to a
customer who was granted a special credit period of 2 years. POIGNANT normally
sells the goods for ₱880,000 with a credit period of one month or with a ₱20,000
discount for cash on delivery (i.e., outright payment in cash).

55
 On December 31, 20x1, POIGNANT sold goods in exchange for a ₱4,000,000,
noninterest-bearing note that matures on December 31, 20x4. The prevailing rate
for a similar instrument as of date of sale is 12%.

How much is the total sales revenue to be recognized from the transactions described
above?
a. 880,000 b. 2,847,120 c. 3,727,120 d. 3,707,120

Exchange of goods
Use the following information for the next two questions:
Fact pattern
ANCILLIARY Co. and SUBORDINATE, Inc. are provincial distributors of oil. ANCILLIARY’s
principal place of business is located at Town One while SUBORDINATE’s principal place of
business is located at Town Two. However, each of the distributors operates in both towns.
During the year, a bridge connecting these two towns has been damaged. Wary of the
disruption of their business operations, ANCILLIARY and SUBORDINATE agreed to
exchange inventories of oil so that each could maintain its operations in both towns.

Case #1: Dissimilar goods


5. ANCILLIARY agreed to exchange its inventory of “premium” oil in Town One with
SUBORDINATE’s inventory of “diesel” oil in Town Two. ANCILLIARY’s “premium” oil has
a fair value of ₱4M while SUBORDINATE’s “diesel” oil has a fair value of ₱3.8M. How
much revenue shall ANCILLIARY recognize from the inventory exchange?
a. 3,800,000 b. 4,000,000 c. 200,000 d. 0

Case #2: Similar goods


6. ANCILLIARY agreed to exchange its inventory of “diesel” oil in Town One with
SUBORDINATE’s inventory of “diesel” oil in Town Two. ANCILLIARY’s “diesel” oil has a
fair value of ₱4M while SUBORDINATE’s “diesel” oil has a fair value of ₱3,992,000. How
much revenue shall ANCILLIARY recognize from the inventory exchange?
a. 3,992,000 b. 4,000,000 c. 8,000 d. 0

Transfer/retention of risks and rewards


7. SEPULCHER TOMB Company had the following transactions during the year:
 Sold goods to Customer W on an installment basis for ₱20,000. Legal title to the
goods was retained by SEPULCHER. The installment receivables are collectible over
the next eight months.
 Sold goods to Customer X on a cash basis for ₱80,000. The sale agreement gives
Customer X the right to return up to 40% of the product sold within the first six
months after the date of sale provided an appropriate reason is given. Customer X
will be refunded for any goods returned. SEPULCHER can reliably estimate that 10%
of the products sold would be returned.
 Sold goods to Customer Y on account basis for ₱40,000. The sale agreement gives
Customer Y the right to return up to 10% of the product sold within the first two
weeks after the date of sale provided an appropriate reason is given. SEPULCHER
can reliably estimate that 5% of the products sold would be returned.
 Sold goods to Customer Z on a cash basis for ₱48,000. The sale agreement gives
Customer Z the right to return the goods and be refunded for the sale price if he is
not satisfied. SEPULCHER cannot reliably estimate the amount of goods to be
returned.
 Sold goods to Customer Voiz&Gurlz on a cash basis for ₱40,000. The sales
representative was paid a 2% commission based on the selling price.

How much is the total net sales revenue to be recognized from the transactions above?
a. 170,000 b. 169,200 c. 218,000 c. 224,000

Transfer/retention of risks and rewards

56
8. YEASTY IMMATURE Co. had the following transactions during the year:
 Sold goods to Customer X for ₱400,000. YEASTY is contractually bound per sales
contract to remediate any unsatisfactory performance, which is not governed by
normal warranty provisions.
 Sold goods to Customer Y for ₱800,000. Customer Y is given the right to rescind the
purchase for a reason not specified in the contract of sale (duly signed by both
parties). YEASTY is uncertain about the outcome of the transaction.

How much total revenue shall be recognized from the transactions described above?
a. 400,000 b. 800,000 c. 1,200,000 d. 0

Bill and hold and Lay away sales


9. SUBJACENT UNDERLYING Co. had the following transactions during the year:
 Billed Customer W ₱40,000 for goods sold. However, delivery has been postponed
per Customer W’s request. The goods have already been segregated from other
inventories on hand and will be delivered after W’s further instruction.
 Billed Customer X ₱80,000 for goods yet to be manufactured. The goods will be
delivered immediately after production is completed.
 Billed Customer Y ₱100,000 for goods sold. Delivery will be made when the billed
price is fully collected. Customer Y made a down payment of ₱1,000.
 Billed Customer Z ₱120,000 for goods sold. Delivery will be made when the billed
price is fully collected. Customer Z made total payments of ₱119,600 during the
year.

How much is the total sales revenue from the transactions above to be included in
SUBJACENT Co.’s. statement of profit or loss and other comprehensive income for the
year?
a. 40,000 b. 160,000 c. 260,000 d. 0

Subscriptions to publications
10. PALTRY INFERIOR Co. sells 12-month mail order subscriptions for its magazine.
Subscriptions are received in advance and credited to sales. However, subscriptions
received during the month are held for mail starting on the following month. The
magazines are of similar value. PALTRY received ₱4,800,000 subscriptions evenly
during the last quarter of the year. How much revenue from the subscriptions received
during the quarter shall be included in PALTRY Co.’s year-end statement of profit or
loss and other comprehensive income?
a. 400,000 b. 1,200,000 c. 2,400,000 d. 3,200,000

Uncollectibility of revenue already recognized


11. DESULTORY UNPLANNED Co. offers repair services for manufacturing equipment and
also sells machinery parts. During the year, DESULTORY had the following transactions.
 In January 20x1, DESULTORY Co. signed a repair service contract with a customer
for ₱4,000,000. The contract price is due on March 31, 20x1. By the end of the year,
the repair service has long been completed. However, the contract price has not yet
been collected. DESULTORY has assessed that the receivable should be written-off.
DESULTORY has no balance in its allowance for uncollectible service contract
receivables.
 In July 20x1, DESULTORY Co. sold various parts to a customer on account for
₱2,000,000. By the end of the year, only half of the receivable from the customer has
been collected. DESULTORY assessed that 5% of the outstanding balance as of
December 31, 20x1 is doubtful of collection.

How much revenue from the transactions described above shall be included in
DESULTORY’s statement of profit or loss and other comprehensive income for 20x1?
a. 6,000,000 c. 1,900,000 c. 1,100,000 d. 0

57
Percentage of completion
12. HEATHEN UNCIVILIZED Co. is a systems developer. During the year, HEATHEN was
engaged to develop an automated system for the billing and collection of an electric
cooperative for a total contract price of ₱24,000,000. As of December 31, 20x1, the
system is only 70% complete. However, HEATHEN Co. had already billed the client for
90% of the contract price or ₱21,600,000 and collected ₱19,440,000 therefrom, after
10% retention by the client to be reverted to HEATHEN at the close-out date of the
contract. How much revenue shall HEATHEN recognize during the year?
a. 16,000,000 b. 19,440,000 c. 16,800,000 d. 19,980,000

Cost-recovery approach (Sunk cost method)


Use the following information for the next two questions:
PERSPICACIOUS KEEN Co. is a systems developer. During the year, PERSPICACIOUS was
engaged to develop a specialized anti-money laundering system for a bank at a total
contract price of ₱24,000,000. This is the first time that PERSPICACIOUS will be developing
such system. As of year-end, the system is not yet complete and PERSPICACIOUS is
uncertain as to the outcome of the systems development. Total costs incurred to date
amounted to ₱12,000,000.

Case #1:
13. If PERSPICACIOUS nonetheless expects to recover the costs incurred, how much
revenue shall be recognized for the year?
a. 24,000,000 b. 12,000,000 c. 4,000,000 d0

Case #2:
14. If PERSPICACIOUS does not expect to recover the costs incurred if the system is not
developed satisfactorily, how much revenue shall be recognized for the year?
a. 24,000,000 b. 12,000,000 c. 4,000,000 d0

Advertising commissions
15. On March 1, 20x1, COMMISERATE SYMPATHIZE Advertising Co. signed a contract to
make a TV commercial for a client at a contract price of ₱20,000,000. The client was
billed on contract date but the contract price was collected only on March 31, 20x1. The
advertisement was aired on TV on April 3, 20x1. How much revenue is recognized
during the first quarter ended March 31, 20x1?
a. 20,000,000 b. 10,000,000 c. 2,000,000 d. 0

Admission fees – One-time event


16. SECERN SEPARATE Production Co. organized a concert in the first quarter of 20x1.
Sales of concert tickets from January to March 20x1 amounted to ₱4,000,000. The
concert was held on April 1, 20x1. How much revenue is recognized during the first
quarter ended March 31, 20x1?
a. 4,000,000 b. 2,000,000 c. 400,000 d. 0

Admission fees – More than one event


17. The PASTORAL RURAL is a major league basketball team. The team has 60 home games
during a regular season. The regular season starts in November and ends in April of the
following year. From November to December 20x1, PASTORAL RURAL sold 3,000
regular season tickets for ₱4,800 each. As of December 31, 20x1, twelve home games
have been played. How much is the revenue to be included in PASTORAL RURAL’s
statement of profit or loss and other comprehensive income for the three months ended
December 31, 20x1?
a. 2,800,000 b. 14,400,000 c. 3,200,00 d. 2,880,000

Royalties
18. CHIASMA Co.’s copyright was licensed to INTERSECTION, Inc. for royalties of 10% of
sales for the copyrighted materials. Royalties are payable semi-annually on April 1 for

58
sales in July through December of the prior year, and on October 1 for sales in January
through June of the same year. CHIASMA received the following royalties from
INTERSECTION.
April 1 October 1
20x1 40,000 48,000
20x2 60,000 72,000

INTERSECTION estimated that sale of the copyrighted materials would total ₱800,000 for
July through December 20x2. How much is the royalty revenue in CHIASMA’s 20x2
statement of profit or loss and other comprehensive income?
a. 152,000 b. 164,000 c. 148,000 d. 0

Customer loyalty program


Use the following information for the next two questions:
DISCALCED BAREFOOTED NAKASAKASAKA Supermarket, Inc. awards loyalty points to
customers who use the card, issued by DISCALCED after payment of membership fee, to
pay for purchases. The award is at the rate of one point for every ₱400 charged to the card.
The points accumulated may be redeemed for awards in the form of appliances, electronics,
groceries and other household items. DISCALCED estimates the fair value of each point at
₱4.00. During the period, DISCALCED made total sales of ₱40M to cardholders.

19. How much sales revenue is recognized?


a. 400,000 b. 40,000,000 c. 39,600,000 d. 0

20. How much is the deferred revenue from loyalty points?


a. 400,000 b. 40,000,000 c. 39,600,000 d. 0

The answers and solutions to the computational problems above


(Multiple choice – Computational (SET B) can be found in the
accompanying Teacher’s Manual.

Chapter 38: Theory of Accounts Reviewer


1. Which of the following statements is incorrect?
a. Income encompasses both revenue and gains.
b. Revenue is income that arises in the course of ordinary activities of an entity.
c. Gains arise from peripheral or incidental activities.
d. PAS 18 applies equally to revenue and gains.

2. According to PAS 18, revenue may arise from


I. sale of goods
II. rendering of services
III. interest
IV. royalties
V. dividends
VI. initial recognition of agricultural produce at fair value less costs to sell
a. all of these b. I and II only c. I, II, III, IV, and V d. I, II and III

3. According to PAS 18, the primary issue in accounting for revenue is


a. when to recognize revenue
b. how much revenue to recognize
c. from whom will the revenue be collected
d. all of these

4. Revenue is recognized
I. when it is probable that future economic benefits will flow to the entity
II. the benefits can be measured reliably

59
a. Yes, yes b. Yes, no c. No, no d. No, yes

5. Revenue is measured
a. at the fair value of the consideration received or receivable taking into account the
amount of any trade discounts and volume rebates allowed by the entity.
b. at the proceeds receivable
c. at the present value of the proceeds receivable
d. at the carrying amount of the proceeds received

6. Amounts collected on behalf of others, including sales taxes, value added taxes and
amounts collected as agent on behalf of a principal,
a. are included in the revenue figure.
b. are excluded from the revenue figure.
c. may be excluded from the revenue figure
d. may be included in the revenue figure

7. When payment is deferred, the amount of revenue recognized is


a. the cash price equivalent of the goods sold or service rendered.
b. the present value of the deferred payments discounted at an imputed rate of
interest.
c. a or b as an accounting policy choice
d. choice (a), if this is not determinable, then choice (b)

8. Under revenue transactions where payment of consideration received is deferred, the


difference between the fair value and the nominal amount of the consideration
a. is recognized as interest revenue computed using the effective interest method.
b. is recognized immediately as interest revenue
c. is recognized as interest revenue computed using the straight line method.
d. a or c

9. The imputed rate of interest is


I. the prevailing rate for a similar instrument of an issuer with a similar credit rating
II. a rate of interest that discounts the nominal amount of the instrument to the current
cash sales price of the goods or services.
a. I c. the more clearly determinable of I and II
b. II d. none of these

10.Which of the following is a transaction regarded to give rise to revenue?


a. The sale of a property, plant and equipment above carrying amount
b. The increase in fair value of a held for trading investment
c. The exchange of similar goods or services
d. The exchange of dissimilar goods or services

11.The revenue from an exchange of dissimilar goods or services is measured


a. at the fair value of the goods or services received, adjusted by the amount of any
cash or cash equivalents transferred.
b. at the fair value of the goods or services given up, adjusted by the amount of any
cash or cash equivalents transferred.
c. choice (a), if this is not determinable, then choice (b)
d. at the carrying amount of the goods or services given up, adjusted by the amount of
any cash or cash equivalents transferred.

12.It is a type of sale in which delivery is delayed at the buyer’s request but the buyer takes
title and accepts billing.
a. Bill and sold b. Lay away c. Hold away d. Bill and hold

60
13.It is a type of sale in which the goods are delivered only when the buyer makes the final
payment in a series of installments.
a. Bill and hold b. Lay away c. Sale away d. Installment sale

14.SACCHARINE EXCESSIVELY SWEET Co. sold equipment to MAWKISH Co. The sale is
subject to installation and inspection. When should SACCHARINE recognize revenue
from the sale?
a. when the MAWKISH accepts delivery
b. when the MAWKISH is billed
c. when the MAWKISH accepts delivery, and installation and inspection are complete.
d. whenever SACCHARINE feels like to

15.According to PAS 18, when the installation process is simple in nature or the inspection
is performed only for purposes of final determination of contract prices, revenue is
recognized
a. immediately upon billing
b. immediately upon the buyer’s acceptance of delivery
c. when installation and inspection are complete
d. in any of these

16.Under a “cash on delivery” sale, revenue is recognized


a. when delivery is made and cash is received by the seller or its agent.
b. when cash is received by the seller or its agent.
c. when delivery is made regardless of whether cash is received by the seller or its
agent.
d. when cash is received by the seller or its agent regardless of whether delivery has
been made

17.Under sale of subscriptions to publications with similar value, revenue is recognized


a. immediately upon receipt of consideration
b. on a straight-line basis over the period in which the items are dispatched.
c. on the basis of the sales value of the item dispatched in relation to the total
estimated sales value of all items covered by the subscription.
d. b or c

18.Under sale of subscriptions to publications with different values, revenue is recognized


a. immediately upon receipt of consideration
b. on a straight-line basis over the period in which the items are dispatched.
c. on the basis of the sales value of the item dispatched in relation to the total
estimated sales value of all items covered by the subscription.
d. b or c

19.According to PAS 18, under installment sales in which the consideration is receivable in
installments
a. Revenue attributable to the sales price, exclusive of interest, is recognized at the
date of sale.
b. Revenue attributable to the sales price, exclusive of interest, is recognized by
multiplying a “realized gross profit percentage” to collections made.
c. Revenue attributable to the sales price, exclusive of interest, is recognized in
proportion to collections received over the total agreed selling price.
d. Revenue attributable to the sales price, exclusive of interest, is recognized only
when the cost of the asset sold has been recovered.

20.According to PAS 18, the sale price of an installment sale


a. is the present value of the consideration, determined by discounting the
installments receivable at the imputed rate of interest.

61
b. is the present value of the consideration, determined by discounting the
installments receivable at the nominal rate of interest.
c. is the amount billed to the customer
d. is the average retail price of the item sold

21.When the outcome of a transaction involving the rendering of services can be estimated
reliably, revenue associated with the transaction shall be recognized
a. using the percentage of completion method
b. using the cost recovery method
c. using the installment method
d. any of these

22.When the collectibility of an amount already recognized as revenue becomes uncertain,


a. the uncollectible amount is recognized as an expense
b. the uncollectible amount is accounted for as an adjustment to the amount of
revenue originally recognized
c. a or b
d. the uncollectible amount continues to be presented in the statement of financial
position

23.Which of the following statements is (are) correct?


I. When the outcome of the transaction involving the rendering of services cannot be
estimated reliably, revenue shall be recognized only to the extent of the expenses
recognized that are recoverable (a "cost-recovery approach").
II. When the outcome of a transaction cannot be estimated reliably and it is not
probable that the costs incurred will be recovered, revenue is not recognized and
the costs incurred are recognized as an expense.
a. I only b. II only c. I and IId. Neither I nor II

24.Which of the following statements is incorrect regarding the provisions of PAS 18?
a. When services are performed by an indeterminate number of acts over a specified
period of time, revenue is recognized on a straight-line basis, unless a specific act is
much more significant than any other acts, in which case revenue is postponed until
the significant act is executed.
b. Progress payments and advances received from customers often do not reflect the
services performed.
c. Surveys of work performed should never be used to estimate the stage of
completion of services being rendered.
d. Services performed to date as a percentage of total services to be performed may be
used to estimate the stage of completion of services being rendered.

25.When the outcome of the transaction involving the rendering of services cannot be
estimated reliably,
a. revenue shall be recognized using the cost-recovery method
b. no revenue is recognized; any costs incurred are recognized immediately as expense
c. revenue shall be recognized only for the present value of the consideration
receivable
d. any of these

26.When the outcome of a transaction cannot be estimated reliably and it is not probable
that the costs incurred will be recovered
a. revenue shall be recognized using the cost-recovery method
b. no revenue is recognized; any costs incurred are recognized immediately as expense
c. revenue shall be recognized only for the present value of the consideration
receivable
d. any of these

62
27.According to PAS 18, media commissions are recognized
a. when the related advertisement or commercial appears before the public.
b. by reference to the stage of completion of the project.
c. over the remaining months of the reporting period using straight line basis
d. immediately upon signing of contract

28.According to PAS 18, production commissions are recognized


a. when the related advertisement or commercial appears before the public.
b. by reference to the stage of completion of the project.
c. over the remaining months of the reporting period using straight line basis
d. immediately upon signing of contract

29.Revenue from artistic performances, banquets and other special events


a. is recognized immediately
b. is recognized when the event takes place
c. by reference to the stage of completion of the service
d. when tickets are sold

30.Revenue from tuition fees is recognized


a. immediately upon enrolment c. after the semester
b. over the period of instruction d. only on graduation day

31.Revenue from consignment sales is recognized


a. upon sale of the consigned goods to external parties
b. upon delivery of the consigned goods to the consignee
c. upon receipt of proceeds from the consignee
d. any of these

32.According to PAS 18, interest revenue is recognized


a. on a time proportion basis that takes into account the effective yield
b. on an accruals basis in accordance with the substance of the relevant agreement
c. when the creditor's right to receive payment is established
d. on a straight-line basis that ignores the effective yield

33.According to PAS 18, revenue from royalty agreements is recognized


a. on a time proportion basis that takes into account the effective yield
b. on an accruals basis in accordance with the substance of the relevant agreement
c. when the grantor's right to receive payment is established
d. on a straight-line basis that ignores the effective yield

34.According to PAS 18, dividend revenue is recognized


a. on dividend declaration date
b. on date of record
c. on date of distribution
d. on the date when the shareholder's right to receive payment is established.

35.When unpaid interest has accrued before the acquisition of an interest-bearing


investment, the subsequent receipt of interest
a. is allocated between pre-acquisition and post-acquisition periods. Only the post-
acquisition portion is recognized as revenue.
b. is allocated between pre-acquisition and post-acquisition periods. Only the pre-
acquisition portion is recognized as revenue.
c. need not be allocated but rather recognized in full as revenue.
d. is recognized using cash basis of accounting.

36.In case of sale with award credits for customer loyalty,

63
a. the fair value of the consideration received or receivable in respect of the initial sale
shall be recognized as revenue from sale of product.
b. the fair value of the consideration received or receivable in respect of the initial sale
shall be recognized as revenue from the award credit granted.
c. the fair value of the consideration received or receivable in respect of the initial sale
shall be allocated between the award credits and the other components of the sale.
d. any of these

37.When award credits for customer loyalty are granted to customers, the entity shall
allocate the fair value of the consideration received or receivable in respect of the initial
sale between the award credits and the other components of the sale. The consideration
allocated to the award credits
a. shall be measured by reference to their fair value, i.e., the amount for which the
award credits could be sold separately.
b. shall be measured by reference to the cost of the items to be redeemed by the award
credits granted.
c. shall be measured as the excess of sales proceeds from initial sale and the present
value of future cash flows from the sale.
d. shall be measured by multiplying the number of award points granted by P1.00.

38.If the entity itself supplies the awards on customer loyalty program, it shall recognize
the consideration allocated to award credits as revenue
a. upon initial sale without regard as to whether the award credits will be redeemed or
not.
b. when award credits are redeemed and it fulfills its obligations to supply awards.
c. a or b as an accounting policy choice
d. at each reporting period by reference to the estimated number of award credits to
be redeemed in the following period.

39.Which of the following bases of revenue recognition reflects the greatest degree of
uncertainty about future events?
a. Sales method applied to sales of a department store
b. Cost recovery method applied to an installment sales contract
c. Production method for a gold mining operation
d. Percentage of completion on a construction contract
(Adapted)

40.“Bill and hold” sales, in which delivery is delayed at the buyer’s request but the buyer
assumes title and accepts invoicing, should be recognized when
a. The buyer makes an order.
b. The seller starts manufacturing the goods.
c. The title has been transferred but the goods are kept on the seller’s premises.
d. It is probable that the delivery will be made, payment terms have been established,
and the buyer has acknowledged the delivery instructions.
(Adapted)

41.OVERCAST, CLOUDED OVER Inc. is a large manufacturer of machines. DARKEN Ltd., a


major customer of OVERCAST, has placed an order for a special machine for which it
has given a deposit of ₱112,500 to OVERCAST. The parties have agreed on a price for
the machine of ₱150,000. As per the terms of the sales agreement, it is an FOB (free on
board) contract and the title passes to the buyer when goods are loaded onto the ship at
the port. When should the revenue be recognized by OVERCAST?
a. When the customer orders the machine.
b. When the deposit is received.
c. When the machine is loaded on the port;
d. When the machine has been received by the customer.
(Adapted)

64
42.Revenue from an artistic performance is recognized once
a. The audiences register for the event online.
b. The tickets for the concert are sold.
c. Cash has been received from the ticket sales.
d. The event takes place.
(Adapted)

43.SURMOUNT EXCEL Company, a large manufacturer of cosmetics, sells merchandise to


OVERCOME Ltd., a retailer, which in turn sells the goods to the public at large through
its chain of retail outlets. OVERCOME Ltd. purchases merchandise from SURMOUNT
under a consignment contract. When should revenue from the sale of merchandise to
OVERCOME Ltd. be recognized by SURMOUNT?
a. When goods are delivered to OVERCOME Ltd.
b. When goods are sold by OVERCOME Ltd.
c. It will depend on the terms of delivery of the merchandise by SURMOUNT to
OVERCOME Ltd. (i.e., CIF [cost, insurance, and freight] or FOB).
d. It will depend on the terms of payment between SURMOUNT Ltd. and OVERCOME
(i.e., cash or credit).
(Adapted)

44.HOMAGE RESPECT Co. a new company manufacturing and selling consumable


products, has come out with an offer to refund the cost of purchase within one month of
sale if the customer is not satisfied with the product. When should HOMAGE recognize
the revenue?
a. When goods are sold to the customers, if HOMAGE can reliably estimate future
returns.
b. After one month of sale.
c. Only if goods are not returned by the customers after the period of one month.
d. At the time of sale along with an offset to revenue of the liability of the same amount
for the possibility of the return.
(Adapted)

45.PHONY COUNTERFEIT, a computer chip manufacturing company, sells its products to


its distributors for onward sales to the ultimate customers. Due to frequent fluctuations
in the market prices for these goods, PHONY has a “price protection” clause in the
distributor agreement that entitles it to raise additional billings in case of upward price
movement. Another clause in the distributor’s agreement is that PHONY can at any time
reduce its inventory by buying back goods at the cost at which it sold the goods to the
distributor. Distributors pay for the goods within 60 days from the sale of goods to
them. When should PHONY recognize revenue on sale of goods to the distributors?
a. When the goods are sold to the distributors.
b. When the distributors pay to PHONY the cost of the goods (i.e., after 60 days of the
sale of goods to the distributors).
c. When goods are sold to the distributor provided estimated additional revenue is
also booked under the “protection clause” based on past experience.
d. When the distributor sells goods to the ultimate customers and there is no
uncertainty with respect to the “price protection” clause or the buyback of goods.
(Adapted)

46.PAEAN SONG OF JOY, Inc. manufacturers and sells standard machinery. One of the
conditions in the sale contract is that installation of machinery will be undertaken by
PAEAN Inc. During December 20x1, PAEAN received a special onetime contract from
HYMN OF PRAISE Ltd. To manufacture, install, and maintain customized machinery. It is
the first time PAEAN Inc. will be producing this kind of machinery, and it is expecting
numerous changes that would need to be made to the machine after the installation is
completed, which one period is described in the contract of sale as the “maintenance

65
period.” The total cost of making the changes during the maintenance period cannot be
reasonably estimated at the time of the installation. When should the revenue from sale
of this special machine be recognized?
a. When the machinery is produced.
b. When the machinery is produced and delivered.
c. When the installation is complete.
d. When the maintenance period as per the contract of sale expires.
(Adapted)

47.One of the conditions that must be satisfied in order to recognize revenue in a


transaction involving the rendering of services is that the stage of completion of the
transaction at the end of the reporting period can be measured reliably. Which of the
following methods for determining the stage of completion of a contract involving the
rendering of services are specifically referred to in PAS18 Revenue, as being acceptable?
I. Costs incurred to date as a percentage of the estimated total costs of the transaction
II. Advances received to date as a percentage of the total amount receivable
III. Surveys of work performed
IV. Revenue to date divided by total contract revenue
a. I and III b. I and II c. I, II, and III d. I and IV
(ACCA)

48.According to PAS18 Revenue, which of the following conditions apply to the recognition
of revenue for transactions involving the rendering of services?
I. The amount of revenue can be measured reliably
II. The entity retains neither continuing managerial involvement nor effective control
over the transaction
III. The significant risks and rewards of ownership have been transferred to the buyer
IV. The costs incurred for the transaction and the costs to complete the transaction can
be measured reliably
a. I and III b. I and II c. I, II, and III d. I and IV
(ACCA)
49.According to PAS18 Revenue, which of the following criteria must be satisfied before
revenue from the sale of goods should be recognized in profit or loss?
I. Revenue can be reliably measured
II. Managerial control over the goods sold has been relinquished
III. Ownership has been transferred to the buyer
IV. The outcome of the transaction is certain
a. I and III b. I and II c. I, II, and III d. I and IV
(ACCA)

50.JUGATE Company placed an order with PAIRED Company for new specialist machinery.
The order was non-cancellable once signed and JUGATE agreed to pay for the
machinery at the time the order was signed on February 1, 20x1. PAIRED held the
machinery to JUGATE's order from June 1, 20x1, the date on which it was completed.
JUGATE commenced using the machinery on August 1, 20x1 when PAIRED completed
the installation process. PAIRED had staff on standby to deal with any operating
problems until the warranty period ended on November 1, 20x1. Under PAS18 Revenue,
PAIRED should recognize the revenue from the sale of this specialist machinery on
a. February 1, 20x1 c. August 1, 20x1
b. June 1, 20x1 d. November 1, 20x1
(ACCA)

51.Which of the following would not contribute to the difference between gross sales and
net sales?
a. Cash refunds for returned products.
b. Discounts from listed prices for delayed shipping.
c. Allowances off selling prices for defective products.

66
d. Sales made on credit rather than for immediate payment.

52.Which of the following is/are revenue recognition methods specifically stated under
PAS 18 Revenue?
I. Cost recovery Method
II. Installment sale Method
III. Percentage of completion Method
IV. Successful efforts Method
a. all of these b. I & II c. I & III d. I, II, & III
(Adapted)

53.Why are certain costs of doing business capitalized when incurred and then depreciated
or amortized over subsequent accounting periods.
a. To reduce income tax liability.
b. To adhere to the concept of conservatism.
c. To aid management in the decision-making process.
d. To match the costs of production with the revenues as earned.
(Adapted)

54.Each of the following are conditions that must be met if revenue is going to be
recognized at the time of sale except
a. seller has no significant obligations for future performance to directly affect resale
of product.
b. product return periods are less than or equal to 30 days from the time of sale.
c. price is substantially fixed or determinable at the sale date.
d. buyer pays the seller or is obligated to pay the seller (not contingent on resale).
(Adapted)

55.During year one, a professional sports team had advance ticket sales of ₱3,000,000 for
games to be played during the period beginning November 1, 20x1 and ending April 30,
20x2. Ten games are played each month. Based on this information, the company
should:
a. Report a liability for unearned revenue of ₱2,000,000 on its balance sheet at
December 31, 20x1.
b. Report a liability for unearned revenue of ₱3,000,000 on its balance sheet at
December 31, 20x1.
c. Report revenue of ₱3,000,000 on its income statement for 20x1.
d. Report revenue of ₱1,000,000 on its income statement for 20x2.
(Adapted)

56.Which is incorrect concerning recognition of revenue?


a. Revenue from rendering of services shall be recognized by reference to the stage of
completion of the transaction at balance sheet date.
b. Interest revenue shall be recognized on a time proportion basis that does not take
into account the effective yield on the asset.
c. Royalty revenue shall be recognized on an accrual basis in accordance with the
substance of the relevant agreement,
d. Dividend revenue shall be recognized when the stockholder’s right to receive
payment is established.
(Adapted)

57.Choose the correct statement.


a. The income statements deal with a description of the financial position at one point
in time, rather than the causes of change in financial position during a period.
b. For all practical purposes, accounting income and economic income are the same.
c. A gain in the value of an unsold piece of land is part of economic income as well as
accounting income.

67
d. Income and revenue have different definitions as used in accounting.
(Adapted)

58.Choose the incorrect statement


a. A restriction of retained earnings, say to meet the requirements of a bond indenture
(to facilitate payment of the bonds at maturity), permanently reduces the balance in
retained earnings
b. A statement of retained earnings is not required by PFRSs.
c. Restrictions on retained earnings must be reported as separate items on the
retained earnings statement or in the notes to the financial statements
d. Intraperiod income tax allocation requires that the total income tax for the current
period be allocated on the current financial statements
(Adapted)

59.Which of the following is expensed under the principle of systematic and rational
allocation?
a. Salespeople's monthly salaries c. Transportation to customers
b. Insurance premiums d. Electricity to light office building

60.Which of the following statements is false?


a. A trucking firm which has contracted to pack, load, transport, and deliver goods
would most appropriately recognize all revenue when they had completed the final
act, delivery.
b. Sales with right of return can be recognized at the point of sale if the amount of
future returns can be reasonably estimated, and other criteria for recognition of a
sale have been met.
c. Current PFRSs allow the recording of sales with right of return until the return
period has elapsed if future returns cannot be estimated at the point of sale.
d. Expenses can be divided into two categories, those directly related to the sales of
products or services and those that result from general and administrative
activities.
(Adapted)

61.Revenue arising from the sale of goods should be recognized when all of the following
conditions have been satisfied, except
I. The seller has transferred to the buyer the significant risks and rewards of
ownership.
II. The seller retains either continuing managerial involvement to the degree
usually associated with ownership or effective control over the goods sold.
III. The amount of revenue can be measured reliably.
IV. It is probable that the economic benefits associated with the transaction will
flow to the seller.
V. The costs incurred or to be incurred in respect of the transaction can be
measured reliably.
a. all of these are included c. II only
b. V only d. I, III, IV and V

62.In a normal sale, generally the most uncertain factor in the revenue recognition process
is
a. the seller's fulfillment of its responsibility in the transaction
b. the measurability of the resource or item received by the seller
c. the realizability of the resource or item received by the seller
d. the relevance of the resource or item received by the seller
(Adapted)

63.On April 30, OGLE, TO EYE AMOROUSLY Corporation sold land with a book value of
₱600,000 to KINDAT SABAY KAGAT LABI Company for its market value of ₱800,000.

68
KINDAT gave OGLE a 12%, ₱800,000 note secured only by the land. At the date of sale,
KINDAT was in a very poor financial position and its continuation as a going concern
was very questionable. OGLE should:
a. record a ₱200,000 gain on the sale of land
b. fully reserve the note
c. use the cost recovery method of accounting
d. record the note at its discounted value
(RPCPA)

64.Which of the following methods of service revenue recognition usually would be most
appropriate for a business engaged in packing, loading, transporting and delivering
freight?
a. Proportional performance method c. Specific performance method
b. Completed performance method d. Collection method
(Adapted)

65.JARGON TECHNICAL TERMINOLOGY Corporation is preparing its annual financial


statements at December 31, 20x1. Because of a recently proven health hazard in one of
its paints, the government has clearly indicated its intention of having the corporation
recall all cans of this paint sold in the last six months. Management estimates that this
recall would cost ₱1,000,000. What accounting recognition, if any, should be accorded
this situation?
a. Operating expense of ₱1,000,000 c. No recognition
b. Non-operating expense of ₱1,000,000 d. Note disclosure
(Adapted)

66.Which of the following is an example of the concept of conservatism?


a. Using the percentage-of-completion method in its first year of a long-term
construction contract
b. Stating inventories using the FIFO method in periods of rising prices
c. Stating inventories at the lower of cost or NRV
d. Using the interest method instead of the straight-line method to record interest in
the first year of a long-term receivable
(Adapted)

67.CACOPHONY HARSH SOUND Co. sells equipment on installment contracts. Which of the
following statements best justifies CACOPHONY’s use of the cost recovery method of
revenue recognition to account for these installment sales?
a. The sales contract provides that title to the equipment only passes to the purchaser
when all payments have been made.
b. No cash payments are due until one year from the date of sale.
c. Sales are subject to a high rate of return.
d. There is no reasonable basis for estimating collectibility.
(AICPA)

68.CLEAVE TO SPLIT Inc. specializes in real estate transactions other than retail land sales.
On January 1, 20x1, CLEAVE consummated a sale of property to TEAR Corporation. The
amount of profit on the sale is determinable and CLEAVE is not obligated to perform
any additional activities to earn the profit. TEAR’s initial and continuing investments
were adequate to demonstrate a commitment to pay for the property. However,
CLEAVE’s receivable will probably be subjected to future subordination. CLEAVE should
account for the sale using the
a. Deposit method. c. Cost recovery method.
b. Reduced recovery method. d. Full accrual method.
(AICPA)

69
69.On January 1, 20x1, MAIM TO CRIPPLE Co. installed cabinets to display its merchandise
in customers’ stores. MAIM expects to use these cabinets for five years. MAIM’s 20x1
multi-step income statement should include
a. One-fifth of the cabinet costs in cost of goods sold.
b. One-fifth of the cabinet costs in selling, general, and administrative expenses.
c. All of the cabinet costs in cost of goods sold.
d. All of the cabinet costs in selling, general, and administrative expenses.
(AICPA)

70.According to PAS 1, a material loss should be presented separately as a component of


income from continuing operations
a. when it is an extraordinary item.
b. when it is a cumulative-effect-type change in accounting principle.
c. when it is unusual in nature and infrequent in occurrence.
d. whether it is unusual in nature or infrequent in occurrence.
(Adapted)

71.In 20x1, hail damaged several of SARDONIC SARCASTIC Co.’s vans. Hailstorms had
frequently inflicted similar damage to SARDONIC’s vans. Over the years, SARDONIC had
saved money by not buying hail insurance and either paying for repairs, or selling
damaged vans and then replacing them. In 20x1, the damaged vans were sold for less
than their carrying amount. How should the hail damage cost be reported in
SARDONIC’s 20x1 financial statements?
a. The actual 2003 hail damage loss as an extraordinary loss, net of income taxes.
b. The actual 2003 hail damage loss in continuing operations, with no separate
disclosure.
c. The expected average hail damage loss in continuing operations, with no separate
disclosure.
d. The expected average hail damage loss in continuing operations, with separate
disclosure.
(Adapted)

72.In 20x1, ELUSION ESCAPE Co. incurred losses arising from its guilty plea in its first
antitrust action, and from a substantial increase in production costs caused when a
major supplier’s workers went on strike. Which of these losses should be reported as an
extraordinary item?
(Item #1) Antitrust action; (Item #2) Production costs
a. No, No b. No, Yes c. Yes, No d. Yes , Yes
(AICPA)

73.Revenue is recognized at the time of sale under the:


a. cost recovery method
b. collection method
c. percentage-of-completion method
d. sales method when goods are sold on credit
(AICPA)

74.Revenue from sale of goods shall be recognized when all of the conditions have been
satisfied, except
a. The entity has transferred to the buyer the significant risks and rewards of
ownership of the goods.
b. The entity retains either continuing managerial involvement or effective control
over the goods sold.
c. The amount of revenue can be measured reliably.
d. It is probable that economic benefits will flow to the entity.

70
75.POLITIC SHREWD Co. operates a catering service that specializes in business luncheons
for large corporations. POLITIC requires customers to place their orders 2 weeks in
advance of the scheduled events. POLITIC bills its customers on the tenth day of the
month following the date of service and requires that payment be made within 30 days
of the billing date. Conceptually, POLITIC should recognize revenue from its catering
services at the date when a
a. Customer places an order. c. Billing is mailed.
b. Luncheon is served. d. Customer’s payment is received.
(Adapted)

76.An entity sells a durable good to a customer on January 1, 20X3, and the customer is
automatically given a 1-year warranty. The customer also buys an extended warranty
package, extending the coverage for an additional 2 years to the end of 20X5. At the
time of the original sale, the company expects warranty costs to be incurred evenly over
the life of the warranty contracts. The customer has only one warranty claim during the
3-year period, and the claim occurs during 20X4. The company will recognize income
from the sale of the extended warranty
a. On January 1, 20X3.
b. In years 20X4 and 20X5.
c. At the time of the claim in 20X4.
d. December 31, 20X5, when the warranty expires.
(Adapted)

77.Using the cost-recovery method of revenue recognition, profit on an installment sale is


recognized
a. On the date of the installment sale.
b. In proportion to the cash collections.
c. After cash collections equal to the cost of goods sold have been received.
d. On the date the final cash collection is received.
(Adapted)

78.On February 1, year 1 a computer software firm agrees to program a software package.
Twelve payments of ₱10,000 on the first of each month are to be made, with the first
payment due on March 1, year 1. The software is accepted by the client on June 1, year
2. How much year 1 revenue should be recognized?
a. 0 b. 100,000 c. 110,000 d. 120,000

79.When a right of return exists, an entity may recognize revenue from a sale of goods at
the time of sale only if
a. The amount of future returns can be reliably estimated.
b. The seller retains the risks and rewards of ownership.
c. The buyer resells the goods.
d. The seller believes returns will not be material.
(AICPA)

80.In which of the following examples of real estate transactions would the seller not
transfer the usual risks and rewards of ownership?
I. The buyer can compel the seller to repurchase the property.
II. The seller guarantees the return of the buyer’s investment.
III. The seller is required to support operations of the buyer and will be reimbursed on
a cost plus 5% basis.
a. I. b. II. c. III. d. I and II.

81.Which of the following is not a correct statement about sales taxes?


a. Sales taxes are an expense of the seller.
b. Many companies record sales taxes in the sales account.

71
c. If sales taxes are included in the sales account, the first step to find the amount of
sales taxes is to divide sales by 1 plus the sales tax rate.
d. All of these are true.
(AICPA)

82.For ₱50 a month, GAUDY SHOWY Co. visits its customers' premises and performs insect
control services. If customers experience problems between regularly scheduled visits,
GAUDY makes service calls at no additional charge. Instead of paying monthly,
customers may pay an annual fee of ₱540 in advance. For a customer who pays the
annual fee in advance, GAUDY should recognize the related revenue
a. When the cash is collected.
b. At the end of the fiscal year.
c. At the end of the contract year after all of the services have been performed.
d. Evenly over the contract year as the services are performed.
(Adapted)

83.Revenue may not be recognized in which of the following transactions


a. A department store operates a customer loyalty program. It grants loyalty points to
its program members whenever they spend a specified amount in one of its stores.
The points can be redeemed for further products in its stores.
b. A restaurant distributes "money-off" vouchers. It grants vouchers free of charge to
people walking on the streets near the restaurant. The vouchers can be redeemed
for discounted meals in the restaurant.
c. A book retailer participates in a customer loyalty program operated by an airline. It
grants loyalty air miles to its program members whenever they spend a specified
amount in one of its stores. The points can be redeemed for air travel with the
airline.
d. Revenue may not be recognized in any of the above transactions.

84.It is proper to recognize revenue prior to the sale of merchandise when


I. The revenue will be reported as an installment sale.
II. The revenue will be reported under the cost recovery method.
a. I only. b. II only. c. Both I and II. d. Neither I nor II.
(AICPA)

85.A real estate broker engaged in the sale of real estate on commission basis should
recognize revenue on the basis of
a. cash collection c. specific performance
b. completed performance d. proportional performance
(AICPA)

86.Which of the following statements is incorrect?


a. For a business engaged in delivering, revenue is normally recognized when the
goods ordered have been delivered.
b. Proportional performance is synonymous with percentage-of-completion.
c. Cost recovery method allows revenue recognition only when the cost has been
recovered.
d. Percentage of completion method allows full recognition of revenue prior to 100%
completion of the act giving rise to revenue recognition.

87.Which of the following statements is incorrect?


a. One of the reasons for recognizing revenue at the time of sale is that the risk of loss
due to price decline or damage to the goods is passed on to the buyer.
b. Revenue is recorded when products are sold, services are provided, or entity
resources are used by others. Revenue is also recorded when an entity sells assets
other than products for proceeds exceeding the assets’ carrying amounts.

72
c. Revenue results only from those types of profit-directed activities that can change
owners’ equity under PFRSs.
d. Revenue can also arise from customer loyalty programs.

88.How would the proceeds received from the advance sale of nonrefundable tickets for a
theatrical performance be reported in the seller’s financial statements before the
performance?
a. Revenue for the entire proceeds
b. Revenue to the extent of related costs expended
c. Unearned revenue to the extent of related costs expended
d. Unearned revenue for the entire proceeds
(AICPA)

Chapter 38 - Suggested answers to theory of accounts questions


1. D 16. A 31. A 46. D 61. C 76. B
2. C 17. B 32. A 47. A 62. C 77. C
3. A 18. C 33. B 48. D 63. C 78. A
4. A 19. A 34. D 49. B 64. B 79. A
5. A 20. A 35. A 50. C 65. A 80. D
6. B 21. A 36. C 51. D 66. C 81. A
7. D 22. A 37. A 52. C 67. D 81. D
8. A 23. C 38. B 53. D 68. C 83. B
9. C 24. C 39. B 54. B 69. B 84. D
10. D 25. A 40. D 55. A 70. D 85. C
11. C 26. B 41. C 56. B 71. B 86. D
12. D 27. A 42. D 57. D 72. A 87. B
13. B 28. B 43. B 58. A 73. D 88. D
14. C 29. B 44. A 59. B 74. B
15. B 30. B 45. D 60. D 75. B

73
Chapter 39
Non-current assets Held for Sale and Discontinued
Operations

Chapter 39: Multiple choice – Computational (SET B) – (For classroom instruction


purposes)
Availability for immediate sale in present condition
Use the following information for the next two questions:
VISAGE APPEARANCE Co. is committed to a plan to sell its headquarters building and has
initiated actions to locate a buyer. As of this date, the building has a carrying amount of
₱5,000,000, a fair value of ₱6,000,000 and estimated costs to sell of ₱200,000.

1. VISAGE Co. has an intention to transfer ownership of a building to a buyer after it


vacates the building. How should VISAGE Co. classify the headquarters building?
a. Included under property, plant and equipment at ₱5,000,000.
b. Included under property, plant and equipment at ₱5,800,000.
c. Classified as held for sale at ₱5,000,000
d. Classified as held for sale at ₱5,800,000

2. VISAGE Co. will continue to use the building until the construction of a new
headquarters is completed. How should VISAGE Co. classify the headquarters building?
a. Included under property, plant and equipment at ₱5,000,000.
b. Included under property, plant and equipment at ₱5,800,000.
c. Classified as held for sale at ₱5,000,000
d. Classified as held for sale at ₱5,800,000

Availability for immediate sale in present condition


Use the following information for the next two questions:
DECORTICATE PEEL, Inc. is committed to a plan to sell a manufacturing facility and has
initiated actions to locate a buyer. As of this date, the building has a carrying amount of
₱6,000,000, a fair value of ₱5,000,000 and estimated costs to sell of ₱200,000. At the plan
commitment date, there is a backlog of uncompleted customer orders.

3. DECORTICATE, Inc. intends to sell the manufacturing facility with its operations. Any
uncompleted customer orders at the sale date will be transferred to the buyer. The
transfer of uncompleted customer orders at the sale date will not affect the timing of
the transfer of the facility. How should DECORTICATE Co. classify the manufacturing
facility?
a. Included under property, plant and equipment at ₱6,000,000.
b. Included under property, plant and equipment at ₱4,800,000.
c. Classified as held for sale at ₱6,000,000
d. Classified as held for sale at ₱4,800,000

4. DECORTICATE, Inc. intends to sell the manufacturing facility, but without its operations.
The entity does not intend to transfer the facility to a buyer until after it ceases all
operations of the facility and eliminates the backlog of uncompleted customer orders.
How should DECORTICATE Co. classify the manufacturing facility?
a. Included under property, plant and equipment at ₱6,000,000.
b. Included under property, plant and equipment at ₱4,800,000.
c. Classified as held for sale at ₱6,000,000
d. Classified as held for sale at ₱4,800,000

Availability for immediate sale in present condition


Use the following information for the next two questions:

74
DEMOTIC POPULAR Co. acquires through foreclosure a property comprising land and
buildings that it intends to sell. The fair value of the land and buildings is ₱6,000,000 and
costs to sell are ₱200,000. The related defaulted receivables have a carrying amount of
₱5,000,000.

5. The entity does not intend to transfer the property to a buyer until after it completes
renovations to increase the property’s sales value. How should DEMOTIC Co. classify
the land and buildings?
a. Included under property, plant and equipment at ₱5,000,000.
b. Included under investment property at ₱5,000,000.
c. Included under investment property at ₱5,800,000.
d. Classified as held for sale at ₱5,800,000

6. After the renovations are completed and the property is classified as held for sale but
before a firm purchase commitment is obtained, the entity becomes aware of
environmental damage requiring remediation. The entity still intends to sell the
property. However, the entity does not have the ability to transfer the property to a
buyer until after the remediation is completed. The costs of renovations made totaled
₱200,000. The estimated costs of remediation are ₱100,000. How should DEMOTIC Co.
classify the land and buildings?
a. Included under property, plant and equipment at ₱5,700,000.
b. Included under investment property at ₱6,000,000.
c. Included under investment property at ₱5,700,000.
d. Classified as held for sale at ₱5,700,000

Highly probable sale


7. PERAMBULATE STROLL Co. is a commercial leasing and finance company. As of year-
end, PERAMBULATE holds equipment that is available either for sale or lease.
PERAMBULATE is not yet decided whether to sell or to lease the equipment. The
equipment has a carrying amount of ₱1,000,000, fair value of ₱1,200,000 and costs to
sell of ₱50,000. How should PERAMBULATE Co. classify the equipment?
a. Inventory, ₱1,000,000 c. Held for sale, ₱1,150,000
b. Investment property, ₱1,250,000 d. Held for sale, ₱1,000,000

Highly probable sale


8. PHILANDERING FLIRTING Co. is committed to a plan to sell a property that is in use.
The transfer of the property will be accounted for as a sale and finance leaseback. The
property has a carrying amount of ₱1,000,000, fair value of ₱1,200,000 and costs to sell
of ₱50,000. How should PHILANDERING Co. classify the equipment?
a. Property, plant and equipment, ₱1,000,000
b. Investment property, ₱1,250,000
c. Held for sale, ₱1,150,000
d. Inventory, ₱1,000,000

Exception to one-year requirement


9. An entity in the power generating industry is committed to a plan to sell a disposal
group that represents a significant portion of its regulated operations. The sale requires
regulatory approval, which could extend the period required to complete the sale
beyond one year. Actions necessary to obtain that approval cannot be initiated until
after a buyer is known and a firm purchase commitment is obtained. However, a firm
purchase commitment is highly probable within one year. The disposal group has a
carrying amount of ₱10,000,000 and fair value less costs to sell of ₱10,600,000. How
should the entity classify the disposal group?
a. Held for sale, ₱10.6M c. Under previous classifications, ₱10M
b. Held for sale, ₱10M d. Under previous classifications, ₱10.6M

75
Exception to one-year requirement
10. An entity is committed to a plan to sell a manufacturing facility in its present condition
and classifies the facility as held for sale at that date. After a firm purchase commitment
is obtained, the buyer’s inspection of the property identifies environmental damage not
previously known to exist. The entity is required by the buyer to make good the
damage, which will extend the period required to complete the sale beyond one year.
However, the entity has initiated actions to make good the damage, and satisfactory
rectification of the damage is highly probable. The manufacturing facility has a carrying
amount of ₱10,000,000 and fair value less costs to sell of ₱10,600,000. How should the
entity classify the manufacturing facility?
a. Held for sale, ₱10.6M c. PPE, ₱10M
b. Held for sale, ₱10M d. PPE, ₱10.6M

Exception to one-year requirement


Use the following information for the next two questions:
In 20x1, FORGETIVE CREATIVE Co. classified a property as held for sale. The carrying
amount prior to classification is ₱400,000 while fair value less cost to sell is ₱360,000. The
property is being sold at ₱360,000.

During 20x1, the market conditions that existed at the date the asset was classified initially
as held for sale deteriorate and, as a result, the asset is not sold by the end of that period.
During that period, FORGETIVE actively solicited but did not receive any reasonable offers
to purchase the asset and, in response, FORGETIVE reduced the price from ₱360,000 to
₱320,000. The fair value less costs to sell on December 31, 20x1 is ₱340,000.

11. How should FORGETIVE Co. classify the property in its 20x1 annual financial
statements?
a. Held for sale, ₱320,000 c. PPE, ₱340,000
b. Held for sale, ₱340,000 d. PPE, ₱400,000

12. During 20x2, the market conditions deteriorate further, and the asset is not sold by
December 31, 20x2. FORGETIVE Co. believes that the market conditions will improve
and has not further reduced the price of the asset. The fair value less costs to sell on
December 31, 20x2 is ₱300,000. If the property was not classified as held for sale in
20x1, its carrying amount by this time would have been ₱350,000.
a. Held for sale, ₱300,000 c. PPE, ₱300,000
b. Held for sale, ₱320,000 d. PPE, ₱350,000

Event after reporting period


13. WAYFARER TRAVELER Co. is preparing its December 31, 20x1, current year financial
statements. A land included in WAYFARER’s property, plant and equipment that did not
qualify as held for sale as of December 31, 20x1 was actually sold on January 5, 20x2.
The financial statements were authorized for issue on March 1, 20x2. On December 31,
20x1, WAYFARER has total current assets of ₱9,000,000. Not included in this amount is
the fair value less costs to sell of the land amounting to ₱1,000,000. How much is the
total current assets current in WAYFARER’s December 31, 20x1 financial statements?
a. ₱8,000,000 c. ₱10,000,000
b. ₱9,000,000 d. ₱11,000,000

Measurement
14. On December 31, 20x1, STRIDENT HARSH-SOUNDING Co. classified its building with a
historical cost of ₱4,000,000 and accumulated depreciation of ₱2,400,000 as held for
sale. All of the criteria under PFRS 5 are complied with. On that date, the land has a fair
value of ₱1,400,000 and cost to sell of ₱80,000. The entry on December 31, 20x1
includes
a. a debit to building for ₱1,320,000
b. a credit to accumulated depreciation for ₱2,400,000

76
c. a debit to impairment loss for ₱280,000
d. No reclassification entry will be made on December 31, 20x1

Reversal of impairment loss


15. On December 31, 20x1, OBSTINACY STUBBORNESS Co. classified its building with a
carrying amount of ₱1,600,000 and fair value less cost to sell of ₱1,320,000 as held for
sale.

The building was not sold in 20x2. However, the exception to the one-year requirement
was met. On December 31, 20x2, the fair value less cost to sell of building is ₱1,240,000.

The building was not sold in 20x3. However, the exception to the one-year requirement
was still met. On December 31, 20x3, the fair value less cost to sell of building increased to
₱1,680,000. How much is the gain on reversal of impairment to be recognized on December
31, 20x3?
a. 440,000 b. 360,000 c. 280,000 d. 0

Allocation of impairment losses and reversals


Use the following information for the next four questions:
On December 31, 20x1, INSOUCIANT CAREFREE Co. plans to dispose of a group of its
assets. Information on these assets is shown below:
Carrying amount as
Carrying amount on remeasured
Dec. 31, 20x1 before immediately before
classification as held classification as held
  for sale for sale
Inventory 9,600,000 8,800,000
Investment in FVOCI 7,200,000 6,000,000
Investment property (at cost model) 22,800,000 22,800,000
PPE (at cost model) 18,400,000 16,000,000
Goodwill 6,000,000 6,000,000
Total 64,000,000 59,600,000

INSOUCIANT Co. entity estimates that the fair value less costs to sell of the disposal group
amounts to ₱52,000,000.

16. How would the reduction in the value of the assets on classification as held for sale be
treated in the financial statements?
a. The entity recognizes a loss of ₱4.4M immediately before classification as held for
sale and then recognizes an impairment loss of ₱7.6M.
b. The entity recognizes an impairment loss of ₱12 million.
c. The entity recognizes an impairment loss of ₱7.6M.
d. The entity recognizes a loss of ₱12M immediately before classifying the disposal
group as held for sale.

17. How much is the carrying amount of the inventory after classification of the disposal
group as held for sale?
a. 8,800,000 b. 7,950,576 c. 7,899,324 d. 7,765,391

18. How much is the carrying amount of the Investment property (at cost model) after
classification of the disposal group as held for sale?
a. 22,800,000 b. 21,859,794 c. 21,786,665 d. 20,766,298

19. How much is the carrying amount of the PPE (at cost model) after classification of the
disposal group as held for sale?
a. 16,000,000 b. 15,780,740 c. 15,340,206 d. 15,211,612

77
Change to a plan of sale
20. On December 31, 20x1, INGENIOUS NATURAL Co. classified its building with a carrying
amount of ₱1,600,000 and fair value less costs to sell of ₱1,320,000 as held for sale.
Impairment loss of ₱280,000 was recognized on that date. The building has a remaining
useful life of 4 years and it was depreciated using the straight-line method.

As of December 31, 20x2, the building was not yet sold and management decided not to sell
the building anymore. The fair value less cost to sell of the building on December 31, 20x2
is ₱1,240,000 while the value in use is ₱1,220,000.

How much is the carrying amount of the building upon reclassification back to property,
plant and equipment?
a. 1,220,000 b. 1,320,000 c. 1,240,000 d. 1,200,000

Discontinued operations
21. On December 31, 20x1, INIMICAL UNFRIENDLY Co. entered into an agreement to sell a
component. On that date, INIMICAL estimated the gain from the disposal to be made in
20x2 at ₱2,000,000 and the operating losses prior to the date of sale to be ₱1,200,000.
As a result of the sale, the component’s operations and cash flows will be eliminated
from the entity’s operations and the entity will not have any significant continuing post-
sale involvement in the component’s operations. Accordingly, the component was
classified as held for sale and discontinued operations.

The component’s actual operating losses in 20x1 and 20x2 were ₱2,800,000 and
₱2,600,000, respectively, and the actual gain on disposal of the component in 20x2 was
₱1,600,000. INIMICAL’s income tax rate is 30%. Any income tax benefit is expected to be
realizable. There were no other temporary differences during the year.

What single, post-tax amounts should be reported for discontinued operations in


INIMICAL’s comparative 20x2 and 20x1 income statements, respectively?
a. (1,960,000), (700,000) c. (650,000), (1,950,000)
b. (560,000), (1,960,000) d. (700,000), (1,960,000)

Discontinued operations - Direct costs of disposal


22. On April 30, 20x1, ABROGATE ABOLISH Co. approved a plan to dispose of a component
of its operations. The disposal meets the requirements for classification as discontinued
operations.

From January 1 to April 30, 20x1, the component earned operating profit of ₱400,000 and
from May 1 to December 31, 20x1, the segment suffered operating losses of ₱200,000.

The net assets of the component has a carrying amount of ₱32,000,000 as of April 30, 20x1.
The fair value less costs to sell of the component is ₱26,000,000. Additional estimated
disposal loss includes severance pay of ₱220,000 and employee relocation costs of
₱100,000, both of which are directly associated with the decision to dispose of the segment.
ABROGATE’s income tax rate is 30%. Any income tax benefit is expected to be realizable.
There were no other temporary differences during the year.

How much is the profit (loss) from discontinued operations to be reported in ABROGATE's
statement of profit or loss and other comprehensive income for the year ended December
31, 20x1?
a. 4,564,000 b. 4,060,000 c. 4,340,000 d. 4,284,000

The answers and solutions to the computational problems above


(Multiple choice – Computational (SET B) can be found in the
accompanying Teacher’s Manual.

78
Chapter 39: Theory of Accounts Reviewer
1. PFRS 5 requires
a. assets that meet the criteria to be classified as held for sale to be measured at fair
value less costs to sell
b. depreciation on held for sale assets to cease but amortization should still be
provided for
c. assets that meet the criteria to be classified as held for sale to be presented
separately on the face of the statement of financial position and the results of
discontinued operations to be presented separately in the statement of profit or loss
and other comprehensive income.
d. assets and liabilities of a disposal group to be off-set and presented as one item on
the face of the statement of financial position

2. Which of the following statements is incorrect?


a. PFRS 5 specifies that assets or disposal groups that are classified as held for sale are
carried at fair value less costs to sell.
b. PFRS 5 specifies that an asset classified as held for sale, or included within a
disposal group that is classified as held for sale, is not depreciated.
c. PFRS 5 specifies that an asset classified as held for sale, and the assets and liabilities
included within a disposal group classified as held for sale, are presented separately
in the statement of financial position.
d. PFRS 5 specifies that assets classified as non-current in accordance with PAS 1
Presentation of Financial Statements shall not be reclassified as current assets until
they meet the criteria to be classified as held for sale in accordance with this PFRS.

3. A non-current asset is an asset that


a. is being depreciated or amortized
b. is not expected to be realized within 12 months
c. is restricted cash
d. does not meet the definition of a current asset.

4. Which of the following assets is outside the scope of PFRS 5?


a. Property, plant and equipment
b. Investment property carried under the Cost model
c. Deferred tax assets
d. Intangible assets

5. Which of the following assets is within the scope of PFRS 5?


a. Assets arising from employee benefits
b. Investment property carried under the fair value model
c. Deferred tax assets
d. Intangible assets

6. To which of the following types of asset do the measurement provisions of PFRS5 Non-
current assets held for sale and discontinued operations apply?
I. Financial assets
II. Intangible development assets
III. Leasehold buildings
IV. Biological assets
V. Contractual rights under insurance contracts
a. I, IV, V b. II, III c. II, III, IV d. III
(ACCA)

7. It is a group of assets to be disposed of, by sale or otherwise, together as a group in a


single transaction, and liabilities directly associated with those assets that will be
transferred in the transaction. The group includes goodwill acquired in a business

79
combination if the group is a cash-generating unit to which goodwill has been allocated
or if it is an operation within such a cash-generating.
a. discontinued operations c. cash generating unit
b. disposal group d. component of an entity

8. Assets which are previously classified as non-current in accordance with PAS 1


Presentation of Financial Statements when they meet the criteria to be classified as held
for sale are to be presented separately on the face of the balance sheet and included as
a. Non-current asset c. Neither current nor noncurrent
b. Current asset d. Any of these

9. An entity shall classify a non-current asset (or disposal group) as held for sale when
a. the non-current asset’s (or disposal group’s) carrying amount will be recovered
principally through continuing use rather than through a sale transaction.
b. the non-current asset’s (or disposal group’s) carrying amount will be recovered
principally through a sale transaction rather than through continuing use.
c. the non-current asset (or disposal group) has actually been disposed of.
d. the non-current asset (or disposal group) is abandoned or taken out temporarily
from use.

10.When the economic benefits from a non-current asset (or disposal group) will be
realized more if it is to be sold rather than continually used
a. the non-current asset (or disposal group) will be classified as “held for sale” and
presented as part of current assets in a classified statement of financial position
b. the non-current asset (or disposal group) will be classified as “held for sale” and
presented as part of noncurrent assets in a classified statement of financial position
c. the non-current asset (or disposal group) will be classified as “held for sale” only if
management expects to earn profit from the disposal
d. the non-current asset (or disposal group) will be classified as “held for sale” only if
management expects to abandon the non-current asset (or disposal group)

11.A non-current asset or a disposal group shall be classified as held for sale if which of the
following conditions are met.
I. the asset or disposal group must be available for immediate sale in its present
condition subject only to terms that are usual and customary
II. the sale must be highly probable.
a. I only b. II only c. I and II d. I or II

12.A non-current asset or a disposal group shall be classified as held for sale if the asset or
disposal group is available for immediate sale in its present condition subject only to
terms that are usual and customary and the sale is highly probable. Sale is highly
probable if
I. an appropriate level of management is committed to a plan to sell the asset
II. an active program to locate a buyer and complete the plan will yet to be initiated
III. the asset or disposal group must be actively marketed for sale at a price that
reflects a reasonable profit
IV. the sale (or exchange with commercial substance) should be expected to qualify
for recognition as a completed sale within two years from the date of
classification
V. actions required to complete the plan should indicate that it is unlikely that
significant changes to the plan will be made or that the plan will be withdrawn:
a. I, V b. I, II, V c. I, II, III, V d. I, II, III, IV, V

13.A non-current asset or a disposal group shall be classified as held for sale if certain
conditions provided under PFRS 5 are met. Which of the following is among those
conditions?

80
a. the asset or disposal group must be available for immediate sale in its present
condition subject only to terms that are usual and customary and the sale must be
probable.
b. an appropriate level of management must have received a purchase commitment for
the sale of the asset
c. an active program to locate a buyer and complete the plan will yet to be initiated
d. actions required to complete the plan should indicate that it is unlikely that
significant changes to the plan will be made or that the plan will be withdrawn:

14.In order for a non-current asset to be classified as held-for-sale the sale must be highly
probable. Highly probable means
a. that the future sale will occur
b. that the future sale might occur
c. management must be committed to selling the asset and must be actively looking for
a buyer
d. the sale contract has been signed
(ACCA)

15.In order for a noncurrent asset to be classified as held for sale, the sale must be highly
probable. “Highly probable” means that
a. The future sale is likely to occur.
b. The future sale is more likely than not to occur.
c. The sale is certain.
d. The probability is higher than more likely than not.
(Adapted)

16.In accordance with PFRS 5 Non-current Assets Held for Sale and Discontinued Operations,
an asset should be classified as held for sale when which of the following criteria are
satisfied?
I. The sale is highly probable
II. The asset has a readily observable market value
III. The sale is expected to be completed within 3 months of the end of the reporting
period
IV. The asset is available for immediate sale in its present condition
a. I, IV b. I, II, IV c. I, III, IV d. I, II, III, IV
(ACCA)

17.The directors of SPOONY FOOLISHLY IN LOVED Company decided at a board meeting


on February 28, 20x1 that a major machine tool should be sold. Trade magazines
reported recent transactions in non-current assets of a similar age at ₱50,000, but the
board decided that the asking price should be ₱75,000. The board also decided that as a
program of repairs to the tool needed to be carried out, an agent should not be
contracted with for the sale of the item until the repairs were completed, which was on
May 31, 20x1.

On July 31, 20x1 the board agreed to reduce the asking price to ₱50,000. A deal was agreed
with a buyer on August 31, 20x1 and completion of the sale took place on November 30,
20x1. In accordance with PFRS 5 Non-current Assets Held for Sale and Discontinued
Operations, the asset should be classified as held for sale on
a. February 28, 20x1 c. July 31, 20x1
b. May 31, 20x1 d. August 31, 20x1
(ACCA)

18.Significantly more likely than probable.


a. certain c. 90% to 99% probability
b. highly probable d. reasonably possible

81
19.More likely than not.
a. Reasonably possible c. Highly probable
b. Probable d. Certain

20.In which of the following instances would classification as “held-for-sale” appropriate?


a. CHARLATAN Co. will sell its building to IMPOSTOR, Inc. after CHARLATAN vacates
the building.
b. CHARLATAN Co. will sell its old building to IMPOSTOR, Inc. but CHARLATAN will
continue to use the building until the construction of CHARLATAN’s new building is
completed.
c. CHARLATAN Co. will sell its manufacturing facility to IMPOSTOR, Inc. until after
CHARLATAN ceases all operations of the facility and eliminates the backlog of
uncompleted customer orders.
d. CHARLATAN Co. will sell its equipment to IMPOSTOR, Inc. after a major overhaul
and replacement of major parts

21.Classification as held-for-sale is appropriate in which of the following instances?


a. PLENARY, Inc. will transfer a property to FULL Co. after PLENARY completes
renovations to increase the property’s sales value.
b. PLENARY, Inc. will transfer a property to FULL Co. after necessary remediation
identified during inspection is completed by PLENARY
c. PLENARY, Inc. will transfer a property to FULL Co. after the notarization of the deed
of transfer
d. PLENARY, Inc. will transfer a property to FULL Co. after PLENARY completes a
purchase transaction for a replacement property

22.A component of an entity is classified as “held for sale” when the following conditions
are met (choose the incorrect one)
a. Management is committed to a plan to sell.
b. The component is available for immediate sale in its present condition.
c. An active program to locate a buyer is initiated.
d. The sale is highly probable within two years from the date of classification as held
for sale.

23.An agreement with an unrelated party, binding on both parties and usually legally
enforceable, that (a) specifies all significant terms, including the price and timing of the
transactions, and (b) includes a disincentive for non-performance that is sufficiently
large to make performance highly probable.
a. forward contract c. futures contract
b. firm purchase commitment d. option contract

24.Events or circumstances may extend the period to complete the sale beyond one year.
An extension of the period required to complete a sale does not preclude an asset (or
disposal group) from being classified as held for sale if:
I. the delay is attributable to events or circumstances beyond the entity’s control
II. there is sufficient evidence that the entity remains committed to its plan to sell the
asset (or disposal group)
a. I only b. II only c. I and II d. none

25.In which of the following situations would an exception to the one-year requirement
under PFRS 5 not apply?
a. An entity in the power generating industry is committed to a plan to sell a disposal
group that represents a significant portion of its regulated operations. The sale
requires regulatory approval, which could extend the period required to complete
the sale beyond one year. Actions necessary to obtain that approval cannot be
initiated until after a buyer is known and a firm purchase commitment is obtained.
However, a firm purchase commitment is highly probable within one year.

82
b. An entity is committed to a plan to sell a manufacturing facility in its present
condition and classifies the facility as held for sale at that date. After a firm purchase
commitment is obtained, the buyer’s inspection of the property identifies
environmental damage not previously known to exist. The entity is required by the
buyer to make good the damage, which will extend the period required to complete
the sale beyond one year. However, the entity has initiated actions to make good the
damage, and satisfactory rectification of the damage is highly probable.
c. In 20X1, ABC Co. classified a property as held for sale. The carrying amount prior to
classification is ₱100 while fair value less cost to sell is ₱90. The property is being
sold at ₱90. During 20X1, the market conditions that existed at the date the asset
was classified initially as held for sale deteriorate and, as a result, the asset is not
sold by the end of that period. During that period, the entity actively solicited but
did not receive any reasonable offers to purchase the asset and, in response,
reduced the price. The selling price was reduced to ₱80.
d. In 20X1, ABC Co. classified a property as held for sale. The carrying amount prior to
classification is ₱100 while fair value less cost to sell is ₱90. The property is being
sold at ₱90. During 20X1, the market conditions that existed at the date the asset
was classified initially as held for sale deteriorate and, as a result, the asset is not
sold by the end of that period. During that period, the entity actively solicited but
did not receive any reasonable offers to purchase the asset. The entity believes that
the market conditions will improve and has not further reduced the price of the
asset. The asset continues to be held for sale, but is still being sold at ₱90, a price in
excess of its current fair value.

26.Which of the following incorrectly relates to the provisions of PFRS 5?


a. Sale transactions include exchanges of non-current assets for other non-current
assets even when the exchange has commercial substance in accordance with PAS
16.
b. Non-current assets (or disposal groups) held for sale must be tested for impairment
at the date of initial classification and subsequently
c. Non-current assets (or disposal groups) should not be depreciated during the
period in which they are classified as held for sale
d. Assets classified as non-current in accordance with PAS 1 shall be reclassified as
current assets if they are sold after the balance sheet date but before the financial
statements are authorized by management for issue.

27.When an entity acquires a non-current asset (or disposal group) exclusively with a view
to its subsequent disposal, it shall classify the non-current asset (or disposal group) as
held for sale at the acquisition date
a. only if the “sale within one-year” requirement is met and it is highly probable that
any other criteria that are not met at that date will be met within a short period
following the acquisition (usually within three months).
b. only if all of requirements in PFRS 5 are met
c. only if the “available for immediate sale” criteria is met and it is highly probable that
any other criteria that are not met at that date will be met within a short period
following the acquisition (usually within 12 months).
d. under no circumstances

28.When an entity acquires a non-current asset (or disposal group) exclusively with a view
to its subsequent disposal, it shall classify the non-current asset (or disposal group) as
held for sale at the acquisition date only if the one-year requirement is met and it is
highly probable that the other criteria as provided in PFRS 5 will be met within
a. 12 months after balance sheet date c. 6 months from date of acquisition
b. 12 months from date of acquisition d. 3 months from date of acquisition

83
29.A non-current asset (or disposal group) that is acquired as part of a business
combination exclusively with a view to its subsequent disposal shall be measured at
initial recognition at
a. fair value c. lower of cost or fair value
b. cost d. fair value less costs to sell

30.An entity acquires a subsidiary exclusively with a view to selling it. The subsidiary
meets the criteria to be classified as held for sale. At the balance sheet date, the
subsidiary has not yet been sold, and six months have passed since its acquisition. How
will the subsidiary be valued in the balance sheet at the date of the first financial
statements after acquisition?
a. At fair value. c. At the lower of its cost and fair value less cost to sell
b. At carrying value. d. In accordance with applicable PFRS.
(Adapted)

31.SNAPPY Co. has acquired an investment in a subsidiary, QUICKLY, Inc., with the view to
dispose of this investment within six months. The investment in the subsidiary has been
classified as held for sale and is to be accounted for in accordance with PFRS 5. The
subsidiary has never been consolidated. How should the investment in the subsidiary
be treated in the financial statements?
a. Purchase accounting should be used.
b. Equity accounting should be used.
c. The subsidiary should not be consolidated but PFRS 5 should be used.
d. The subsidiary should remain off balance sheet.
(Adapted)

32.On December 31, 20x1, TANTAMOUNT EQUIVALENT Co. intends to sell its land.
However, TANTAMOUNT assessed that it will not be able to sell the asset within 12
months after the reporting period, so the land was still classified under investment
property. On January 15, 20x2, the land was actually sold. The financial statements
were authorized for issue on March 31, 20x2. Which of the following statements is
correct?
a. The land should be presented in the statement of financial position under
investment property with disclosures of the disposal.
b. The land should be separately presented in the statement of financial position as
“Non-current Asset Held For Sale.”
c. The land should be presented in the statement of financial position under
investment property with no further disclosures in the notes.
d. The land should be separately presented in the statement of financial position as
“Non-current Asset Held For Sale” and the financial statements should be adjusted
for any effects of the reclassification since this would qualify as an adjusting event
after reporting period.

33.If the criteria for classification as held for sale are met after the reporting period but
before the authorization of the financial statements for issue,
a. an entity shall classify a non-current asset (or disposal group) as held for sale in the
financial statements when issued; no further disclosures required
b. an entity shall classify a non-current asset (or disposal group) as held for sale in the
financial statements when issued; disclosure should be made in the notes.
c. an entity shall not classify a non-current asset (or disposal group) as held for sale in
the financial statements when issued; no further disclosures are required
d. an entity shall not classify a non-current asset (or disposal group) as held for sale in
the financial statements when issued; disclosure should be made in the notes.

34.LANGUID WEAK Co. is preparing its December 31, 20x1, current year, financial
statements. Included in the assets of LANGUID is an intangible asset that was sold on
February 1, 20x2 before the financial statements were authorized for issue.

84
I. If the intangible asset did not qualify as non-current asset “held for sale” as of
December 31, 20x1, no reclassification should be made.
II. Since the intangible asset, was actually sold after reporting period and before
authorization of financial statements for issue, the intangible asset should be
reclassified to” held for sale”
a. true, true b. true, false c. false, true d. false, false

35.Are the following statements about the requirements of PFRS 5 Non-current assets held
for sale and discontinued operations true or false?
I. An asset that meets the criteria for classification as held for sale after the end of the
reporting period but before the authorization of the financial statements should be
measured in the statement of financial position at the lower of carrying amount and
fair value less costs to sell.
II. To be classified as an asset held for sale, the sale must be expected to be completed
within 12 months from the end of the financial year.
a. true, true b. true, false c. false, true d. false, false
(ACCA)

36.An entity shall not classify as held for sale a non-current asset (or disposal group) that
is to be abandoned. This is because
a. the realizable value of such asset may be zero or negative
b. the asset’s carrying amount will be recovered principally through continuing use
c. the asset’s carrying amount may exceed the recoverable amount, in which case the
continued use may be detrimental to the company
d. the asset may be obsolete and the replacement cost may be so material not to
warrant replacement

37.According to PFRS 5, non-current assets (or disposal groups) to be abandoned include


I. non-current assets (or disposal groups) that are to be used to the end of their
economic life
II. non-current assets (or disposal groups) that are to be closed rather than sold
a. True, true b. True, false c. False, true d. False, false

38.Which of the following statements is incorrect?


a. An entity shall not account for a non-current asset that has been temporarily taken
out of use as if it had been abandoned.
b. An entity shall measure a non-current asset (or disposal group) classified as held for
sale at fair value less costs to sell.
c. When the sale is expected to occur beyond one year, the entity shall measure the
costs to sell at their present value. Any increase in the present value of the costs to
sell that arises from the passage of time shall be presented in profit or loss as a
financing cost.
d. An entity shall recognize an impairment loss for any initial or subsequent write-
down of the asset (or disposal group) to fair value less costs to sell.
e. An entity shall recognize a gain for any subsequent increase in fair value less costs
to sell of an asset, but not in excess of the cumulative impairment loss that has been
recognized either in accordance with PFRS 5 or previously in accordance with PAS
36 Impairment of Assets.

39.A non-current asset (or disposal group) classified as held-for-sale should be measured
at
a. acquisition cost plus necessary costs
b. at depreciated cost
c. at the lower of carrying amount and fair value
d. at the lower of carrying amount and fair value less cost to sell

85
40.The amount for which an asset could be exchanged or a liability settled, between
knowledgeable, willing parties in an arm’s length transaction.
a. fair value c. net realizable value
b. fair value less cost to sell d. replacement cost

41.According to PFRS 5, this refers to the incremental costs directly attributable to the
disposal of an asset (or disposal group), excluding finance costs and income tax
expense.
a. costs to sell b. relevant cost c. variable cost d. opportunity cost

42.When an entity expects that the sale of a non-current asset (or disposal group) extends
beyond one year and the exceptions to the one-year requirement are met,
a. the non-current asset (or disposal group) should be reclassified back to its original
classification without exceptions
b. the comparative financial statements should be restated to reflect a valid and
relevant comparative information
c. cost to sell should be discounted to its present value
d. the sale is not highly probable, classifying the non-current asset (or disposal group)
would adversely affect the entity’s current ratio and working capital

43.All of the following correctly relate to the provisions of PFRS 5 Non-current Assets Held
for Sale and Discontinued Operations, except
a. Costs to sell exclude finance costs and income tax expense.
b. When the sale is expected to occur beyond one year, the entity shall measure the
costs to sell at their present value. Any increase in the present value of the costs to
sell that arises from the passage of time shall be presented in profit or loss as a
financing cost.
c. An entity shall measure a non-current asset (or disposal group) classified as held for
distribution to owners at the lower of its carrying amount and fair value less costs to
distribute.
d. If the non-current asset (or disposal group) is acquired as part of a business
combination, it shall be measured at fair value less costs to sell, not at fair value as
required by PFRS 3 Business Combination.
e. Events after reporting period involving classification as held for sale are treated as
adjusting events after reporting period.

44.Any gain or loss on the remeasurement of a non-current asset (or disposal group)
classified as held for sale that does not meet the definition of a discontinued operation
shall be included
I. in profit or loss from continuing operations
II. in profit or loss from discontinued operations
III. adjustment to the beginning balance of retained earnings
a. I only b. Either II or III c. Either I or III d. Either I, II or III

45.Which of the following statements regarding presentation of remeasurement gains and


losses on non-current assets or disposal groups?
I. Changes in fair values less costs to sell of non-current assets (or disposal group)
held for sale not meeting the conditions of a discontinued operation and subject to
the limitations on reversal of impairment losses are recognized as gains or losses
and included in profit or loss from continuing operation.
II. Changes in fair values less costs to sell of disposal groups qualifying as discontinued
operations are recognized as gains or losses and included in profit or loss from
discontinued operation.
a. True, true b. True, false c. False, true d. False, false

46.Any gain on a subsequent increase in the fair value less cost to sell of a noncurrent asset
classified as held for sale should be treated as follows:

86
a. The gain should be recognized in full.
b. The gain should not be recognized.
c. The gain should be recognized but not in excess of the cumulative impairment loss.
d. The gain should be recognized but only in retained earnings.
(Adapted)

47.Which of the following statements is true?


a. Interest and other expenses attributable to the liabilities of a disposal group
classified as held for sale shall continue to be recognized.
b. If an entity has classified an asset (or disposal group) as held for sale, but the
criteria in for classification to held for sale are no longer met, the entity shall cease
to classify the asset (or disposal group) as held for sale. Such change is treated
retrospectively.
c. The entity shall include any required adjustment to the carrying amount of a non-
current asset that ceases to be classified as held for sale in the beginning balance of
the retained surplus and in the income from continuing operations in the period in
which the criteria for classification to held for sale are no longer met.
d. If an entity removes an individual asset or liability from a disposal group classified
as held for sale, the remaining assets and liabilities of the disposal group should also
cease to be classified as held for sale.

48.A non-current asset which has been classified as held-for-sale should


a. be valued at depreciated historical cost
b. be depreciated over its remaining useful life
c. not be depreciated
d. be tested monthly for impairment
(ACCA)

49.The entity shall measure a non-current asset that ceases to be classified as held for sale
(or ceases to be included in a disposal group classified as held for sale) at:
I. its carrying amount before the asset (or disposal group) was classified as held for sale,
adjusted for any depreciation, amortization or revaluations that would have been
recognized had the asset (or disposal group) not been classified as held for sale
II. its recoverable amount at the date of the subsequent decision not to sell.
a. I or II b. I and II c. Lower of I and II d. Neither I nor II

50.The DEBAUCH TO CORRUPT Company classified a non-current asset accounted for


under the cost model as held for sale on December 31, 20x1. Because no offers were
received at an acceptable price, DEBAUCH decided on July 20x2 not to sell the asset, but
to continue to use it. In accordance with PFRS5 Non-current assets held for sale and
discontinued operations, the asset should be measured on July 1, 20x2 at
a. the lower of its carrying amount and its recoverable amount
b. the higher of its carrying amount and its recoverable amount
c. the lower of its carrying amount on the basis that it had never been classified as held
for sale and its recoverable amount
d. the higher of its carrying amount on the basis that it had never been classified as
held for sale and its recoverable amount
(Adapted)

51.The adjustment in the carrying amount of the non-current asset ceasing to be classified
as held for sale
a. shall be in profit or loss from continuing operation in the period in which the
criteria are no longer met
b. shall be in profit or loss from discontinued operation in the period in which the
criteria are no longer met
c. shall be charged to the beginning balance of accumulated profits and losses

87
d. shall be made in the next accounting period subsequent to the change of
classification

52.An entity has a non-current asset that has been classified as held-for-sale but the asset
no longer meets the criteria to be held for sale. The entity should, therefore
a. leave the non-current asset in the financial statements at its current carrying value
b. measure the non-current asset at the lower of its carrying amount before its asset
was classified as held-for-sale (as adjusted for subsequent depreciation,
amortization, or revaluations) and its recoverable amount at the date of the decision
not to sell
c. re-measure in accordance with applicable PFRS
d. re-measure the non-current asset at fair value
(ACCA)

53.The higher of an asset’s fair value less costs to sell and its value in use.
a. the measurement for non-current assets held for sale
b. the measurement for impairment losses of assets and liabilities
c. the measurement for impairment losses of assets but not liabilities
d. recoverable amount

54.The present value of estimated future cash flows expected to arise from the continuing
use of an asset and from its disposal at the end of its useful life.
a. discounted financial position c. value in use
b. discounted future cash flows d. fair value

55.If an entity removes an individual asset or liability from a disposal group classified as
held for sale,
a. the remaining assets and liabilities of the disposal group to be sold shall continue to
be measured as a group only if the group meets the criteria for classification as
disposal group.
b. the remaining assets and liabilities should not be presented as held for sale
c. the remaining assets and liabilities of the disposal group should be reclassified back
to their previous classification; disclosure should be made
d. the remaining assets and liabilities of the disposal group should be reclassified back
to their previous classification since the disposal group cannot be sold as a group
anymore; previous financial statements should be restated

56.If after removal an individual asset or liability from a disposal group classified as held
for sale, the remaining assets do not qualify to be classified as disposal group, (choose
the incorrect statement)
a. the remaining non-current assets of the group that individually meet the criteria to
be classified as held for sale shall still be classified as held for sale
b. the remaining non-current assets of the group that individually meet the criteria to
be classified as held for sale shall be measured individually at the lower of their
carrying amounts and fair values less costs to sell at that date.
c. the remaining non-current assets of the group that individually meet the criteria to
be classified as held for sale should be reclassified back to their previous
classification since the disposal group failed to be sold as a group.
d. Any non-current assets that do not meet the criteria shall cease to be classified as
held for sale.

57.It is a component of an entity that either has been disposed of, or is classified as held for
sale, and
i. represents a separate major line of business or geographical area
of operations,
ii. is part of a single coordinated plan to dispose of a separate major
line of business or geographical area of operations or

88
iii. is a subsidiary acquired exclusively with a view to resale.
a. Discontinued operation c. Disposal group
b. Non-current assets held for sale d. Non-going concern entity

58.According to PFRS 5, this comprises operations and cash flows that can be clearly
distinguished, operationally and for financial reporting purposes, from the rest of the
entity.
a. Responsibility center c. Division within an entity
b. Component of an entity d. Cash generating unit

59.This refers to the smallest identifiable group of assets that generates cash inflows that
are largely independent of the cash inflows from other assets or groups of assets.
a. component of an entity c. division of an entity
b. cash-generating unit d. responsibility center

60.A component unit may be (choose the incorrect)


a. a reportable segment or an operating segment
b. a reporting unit
c. a subsidiary
d. an asset group
e. a non-current asset used in conjunction with other assets to produce revenue

61.It is a distinguishable component of an enterprise that is engaged in providing an


individual product or service or a group of related products or service and that is
subject to risks and returns that are different from those of other segments.
a. Geographical segment c. Reportable segment
b. Operating (Business) segment d. None
(Adapted)

62.Discontinued operations occur at the


I. date the component is actually disposed of
II. date the criteria for classification as held for sale are met by the component
a. I only b. II only c. earlier of I or II d. later of I or II

63.When a discontinued operation occur


I. a company eliminates (or will eliminate) the results of operations and cash flows of
a component of an entity from its ongoing operations, and
II. there is no significant continuing involvement in that component after the disposal
transaction.
a. I only b. II only c. I and II d. none

64.A component unit that is considered a discontinued operation is


a. An extraordinary item.
b. A separate line of business or a class of customer.
c. An unrelated group of assets held for sale.
d. A set of operations and cash flows clearly distinguishable from the rest of the entity
for operational and financial reporting purposes.
(AICPA)

65.RAUNCHY DIRTY Fast Foods has subsidiaries in North Carolina, Georgia and Maine. In
20X5, RAUNCHY committed to a plan to sell the subsidiaries in Georgia and North
Carolina. The subsidiaries are classified as held for sale. RAUNCHY will not have any
post-disposal involvement with the Georgia subsidiary and their operations and cash
flows will be eliminated from RAUNCHY’s consolidated statements. The North Carolina
subsidiary will continue to be significantly involved with RAUNCHY after the sale is
completed. Which subsidiaries would RAUNCHY report as discontinued operations?
(Item #1) Georgia; (Item #2) North Carolina

89
a. Yes, Yes b. Yes, No c. No, Yes d. No, No
(AICPA)

66.Which of the following criteria is not required for a component’s results to be classified
as discontinued operations?
a. Management must have entered into a sales agreement.
b. The component is available for immediate sale.
c. The operations and cash flows of the component will be eliminated from the
operations of the entity as a result of the disposal.
d. The entity will not have any significant continuing involvement in the operations of
the component after disposal.
(AICPA)

67.Which of the following criteria do not have to be met in order for an operation to be
classified as discontinued?
a. The operation should represent a separate line of business or geographical area.
b. The operation is part of a single plan to dispose of a separate major line of business
or geographical area.
c. The operation is a subsidiary acquired exclusively with a view to resale.
d. The operation must be sold within three months of the year-end.
(Adapted)

68.A disposal group that is a component of an entity is presented as discontinued


operation when the disposal group (choose the incorrect)
a. is classified as held for sale or actually disposed of and represents a major line of
business or geographical area of operations
b. is part of a single coordinated plan to dispose of a separate major line of business or
geographical area of operations
c. a subsidiary acquired exclusively with a view to resale
d. is sold after the balance sheet date but before the financial statements are
authorized for issue

69.A non-current asset or disposal group held for sale or actually disposed of that is not a
component of an entity
a. shall be presented as continuing operation
b. shall be presented as discontinued operation
c. shall not be presented in the financial statements separately
d. shall be disclosed only

70.How should profit or loss from discontinued operations be shown in the statement of
profit or loss and other comprehensive income?
a. The amounts from discontinued operations should be broken down over each
category of revenue and expense.
b. Discontinued operations should be shown as a line item after gross profit with the
taxation being shown as part of income tax expense
c. Discontinued operations should be shown as a movement in the statement of
changes in equity only
d. The entity should disclose a single amount on the face of the income statement with
analysis in the notes or a section of the income statement separate from continuing
operations
(ACCA)

71.Profit or loss for the period from discontinued operations does not include
a. the post-tax profit or loss of discontinued operations
b. the post-tax expected gain on future disposal of the operation
c. the post-tax gain or loss recognized on the measurement to fair value less costs to
sell

90
d. the post-tax gain or loss recognized on the disposal of the assets or disposal
group(s) constituting the discontinued operation

72.Profit or loss for the period from discontinued operations


a. does not include severance pay or employee termination costs
b. does not include expected losses on eventual disposal of the component
c. does not include employee relocation expenses and future rentals on long-term
leases
d. does not include expected gains on eventual disposal of the component

73.The single amount of loss for the period from discontinued operations
a. is presented after profit or loss from continuing operations
b. is presented after gross income but before other income
c. is presented after profit before taxes
d. is presented only in the notes

74.Presenting profit or loss from discontinued operations at net of tax is an application of


a. interperiod tax allocation c. substance over form
b. intraperiod tax allocation d. materiality

75.If disposal occurs in the same period that the component meets the criteria to be
classified as held for sale
a. PFRS 5 Non-current Assets Held for Sale and Discontinued Operations prohibits re-
presenting the disclosures for prior periods presented in the financial statements so
that the disclosures relate to all operations that have been discontinued by the
reporting period for the latest period presented
b. The gain or loss on disposal of discontinued operations is the actual gain or loss
c. PFRS 5 does not require classification as discontinued operation in the current
financial statements
d. a single amount of pre-tax profit or loss from discontinued operations should be
presented after profit for the year from continuing operations

76.If the criteria to classify the component as “held for sale” is met in a period before it is
disposed of,
a. the amount of the loss (if applicable) on disposal is an estimated loss resulting from
the write-down of the group of assets to their estimated fair value less costs to sell
b. PFRS 5 does not require the one-year requirement anymore for as long as the
component is available for immediate sale in its present condition
c. The gain or loss on disposal of discontinued operations is the actual gain or loss
d. an impairment loss should be recognized
77.Costs or adjustments directly associated with the decision to dispose of a component of
an entity
a. should be recognized and shown as part of discontinued operations only if they
relate to impairment losses
b. should not be recognized and shown as part of discontinued operations
c. should be recognized and shown as part of discontinued operations
d. should be recognized and shown as part of continuing operations

78.Which of the following costs cannot be recognized and shown as part of discontinued
operations?
a. severance pay or employee termination costs
b. employee relocation expenses
c. future rentals on long-term leases
d. future operating losses

79.A component of HUBRIS ARROGANCE, Inc. was discontinued during 20x1. HUBRIS’ loss
on disposal should

91
a. Exclude the associated employee relocation costs.
b. Exclude operating losses for the period.
c. Include associated employee termination costs.
d. Exclude associated lease cancellation costs.
(AICPA)

80.If a component of an entity qualified as a discontinued operation during the year


a. all of its results of operations, before and after date of classification as discontinued
operation, will be classified as discontinued operations
b. results of operations of the component prior to classification as discontinued
operation will be shown under continuing operation, only results of operations after
classification as discontinued operation will be presented under discontinued
operations
c. PFRS 5 Non-current Assets Held for Sale and Discontinued Operations prohibits a
component to be classified as discontinued operations during the year. Such
classification should only be made at the beginning of the year or at the end of the
financial reporting period.
d. the component is deemed sold only when a purchase commitment has been
obtained from a financially capable buyer

81.When a component of a business has been discontinued during the year, the loss on
discontinued operations should
a. Include operating losses of the current period.
b. Exclude operating losses during the period.
c. Be an extraordinary item.
d. Be an operating item.
(AICPA)

82.If, in the current year, a disposal group (component of an entity) is classified as


discontinued operation,
a. an entity shall re-present the disclosures for prior periods presented in the financial
statements so that the disclosures relate to all operations that have been
discontinued by the reporting period for the latest period presented.
b. an entity shall not re-present the disclosures for prior periods presented in the
financial statements so that the disclosures relate to all operations that have been
discontinued by the reporting period for the latest period presented.
c. only the current year financial statements should disclose the component as
discontinued operation; comparative financial statements should disclose the
component as continuing operation
d. presenting profit or loss from discontinued operation at post-tax is prohibited

83.If the criteria for classification as discontinued operation are met after the reporting
period but prior to authorization of financial statements for issue, an entity
a. shall classify the component of an entity as discontinued operation in those financial
statements when issued.
b. shall disclose only the information in the notes
c. shall classify the component of an entity as discontinued operation in those financial
statements when issued and shall make the necessary disclosures in the notes
d. shall do nothing

84.Which of the following statements correctly relates to the provisions of PFRS 5?


I. Once a component unit qualifies as held for sale, the assets are required to be tested
for an impairment loss.
II. The impairment loss on the remeasurement of a component is netted with the
operational results to calculate the amount shown under discontinued operations.
III. PFRS 5 prohibits the retrospective classification of as a discontinued operation
when the discontinued operation criteria are met after the reporting period.

92
IV. If a component of an entity qualifies as discontinued operation during the year, the
results of its operations for the months prior to date of qualification is shown as part
of continuing operations.
a. II, III b. I, II, III c. II, III, IV d. I, II, III, IV

85.Comprehensive income includes which of the following?


(Item #1) Loss on Discontinued Operations; (Item #2) Investment by Owners
a. Yes, Yes b. Yes, No c. No, Yes d. No, No
(AICPA)

86.One of the elements of a financial statement is comprehensive income. Comprehensive


income excludes changes in equity resulting from which of the following?
a. Loss from discontinued operations.
b. Prior period error correction.
c. Dividends paid to stockholders.
d. Unrealized loss on investments in noncurrent marketable equity securities.
(AICPA)

87.When a component of a business has been discontinued during the year, this
component’s operating losses of the current period should be included in the
a. Income statement as part of revenues and expenses.
b. Income statement as part of the loss on disposal of the discontinued component.
c. Income statement as part of the income (loss) from continuing operations.
d. Retained earnings statement as a direct decrease in retained earnings.
(AICPA)

88.On January 1, 20x1, ABJURE TO RENOUNCE Co. agreed to sell a business component on
March 1, 20x1. The gain on the disposal should be
a. Presented as an extraordinary gain.
b. Presented as an adjustment to retained earnings.
c. Netted with the loss from operations of the component as a part of discontinued
operations.
d. None of the above.
(AICPA)

89.Adjustments in the current period to amounts previously presented in discontinued


operations that are directly related to the disposal of a discontinued operation in a prior
period shall be classified
a. separately in discontinued operations
b. in income from continuing operations
c. in income from continuing operations with additional disclosure in the notes
d. as an adjustment of the beginning balance of the retained earnings account

90.If an entity ceases to classify a component of an entity as held for sale, the results of
operations of the component previously presented in discontinued operations in
accordance with PFRS 5
a. shall be reclassified and included in income from continuing operations for all
periods presented
b. shall be reclassified and included in income from continuing operations for results
of operations of the current year and included as an adjustment to the beginning
balance of retained earnings the comparative financial statements
c. shall be reclassified and included as an adjustment to the beginning balance of
retained earnings for all periods presented
d. shall not be reclassified and included in income from discontinued operations for
comparative financial statements and included in income from continuing
operations for current period

93
91.If non-current assets (disposal groups) held for sale ceases to be classified as held for
sale,
a. the previous presentation and disclosures are retained for the previous financial
statements
b. the previous financial statements are restated to provide disclosures which are
comparable and relevant
c. adjustment to the carrying amount of the non-current asset (or disposal group)
shall be made in retained earnings
d. adjustment to the carrying amount the non-current asset (or disposal group) shall
be made in discontinued operations

92.How should the assets and liabilities of a disposal group classified as held-for-sale be
shown in the balance sheet?
a. The assets and liabilities should be presented as a single amount and as a deduction
from equity
b. There should be no separate disclosure of the assets and liabilities
c. The major classes of assets and liabilities classified as held for sale shall be
separately disclosed either in the statement of financial position or in the notes,
except when the disposal group is a newly acquired subsidiary that meets the
criteria to be classified as held for sale on acquisition.
d. The assets and liabilities should be netted off and presented as a single amount in
current assets or liabilities
(ACCA)

93.Which of the following statements is incorrect?


a. An entity shall present a non-current asset classified as held for sale and the assets
of a disposal group classified as held for sale separately from other assets in the
balance sheet.
b. Those assets and liabilities (of a disposal group) shall be offset and presented as a
single amount.
c. The major classes of assets and liabilities classified as held for sale shall be
separately disclosed either on the face of the statement of financial position or in the
notes, subject to exceptions as provided in PFRS 5.
d. An entity shall present separately any cumulative income or expense recognized
directly in equity relating to a non-current asset (or disposal group) classified as
held for sale.
e. If the disposal group is a newly acquired subsidiary that meets the criteria to be
classified as held for sale on acquisition, disclosure of the major classes of assets and
liabilities is not required

94.Non-current assets held for sale and assets and liabilities of disposal groups shall be
presented in the classified statement of financial position as part of
a. current items c. neither current nor non-current
b. non-current items d. equity

95.PFRS 5 prohibits assets that will be abandoned from being classified as held for sale.
However, if the assets to be abandoned are a major line of business or geographical area
of operations, they are reported in discontinued operations
a. at the date at which they are abandoned.
b. at the date at which they are sold
c. not reported in discontinued operations
d. earlier of a or b

96.An entity has bought a 25% share in another entity with a view to selling that
investment within six months. The investment has been classified as held for sale in
accordance with PFRS 5. How should the investment be treated in the final year
accounts?

94
a. The investment shall be accounted for under PAS 28.
b. The investment shall be accounted for under PFRS 5.
c. The investment should be dealt with under PAS 29.
d. Purchase accounting should be used for this investment.
(Adapted)

97.PAS 28 does not require the equity method to be applied when the associate has been
acquired and held with a view to its disposal within a certain time period. What is the
period within which the associate must be disposed of?
a. Six months. b. Twelve months. c. Two years. d. In the near future.
(Adapted)

98.When an investment in an associate previously classified as held for sale no longer


meets the criteria to be so classified,
a. it shall be accounted for using the equity method as from the date of its classification
as held for sale.
b. it shall be accounted for at fair value in the group statements
c. it shall be accounted for at cost less impairment
d. it shall be accounted for at amortized cost

99.The financial results from a discontinued operation should be reflected on a company’s


financial statements by:
a. discussing the effects in a footnote.
b. adjusting the company’s reported costs.
c. adjusting the company’s reported net revenue.
d. reporting the effects separately on the income statement.
(Adapted)

100. When a business segment is discontinued during the year, the gain or loss on
disposal
a. is reported as an extraordinary item.
b. should include operating losses of the current period up to the measurement date
only.
c. excludes operating losses during the phase-out period.
d. should be shown net of applicable income taxes.
(Adapted)

101. The objective of PFRS 5 is to specify the accounting for assets held for sale, and the
presentation and disclosure of discontinued operations. In particular, PFRS 5 requires
all of the following, except
a. assets that meet the criteria to be classified as held for sale are to be measured at
fair value less costs to sell
b. depreciation on assets that meet the criteria to be classified as held for sale shall
cease
c. assets that meet the criteria to be classified as held for sale are to be presented
separately on the face of the balance sheet
d. the results of discontinued operations are to be presented separately in the income
statement.

102. Which of the following statements correctly relate to the classification of a non-
current asset (or disposal group) to held for sale?
a. An entity shall classify a non-current asset (or disposal group) as held for sale if its
carrying amount will be recovered principally through continuing use rather than
through a sale.
b. For an asset to be classified as a non-current asset (or disposal group) as held for
sale, the asset (or disposal group) must be available for immediate sale in its present

95
condition subject only to terms that are usual and customary for sales of such assets
(or disposal groups) and its sale must be probable.
c. The appropriate level of management must be committed to a plan to sell the asset
(or disposal group), and an active program to locate a buyer and complete the plan
will be initiated at least 12 months from the date of reclassification.
d. The asset (or disposal group) must be actively marketed for sale at a price that
reflects a reasonable profit.
e. The sale should be expected to qualify for recognition as a completed sale within
one year from the date of classification.
f. Sale transactions do not include exchanges of non-current assets for other non-
current assets even when the exchange has commercial substance in accordance
with PAS 16 Property, Plant and Equipment.
g. all of these

103. On December 15, 20x1, WHIM FANCY Corp. acquired a non-current asset which it
intends to sell. On December 31, 20x1, WHIM Corp. assessed that it will not be able to
sell the asset within 12 months after the balance sheet date, so the non-current asset
was not reclassified to held for sale. However, on January 31, 20x2, all the requirements
for reclassification to held for sale was met. The financial statements were authorized
for issue on March 31, 20x2. Which of the following statements is correct?
a. The non-current asset should be separately presented in the balance as Non-current
Asset Held For Sale.
b. The non-current asset should be presented in the balance sheet under non-current
assets with disclosures of the facts and circumstances leading to the expected
disposal and the expected manner and timing of the expected disposal.
c. The non-current asset should be presented in the balance sheet under non-current
assets with no further disclosures in the notes.
d. The non-current asset should be separately presented in the balance as Non-current
Asset Held For Sale and the financial statements should be adjusted for any effects
of the reclassification since this would qualify as an adjusting event after balance
sheet date.

104. Which of the following statements is true?


a. An entity shall not depreciate a non-current asset while it is classified as held for
sale or while it is part of a disposal group classified as held for sale. However,
intangible assets classified as held for sale should still be amortized.
b. Interest and other expenses attributable to the liabilities of a disposal group
classified as held for sale shall continue to be recognized.
c. If an entity has classified an asset (or disposal group) as held for sale, but the
criteria in for classification to held for sale are no longer met, the entity shall cease
to classify the asset (or disposal group) as held for sale. Such change is treated
retrospectively.
d. The entity shall include any required adjustment to the carrying amount of a non-
current asset that ceases to be classified as held for sale in the beginning balance of
the retained surplus and in the income from continuing operations in the period in
which the criteria for classification to held for sale are no longer met.
e. If an entity removes an individual asset or liability from a disposal group classified
as held for sale, the remaining assets and liabilities of the disposal group should also
cease to be classified as held for sale.

105. In its 20x1 year-end balance sheet, CULMINATE TO CLIMAX Company properly
classified one of its non-current assets as held for sale at P100,000. During 20x2, the
held for sale asset was not sold and the other criteria for classification as “held for sale”
were not met, hence, CULMINATE’s management decided to reclassify the held for sale
asset to Investment Property. The appropriate valuation of the asset as of December 31,
20x2 was determined to be ₱80,000. The cost of the asset before reclassification to Non-

96
current asset held for sale was ₱120,000. In its 20x2 comparative balance sheets what
would be the correct classification of such asset?
20x2 20x1
a. Investment Property ₱ 80,000 ₱ 100,000
b. Non-current asset held for sale ₱ 80,000 ₱ 100,000
c. Investment Property ₱ 80,000
Non-current asset held for sale ₱ 100,000
d. Investment Property ₱ 80,000
e. Non-current asset held for sale ₱ 120,000
Investment Property ₱ 80,000 ₱ 120,000

106. An asset that does not meet the definition of a current asset.
a. Not recognized but disclosed c. Non-current asset
b. Recognized and disclosed d. Current liability

107. On March 1, 20x0, LUCENT CLEAR Co. issued 20-year bonds at a discount. By
September 1, 20x5, the bonds were quoted at 106 when LUCENT exercised its right to
retire the bonds at 105. How should LUCENT report the bond retirement on its 20x5
income statement?
a. A gain in continuing operations. c. An extraordinary gain.
b. A loss in continuing operations. d. An extraordinary loss.

108. Are the following statements relating to a discontinued operation true or false,
according to PFRS 5 Non-current assets held for sale and discontinued operations?
I. When the discontinued criteria are met after the end of the reporting period, the
operation shall retrospectively be separately presented as a discontinued operation.
II. The net cash flows attributable to the operating, investing, and financing activities of
a discontinued operation shall be separately presented.
a. False, False b. False, True c. True, False d. True, True
(ACCA)

109. Which of the following is a requirement for a component of an entity to be classified


as a discontinued operation in accordance with PFRS 5 Noncurrent assets held for sale
and discontinued operations?
a. Its activities must cease permanently prior to the financial statements being
authorized for issue by management
b. It must comprise a separately reported segment in accordance with PFRS 8
Operating Segments
c. Its assets must have been classified as held for sale in the previous financial
statements
d. It must have been a cash-generating unit or a group of cash generating units while
being held for use
(ACCA)

110. PLACABLE TOLERANT Company has correctly classified its manufacturing


operations as a disposal group held for sale and as discontinued operations during the
year ended December 31, 20x2. Are the following statements true or false, according to
PFRS 5 Non-current Assets Held for Sale and Discontinued Operations?
I. The disposal group's results for the year ended 31 December 20x1 should be re-
presented as relating to discontinued operations in the comparative figures for the
20x2 statement of profit or loss and other comprehensive income.
II. The disposal group's assets at December 31, 20x1 should be represented as held for
sale in the comparative figures for the 20x2 statement of financial position.
a. False, False b. False, True c. True, False d. True, True
(ACCA)

97
111. FACTITIOUS ARTIFICIAL Company has correctly classified its packaging operations
as a disposal group held for sale and as discontinued operations. In the year ended
December 31, 20x1 this disposal group incurred trading losses after tax of ₱20 million
and the loss on remeasuring it to fair value less costs to sell was ₱15 million. How
should the disposal group's losses for the year ended December 31, 20x1 be presented,
according to PFRS5 Non-current Assets Held for Sale and Discontinued Operations?
(Item #1) In profit or loss (Item #2) In other comprehensive income
a. ₱35 million loss; Nil c. Nil; ₱35 million loss
b. ₱20 million loss; ₱15 million loss d. ₱15 million loss; ₱20 million loss
(ACCA)

112. How should the income from discontinued operations be presented in the income
statement?
a. The entity should disclose a single amount on the face of the income statement with
analysis in the notes or a section of the income statement separate from continuing
operations.
b. The amounts from discontinued operations should be broken down over each
category of revenue and expense.
c. Discontinued operations should be shown as a movement on retained earnings.
d. Discontinued operations should be shown as a line item after gross profit with the
taxation being shown as part of income tax expense.
(Adapted)

113. How should the assets and liabilities of a disposal group classified as held for sale
be shown in the balance sheet?
a. The assets and liabilities should be offset and presented as a single amount.
b. The assets of the disposal group should be shown separately from other assets in
the balance sheet, and the liabilities of the disposal group should be shown
separately from other liabilities in the balance sheet.
c. The assets and liabilities should be presented as a single amount and as a deduction
from equity.
d. There should be no separate disclosure of assets and liabilities that form part of a
disposal group.
(Adapted)

114. An entity is planning to dispose of a collection of assets. The entity designates these
assets as a disposal group. The carrying amount of these assets immediately before
classification as held for sale was ₱20 million. Upon being classified as held for sale, the
assets were revalued to ₱18 million. The entity feels that it would cost ₱1 million to sell
the disposal group. What would be the carrying amount of the disposal group in the
entity’s accounts after its classification as held for sale?
a. ₱20 million. b. ₱18 million. c. ₱17 million. d. ₱19 million.
(Adapted)

115. An entity is planning to dispose of a collection of assets. The entity designates these
assets as a disposal group, and the carrying amount of these assets immediately before
classification as held for sale was ₱20 million. Upon being classified as held for sale, the
assets were revalued to ₱18 million. The entity feels that the fair value less cost to sell
would be ₱17 million. How would the reduction in the value of the assets on
classification as held for sale be treated in the financial statements?
a. The entity recognizes a loss of ₱2 million immediately before classification as held
for sale and then recognizes an impairment loss of ₱1 million.
b. The entity recognizes an impairment loss of ₱3 million.
c. The entity recognizes an impairment loss of ₱2 million.
d. The entity recognizes a loss of ₱3 million immediately before classifying the disposal
group as held for sale.
(Adapted)

98
116. In order for a noncurrent asset to be classified as held for sale, the sale must be
highly probable. “Highly probable” means that
a. The future sale is likely to occur.
b. The future sale is more likely than not to occur.
c. The probability is higher than more likely than not.
d. The sale is certain.
(Adapted)

117. An entity has an asset that was classified as held for sale. However, the criteria for it
to remain as held for sale no longer apply. The entity should therefore
a. Leave the noncurrent asset in the financial statements at its current carrying value.
b. Remeasure the noncurrent asset at fair value.
c. Measure the noncurrent asset at the lower of its carrying amount before the asset
was classified as held for sale (as adjusted for subsequent depreciation,
amortization, or revaluations) and its recoverable amount at the date of the decision
not to sell.
d. Recognize the noncurrent asset at its carrying amount prior to its classification as
held for sale as adjusted for subsequent depreciation, amortization, or revaluations.
(Adapted)

118. PFRS 5 states that a noncurrent asset that is to be abandoned should not be
classified as held for sale. The reason for this is because
a. Its carrying amount will be recovered principally through continuing use.
b. It is difficult to value.
c. It is unlikely that the noncurrent asset will be sold within 12 months.
d. It is unlikely that there will be an active market for the noncurrent asset.
(Adapted)

119. An entity has publicly announced a detailed, formal plan to dispose in its entirety of
a component of the entity that represents a separate major line of business that is
distinct operationally and financially. Which of the following is the proper treatment of
the disclosures that should be made after the announcement?
a. As part of continuing operations. c. As an extraordinary item.
b. As a discontinued operation. d. As a prior-period item.
(Adapted)

120. If a loss is expected from the proposed sale or abandonment of a segment of


business, the estimated loss shall be provided
a. at disposal date
b. at balance sheet date
c. date the criteria for classification as held for sale are met
d. earlier of (a) and (c)

121. Which of the following asset disposals would qualify as discontinued operations?
a. Sale of assets due to technological improvements
b. Sale by a manufacturer of children’s wear of all assets in Hongkong due to
discontinuance of all operations in that city. The Hongkong’s operations are
subsumed in the Singapore’s operations both operationally and for financial
reporting purposes.
c. Sale by a meat packing entity its furniture business. After the sale, all activities
relating to furniture will be eliminated.
d. Disposal of assets due to the phasing out of a product line. The product line will be
continued after the assets disposal but with limited production.
(RPCPA)

122. The order in which sections of the income statement appear is

99
a. continuing operations, effect of accounting changes, discontinued. operations, EPS
b. continuing operations, discontinued operations, effect of accounting changes, EPS
c. continuing operations, discontinued operations, profit for the year, other
comprehensive income, total comprehensive income, EPS
d. continuing operations, discontinued operations, profit for the year, EPS
(RPCPA)

123. When a segment of a business has been discontinued during the year, this segment's
operating income of the current period up to the date of classification as discontinued
operation should be included in the
a. retained earnings statement as a direct increase in retained earnings.
b. income statement as part of the income (loss) from continuing operations.
c. income statement as part of the gain (loss) on disposal of the discontinued segment.
d. income statement as part of the income (loss) from operations of the discontinued
segment.
(AICPA)

124. Which of the following statements is (are) correct?


I. In the income statement, presentation and disclosure of income taxes applicable to
discontinued operations need not be disclosed on the face of the income statement.
Disclosure may be made in the notes to the financial statement.
II. The disposal of two or more unrelated assets that individually do not constitute a
component of an entity shall not be combined and accounted for as disposal of a
segment of a business.
a. I only b. II only c. I and II d. Neither I nor II

Chapter 39 - Suggested answers to theory of accounts questions


1. C 21. C 41. A 61. B 81. A 101. A 121. C
2. A 22. D 42. C 62. C 82. A 102. E 122. D
3. D 23. B 43. E 63. C 83. B 103. B 123. D
4. C 24. C 44. A 64. D 84. B 104. B 124. C
5. D 25. D 45. A 65. B 85. B 105. C
6. B 26. D 46. C 66. A 86. C 106. C
7. B 27. A 47. A 67. D 87. B 107. B
8. B 28. D 48. C 68. D 88. C 108. B
9. B 29. D 49. C 69. A 89. A 109. D
10. A 30. C 50. C 70. D 90. A 110. C
11. C 31. C 51. A 71. B 91. A 111. A
12. A 32. A 52. B 72. D 92. C 112. A
13. D 33. D 53. D 73. A 93. B 113. B
14. C 34. B 54. C 74. B 94. A 114. C
15. D 35. D 55. A 75. B 95. A 115. A
16. A 36. B 56. C 76. A 96. B 116. C
17. C 37. A 57. A 77. C 97. B 117. C
18. B 38. B 58. B 78. D 98. A 118. A
19. B 39. D 59. B 79. C 99. D 119. B
20. A 40. A 60. E 80. A 100. D 120. D

100
Chapter 40
Accounting Policies, Changes in Estimates and Errors

Chapter 40: Multiple choice – Computational (SET B) – (For classroom instruction


purposes)
Change of cost formulas
1. During 20x1, ALBEIT ALTHOUGH Company decided to change from the Average cost
formula for inventory valuation to the FIFO cost formula. Inventory balances under
each method were as follows:
Average FIFO
January 1 4,000,000 4,800,000
December 31 8,000,000 8,400,000

Income tax rate is 30%. What is the net cumulative effect of the accounting change in
ALBEIT’s opening retained earnings balance?
a. 400,000 increase c. 280,000 increase
b. 560,000 decrease d. 560,000 increase

Change in depreciation method, EUL, and salvage value


2. On January 1, 20x1, PRISTINE UNCORRUPTED Co. acquired an equipment for
₱4,000,000. The equipment will be depreciated using the straight-line method over 20
years. The estimated residual value is ₱400,000.

In 20x6, following a reassessment of the realization of the expected economic benefits from
the equipment, PRISTINE Co. changed its depreciation method to sum-of-the-years digits
(SYD). The remaining useful life of the asset is estimated to be 4 years and the residual
value is changed to ₱200,000. How much is the depreciation expense in 20x6?
a. 1,160,000 b. 1,140,000 c. 1,233,560 d. 1,110,669

Change in provisions
3. On December 31, 20x1, LIBERATED FREE Company recognized the following
provisions:
Dec. 31, Warranty expense 200,000
20x1 Estimated warranty obligation 200,000
Dec. 31, Probable loss on litigation 400,000
20x1 Estimated liability on pending 400,000
litigation

In 20x2, LIBERATED incurred ₱260,000 in discharging its warranty obligation and settled
the pending lawsuit for ₱360,000. How much is the net effect of the settlements of the
provisions on profit or loss in 20x2?
a. 20,000 increase c. 60,000 decrease
b. 20,000 decrease d. 0

Current period error


Use the following information for the next two questions:
On January 10, 20x2, prior to the authorization of LIBERTINE IMMORAL Co.’s December 31,
20x1 financial statements for issue, the accountant of LIBERTINE Co. received a bill for an
advertisement made in the month of December 20x1 amounting to ₱1,600,000. This
expense was not accrued as of December 31, 20x1.

4. The correcting entry, if the books are still open, includes


a. a debit to advertising expense for ₱1,600,000
b. a credit to advertising income for ₱1,600,000

101
c. a debit to retained earnings for ₱1,600,000
d. a credit to retained earnings for ₱1,600,000

5. The correcting entry, if the books are already closed, includes


a. a debit to advertising expense for ₱1,600,000
b. a credit to advertising income for ₱1,600,000
c. a debit to retained earnings for ₱1,600,000
d. a credit to retained earnings for ₱1,600,000

Prior period error


6. On January 15, 20x3 while finalizing its 20x2 financial statements, DIAPHANOUS
TRANSPARENT Co. discovered that depreciation expense recognized in 20x1 is
overstated by ₱1,600,000. Ignoring income tax, the entry to correct the prior period
error includes
a. a debit to depreciation expense for ₱1,600,000
b. a debit to retained earnings for ₱1,600,000
c. a credit to depreciation expense for ₱1,600,000
d. a debit to accumulated depreciation for ₱1,600,000

Counterbalancing and non-counterbalancing errors


Use the following information for the next four questions:
GULOSITY GREEDINESS Co. reported profits of ₱4,000,000 and ₱8,000,000 in 20x1 and
20x2, respectively. In 20x3, the following prior period errors were discovered:
 The inventory on December 31, 20x1 was understated by ₱200,000.
 An equipment with an acquisition cost of ₱1,200,000 was erroneously charged as
expense in 20x1. The equipment has an estimated useful life of 5 years with no residual
value. GULOSITY Co. provides full year depreciation in the year of acquisition.

The unadjusted balances of retained earnings are ₱8,800,000 and ₱16,800,000 as of


December 31, 20x1 and 20x2, respectively.

7. How much is the correct profit in 20x1?


a. 7,560,000 b. 5,610,000 c. 4,760,000 d. 5,160,000

8. How much is the correct profit in 20x2?


a. 7,560,000 b. 5,160,000 c. 5,720,000 d. 5,610,000

9. How much is the correct retained earnings in 20x1?


a. 9,960,000 b. 17,520,000 c. 9,860,000 d. 18,420,000

10. How much is the correct retained earnings in 20x2?


a. 9,960,000 b. 17,520,000 c. 9,860,000 d. 18,420,000

Counterbalancing and non-counterbalancing errors


Use the following information for the next four questions:
HELICAL SPIRAL Co. reported profits of ₱1,600,000 and ₱2,400,000 in 20x1 and 20x2,
respectively. In 20x3, the following prior period errors were discovered:
 Prepaid supplies in 20x1 were overstated by ₱80,000.
 Accrued salaries payable in 20x1 were understated by ₱160,000.
 Repairs and maintenance expenses in 20x1 amounting to ₱400,000 were erroneously
capitalized and being depreciated over a period of 4 years.

The unadjusted balances of retained earnings are ₱6,400,000 and ₱8,800,000 as of


December 31, 20x1 and 20x2, respectively.

11. How much is the correct profit in 20x1?


a. 1,006,000 b. 1,610,000 c. 1,720,000 d. 1,060,000

102
12. How much is the correct profit in 20x2?
a. 2,704,000 b. 2,160,000 c. 2,740,000 d. 2,610,000

13. How much is the correct retained earnings in 20x1?


a. 5,806,000 b. 5,520,000 c. 5,860,000 d. 5,420,000

14. How much is the correct retained earnings in 20x2?


a. 8,960,000 b. 8,600,000 c. 8,860,000 d. 8,420,000

Counterbalancing and non-counterbalancing errors


Use the following information for the next sixteen questions:
THRALL SLAVE Company made the following errors:
a. December 31, 20x1 inventory was understated by ₱100,000.
b. December 31, 20x2 inventory was overstated by ₱160,000.
c. Purchases on account in 20x1 were understated by ₱400,000 (not included in physical
count).
d. Advances to suppliers in 20x2 totaling ₱520,000 were inappropriately charged as
purchases.
e. December 31, 20x1 prepaid insurance was overstated by ₱20,000.
f. December 31, 20x1 unearned rent income was overstated by ₱104,000.
g. December 31, 20x2 interest receivable was understated by ₱68,000.
h. December 31, 20x2 accrued salaries payable was understated by ₱120,000.
i. Advances from customers in 20x2 totaling ₱240,000 were inappropriately recognized
as sales but the goods were delivered in 20x3.
j. Depreciation expense in 20x1 was overstated by ₱28,800
k. In 20x2, the acquisition cost of a delivery truck amounting to ₱360,000 was
inappropriately charged as expense. The delivery truck has a useful life of five years.
THRALL’s policy is to provide a full year’s straight line depreciation in the year of
acquisition and none in the year of disposal.
l. A fully depreciated equipment with no residual value was sold in 20x3 for ₱200,000 but
the sale was recorded in the following year.

Profits before correction of errors were ₱492,000, ₱624,000, and ₱840,000 in 20x1, 20x2,
and 20x3, respectively.

Retained earnings before correction of errors were ₱4,492,000, ₱5,116,000 and ₱5,956,000
in 20x1, 20x2, and 20x3, respectively.

15. What is the net effect of the errors on the 20x1 profit? (over) understatement
a. (187,200) b. 187,200 c. (164,200) d. 164,200

16. What is the net effect of the errors on the 20x2 profit? (over) understatement
a. (572,000) b. 572,000 c. 563,400 d. (563,400)

17. What is the net effect of the errors on the 20x3 profit? (over) understatement
a. (78,000) b. 78,000 c. (60,000) d. 60,000

18. How much is the correct profit (loss) in 20x1?


a. (348,000) b. 348,000 c. 324,800 d. 304,800

19. How much is the correct profit (loss) in 20x2?


a. 1,196,000 b. 1,296,000 c. 1,684,800 d. 1,286,000

20. How much is the correct profit (loss) in 20x3?


a. 900,000 b. 926,000 c. 968,400 d. 986,000

103
21. What is the net effect of the errors on the 20x1 retained earnings? (over)
understatement
a. (182,700) b. 182,700 c. (165,200) d. (187,200)

22. What is the net effect of the errors on the 20x2 retained earnings? (over)
understatement
a. 348,800 b. (348,800) c. (384,800) d. 384,800

23. What is the net effect of the errors on the 20x3 retained earnings? (over)
understatement
a. 444,800 b. (444,800) c. 524,800 d. (524,800)

24. How much is the correct retained earnings in 20x1?


a. 4,304,800 b. 4,404,800 c. 4,524,400 d. 4,340,800

25. How much is the correct retained earnings in 20x2?


a. 5,500,800 b. 5,756,800 c. 5,246,400 d. 5,340,400

26. How much is the correct retained earnings in 20x3?


a. 6,340,800 b. 6,400,800 c. 6,479,800 d. 7,004,400

27. Assuming that the errors were discovered late in 20x3 when the books are still open,
the compound correcting entry will include
a. 384,400 net debit to retained earnings
b. 384,400 net credit to retained earnings
c. 444,800 net debit to retained earnings
d. 444,800 net credit to retained earnings

28. Assuming that the errors were discovered late in 20x3 when the books are still open,
the compound correcting entry will include
a. 444,800 net debit to retained earnings
b. 444,800 net credit to retained earnings
c. 60,000 net debit to retained earnings
d. 60,000 net credit to retained earnings

29. What is the net effect of the errors on the 20x1 working capital? (over) understatement
a. (216,000) b. 216,000 c. 80,000 d. (80,000)

30. What is the net effect of the errors on the 20x2 working capital? (over) understatement
a. 228,000 b. (228,000) c. (68,000) d. 68,000

31. What is the net effect of the errors on the 20x3 working capital? (over) understatement
a. No effect b. 132,000 c. 200,000 d. (200,000)

The answers and solutions to the computational problems above


(Multiple choice – Computational (SET B) can be found in the
accompanying Teacher’s Manual.

Chapter 40: Theory of Accounts Reviewer


1. Philippine Financial Reporting Standards (PFRSs) are Standards and Interpretations
adopted by the Financial Reporting Standards Council (FRSC). They comprise:
a. Philippine Financial Reporting Standards (PFRSs)
b. Philippine Accounting Standards (PASs)
c. Interpretations
d. all of these

104
2. The correct arrangement of the following in accordance with the hierarchy of financial
reporting standards is
I. Conceptual framework
II. PFRSs comprising PASs, PFRSs, and Interpretations
III. Management’s judgment
IV. Standards issued by other standard-setting bodies that use a similar framework
a. I, II, IV, III b. II, III, I, IV c. II, III, IV, I d. II, I, III, IV

3. The types of accounting changes under PAS 8 are


I. change in accounting policy
II. change in accounting estimate.
III. change in reporting entity
IV. prior period errors
a. I and II b. I, II and III c. I, II, and IV d. all of these

4. This type of accounting change results from change in measurement basis


a. Change in accounting policy c. Prior period error
b. Change in accounting estimate d. Change in reporting entity

5. This type of accounting change results from changes in the realization (or incurrence)
of expected inflow (or outflow) of economic benefits from assets (or liabilities).
a. Change in accounting policy c. Prior period error
b. Change in accounting estimate d. Change in reporting entity

6. According to PAS 8, these are those adopted by an entity in preparing and presenting its
financial statements which shall be applied consistently.
a. Accounting estimates c. PFRSs
b. Accounting policies d. Debit credit

7. Accounting policies shall be changed


I. when the change is required by PFRSs
II. when the change results in a more relevant and reliable information
a. I only b. II only c. I or II d. none

8. Changes in accounting policies are accounted for


a. based on specific transitional provisions of relevant PFRS
b. by retrospective application by restatement of the beginning balance of retained
earnings and by restatement of previously presented financial statements, unless
impracticable.
c. by prospective application
d. choice (a) or in the absence thereof then choice (b)

9. A voluntary change in accounting policy is accounted for


a. based on specific transitional provisions of relevant PFRS
b. by retrospective application by restatement of the beginning balance of retained
earnings and by restatement of previously presented financial statements, unless
impracticable.
c. by prospective application
d. choice (a) or in the absence thereof then choice (b)

10.Early application of a PFRS is


a. a voluntary change in accounting policy.
b. not a voluntary change in accounting policy.
c. an involuntary change in accounting policy.
d. a prior period error

11.Changes in accounting estimates are accounted for

105
a. based on specific transitional provisions of relevant PFRS
b. by retrospective application by restatement of the beginning balance of retained
earnings and by restatement of previously presented financial statements, unless
impracticable.
c. by prospective application
d. choice (a) or in the absence thereof then choice (b)

12.Which of the following is incorrect regarding the accounting for a change in accounting
estimate?
a. The effect of a change in accounting estimate affects profit or loss in the current
period or future periods, or both.
b. No adjustments are made to the beginning balance of retained earnings or to
previously presented financial statements.
c. The effect of a change in accounting estimate affects profit or loss in the current
period only.
d. Previous financial statements need not be adjusted to apply the changed estimate in
prior periods.

13.A correction of prior period error is accounted for by (Item #1) while a change in
accounting policy is accounted for by (Item #2)
a. Retrospective restatement, Retrospective application
b. Retrospective application, Retrospective restatement
c. Prospective application, Retrospective restatement
d. None of these

14.If an entity adjusts the beginning balance of its retained earnings, then there must be a
a. change in accounting policy c. change in accounting estimate
b. correction of prior period error d. a or b

15.Adjustments to the beginning balance of retained earnings


a. are also presented in the income statement
b. are made because of a current period error
c. are made net of tax
d. are presented in the statement of financial position

16.Retrospective restatement and retrospective application (choose the incorrect


statement)
a. involves adjustments to the beginning balance of retained earnings
b. are presented in the statement of changes in equity
c. affects profit or loss in the current period
d. are not presented in the statement of financial position

17.Prior period errors in items of other comprehensive income


a. require net of tax adjustments to the beginning balance of retained earnings
b. require gross of tax adjustments to the beginning balance of retained earnings
c. may not require any adjustment to the beginning balance of retained earnings
d. a or b as an accounting policy choice in accordance with PAS 1

18.Which of the following errors is not a counterbalancing error?


a. misstatement of unearned income and prepaid expenses
b. misstatement of accrued income and accrued expenses
c. misstatement of inventory
d. misstatement of depreciation

19.Which of the following statements is incorrect?


a. Counterbalancing errors automatically reverse in subsequent period if not
corrected.

106
b. Non-counterbalancing errors do not automatically reverse in subsequent period.
c. Some errors affect only the statement of financial position.
d. Errors in financial statement elements may be rectified by appropriate disclosures
in the notes.

20.If ending inventory is understated,


a. profit for the year is overstated
b. cost of sales for the year is also understated
c. working capital is overstated
d. profit for the year is also understated

21.If an asset-related account (e.g., prepaid insurance) is understated,


a. profit for the year is overstated
b. cost of sales for the year is also understated
c. working capital is overstated
d. profit for the year is also understated

22.If current assets are understated, working capital is


a. overstated c. effect is indeterminable
b. understated d. not affected

23.The effect of prior period errors in retained earnings


a. is none
b. is always counterbalancing
c. is equal to the sum of the effects of prior period errors to profit
d. is limited to the effect of non-counterbalancing errors only

24.Which of the following items are included in the statement of changes in equity?
a. Total comprehensive income for the period, dividends declared, and proceeds from
issuance of share capital.
b. Proceeds from issuance of shares, loan notes issued or repaid, retained profit for the
period, surplus on revaluation of non-current assets.
c. Profit on ordinary activities, income tax expense, extraordinary items.
d. Accumulated profits, reserves, issued share capital, and profit sharing of employees.

25.The initial adoption of the revaluation model for PPE is


a. a change in accounting policy accounted for under PAS 16 Property, plant and
equipment
b. a change in accounting policy accounted for under PAS 8 Accounting Policies,
Changes in Accounting Estimates and Errors
c. a change in accounting estimate accounted for prospectively
d. a change in accounting estimate accounted for retrospectively

26.According to PAS 8, these are the specific principles, bases, conventions, rules and
practices applied by an entity in preparing and presenting financial statements.
a. Accounting principles c. Accounting assumptions
b. Accounting policies d. Financial Reporting Standards

27.It is an adjustment of the carrying amount of an asset or a liability, or the amount of the
periodic consumption of an asset, that results from the assessment of the present status
of, and expected future benefits and obligations associated with, assets and liabilities.
a. Change in accounting estimate c. Prior period error
b. Change in accounting policy d. Fundamental error

28.Omissions or misstatements of items are material if they could, individually or


collectively; influence the economic decisions of users taken on the basis of the financial
statements. Materiality depends on the

107
a. judgment of users, specially regulatory bodies
b. size and nature of the omission or misstatement judged in the surrounding
circumstances
c. relevance of the matter
d. judgment of the auditor and provisions of standards

29.These are omissions from, and misstatements in, the entity’s financial statements for
one or more prior periods arising from a failure to use, or misuse of, reliable
information that: (a) was available when financial statements for those periods were
authorized for issue; and (b) could reasonably be expected to have been obtained and
taken into account in the preparation and presentation of those financial statements.
a. Change in accounting estimate c. Prior period error
b. Change in accounting policy d. Fundamental error

30.According to PAS 8, prior period errors include the effects of the following
I. mathematical mistakes
II. mistakes in applying accounting policies
III. oversights or misinterpretations of facts
IV. fraud
a. II, IV b. I, II, III c. III, IV d. all of these

31.This refers to applying a new accounting policy to transactions, other events and
conditions as if that policy had always been applied.
a. Retrospective restatement c. Change in accounting policy
b. Prospective application d. Retrospective application

32.It results when the entity cannot apply it after making every reasonable effort to do so.
a. Impracticable application
b. Impractical application
c. Accounting failure
d. Financial Statement Preparation Failure

33.This refers to (a) applying the new accounting policy to transactions, other events and
conditions occurring after the date as at which the policy is changed; and (b)
recognizing the effect of the change in the accounting estimate in the current and future
periods affected by the change.
a. Prospective restatement c. Change in accounting estimate
b. Prospective application d. Pro forma financial statements

34.This refers to correcting the recognition, measurement and disclosure of amounts of


elements of financial statements as if a prior period error had never occurred.
a. Retrospective restatement c. Correction of Prior Period Error
b. Correction of Fundamental Error d. Retrospective application

35.For a particular prior period, it is impracticable to apply a change in an accounting


policy retrospectively or to make a retrospective restatement to correct an error if:
I. the effects of the retrospective application or retrospective restatement are not
determinable
II. the retrospective application or retrospective restatement requires assumptions
about what management’s intent would have been in that period
III. the retrospective application or retrospective restatement requires significant
estimates of amounts and it is impossible to distinguish objectively information
about those estimates.
a. I or II b. I or III c. I, II or III d. II and III

36.Which of the following statements is incorrect?

108
a. Implementation Guidance for Standards issued by the IASB does not form part of
those Standards, and therefore does not contain requirements for financial
statements.
b. Changes in accounting estimates result from new information or new developments
and, accordingly, are not corrections of errors.
c. The initial application of a policy to revalue assets in accordance with PAS 16
Property, Plant and Equipment or PAS 38 Intangible Assets is a change in an
accounting policy.
d. An entity shall account for a change in accounting policy resulting from the initial
application of a Standard or an Interpretation in accordance with the specific
transitional provisions, if any, in that Standard or Interpretation
e. When an entity changes an accounting policy upon initial application of a Standard
or an Interpretation that does not include specific transitional provisions applying
to that change, or changes an accounting policy voluntarily, it shall apply the change
prospectively.

37.Which of the following is a change in accounting policy?


I. the application of an accounting policy for transactions, other events or conditions
that differ in substance from those previously occurring
II. the application of a new accounting policy for transactions, other events or
conditions that did not occur previously or were immaterial.
a. I and II b. I or II c. I only d. Neither I nor II

38.Which of the following statements is incorrect?


a. For the purpose of PAS 8, early application of a Standard or an Interpretation is not
a voluntary change in accounting policy.
b. When an entity applies a new accounting policy retrospectively, it applies the new
accounting policy to comparative information for prior periods as far back as is
practicable.
c. The amount of the resulting adjustment relating to periods before those presented
in the financial statements is made to the opening balance of each affected
component of equity of the earliest prior period presented. Usually the adjustment
is made to retained earnings.
d. When it is impracticable for an entity to apply a new accounting policy
retrospectively because it cannot determine the cumulative effect of applying the
policy to all prior periods, the entity applies the new policy prospectively from the
start of the earliest period practicable. It therefore disregards the portion of the
cumulative adjustment to assets, liabilities and equity arising before that date.
e. The use of reasonable estimates is not an essential part of the preparation of
financial statements and may undermine their reliability.

39.The effect of a change in an accounting estimate shall be recognized prospectively by


including it in profit or loss in:
I. the period of the change, if the change affects that period only
II. the period of the change and future periods, if the change affects both
a. II only b. I or II c. I only d. Neither I nor II

40.To the extent that a change in an accounting estimate gives rise to changes in assets and
liabilities, or relates to an item of equity, it shall be recognized by adjusting the carrying
amount of the related asset, liability or equity item in
I. the period of the change
II. the period of the change and future periods
a. I and II b. I or II c. I only d. Neither I nor II

41.Which of the following statements is(are) correct?


I. An estimate may need revision if changes occur in the circumstances on which the
estimate was based or as a result of new information or more experience. By its

109
nature, the revision of an estimate does not relate to prior periods and is not the
correction of an error.
II. A change in the measurement basis applied is a change in an accounting policy, and
is not a change in an accounting estimate. When it is difficult to distinguish a change
in an accounting policy from a change in an accounting estimate, the change is
treated as a change in an accounting estimate.
III. Omission or misstatement of required disclosures in prior period financial
statements are not considered prior period errors.
IV. PFRSs apply only to material items, thus, financial statements do not comply with
PFRSs if they contain material errors but not immaterial errors made intentionally
to achieve a particular presentation of an entity’s financial position, financial
performance or cash flows.
V. The correction of a prior period error is excluded from profit or loss for the period
in which the error is discovered. Any information presented about prior periods,
including any historical summaries of financial data, is restated as far back as is
practicable.
a. I, II, V b. I, II, III, V c. I, II, IV, V d. I, II, III, IV, V
42.Retrospectively applying a new accounting policy or correcting a prior period error
requires distinguishing information that
I. provides evidence of circumstances that existed on the date(s) as at which the
transaction, other event or condition occurred
II. would have been available when the financial statements for that prior period were
authorized for issue from other information
a. I b. II c. I, II d. Neither I nor II

43.Which of the following items requires a prior period adjustment to retained earnings?
a. Purchases of inventory this year were overstated by ₱5 million.
b. FVOCI securities were improperly valued last year by ₱20 million.
c. Revenue of ₱5 million that should have been deferred was recorded in the previous
year as earned.
d. The prior year’s foreign currency translation gain of ₱2 million was never recorded.
(Adapted)

44.At the beginning of the current year, GRATUITOUS FREE Co. sold equipment with a two-
year service contract for a single payment of ₱20,000. The fair value of the equipment
was ₱18,000. GRATUITOUS recorded this transaction with a debit of ₱20,000 to cash
and a credit of ₱20,000 to sales revenue. Which of the following statements is correct
regarding GRATUITOUS’ current-year financial statements?
a. The financial statements are correct.
b. Net income will be overstated.
c. Total assets will be overstated.
d. Total liabilities will be overstated.
(AICPA)

45.Which of the following describes the appropriate reporting treatment for a change in
accounting estimate?
a. In the period of change with no future consideration
b. By reporting pro forma amounts for prior periods
c. By restating amounts reported in financial statements of prior periods
d. In the period of change and future periods if the change affects both
(AICPA)

46.During December 20x0, FOOTLOOSE FREE Co. incurred special insurance costs but did
not record these costs until payment was made during 20x1. These insurance costs
related to inventory that had been sold by December 31, 20x0. What is the effect of the
omission on FOOTLOOSE's accrued liabilities and retained earnings at December 31,
20x0?

110
(Item #1) Accrued liabilities; (Item #2) Retained earnings
a. No effect, No effect c. Understated, Overstated
b. No effect, Overstated d. Understated, No effect
(AICPA)

47.A material overstatement in ending inventory was discovered after the year-end
financial statements of a company were issued to the public. What effect did this error
have on the year-end financial statements? (Item #1) Current assets; (Item #2) Gross
profit
a. Understated, Overstated c. Understated, Understated
b. Overstated, Overstated d. Overstated, Understated
(AICPA)

48.Which of the following correctly relate to the effects of failure to recognize


adjustments?
Failure to Effect on Effect on statement of
Recognize profit financial position
I. Consumption of Understates Overstates Overstates
the benefits of profit assets retained
an asset earnings
II. Earning of Understates Overstates Understates
previously profit liabilities retained
unearned earnings
revenues
III. Accrual of assets Understates Understates Overstates
profit assets retained
earnings
IV. Accrual of Overstates Overstates Overstates
liabilities profit liabilities retained
earnings

a. IV b. I and III c. II d. II and III

49.An understatement in reported net income may result from failure to record:
a. amortization of discount on bonds payable c. a prepaid expense
b. an accrued liability d. a deferred revenue
(Adapted)

50.Are the following statements in relation to a change in accounting estimate true or false,
according to PAS 8 Accounting Policies, Changes in Accounting Estimates and Errors?
I. Changes in accounting estimates are accounted for retrospectively.
II. Changes in accounting estimates result from new information or new developments.
a. False, False b. False, True c. True False d. True, True
(ACCA)

51.According to PAS 8 Accounting Policies, Changes in Accounting Estimates and Errors,


which of the following statements best describes “prospective application?”
a. Recognizing a change in accounting estimate in the current and future periods
affected by the change
b. Correcting the financial statements as if a prior period error had never occurred
c. Applying a new accounting policy to transactions occurring after the date at which
the policy changed
d. Applying a new accounting policy to transactions as if that policy had always been
applied
a. I only b. I and II c. I and III d. I and IV
(ACCA)

111
52.According to PAS 8 Accounting Policies, Changes in Accounting Estimates and Errors,
which of the following terms best describes applying a new accounting policy to
transactions as if that policy had always been applied?
a. Retrospective application c. Prospective application
b. Retrospective restatement d. Prospective restatement
(ACCA)

53.Are the following statements true or false, according to PAS 8 Accounting Policies,
Changes in Accounting Estimates and Errors? An entity changes its accounting policy if
I. it is required to do so by law.
II. the change will result in providing reliable and more relevant information.
a. False, False b. False, True c. True False d. True, True
(ACCA)

54.Which of the following should be treated as a change of accounting policy according to


PAS 8 Accounting Policies, Changes in Accounting Estimates and Errors?
I. A new accounting policy of capitalizing development costs as a project has become
eligible for capitalization for the first time
II. A new policy resulting from the requirements of a new IFRS
III. To provide more relevant information, items of property, plant and equipment are
now being measured at fair value, whereas they had previously been measured at
cost
IV. A company engaging in construction contracts for the first time needs an accounting
policy to deal with this
a. I only b. I and II c. II and III d. I and IV
(ACCA)

55.How should the following changes be treated, according to PAS 8 Accounting Policies,
Changes in Accounting Estimates and Errors?
1) A change is to be made in the method of calculating the provision for uncollectible
receivables.
2) Investment properties are now measured at fair value, having previously been
measured at cost.
Change (1) Change (2)
a. Change of accounting policy Change of accounting policy
b. Change of accounting policy Change of accounting estimate
c. Change of accounting estimate Change of accounting policy
d. Change of accounting estimate Change of accounting estimate
(ACCA)

56.Failure to record the receipt of a utility bill for services already received will result in:
a. An overstatement of assets. c. An overstatement of equity.
b. An overstatement of liabilities. d. An understatement of assets.
(Adapted)

57.A voluntary change in an accounting policy in the current period should be accounted
for in comparative reports by
a. an adjustment directly to equity balances for the earliest period presented and
restatement of other comparative amounts
b. a line item on the current income statement for the cumulative effect of the change
c. presentation of pro forma comparative information
d. note disclosure only in the current period

58.If ending inventory is underestimated due to an error in the physical count of items on
hand, the cost of goods sold for the period will be <List A> and profit will be <List B>.
a. Understated, Understated c. Overstated, Understated
b. Understated, Overstated d. Overstated, Overstated

112
59.On June 15, 20x1 UPKEEP MAINTENANCE Corporation accepted delivery of
merchandise which it purchased on account. As of June 30 UPKEEP had not recorded
the transaction or included the merchandise in its inventory. The effect of this error on
its balance sheet for June 30, 20x1 would be
a. assets and stockholders’ equity were overstated but liabilities were not affected.
b. stockholders’ equity was the only item affected by the omission.
c. assets and liabilities were understated but stockholders’ equity was not affected.
d. assets and stockholders’ equity were understated but liabilities were not affected.
(Adapted)

60.How does failure to record accrued revenue distort the financial reports?
a. It understates revenue, net income, and current assets.
b. It understates net income, stockholders’ equity, and current liabilities.
c. It overstates revenue, stockholders’ equity, and current liabilities.
d. It understates current assets and overstates stockholders’ equity.

61.METE BOUNDARY Co. changed its depreciation method from the straight line method to
the sum-of-the-years’ digits method. Which is the correct sequence of accounting for the
change?
I. Compute for the carrying amount of the asset as of date of change using the previous
depreciation method.
II. Compute for the carrying amount of the asset as of date of change using the new
depreciation method as if it was used all along.
III. Get the difference between the amounts computed in steps I and II.
IV. Adjust the beginning balance of the retained earnings for the amount determined in
step III after deducting the related income tax effect.
V. Depreciate the amount computed in step I using the new depreciation method for
the current and future periods.
a. I, II, III, IV, V b. I, II, V c. I, II, III, IV d. I, V

62.Which of the following is not a retrospective-type accounting change?


a. Completed-contract method to the percentage-of-completion method for long-term
contracts
b. Average method to the FIFO method for inventory valuation
c. Sum-of-the-years'-digits method to the straight-line method
d. "Full cost" method to another method in the extractive industry
(Adapted)

63.ELAPSE TO PASS Company changed its method of pricing inventories from FIFO to
LIFO. What type of accounting change does that represent?
a. a change in accounting estimate for which the financial statements for prior periods
included for comparative purposes should be presented as previously reported.
b. a change in accounting estimate for which the financial statements for prior periods
included for comparative purposes should be presented as previously reported.
c. a change in accounting principle for which the financial statements for prior periods
included for comparative purposes should be restated.
d. none of these
(Adapted)

64.If a company incorrectly includes consignment items in the ending inventory, the net
effects on the next period’s cost of goods sold and profit, respectively are
a. Overstatement, Understatement
b. Understatement, Overstatement
c. Overstatement, overstatement
d. The next period’s account will be correct

113
65.As of year-end, a company estimated its uncollectible accounts to be P100,000. In the
following year, it was proved that the estimate was very insufficient since bad debts
incurred were almost 100% greater than the estimate made in the previous year. The
company should
a. make an adjustment classified as a prior period error
b. make an adjustment classified as a change in estimate
c. make an adjustment classified as a change in accounting policy
d. do nothing

66.Which of the following statements is incorrect?


a. When facts subsequently become available to indicate that the amount provided as
an allowance for uncollectible accounts was incorrect, an adjustment classified as
prior period error is made.
b. The percentage(s) used previously in determining provisions for uncollectible
accounts should be reviewed periodically for their continued applicability.
c. The depreciation (amortization) method, residual value, and useful lives of
depreciable assets should be reviewed at least annually, and if there are indicators
that any of them should be changed, the change is treated as a change in estimate
which is accounted for prospectively.
d. Only if an actual clerical or mechanical error occurred in the recording of allowance
for uncollectible accounts or when management was grossly negligent that
information which was present as of the time the estimates were made were not
considered properly would a subsequent change in the amounts recognized for
uncollectibility of receivables treated as correction of an error.

67.If an entity incorrectly accrues sales commission expense, which of the following best
describes the effects of the error on the current year’s profit, total assets and retained
earnings?
(Item #1) Profit; (Item #2) Total Assets; (Item #3) Retained Earnings
a. Understated, Understated, Understated
b. Understated, Overstated, Understated
c. Overstated, Understated, Overstated
d. Understated, No effect, Understated

68.Gary Snail Inc., received a 3-year non-interest bearing trade note for P50,000 on
January 1, 1985. The current interest rate at that time was 15% for similar notes. Gary
Snail recorded the receipt of the note as follows:
(Dr) Notes receivable – trade ₱50,000
(Cr) Sales ₱50,000

The effect of this accounting for the note receivable in Gary Snail’s profit for years 1985,
1986 and 1987 and retained earnings at the end of 1987, respectively, shall to
a. overstate, overstate, understate, no effect
b. overstate, understate, understate, no effect
c. overstate, understate, understate, understate
d. no effect on any of these
(RPCPA)

69.The purchase of certain goods by a company was recorded twice, in error. Because no
physical count of items on hand was conducted, inventory was also overstated.
Assuming that the company has a positive net working capital, the result of the error at
the end of the current period is:
a. The current ratio will be understated.
b. The current ratio will be overstated.
c. Working capital will be understated.
d. Working capital will be overstated.

114
70.Error of commission arises when
a. Any transaction is incorrectly entered partially
b. Any transaction is incorrectly entered wholly
c. Any transaction is not recorded either wholly or partially
d. Any transaction is incorrectly entered either wholly or partially
e. Any transaction is recorded in a fundamentally incorrect manner

71.If the opening inventory is understated and closing inventory is overstated


a. The profitability will increase and currents assets will also increase
b. The profitability will reduce and current assets will decrease
c. The profitability will increase but current assets will decrease
d. The profitability will reduce but current assets will increase
e. There will not be any change in profitability or current assets
(Adapted)

72.If machinery account is debited with the amount of repairs incurred on the machine,
this is an example of
a. Error of principle c. Error of omission
b. Error of commission d. Error of partial omission
(Adapted)

73.MIASMA HARMFUL VAPOR Inc. changes its method of valuation of inventories from
weighted-average method to first-in, first-out (FIFO) method. MIASMA should account
for this change as
a. A change in estimate and account for it prospectively.
b. A change in accounting policy and account for it prospectively.
c. A change in accounting policy and account for it retrospectively.
d. Account for it as a correction of an error and account for it retrospectively.
(Adapted)

74.Change in accounting policy does not include


a. Change in useful life from 10 years to 7 years.
b. Change of method of valuation of inventory from FIFO to weighted-average.
c. Change of method of valuation of inventory from weighted-average to FIFO.
d. Change from the practice (convention) of paying as Christmas bonus one month’s
salary to staff before the end of the year to the new practice of paying one-half
month’s salary only.
(Adapted)

75.When a public shareholding company changes an accounting policy voluntarily, it has to


a. Inform shareholders prior to taking the decision.
b. Account for it retrospectively.
c. Treat the effect of the change as an extraordinary item.
d. Treat it prospectively and adjust the effect of the change in the current period and
future periods.
(Adapted)

76.When it is difficult to distinguish between a change of estimate and a change in


accounting policy, then an entity should
a. Treat the entire change as a change in estimate with appropriate disclosure.
b. Apportion, on a reasonable basis, the relative amounts of change in estimate and the
change in accounting policy and treat each one accordingly.
c. Treat the entire change as a change in accounting policy.
d. Since this change is a mixture of two types of changes, it is best if it is ignored in the
year of the change; the entity should then wait for the following year to see how the
change develops and then treat it accordingly.
(Adapted)

115
77.When an independent valuation expert advises an entity that the salvage value of its
plant and machinery had drastically changed and thus the change is material, the entity
should
a. Retrospectively change the depreciation charge based on the revised salvage value.
b. Change the depreciation charge and treat it as a correction of an error.
c. Change the annual depreciation for the current year and future years.
d. Ignore the effect of the change on annual depreciation, because changes in salvage
values would normally affect the future only since these are expected to be
recovered in future.
(Adapted)

78.Failure to record a liability probably will


a. result in an overstated profit.
b. result in overstated total liabilities and owners’ equity.
c. have no effect on profit.
d. result in overstated total assets.

79.Which of the following statements is incorrect?


a. the accounting policies in PFRSs need not be applied when the effect of applying
them is immaterial.
b. disclosures required by PFRSs need not be made if the information is immaterial.
c. financial statements do not comply with PFRSs if they contain material errors.
d. material prior period errors are to be corrected prospectively in the first set of
financial statements authorized for issue after their discovery.

80.When accounting for changes in accounting policies, including voluntary changes, and
prior period errors, which of the following (is an) are allowed alternative(s) for the
entity under PAS 8?
I. to include in profit or loss for the current period the adjustment resulting from
changing an accounting policy or the amount of a correction of a prior period error
II. to present unchanged comparative information from financial statements of prior
periods.
a. I only b. II only c. I and II d. Neither I nor II

81.Choose the correct statement.


a. A change in the useful life of an asset requires an accounting estimate change which
affects only periods after the change is made
b. A change from a units of production to a service hours method of depreciation
would require that a prior period adjustment be recorded
c. When an accounting change is recorded and reported using the retroactive
approach, the cumulative effect on retained earnings is recorded in that account
d. Accounting changes reported by using the current approach require that "catch up
adjustment" include the effect of earnings in the year of the change
(Adapted)

82.Choose the correct statement


a. when an asset's residual value estimate is changed, the entire depreciation schedule
for the asset is recomputed all the way back to the date of acquisition
b. a change in the useful life of an asset requires an accounting estimate change which
affects only periods after the change is made
c. if an error in computing depreciation for a previous year is discovered in a later
year, a prior period adjustment is recorded in the year of discovery for the effect of
the error on all previous years
d. if a corporation changes from reporting investments on the amortized cost basis to
reporting on the fair value basis, the retrospective approach must be used

116
83.Which of the following statements is true?
a. In all situations, income tax expense is allocated only to (a) profit from continuing
operations, (b) discontinued operations, and (c) prior period adjustments
b. To compute EPS, share splits are treated as if they had occurred at the end of the
period
c. The effect of a change in an accounting estimate, like a prior period adjustment, is
reported in the retained earnings statement.
d. Intraperiod tax allocation relates to the allocation of income tax expense during the
period to various items of income that brought about the tax.
(Adapted)

84.In determining generally accepted accounting principles applicable to Philippine


practice, the pronouncements of the Financial Accounting Standards Board (FASB) in
the United States are
a. not relevant since the Philippines is developing country while the United States is
highly developed and industrialized
b. highly authoritative because accounting practice in the Philippines is patterned after
that in the United States
c. acceptable because of the worldwide recognition accorded the FASB
d. persuasive in the absence of more authoritative pronouncements by FRSC and the
IASB
(RPCPA)

85.Earnings per share figures must be presented for which of the following:
a. income after considering discontinued operations
b. income after considering cumulative effect of change in accounting principle
c. income before extraordinary item
d. income after taking into account other comprehensive income
86.A loss from a disposal of a component of an entity should be reported separately in the
income statement
a. after the cumulative effect of changes in accounting principles and before
extraordinary items
b. before cumulative effect of changes in accounting principles and after extraordinary
items
c. before other comprehensive income but after continuing operations
d. after extraordinary items and cumulative effect of changes in accounting principles
(Adapted)

87.A change in the periods benefited by a deferred cost because additional information has
been obtained is
a. A correction of an error.
b. An accounting change that should be reported by restating the financial statements
of all prior periods presented.
c. An accounting change that should be reported in the period of change and future
periods if the change affects both.
d. Not an accounting change.

88.At the end of 20x1, STOCHASTIC RANDOM Co. failed to accrue sale commissions
incurred during 20x1 but paid in 20x2. The error was not repeated in 20x2. What was
the effect of this error on 20x1 ending working capital and on the 20x2 ending retained
earnings balance?
(Item #1) 20x1 ending working capital (Item #2) 20x2 ending retained earnings
a. Overstated, Overstated c. No effect, No effect
b. No effect, Overstated d. Overstated, No effect
(Adapted)

117
89.On August 31, 20x1, CONTUMACIOUS REBELLIOUS Co. decided to change from the FIFO
periodic inventory system to the weighted average periodic inventory system.
REBELLIOUS is on a calendar year basis. The cumulative effect of the change is
determined
a. As of January 1, 20x1.
b. As of August 31, 20x1.
c. During the eight months ending August 31, 20x1, by a weighted average of the
purchases.
d. During 20x1 by a weighted average of the purchases.
(Adapted)

90.The normal approach in accounting for a change in accounting principle is the


retrospective application method. Which of the following changes is the exception to
that approach?
a. Straight-line method of depreciation for previously recorded assets to the sum-of-
the-years'-digits method.
b. Average method of inventory pricing to the FIFO method.
c. Percentage-of-completion method of accounting for long-term construction-type
contracts to the completed-contract method.
d. Change in financial reporting framework.
(Adapted)

91.Is the cumulative effect of an inventory pricing change on prior years’ earnings
reported separately between profit for the year and other comprehensive income for a
change from
(Item #1) Specific identification to FIFO; (Item #2) FIFO to weighted average?
a. Yes, Yes b. Yes, No c. No, No d. No, Yes
(Adapted)

92.When a company changes the expected service life of an asset because additional
information has been obtained, which of the following should be reported?
(Item #1) Pro forma effect of retroactive application; (Item #2) Cumulative effect of a
change in accounting policy
a. Yes, Yes b. No, Yes c. Yes, No d. No, No
(Adapted)

93.During 20x1, COGENT CONVINCING Co. increased the estimated quantity of copper
recoverable from its mine. COGENT uses the units of production depletion method. As a
result of the change, which of the following should be reported in COGENT’s 20x1
financial statements?
I. Cumulative effect of a change in accounting principle
II. Pro forma effects of retroactive application of new depletion base
a. Yes, Yes b. Yes, No c. No, No d. No, Yes
(Adapted)

94.DOGLEG CROOKED Co. estimated its two-year equipment warranty costs based on ₱100
per unit sold in 20x1. Experience during 20x2 indicated that the estimate should have
been based on ₱110 per unit. The effect of this ₱10 difference from the estimate is
reported
a. In 20x2 income from continuing operations.
b. As an accounting change, net of tax, below 20x2 income from continuing operations.
c. As an accounting change requiring 20x1 financial statements to be restated.
d. As a correction of an error requiring 20x1 financial statements to be restated.
(Adapted)

95.Presenting consolidated financial statements this year when statements of individual


companies were presented last year is

118
a. A correction of an error.
b. An accounting change that should be reported prospectively.
c. An accounting change that should be reported by restating the financial statements
of all prior periods presented.
d. Not an accounting change.
(Adapted)

96.ELOQUENT FLUENT Co. included a foreign subsidiary in its 20x8 consolidated financial
statements. The subsidiary was acquired in 20x2 and was excluded from previous
consolidations. The change was caused by the elimination of foreign exchange controls.
The obtaining of control in 20x8 was accounted for under the acquisition method
prescribed under PFRS 3 Business Combinations. The transaction shall be accounted for
a. By note disclosure only.
b. Currently and prospectively.
c. Currently with note disclosure of pro forma effects of a retrospective application.
d. This is a change in reporting entity accounted for retrospectively by restating the
financial statements of all prior periods presented.
(Adapted)

97.Which of the following should be reported as a prior period adjustment?


(Item #1) Change in estimated lives of depreciable assets
(Item #2) Change from unaccepted principle to accepted principle
a. Yes, Yes b. No, Yes c. Yes, No d. No, No
(Adapted)

98.How should the effect of a change in accounting principle that is inseparable from the
effect of a change in accounting estimate be reported?
a. As a component of income from continuing operations.
b. By restating the financial statements of all prior periods presented.
c. As a correction of an error.
d. By footnote disclosure only.
(Adapted)

99.On January 2, 20X9, to better reflect the variable use of its only machine, COLLUDE TO
CONSPIRE, Inc. elected to change its method of depreciation from the straight-line
method to the units-of-production method. The original cost of the machine on January
2, 20X7, was ₱50,000, and its estimated life was 10 years. COLLUDE estimates that the
machine's total life is 50,000 machine hours. Machine hours usage was 8,500 during
20X8 and 3,500 during 20X7. COLLUDE's income tax rate is 30%. COLLUDE should
report the accounting change in its 20X9 financial statements as a(an)
a. Change in accounting estimate accounted for prospectively [affecting current (20X9)
and future years only].
b. Adjustment to beginning retained earnings of ₱2,000.
c. Cumulative effect of a change in accounting principle of ₱1,400 in its income
statement.
d. Adjustment to beginning retained earnings of ₱1,400.
(Adapted)

100. Which of the following statements is correct regarding accounting changes that
result in financial statements that are, in effect, the statements of a different reporting
entity?
a. Cumulative-effect adjustments should be reported as separate items on the financial
statements pertaining to the year of change.
b. No restatements or adjustments are required if the changes involve consolidated
methods of accounting for subsidiaries.
c. No restatements or adjustments are required if the changes involve the cost or
equity methods of accounting for investments.

119
d. The financial statements of all prior periods presented should be restated.
(AICPA)

101. According to PAS 8, retrospective application of a change in accounting policy is


impracticable when
I. The costs of applying the new policy to prior period financial statements are
material.
II. The entity cannot apply the new policy after making every reasonable effort.
a. I only b. II only c. Both I and II d. Neither I nor II
(Adapted)

102. Which of the following statements is (are) correct?


I. When a component of an entity is eliminated, it is referred to as a discontinued
operation.
II. When a company changes its depreciation method from an accelerated method to
the straight-line method, it is considered to be a change in accounting policy.
III. If a company's stock is publicly traded, it is a requirement that the earnings per
share appear on the statement of profit or loss and other comprehensive income (or
separate income statement).
IV. The notes to the financial statements is considered an integral part of the financial
statements.
V. It is acceptable that some of the expenses reported on the income statement are
estimates.
a. I, III and IV b. III, IV and V c. I, III, IV and V d. all of these

103. Proper application of accounting principles is most dependent upon the


a. existence of specific guidelines.
b. oversight of regulatory bodies.
c. external audit function.
d. professional judgment of the accountant.
(Adapted)

104. Which of the following are correctly stated?


I. Changes in accounting estimates are to be reported only in current and future
periods and no retroactive adjustments are be made.
II. A change in reporting entity is effected by restating all prior period financial
statements in accordance with the method of presenting the current financial
statements of the new reporting entity.
III. A change in accounting policy includes not only changes in accounting principles
and practices but also changes in methods of applying them.
IV. A company fails to record a sale of merchandise on account and the goods sold are
still included in the ending inventory, then net income is understated and current
assets are overstated.
V. Changes in accounting policy, practices or the methods of applying them, together
with the financial effect, and the justification for the change should be disclosed in
the financial statement or in a notes thereto.
a. II, III, IV, and V b. I, II, III, and V c. I, II, III, IV, and V d. I, II, III
(Adapted)

105. IMPERIUM SUPREME POWER Company decided to change its method of inventory
valuation from Average method to FIFO beginning January 1, 20x1. GAAP require that
this change in accounting method be handled by
a. disclosing the reason for the change in the “significant accounting policies” note for
20x1 but not restating prior year financial statements
b. applying retrospectively the new method by restating prior years and providing
appropriate note disclosures
c. showing the cumulative effect of the change in the 20x1 financial statements only

120
d. disclosing the reason for the change in 20x1’s note along with pro forma effect on
future earnings for succeeding years
(Adapted)

106. According to PAS 8, one of the types of accounting changes is change in accounting
policy. Which of the following is not a change in accounting policy?
a. a change from the Average method to FIFO method of inventory pricing
b. a change in the estimated period benefited by a deferred cost
c. a change from the cost model to fair value model of accounting for investment
property
d. initial adoption of revaluation method for property, plant and equipment

107. Potential obligations involving uncertainty as to possible losses are known as:
(Item #1) Contingent liability; (Item #2) Estimated liability
a. yes, yes b. yes, no c. no, yes d. no, no
(Adapted)

108. The value of the inventory of work-in-process was overstated at the end of the
period, the overstatement shall result in
a. understatement of the net income for the period
b. overstatement of the net income for the period
c. overstatement of the cost of goods sold for the period
d. understatement of working capital
(Adapted)

109. Reporting the effect of change in accounting estimate should be accounted for in
a. period of change if the change affect that period only
b. period of change and future periods if the change affects both
c. a or b
d. future periods only

110. Changes in accounting policy, such as a change from Average method to FIFO
method of inventory cost flow assumption, are accounted for retrospectively. The
primary reason for using a retroactive approach is:
a. to meet users’ needs for comparable financial statements
b. to avoid distorting net income due to the significance of the amounts involved
c. to prevent a dilution of public confidence in financial statements
d. to conform with the current operating performance concept of income
measurement

111. The following statements relate to accounting changes:


I. retrospective treatment is not appropriate for changes in accounting estimate
because these are normal recurring adjustments that are the natural result of the
accounting process.
II. a cumulative effect is not reported when an entity adopts a new depreciation
method for newly acquired assets but a pro forma schedule should be disclosed in
the notes.
III. when a cumulative effect type of change in accounting policy is made during the
year, the cumulative effect of the change is determined as far back as practicable.
IV. a retrospective, rather than a prospective adjustment is used when the accounting
change is from FIFO to any other allowed method of inventory costing.

State whether the foregoing statements are false:


a. all of the statements are false c. only two statements are false
b. only one statement is false d. three statements are false
(RPCPA)

121
112. During 20x1, VITUPERATION ABUSE Co. collected ₱1,200 as advance payments
from customers. The bookkeeper credited the sales revenue account as the cash was
received. The goods were delivered in 20x2. No adjusting entry was made at the end of
20x1 on the payments made in advance. The failure to adjust the books would cause
a. liabilities to be understated by ₱1,200 on the Dec. 31, 20x2 balance sheet
b. owners’ equity understated by ₱1,200 on the Dcc. 31, 20x1 balance sheet
c. assets overstated by ₱1,200 on the Dec. 31, 20x1 balance sheet
d. owners’ equity overstated by ₱1,200 on the Dec. 31, 20x1 balance sheet
(Adapted)

113. The following list describes some situations that occur in accounting. Which of the
following does not involve a correction of an error?
a. change from recording all research and development costs as an asset and
amortizing them over 30 years to expensing them in the period of expenditure
b. change in the useful life of a machine because it had mistakenly been classified as a
truck. The firm depreciates trucks over a four year useful life and machinery over a
ten year useful life
c. change in depreciation expense because the useful life of several assets were
changed. They were wearing out faster than assumed
d. charge in 20x1 net income because of an arithmetic error in recording interest
expense

114. Which of the following changes is accounted for on a prospective basis?


a. change from FIFO to average method of inventory cost formula
b. change from PFRS for SMEs to full PFRSs
c. change from the percentage of receivables to the aging of receivables method of
determining uncollectible accounts
d. voluntary change in accounting policy

115. Restatement of prior period financial statements is required when the change or
correction is handled on a
a. prospective basis c. retrospective basis
b. current basis d. current and future basis

116. Which of the following is not reported on a statement of changes in equity?


a. dividends declared and paid c. extraordinary items
b. comprehensive income in total d. correction of prior period error
(Adapted)

117. On December 31, 20x1, TRANSPICUOUS CLEAR Company understated its ending
inventory by ₱4,000. Assuming no correcting entry has yet been made, the effects on
the 20x1 income statement items are: (Item #1) Net income; (Item #2) Sales; (Item #3)
Cost of goods sold
a. overstated, no effect, understated c. overstated, overstated, understated
b. understated, no effect, overstated d. understated, no effect, understated
(Adapted)

118. Which of the following statements is correct?


a. When an entity capitalizes borrowing costs, it should apply this policy to all
qualifying assets.
b. A change in accounting of a reporting entity which does not have a material effect in
the period of change but is reasonably certain to have a material effect in later
periods need not be disclosed whenever the financial statements of the period of
change are presented.
c. An error that understates the ending inventory will overstate net income in the year
of the error.
d. An overstated ending inventory leads to understated net income.

122
119. Which of the following statements is incorrect?
a. When a company fails to record a purchase of merchandise on account and the
goods purchased are included in the ending inventory then net income is overstated
and current liabilities are understated.
b. Remeasurements, new data, corrections, or other adjustments are often required
after the events have been initially recorded, classified and summarized.
c. Errors in determining the cost of inventory at the end of the period lead to a balance
sheet that does not balance and are thus readily observed.
d. The matching principle implies that a relationship exists between expenses and
revenues.

120. Which of the following statements is correct?


a. An error in determining the cost of the ending inventory of a period generally
results in misstated income for two periods because such error is a
counterbalancing error.
b. Because of the importance of consistency, a company cannot change inventory
costing methods unless the prior method is one that is acceptable to the profession
of accountancy.
c. The consistency principle implies that a relationship exists between expenses and
revenues.
d. The correction of error in previously issued financial statements is an accounting
change.

121. Disclosure of accounting policies should identify and describe the accounting
principles followed by the reporting entity and the methods of applying those principles
that materially affect: (Item #1) Determination of financial position; (Item #2) Changes
in financial position; (Item #3) Results of operations
a. No, Yes, No c. No, Yes, Yes
b. Yes, Yes, Yes d. No, Yes, Yes

Chapter 40 - Suggested answers to review theory questions


1. D 16. C 31. D 46. C 61. D 76. A 91. C 106. B
2. B 17. C 32. A 47. B 62. C 77. C 92. D 107. B
3. A 18. D 33. B 48. C 63. D 78. A 93. C 108. B
4. A 19. D 34. A 49. C 64. A 79. D 94. A 109. C
5. B 20. D 35. C 50. B 65. B 80. D 95. D 110. A
6. B 21. D 36. E 51. C 66. A 81. C 96. B 111. B
7. C 22. B 37. D 52. A 67. D 82. C 97. B 112. D
8. D 23. C 38. E 53. B 68. B 83. D 98. A 113. C
9. B 24. A 39. B 54. C 69. A 84. D 99. A 114. C
10. B 25. A 40. C 55. C 70. D 85. A 100. D 115. C
11. C 26. B 41. A 56. C 71. A 86. C 101. B 116. C
12. C 27. A 42. C 57. A 72. A 87. C 102. C 117. B
13. A 28. B 43. C 58. C 73. C 88. D 103. D 118. A
14. D 29. C 44. B 59. C 74. A 89. A 104. C 119. C
15. C 30. D 45. D 60. A 75. B 90. A 105. B 120. A
121. B

123
Chapter 41
Statement of Cash Flows

Chapter 41: Multiple choice – Computational (SET B) – (For classroom instruction


purposes)
Analysis of Cash account Method
Use the following information for the next three questions:
The movements in the cash account of DEADLOCK STANDSTILL Co. during 20x2 are shown
below.

Cash
beg. 400
Sales 12,000 7,600 Purchases
Interest income 40 2,400 Operating expenses
Rent income 540 60 Interest expense
Dividend income 80 140 Income taxes
Sale of held for trading
securities 1,600 200 Investment in FVOCI
Sale of old building 1,040 2,200 Purchase of equipment
Loan granted to
Collection of non-trade note 120 260 employee
Proceeds from loan with a Payment of loan
bank 3,200 480 borrowed
Issuance of shares 1,940 400 Reacquisition of shares
180 Dividends
7,040 end.

1. How much is the cash flows from operating activities?


a. 4,600 b. 4,840 c. 5,040 d. 4,060

2. How much is the cash flows from investing activities?


a. (1,500) b. 1,500 c. 1,240 d. (1,240)

3. How much is the cash flows from financing activities?


a. 4,800 b. (4,800) c. 4,240 d. 4,080

T-Account Method
BLITHE JOYFUL Co. had the following information during 20x2:
Accounts receivable, January 1, 20x2 2,400
Accounts receivable, December 31, 20x2 1,600
Sales on account and cash sales 32,000
Bad debts expense 800
Accounts payable, January 1, 20x2 1,400
Accounts payable, December 31, 20x2 800
Cost of sales 16,000
Increase in inventory 3,600

Operating expenses on accrual basis 4,880


Increase in accrued payables for operating expenses 1,640
Decrease in prepaid operating expenses 1,560

Property, plant, and equipment, January 1, 20x2 7,200


Property, plant, and equipment, December 31, 20x2 10,800

124
Additional information:
 There were no write-offs of accounts receivable during the year.
 Equipment with an accumulated depreciation of ₱800 was sold during the year for
₱480 resulting to a gain on sale of ₱60.

4. How much is the cash receipts from customers?


a. 38,200 b. 37,400 c. 35,400 d. 32,800

5. How much is the cash payments to suppliers?


a. 19,000 b. 20,200 c. 22,000 d. 23,400

6. How much is the cash payments for operating expenses?


a. 1,680 b. 4,800 c. 4,960 d. 8,080

7. How much is the cash payments for acquisition of property, plant, and equipment?
a. 3,600 b. 4,820 c. 4,080 d. 4,940

T-Account Method – Cash receipts (w/ write-off)


8. ABC Co. has the following information as of December 31, 20x1:
Jan. 1 Dec. 31
Accounts receivable 100,000 250,000
Allowance for bad debts 15,000 20,000
Net credit sales 850,000
Bad debt expense 60,000
Recoveries 20,000

How much is the total cash receipts from customers during the period?
a. 970,000 b. 879,000 c. 907,000 d. 897,000

Cash flow from operating activities – Indirect method


9. BLUFF DECEIVE Co. has the following information as of December 31, 20x2:

Jan. 1 Dec. 31
Accounts receivable 16,000 20,000
Allowance for bad debts (400) (1,000)
Prepaid rent 3,840 3,200
Accounts payable 6,800 8,800

BLUFF reported profit of ₱8,800 for the year, after depreciation expense of ₱200, gain on
sale of equipment of ₱240, and restructuring and other provisions of ₱400. None of the
provisions recognized during the period affected cash.

How much is the cash flows from operating activities?


a. 4,800 b. 5,600 c. 8,800 d. 8,400

Cash flow from investing and financing activities


Use the following information for the next two questions:
INORDINATE EXCESSIVE Co. had the following information for 20x2:
 Acquired 3-month treasury bills for ₱200,000.
 Acquired equipment with a purchase price of ₱4,000,000 by paying 20% in cash and
issuing a note payable for the balance. There were no payments made on the note
during the year.
 Acquired land with fair value of ₱3,200,000 by issuing shares with aggregate par value
of ₱2,400,000. The excess is credited to share premium.
 Extended a ₱1,600,000 loan to a director.
 Borrowed ₱1,280,000 from a bank. Used the cash proceeds as follows: ₱800,000 for
additional working capital and ₱480,000 to settle scrip dividends declared in 20x1.

125
 Settled an outstanding note payable by issuing shares with aggregate par value of
₱800,000. Share premium resulted from the transaction amounted to ₱280,000.

10. How much is the net cash flows from (used in) investing activities?
a. (2,400,000) b. 2,400,000 c. 800,000 d. (800,000)

11. How much is the net cash flows from (used in) financing activities?
a. (800,000) b. 800,000 c. (2,400,000) d. 2,400,000

Comprehensive
Information on LA-DI-DA SHOWY Co.'s financial position and performance as of December
31, 20x2 and 20x1 are presented below.

LA-DI-DA SHOWY Company


Statement of financial position
As of December 31, 20x2
ASSETS 20x2 20x1
Current assets
Cash and cash equivalents 1,000,000 600,000
Held for trading securities 480,000 -
Accounts receivable – net 1,520,000 1,240,000
Rent receivable 100,000 40,000
Inventory 2,000,000 3,600,000
Prepaid insurance 200,000 160,000
Total current assets 5,300,000 5,640,000
Noncurrent assets
Investment in bonds 360,000 340,000
Buildings 10,000,000 4,000,000
Accumulated depreciation (800,000) (800,000)
Goodwill 360,000 400,000
Total noncurrent assets 9,920,000 3,940,000
TOTAL ASSETS 15,220,000 9,580,000
LIABILITIES AND EQUITY
Current liabilities
Accounts payable 480,000 320,000
Unearned rent 80,000 120,000
Insurance payable 240,000 180,000
Dividends payable 920,000 480,000
Income tax payable 60,000 140,000
Short-term loan payable - 200,000
Total current liabilities 1,780,000 1,440,000
Noncurrent liabilities
Bonds payable 4,000,000 4,000,000
Discount on bonds (380,000) (400,000)
Deferred tax liability 60,000 40,000
Total noncurrent liabilities 3,680,000 3,640,000
TOTAL LIABILITIES 5,460,000 5,080,000
Equity
Share capital 8,000,000 4,000,000
Retained earnings 1,760,000 500,000
 TOTAL EQUITY 9,760,000 4,500,000
TOTAL LIABILITIES AND EQUITY 15,220,000 9,580,000
     

126
LA-DI-DA SHOWY Company
Statement of profit or loss
For the year ended December 31, 20x2
Sales 20,000,000
Cost of sales (12,000,000)
Gross income 8,000,000
Rent income 1,800,000
Interest income 80,000
Insurance expense (400,000)
Bad debts expense (60,000)
Interest expense (400,000)
Loss on sale of building (160,000)
Unrealized gain on investment 80,000
Other expenses (4,800,000)
Profit before tax 4,140,000
Income tax expense (1,200,000)
Profit for the year 2,940,000

Additional information:
 During 20x2, LA-DI-DA purchased held for trading securities for ₱400,000. The fair value
of the shares on December 31, 20x2 is ₱480,000.
 The allowance for doubtful accounts has balances of ₱80,000 and ₱40,000 as of
December 31, 20x2 and 20x1, respectively.
 During 20x2, LA-DI-DA sold an old building with historical cost of ₱3,200,000 for
₱1,040,000.
 LA-DI-DA inadvertently included depreciation expense in the “Other expenses” line item.
 There were no acquisitions or disposals of investment in bonds during the period.
 During 20x2, LA-DI-DA issued shares with an aggregate par value of ₱4,000,000 for
₱4,000,000 cash.

12. How much is the net cash flows from (used in) operating activities?
a. (6,000,000) b. 6,000,000 c. 6,600,000 d. (7,600,000)

13. How much is the net cash flows from (used in) investing activities?
a. (8,160,000) b. 8,460,000 c. (9,200,000) d. 8,160,000

14. How much is the net cash flows from (used in) financing activities?
a. (2,560,000) b. 2,560,000 c. (2,960,000) d. 2,960,000

Reconstruction of statement of cash flows


15. REMNANT REMAINDER Co.’s cash balances as of December 31, 20x2 and 20x1 were
₱7,040,000 and ₱400,000 respectively. REMNANT’s December 31, 20x2 statement of
cash flows reported net cash used in investing activities of ₱1,500,000 and net cash
from financing activities of ₱4,080,000.

How much is the net cash flows from (used in) operating activities?
a. (4,060,000) b. 4,060,000 c. 4,600,000 d. (4,600,000)

The answers and solutions to the computational problems above


(Multiple choice – Computational (SET B) can be found in the
accompanying Teacher’s Manual.
Chapter 41: Theory of Accounts Reviewer
1. Which of the following statements is incorrect?

127
a. The statement of cash flows shows the historical sources and utilization of cash
flows during the period.
b. The statement of cash flows is an integral part of a complete set of financial
statements.
c. The statement of cash flows is the only source of information used in assessing the
ability of an entity to generate cash and cash equivalents, the timing and certainty of
their generation, and the needs of the entity to utilize those cash flows.
d. The historical changes of cash flows shall be classified into three activities in the
statement of cash flows.

2. Which of the following statements is correct?


a. The historical changes of cash flows shall be classified into operating activities,
investing activities, financing activities, and cash and cash equivalents activities.
b. The statement of cash flows is only one source of information used in assessing the
ability of an entity to generate cash and cash equivalents, the timing and certainty of
their generation, and the needs of the entity to utilize those cash flows.
c. Operating activities include transactions that affect long-term assets and other non-
operating assets.
d. Financing activities include transactions that enter into the determination of profit
or loss. These transactions normally affect income statement accounts.

3. Which of the following statements is incorrect?


a. The historical changes of cash flows are classified into operating activities, investing
activities, and financing activities in the statement of cash flows.
b. The statement of cash flows is used in conjunction with the other financial
statements in assessing the ability of an entity to generate cash and cash
equivalents, the timing and certainty of their generation, and the needs of the entity
to utilize those cash flows.
c. All transactions that result to changes in financial position are included in the
statement of cash flows.
d. Operating activities include transactions that enter into the determination of profit
or loss. These transactions normally affect income statement accounts.

4. Which of the following statements is incorrect?


a. Cash flows from investing activities may be reported using either the direct method
or the indirect method.
b. Only transactions that have affected cash during the reporting period are included
in the statement of cash flows. Non-cash transactions are excluded from the
statement of cash flows and disclosed only.
c. Investing activities include transactions that affect long-term assets and other non-
operating assets.
d. Financing activities include transactions that affect equity and non-operating
liabilities.

5. Which of the following statements is correct?


a. Cash flows from financing activities may be reported using either the direct method
or the indirect method.
b. The indirect method is encouraged under PAS 7.
c. The direct method shows each major class of gross cash receipts and gross cash
payments.
d. The direct method is required under PAS 7.

6. Which of the following is correct regarding the classification of certain items in the
statement of cash flows by a non-financial institution?
Option 1 Option 2
I. Interest received Operating activities Investing activities
II. Interest paid Operating activities Financing activities

128
III Dividends Operating activities Investing activities
. received
IV. Dividends paid Financing activities Operating activities

a. I only b. I, II and III c. II, III, and IV d. all of these

7. Interest income received is classified in the statement of cash flows as


a. Operating activities c. Investing activities
b. Financing activities d. a or c

8. Interest expense paid is classified in the statement of cash flows as


a. Operating activities c. Investing activities
b. Financing activities d. a or b

9. Dividend income received is classified in the statement of cash flows as


a. Operating activities c. Investing activities
b. Financing activities d. a or c

10. Dividends paid is classified in the statement of cash flows as


a. Operating activities c. Investing activities
b. Financing activities d. a or c

11. A financial institution shall classify dividend and interest income received and interest
expense paid as
a. Operating activities c. Investing activities
b. Financing activities d. any of these

12. If the cash flows from an activity in the statement of cash flows results to a negative
amount, it is described as
a. use of cash c. broke
b. source of cash d. ask mama to make padala

13. Under this method, profit or loss computed under the accrual basis is adjusted for non-
cash income and non-cash expenses and changes in current operating assets and
liabilities in order to derive the profit or loss under the cash basis, which is normally the
cash flow from operating activities.
a. Direct method c. Short-cut method
b. Indirect method d. Investing method

14. Under the indirect method, accrual basis profit is converted to cash basis profit as
follows (choose the incorrect statement)
a. Non-cash expense is added back to and non-cash income is deducted from accrual
basis profit.
b. Decreases in current operating assets are added back to and increases in current
operating assets are deducted from accrual basis profit.
c. Decreases in current operating liabilities are added back to and increases in current
operating liabilities are deducted from accrual basis profit.
d. Increases in current operating liabilities are added back to and decreases in current
operating liabilities are deducted from accrual basis profit.

15. Which of the following statements is incorrect?


a. The statement of cash flows may be prepared by analyzing the various debits and
credits in the cash and cash equivalents accounts.
b. The statement of cash flows may be prepared by performing T-account analyses.
c. Cash flows from (used in) investing and financing activities are presented separately
at gross amounts, unless they qualify for net presentation under PAS 7.

129
d. Cash flows from (used in) investing and financing activities are presented on a net
basis, unless they qualify for gross presentation under PAS 7.

16. When computing for the cash receipts from collections of accounts receivable
a. Increase in accounts receivable is added to total net credit sales
b. Decrease in accounts receivable is deducted from total net credit sales
c. Increase in accounts receivable is deducted from total net credit sales
d. Sales returns from cash sales affect the computation.

17. Under PAS 7, cash flows from operating activities are reported using
a. direct method b. indirect method c. a or b d. none

18. Which of the following advantages does not relate to the direct method?
a. Simple to produce
b. Allows for realistic projections of future operating cash flow
c. Greater transparency
d. Better insights into the current cash position
(ACCA)

19. An entity purchases a building and pays in equity shares of the entity. This transaction
should be treated in the statement of cash flows as follows:
a. The purchase of the building should be financing cash outflow and the issuance of
shares as financing cash outflows
b. The purchase of the building should be investing cash outflow and the issuance of
shares as financing cash outflows
c. The purchase of the building should be operating cash outflow and the issuance of
shares financing cash outflows
d. This does not belong in a cash flow statement and should be disclosed only in the
notes to the financial statements
(ACCA)

20. Most companies use this method of reporting cash flows from operating activities.
a. Direct method c. Cooking the books method
b. Indirect method d. No one in particular

21. Both the direct and indirect methods require cash flows to be classified according to
operating, investing, and financing activities. The difference in presentation between
the two methods:
a. affects the investing section only. The operating and financing sections do not differ
between the two presentations
b. affects the financing section only. The investing and operating sections do not differ
between the two presentations
c. affects the operating and investing sections. The financing section does not differ
between the two presentations
d. affects the operating section only. The investing and financing sections do not differ
between the two presentations
(ACCA)

22. An entity (other than a financial institution) receives dividends from its investment in
shares. How should it disclose the dividends received in the statement of cash flow
under PAS 7?
a. Operating cash inflow
b. Financing cash flow
c. Either as operating cash inflow or as financing cash inflow
d. Either as operating cash inflow or as investing cash inflow
(ACCA)

130
23. On December 1, 20x1, LEVIATHAN Co. received notice of a dividend declaration from
GIGANTIC, Inc. to be paid on January 5, 20x2. LEVIATHAN holds shares of GIGANTIC as
investment. In LEVIATHAN’s December 31, 20x1 statement of cash flow, the dividend
income should be presented under
I. Operating activities II. Investing activities III. Financing activities
a. I only b. either I or II c. any of the choices d. none

24. Cash dividends declared but remain unpaid is presented in the statement of cash flows
as
a. operating activity as an outflow
b. financing activity as an inflow
c. either operating activity or financing activity
d. not presented

25. In relation to cash flow statements, which, if any, of the following are correct?
I. The direct method of calculating net cash from operating activities leads to a
different figure from that produced by the indirect method, but this is balanced
elsewhere in the cash flow statement.
II. A company making high profits must necessarily have a net cash inflow from
operating activities.
III. Profits and losses on disposals of non-current assets appear as items under cash
flows from investing activities in the cash flow statement or a note to it.
a. Item 1 only b. Item 2 only c. Item 3 only d. None of the items.
(ACCA)

26. A cash flow statement prepared in accordance with PAS 7 opens with the calculation of
cash flows from operating activities from the net profit before taxation. Which of the
following lists of items consists only of items that would be added to net profit before
taxation in that calculation?
a. Decrease in inventories, depreciation, profit on sale of non-current assets.
b. Increase in trade payables, decrease in trade receivables, profit on sale of non-
current assets.
c. Loss on sale of non-current assets, depreciation, increase in trade receivables.
d. Decrease in trade receivables, increase in trade payables, loss on sale of non-current
assets.
(ACCA)

27. How should the amortization of bond discount on long-term debt be reported in a
statement of cash flows prepared using the indirect method?
a. As a financing activities inflow
b. As a financing activities outflow
c. In operating activities as a deduction from income
d. In operating activities as an addition to income
(ACCA)

28. Which of the following information should be disclosed as supplemental information in


the statement of cash flows? (Item #1) Cash flow per share; (Item #2) Conversion of
debt to equity
a. Yes, Yes b. Yes, No c. No, Yes d. No, No
(AICPA)

29. Which of the following would be subtracted in converting net earnings to cash provided
by operations in the current period in a consolidated statement of cash flows?
a. Amortization of premium on bonds payable.
b. Amortization of patent.
c. Increase in deferred income tax liability.
d. Minority interest's share of net earnings.

131
(AICPA)

30. In a statement of cash flows of a financial institution, which of the following items is
reported as a cash outflow from financing activities?
I. Payments to retire mortgage notes.
II. Interest payments on mortgage notes.
III. Dividend payments
a. I, II, and III. b. II and III. c. I only. d. I and III.
(AICPA)

31. How should a gain from the sale of used equipment for cash be reported in a statement
of cash flows using the indirect method?
a. In investment activities as a reduction of the cash inflow from the sale.
b. In investment activities as a cash outflow.
c. In operating activities as a deduction from income.
d. In operating activities as an addition to income.
(AICPA)

The next two items are based on the following:


A company acquired a building, paying a portion of the purchase price in cash and issuing a
mortgage note payable to the seller for the balance.
32. In a statement of cash flows, what amount is included in investing activities for the
above transaction?
a. Cash payment. c. Zero.
b. Acquisition price. d. Mortgage amount.
(AICPA)

33. In a statement of cash flows, what amount is included in financing activities for the
above transaction?
a. Cash payment. c. Zero.
b. Acquisition price. d. Mortgage amount.
(AICPA)

34. In a statement of cash flows, receipts from sales of property, plant, and equipment and
other productive assets should generally be classified as cash inflows from
a. Operating activities. c. Investing activities.
b. Financing activities. d. Selling activities.
(AICPA)

35. In a statement of cash flows, which of the following would increase reported cash flows
from operating activities using the direct method? (Ignore income tax considerations.)
a. Dividends received from investments.
b. Gain on sale of equipment.
c. Gain on early retirement of bonds.
d. Change from straight-line to accelerated depreciation.
(AICPA)

36. Which of the following cash flows per share should be reported in a statement of cash
flows?
a. Primary cash flows per share only.
b. Fully diluted cash flows per share only.
c. Both primary and fully diluted cash flows per share.
d. Cash flows per share should not be reported.
(AICPA)

37. LIBIDINOUS LUSTFUL Co. uses the direct write-off method to account for uncollectible
accounts receivable. During an accounting period, LIBIDINOUS’ cash collections from

132
customers equal sales adjusted for the addition or deduction of the following amounts:
(Item #1) Accounts written-off; (Item #2) Increase in accounts receivable balance
a. Deduction, Deduction c. Deduction, Addition
b. Addition, Deduction d. Addition, Addition
(AICPA)

38. On September 1, 20x1, PRESUMPTUOUS OVERSTEPPING Co. sold used equipment for a
cash amount equaling its carrying amount for both book and tax purposes. On
September 15, 20x1, PRESUMPTUOUS replaced the equipment by paying cash and
signing a note payable for new equipment. The cash paid for the new equipment
exceeded the cash received for the old equipment. How should these equipment
transactions be reported in PRESUMPTUOUS’s 20x1 statement of cash flows?
a. Cash outflow equal to the cash paid less the cash received.
b. Cash outflow equal to the cash paid and note payable less the cash received.
c. Cash inflow equal to the cash received and a cash outflow equal to the cash paid and
note payable.
d. Cash inflow equal to the cash received and a cash outflow equal to the cash paid.
(AICPA)

39. APERCU OUTLINE Co. purchased a three-month Treasury bill. How should this
purchase be reported in APERCU's statement of cash flows?
a. As an outflow from operating activities.
b. As an outflow from investing activities.
c. As an outflow from financing activities.
d. Not reported in activities.
(AICPA)

40. Which of the following is not disclosed on the statement of cash flows when prepared
under the direct method, either on the face of the statement or in a separate schedule?
a. The major classes of gross cash receipts and gross cash payments.
b. The amount of income taxes paid.
c. A reconciliation of the components of cash and cash equivalents.
d. A reconciliation of ending retained earnings to net cash flow from operations.
(AICPA)

41. Either the direct method or the indirect method of presenting cash flows is used in
presenting cash flows from
a. operating activities only c. financing activities only
b. investing activities only d. any of these activities

42. With regards to the preparation of statement of cash flows using the indirect method,
which of the following statements is correct?
I. Amortization of bond discount on long term debt should be presented in a
statement of cash flows as subtraction from profit.
II. Amortization of bond premium on long term debt should be presented in a
statement of cash flows as subtraction from profit.
III. Amortization of bond discount on a bond investment should be presented in a
statement of cash flows as subtraction to from profit.
IV. Amortization of bond premium on a bond investment should be presented in a
statement of cash flows as subtraction to from profit
a. I and III b. I and IV c. II and III d. II and IV

43. During the year, REVERIE DAYDREAM Co.’s unamortized bond discount account
decreased by P30,000. If REVERIE prepares its statement of cash flows using the
indirect method, how is the change in unamortized bond discount shown in the
statement of cash flows?
a. As an addition to profit in the operating activities section.

133
b. As a subtraction from profit in the operating activities section.
c. As a financing cash inflow.
d. As a financing cash outflow.

44. Which of the following financial statements readily shows information on whether the
reporting entity obtained cash financing during the period by issuing debt of equity
instruments?
a. Statement of financial position
b. Statement of cash flows.
c. Statement of changes in equity.
d. Statement of profit or loss and other comprehensive income

45. During the current year, INCIPIENCY BEGINNING Co. amortized a bond discount.
INCIPIENCY prepares its statement of cash flows using the indirect method. In which
section of the statement should INCIPIENCY report the amortization of the bond
discount?
a. Financing activities. c. Investing activities.
b. Operating activities. d. Cash equivalents.
(AICPA)

46. Which of the following items is specifically included in the body of a statement of cash
flows?
a. Operating and non-operating cash flow information
b. Conversion of debt to equity
c. Acquiring an asset through a finance lease
d. Purchasing a building by giving a mortgage to the seller.
(Adapted)

47. On the face of the statement of cash flows, all of the following would be specifically
identified as cash in(out)flows from investing activities except for:
a. purchase of fixed assets. c. gain on sale of equipment.
b. sale of fixed assets. d. none

48. The statement of cash flows provides us with important clues on all of the following
except:
a. Financial practices of management.
b. Projected changes for the following year in current liability accounts.
c. Feasibility of financing capital expenditures.
d. Ability to meet debt service requirements.
(Adapted)

49. The primary purpose of the statement of cash flows is to


a. provide information about a company's cash receipts and cash payments during the
accounting period
b. measure the change in the company's assets
c. state the company's financial position at period-end
d. analyze net income during the accounting period
(Adapted)

50. The amortization of a copyright may be presented in


a. operating activity. c. an investing activity.
b. financing activity. d. none of these.
(Adapted)

51. Purchasing treasury share for cash is a(n)


a. operating activity. c. investing activity.
b. financing activity. d. none of these.

134
(Adapted)

52. In using a work sheet approach to preparing a statement of cash flows, after all the
changes in the balance sheet accounts have been explained and entered on the work
sheet, the
a. cash account is adjusted to an accrual basis.
b. change in the Cash account is also explained.
c. changes in the income statement accounts are also explained.
d. general journal entries are prepared.
(Adapted)

53. Which of the following items would not be found on a statement of cash flows prepared
using the direct method?
a. Interest Received c. Interest Paid
b. Profit d. Issue of Bonds Payable for Cash
(Adapted)

54. Declaring and paying a cash dividend is normally a(n)


a. operating activity. c. investing activity.
b. financing activity. d. none of these.
(Adapted)

55. Purchasing a new plant by issuing a mortgage is a(n)


a. operating activity. c. investing activity.
b. financing activity. d. none of these.
(Adapted)

56. Receiving dividends on securities held is normally a(n)


a. operating activity. c. investing activity.
b. financing activity. d. none of these.
(Adapted)

57. Purchasing long-term investments for cash is


a. operating activity. c. investing activity.
b. financing activity. d. none of these.
(Adapted)

58. Purchasing land for cash is a(n)


a. operating activity. c. investing activity.
b. financing activity. d. none of these.
(Adapted)

59. An entity needs to purchase 20 warehouses in various states. To finance the cash
purchases, it issues new shares in a seasoned equity offering. It also issues high coupon
bonds on which interest must be paid semi-annually. If the entity purchases the
warehouses at the beginning of the year, which of the cash flows will be affected in the
year-end statements?
Investing Operating Financing
I. affected affected affected
II. unaffected affected affected
III. affected unaffected affected
IV. unaffected unaffected unaffected

a. I. b. II. c. IV. d. III.


(Adapted)

60. Which of the following does not affect an entity’s cash flows:

135
a. warranty expenses.
b. sale of an "impaired" asset.
c. purchase of a trademark financed by stock issuance.
d. change of depreciation method.
(Adapted)

61. Which of the following statements is (are) correct?


I. PAS 7 allows nonmonetary exchanges of non-cash assets to be included in the
statement of cash flows if the exchange is with commercial substance.
II. PAS 7 allows nonmonetary exchanges of non-cash assets to be included in the
statement of cash flows if the exchange is without commercial substance.
a. I only b. II only c. I and II d. Neither I nor II

62. Which of the following best describes a statement of cash flows?


a. A statement of cash flows details the cash inflows and outflows that are related to a
company's operating, investing, and financing activities over a period of time.
b. A statement of cash flows summarizes the financial position of a company at a given
point in time.
c. A statement of cash flows measures a company's financial performance over a
specified period of time.
d. A statement of cash flows reports changes over a period of time in component
accounts that comprise the ownership of a firm.
(Adapted)

63. Which of the following is an operating cash flow for a financial institution?
a. Dividends paid out to shareholders.
b. Dividends received on investments in FVOCI.
c. Cash used to replace the machine tool used in operational activities.
d. Cash proceeds from sale of investment in FVPL.

64. Which of the following is/are correct?


I. The cash flow from operations are higher when expenses are capitalized.
II. Total cash flows are higher when the expenses are capitalized.
III. Capitalization of interest costs leads to higher net income.
IV. Capitalization of interest costs is not allowed under GAAP.
a. II, III & IV b. I, II, III & IV c. III & IV d. I & III
(Adapted)

65. When preparing a statement of cash flow, one of the relationships between balance
sheet changes and cash flows can be summarized as
a. decreases in liabilities represent net cash inflows.
b. increases in liabilities represent net cash outflows.
c. increases in assets represent net cash outflows.
d. decreases in assets represent net cash outflows.
(Adapted)

66. All of the following statements are true except for:


a. When inventory balances decrease, cash flows increase.
b. When short-term debt increases, cash flows increase.
c. When accounts receivable balances increase, cash flows increase.
d. When accounts payable balances increase, cash flows increase.
(Adapted)

67. Which of the following is not a financing activity in the statement of cash flows of a
financial institution?
a. repurchase of ordinary shares c. payment of interest on debt
b. issuance of new debt d. cash dividend

136
(Adapted)

68. Which of the following statements about the cash flow statement is/are true?
I. The cash flow statement provides information on the liquidity of the firm.
II. The income statement is more susceptible to management manipulation than the
cash flow statement.
III. The cash flow statement serves as a check on the inherent assumptions of the
income statement.
IV. The cash flow statement is based on actual events while the income statement is
based on the allocation of the effects of these events over time.

a. I & III b. I, II, III & IV c. I, II & III d. II & IV


(Adapted)

Use the following information for the next two questions:


EURYTHMIC Co. owns 2% of HARMONIOUS Co. A property dividend by HARMONIOUS
consisted of merchandise with a fair value lower than the listed retail price. EURYTHMIC in
turn gave the merchandise to its employees as a holiday bonus.

69. How should EURYTHMIC report the receipt and distribution of the merchandise in its
income statement?
a. At fair value for both dividend revenue and employee compensation expense.
b. At listed retail price for both dividend revenue and employee compensation
expense.
c. At fair value for dividend revenue and listed retail price for employee compensation
expense.
d. By disclosure only.
(AICPA)

70. How should EURYTHMIC report the receipt and distribution of the merchandise in its
statement of cash flows?
a. As both an inflow and outflow for operating activities.
b. As both an inflow and outflow for investing activities.
c. As an inflow for investing activities and outflow for operating activities.
d. As a noncash activity.
(AICPA)

71. Which of the following should not be disclosed in an enterprise's statement of cash
flows prepared using the indirect method?
a. Interest paid, net of amounts capitalized.
b. Income taxes paid.
c. Cash flow per share.
d. Dividends paid on preferred share.
(AICPA)

72. The primary purpose of a statement of cash flows is to provide relevant information
about:
a. Differences between net income and associated cash receipts and disbursements.
b. An entity's ability to generate future positive net cash flows.
c. The cash receipts and cash disbursements of an entity during a period.
d. An entity's ability to meet cash operating needs.
(AICPA)

73. In a statement of cash flows, if used equipment is sold at a gain, the amount shown as a
cash inflow from investing activities equals the carrying amount of the equipment:
a. Plus the gain.
b. Plus the gain and less the amount of tax attributable to the gain.

137
c. Plus both the gain and the amount of tax attributable to the gain.
d. With no addition or subtraction.
(AICPA)

74. On July 1, 20x1, EXPATIATE WANDER Co. signed a 20-year building lease that it
reported as a finance lease. EXPATIATE paid the monthly lease payments when due.
How should EXPATIATE report the effect of the lease payments in the financing
activities section of its 20x1 statement of cash flows?
a. An inflow equal to the present value of future lease payments at July 1, 20x1, less
20x1 principal and interest payments.
b. An outflow equal to the 20x1 principal and interest payments on the lease.
c. An outflow equal to the 20x1 principal payments only.
d. The lease payments should not be reported in the financing activities section.
(AICPA)

75. MISSIVE LETTER Co. prepares its statement of cash flows using the indirect method.
MISSIVE's unamortized bond premium account decreased by ₱25,000 during the year.
How should MISSIVE report the change in unamortized premium discount in its
statement of cash flows?
a. As a financing cash inflow.
b. As a financing cash outflow.
c. As an addition to net income in the operating activities section.
d. As a subtraction from net income in the operating activities section.
(AICPA)

76. Which of the following items is included in the financing activities section of the
statement of cash flows?
a. Cash effects of transactions involving making and collecting loans.
b. Cash effects of acquiring and disposing of investments and property, plant, and
equipment.
c. Cash effects of transactions obtaining resources from owners and providing them
with a return on their investment.
d. Cash effects of transactions that enter into the determination of net income.
(AICPA)

77. Which combination below explains the impact of credit card interest incurred and paid
during the period on (1) equity on the balance sheet and (2) the statement of cash
flows? (Item #1) Effect on Equity on Balance Sheet; (Item #2) Reflected on Statement
Cash Flows as a(n)
a. Decrease, Investing outflow
b. Decrease, Operating or financing outflow
c. No effect, Financing or investing outflow
d. No effect, Operating outflow
(Adapted)

78. A reader of a statement of cash flows wishes to analyze the major classes of gross cash
receipts and tress cash payments from operating activities. Which methods of reporting
cash flows from operating activities will supply that information?
a. Both the direct and indirect methods c. Only the indirect method
b. Only the direct method d. Neither method.

79. In the statement of cash flows, the payment of cash dividends appears in the <List A>
activities section as a <List B> of cash.
List A List B List A List B
a. Operating or investing Source c. Investing or financing Use
b. Operating or financing Use d. Investing Source
(Adapted)

138
80. In the determination of cost of goods sold, <List A> must be <List B> cash payments for
goods along with other adjustments.
List A List B
a. An increase in accounts payable Added to
b. A decrease in accounts payable Added to
c. An increase in inventory Added to
d. A decrease in inventory Subtracted from
(Adapted)

81. In reconciling net profit on an accrual basis to net cash from operating activities, what
adjustments are needed to net profit because of (1) an increase during the period in
prepaid expenses and (2) the periodic amortization of premium on bonds payable?
(Item #1) Increase in Prepaid Expenses; (Item #2) Amortization of Premium on Bonds
Payable
a. Add, Add b. Add, Deduct c. Deduct, Add d. Deduct, Deduct
(Adapted)

82. Which of the following would least likely to be shown in one of the three activity
sections of the statement of cash flows?
a. Refinancing a bond issue currently due with a new bond issue
b. An increase in trade accounts receivable over the period
c. Purchase of a subsidiary corporation
d. A decrease in long-term notes payable over the period
(Adapted)

83. Which of the following items is included in the adjustment of net income to obtain cash
flow from operating activities?
a. Depreciation expense for the period
b. The change in deferred taxes
c. The amount by which equity income recognized exceeds cash received
d. All of these
(Adapted)

84. Which statement is true for gains and losses from capital asset sales?
a. They do not affect cash and are excluded from the statement of cash flows
b. There are included in cash flows from operating activities
c. There are included in cash flows from investing activities
d. There are included in cash flows from financing activities
(Adapted)

85. Why has cash flow from operations become increasingly important as an analytical
tool?
a. Inflation has distorted the meaningfulness of net income
b. High interest rates can put the cost of borrowing to cover short term cash needs out
of reach for many firms
c. Firms may have uncollected accounts receivable and unsalable inventory on the
books
d. All of these
(AICPA)

86. Which of the following could lead to cash flow problems?


a. Obsolete inventory, accounts receivable of inferior quality, easing of credit by
suppliers
b. Slow-moving inventory, accounts receivable of inferior quality, tightening of credit
by suppliers
c. Obsolete inventory, increasing notes payable, easing of credit of suppliers

139
d. Obsolete inventory, improved quality of accounts receivable, easing of credit by
suppliers
(AICPA)

87. In a statement of cash flows (using indirect approach for operating activities) an
increase in inventories should be presented as a(n)
a. Outflow of cash. c. Addition to net income.
b. Inflow and outflow of cash. d. Deduction from net income.

88. Would the following be added back to profit when reporting operating activities’ cash
flows by the indirect method? (Item #1) Excess of treasury share reacquisition cost
over reissuance proceeds; (Item #2) Bond payable discount amortization
a. Yes, Yes b. No, No c. No, Yes d. Yes, No

89. Using the indirect method, which of the following may be deducted from profit in
determining the net cash flow from operating activities?
a. decrease in accounts receivable
b. increase in accounts payable
c. amortization of discount on investment in bonds
d. compensation expense under an equity-settled compensation plan

90. Under the indirect method, which of the following is added to profit before tax to
determine the net cash flows from operating activities?
a. Increase in bond payable discount c. Collection of receivables
b. Increase in deferred tax liabilities d. Increase in accounts payable

91. Which of the following is true?


I. Cash flow presentation is optional in the presentation of financial statements
II. Equity investments are normally excluded as cash equivalent unless they are in
substance cash equivalents
a. I is true b. II is true c. I and II are true d. I and II are not true

92. Select the incorrect statement.


a. Bank overdrafts which are repayable on demand and which form an integral part of
an entity’s cash management are also included as a component of cash and cash
equivalents.
b. Cash and cash equivalents comprises cash on hand and demand deposits, together
with short-term, highly liquid investments that are readily convertible to a known
amount of cash, and that are subject to an insignificant risk of changes in value.
c. The indirect method of presenting cash flows shows the major class of gross cash
receipts and gross cash payments
d. The direct method of presentation for operating cash flows is encouraged but the
indirect method is also acceptable

93. Which of the following may be presented as an investing cash flow?


(Item #1) Dividends received; (Item #2) Interest received; (Item #3) Transaction taxes
paid
a. Yes, Yes, Yes b. Yes, No, No c. Yes, Yes, No d. No, No, No

94. Which of the following is presented as a deduction in determining net cash flows from
investing operation? (Item #1) Increase in accounts receivable; (Item #2) Loss in
disposal of equipment; (Item #3) Gain on sale of land; (Item #4) Payment for land
improvement
a. Yes, No, Yes, No c. No, No, Yes, No
b. Yes, Yes, No, Yes d. No, No, No, Yes

95. Select the incorrect statement.

140
a. Cash flows of foreign subsidiaries should be translated at the exchange rates
prevailing when the cash flows took place
b. The exchange rate used for translation of transactions denominated in a foreign
currency and the cash flows of a foreign subsidiary should be the rate in effect at the
date of the cash flows
c. Investing activities which do not require the use of cash should be excluded from
the cash flow statements.
d. Investing activities which do not require the use of cash should be included in the
cash flow statement

96. MITIGATE TO EASE Corp. has an investment in a joint venture, ALLEVIATE Corp. Which
of the following appropriately describe MITIGATE’s treatment of the relevant cash
flows from ALLEVIATE Corp?
a. When equity method is used, the cash flow statement should report only a
proportionate share in the cash flows of the joint venture
b. When proportionate consolidation is used, the cash flow statement should report
only cash flows between the investor and investee
c. When equity method is used, the cash flow statement should report only cash flows
between the investor and investee
d. None of these

97. The following may be presented on a net basis, except


a. cash flows from acquisitions and disposals of subsidiaries and other business units
b. cash receipts and payments for items in which the turnover is quick, the amounts
are large, and the maturities are short (generally less than three months)
c. cash receipts and payments relating to fixed maturity deposits
d. cash advances and loans made to customers and repayment thereof

98. MOROSE GLOOMY Bank classifies cash advances and loans from other financial
institution as operating activities. Which of the following statement in relation to this is
the most correct?
a. the practice is an alternative to presenting the same transaction as financing activity
b. the practice is unacceptable since all items affecting equity should be financing
activities
c. the practice is proper since these items relates to the primary revenue generating
activities of MOROSE
d. None of the above.

99. Advances made by an entity to its associate are appropriately included in which activity
classification in its cash flow statement?
a. operating activities c. financing activities
b. investing activities d. either a or b

100. Select the incorrect statement.


a. interest and dividends received or paid are usually classified as investing and
financing cash flows for a financial institution
b. dividends paid may be classified as a financing cash flow because they are cost of
obtaining financial resources from the owners
c. interest paid and interest and dividends received may be classified as operating
cash flows because they enter into the determination of profit or loss
d. interest paid and interest and dividends received may be classified as financing cash
flows and investing cash flows respectively because they are costs of obtaining
financial resources or returns on investments

101. An entity purchases a building and the seller accepts payment partly in equity
shares and partly in debentures of the entity. This transaction should be treated in the
cash flow statement as follows:

141
a. The purchase of the building should be investing cash outflow and the issuance of
shares and the debentures financing cash outflows.
b. The purchase of the building should be investing cash outflow and the issuance of
debentures financing cash outflows while the issuance of shares investing cash
outflow.
c. This does not belong in a cash flow statement and should be disclosed only in the
notes.
d. Ignore the transaction totally since it is a noncash transaction. No mention is
required in either the cash flow statement or anywhere else in the financial
statements.
(Adapted)

102. An entity (other than a financial institution) receives dividends from its investment
in shares. How should it disclose the dividends received in the cash flow statement
prepared under PAS 7?
a. Operating cash inflow.
b. Either as operating cash inflow or as investing cash inflow.
c. Either as operating cash inflow or as financing cash inflow.
d. As an adjustment in the “operating activities” section of the cash flow because it is
included in the net income for the year and as a cash inflow in the “financing
activities” section of the cash flow statement.
(Adapted)

103. How should gain on sale of an office building owned by the entity be presented in a
cash flow statement?
a. As an inflow in the investing activities section of the cash flow because it pertains to
a long-term asset.
b. As an inflow in the “financing activities” section of the cash flow statement because
the building was constructed with a long-term loan from a bank that needs to be
repaid from the sale proceeds.
c. As an adjustment to the net income in the “operating activities” section of the cash
flow statement prepared under the indirect method.
d. Added to the sale proceeds and presented in the “investing activities” section of the
cash flow statement.
(Adapted)

104. How should an unrealized gain on the translation of a foreign currency denominated
transaction be presented in a cash flow statement?
a. As an inflow in the “financing activities” section of the cash flow statement because
it arises from a foreign currency translation.
b. It should be ignored for the purposes of the cash flow statement as it is an
unrealized gain.
c. It should be ignored for the purposes of the cash flow statement as it is an
unrealized gain but it should be disclosed in the footnotes to the financial
statements by way of abundant precaution.
d. As an adjustment to the net income in the “operating activities” section of the
statement of cash flows.
(Adapted)

105. How should repayment of a long-term loan comprising repayment of the principal
amount and interest due to date on the loan be treated in a cash flow statement?
a. The repayment of the principal portion of the loan is a cash flow belonging in the
“financing activities” section; the interest payment belongs either in the “operating
activities” section or the “financing activities” section.
b. The repayment of the principal portion of the loan is a cash flow belonging in the
“investing activities” section; the interest payment belongs either in the “operating
activities” section or the “investing activities” section.

142
c. The repayment of the principal portion of the loan is a cash flow belonging in the
“investing activities” section; the interest payment belongs in the “operating
activities” section (because PAS 7 does not permit any alternatives in case of
interest payments).
d. The repayment of the principal portion of the loan is a cash flow belonging in the
“investing activities” section; the interest payment should be netted against interest
received on bank deposits, and the net amount of interest should be disclosed in the
“operating activities” section.
(Adapted)

106. Which of the following transactions would be presented in a statement of cash


flows, according to PAS 7 Statement of cash flows?
I. Conversion of loans into shares
II. Loan interest received
III. Loan interest owed
IV. Proceeds of loan issue
a. I and II b. II and IV c. II, III, and IV d. all of these
(ACCA)

107. Which of the following items should be presented under Cash flows from investing
activities, according to PAS 7 Statement of cash flows?
a. Employee costs c. Redemption of debentures
b. Property revaluation d. Development costs capitalized in the period
(ACCA)

108. Which of the following items should be presented under Cash flows from financing
activities, according to PAS 7 Statement of cash flows?
a. Employee costs c. Redemption of debentures
b. Property revaluation d. Development costs capitalized in the period
(ACCA)

109. Which of the following items should be presented under Cash flows from operating
activities, according to PAS7 Statement of cash flows?
a. Employee costs c. Redemption of debentures
b. Property revaluation d. Development costs capitalized in the period
(ACCA)

110. Which of the following can be classified as Cash and cash equivalents under PAS 7
Statement of cash flows?
I. Redeemable preference shares due in 180 days
II. Loan notes held due for repayment in 90 days
III. Equity investments
IV. A bank overdraft
a. I and II b. II and IV c. II, III, and IV d. none of these
(ACCA)

111. In accordance with PAS 7 Statement of cash flows, and treating it as a nonrecurring
event, which classification of the cash flow arising from the proceeds from an
earthquake disaster settlement would be most appropriate?
a. Cash flows from operating activities
b. Cash flows from investing activities
c. Cash flows from financing activities
d. Does not appear in the cash flow statement
(ACCA)

143
112. In accordance with PAS 7 Statement of cash flows, and treating it as a nonrecurring
event, which classification of the cash flow arising from the proceeds of sale of a
subsidiary would be most appropriate?
a. Cash flows from operating activities
b. Cash flows from investing activities
c. Cash flows from financing activities
d. Does not appear in the cash flow statement
(ACCA)

113. In accordance with PAS 7 Statement of cash flows, and treating it as a nonrecurring
event, which classification of the cash flow arising from the disposal proceeds of a
major item of plant would be most appropriate?
a. Cash flows from operating activities
b. Cash flows from investing activities
c. Cash flows from financing activities
d. Does not appear in the cash flow statement
(ACCA)

114. According to PFRS classification of cash flows, all of the following are financing cash
flows except:
a. share repurchase. c. losses on debt retirement.
b. interest accrued on new debt. d. dividends paid out.
(Adapted)

115. The aggregate cash flows arising from acquisitions and from disposals of
subsidiaries or other business units resulting to loss or obtaining of control are
presented separately and classified as
a. Operating activities c. Financing activities
b. Investing activities d. Disclosed only

116. Cash flows arising from changes in ownership interests in a subsidiary that do not
result in a loss of control are classified as cash flows from
a. Operating activities c. Financing activities
b. Investing activities d. Disclosed only

117. On which of the following statements may the amount of an entity’s profit for the
year appear?
I. statement of financial position
II. separate income statement
III. single statement of profit or loss and other comprehensive income
IV. statement of changes in equity
V. statement of cash flows
a. all of these c. I, II, III, and IV
b. II and III d. II, III, IV, and V

118. The financial statement that properly describes all important aspects of the
company’s financing and investing activities
a. statement of changes in retained earnings c. income statement
b. statement of financial position d. cash flow statement
(Adapted)

119. Interest expense paid to finance the acquisition of a qualifying asset is presented in
the statement of cash flows as
a. operating activities c. investing activities
b. financing activities d. not reported

144
120. PROSELYTE CONVERT Company decided to dispose some of its non-current assets
during 1987. The assets were sold for ₱400,000. The same assets were acquired at an
original cost of ₱900,000 with an accumulated depreciation at the date of sale of
₱650,000. The sale of the non-current assets of PROSELYTE should be shown on the
statement of cash flows as:
a. a subtraction from net income of ₱150,000 and a source of ₱400,000
b. a subtraction from net income of ₱150,000 and a source of ₱250,000
c. an addition to net income of ₱150,000 and a source of ₱250,000
d. a source of ₱250,000
(RPCPA)

121. Which of the following is a source of working capital?


a. sale of equipment at a loss c. declaration of stock dividend
b. net loss from operations d. impairment of goodwill
(RPCPA)

122. Sources and uses of funds that are stated separately from funds provided from or
used in the operations
a. proceeds from the sale of long-term debt
b. redemption and repayment of long-term debt
c. issue of shares for cash and other assets
d. all of the above
(RPCPA)

123. The declaration of a 10% stock dividend has to be presented in a statement of cash
flows as (Item #1) Source of funds; (Item #2) Use of funds
a. No, Yes b. Yes, No c. No, No d. Yes, Yes
(RPCPA)

124. Which of the following is incorrect about the statement of cash flows?
a. it is the 5th basic financial statement
b. it provides information about cash receipts and cash payments of an entity during
the period
c. it provides a general analysis of changes in the cash account during the year
d. it provides information about the investing and financing activities of the business
(Adapted)

125. In determining the cash provided by operations it is incorrect to


a. eliminate noncash revenues from net income
b. eliminate noncash expenses from net income
c. include the issuance of the company’s bonds for cash
d. convert accrual basis profit to cash basis profit
(Adapted)

126. Which of the following statements is incorrect?


a. A statement of cash flow is required as part of a complete set of financial statements
prepared in conformity with Philippine Financial Reporting Standards (PFRS).
b. Entities are encouraged to report cash flows from operating activities using the
direct method.
c. Although the classification of cash flows into the three main categories is important,
classification guidelines are arbitrary.
d. An entity acquires equipment by paying down payment equal to 10% of the contract
price, issuing ordinary shares for 30% of the contract price and issuing note payable
for the 70% balance. The transaction price of the equipment is presented as an
outflow in the investing activities section of a statement of cash flows.

127. Which of the following statements is (are) correct?

145
I. Issues may arise because there is no standard definition of operating activities. The
IASB has taken the position that operating activities are not investing or financing
activities. At the same time the opinion that the association of a cash flow with profit
is the primary criterion for classifying the flow as operating is expressed.
II. An entity borrows funds from a bank, the proceeds from which were used to finance
the construction of a building. The transaction shall be included only in the financing
activities of the statement of cash flows.
a. I only b. II only c. I and II d. Neither I nor II

128. Which of the following statements is (are) correct?


I. An entity declares cash dividends on December 1, 20x1 to shareholders on record as
of December 31, 20x1 to be distributed on February 1, 20x2. This transaction shall
be included in the financing activities of the entity’s statement of cash flows for
20x1, if the entity is a financial institution.
II. An entity declares cash dividends on December 1, 20x1 to shareholders on record as
of December 31, 20x1 to be distributed on February 1, 20x2. This transaction shall
be included in the financing activities of the entity’s statement of cash flows for
20x1, if the entity is not a financial institution.
a. I only b. II only c. I and II d. Neither I nor II

129. Which of the following are considered benefits of cash flow information?
I. In conjunction with the rest of the financial statements, provides information that
enable users to evaluate the changes in net assets of an entity, its financial structure
and its ability to affect the amounts and timing of cash flows in order to adopt to
changing circumstances and opportunities
II. Useful in assessing the ability of the entity to generate cash and cash equivalent and
enables users to develop models to assess and compare the present value of the
future cash flows of different entities
III. Enhances the comparability of the reporting of operating performance by different
entities because it eliminates the effects of using different accounting treatments for
the same transactions and events
IV. Useful in checking the accuracy of past assessment of future cash flows and in
examining the relationship between profitability and net cash flows and the impact
of changing prices
a. I, II, III b. I, II c. I, III d. all of these

130. Which of the following statements is (are) correct?


I. The schedule for noncash investing and financing transactions might include line
items for depreciation, depletion, and amortization recorded during the period.
II. Free cash flow does not include a deduction for dividends because dividend
payment is never required.
III. A net positive figure for cash flows from investing activities implies that the
business is generally expanding.
IV. Cash obtained from bank borrowing is generally considered a financing activity,
whether the debt is classified as short-term or long-term.
V. Management may use the statement of cash flows to assess the liquidity of the
business.
a. I, II, and III b. IV and V c. I, III, IV, and V d. all of these
(Adapted)

131. Which of the following statements is (are) correct?


I. The profit for the period would appear in the cash flows from financing activities
section of the statement of cash flows prepared using the indirect method.
II. In a statement of cash flows work sheet, Accumulated Depreciation is listed not with
assets but with credits.
III. The statement of cash flows discloses significant events related to the operating,
investing, and financing activities of a business.

146
IV. Payment on an account payable is considered a financing activity.
V. The cash flow statement cannot be manipulated as easily as the income statement
since it is based on actual cash flows while income statement has to follow the
accrual principle, which allocates non-cash expenses and revenues to the current
and future periods, leaving open the avenue for earnings manipulation. For this
reason, the cash flow statement acts as a check on the income statement.
a. II, III, and V b. I, II, III, and V c. III only d. all of these

132. Are the following statements true or false?


I. Funds provided from or used in the operations of an entity should be shown
separately in the statement of cash flows, to show the extent to which an entity has
generated funds from or used funds in its operations during the period.
II. Amortization of bond discount on long term debt should be presented in a
statement of cash flows as addition to profit.
a. True, True b. False, True c. True, False d. False, False
(Adapted)

133. Which of the following are non-cash transactions that require disclosure under PAS
7?
a. the acquisition of assets either by assuming directly related liabilities or by means of
a finance lease
b. the acquisition of an entity by means of an equity issue
c. the conversion of debt to equity.
d. all of these

134. Are the following statements true or false?


I. The conversion of preference shares into ordinary shares should be presented in a
statement of cash flows as a source and use of funds.
II. The conversion of debt to equity should be presented in a statement of cash flows as
a source and use of funds.
a. True, True b. False, True c. True, False d. False, False

135. An entity’s operating cash flow is overstated by 90. Its non-cash expenses were
correctly stated, noncash revenues were overstated by 55 and a total of 35 went into
reducing outstanding current debt. The firm's tax rate is 40%. Then, the firm's net
income is
a. overstated by 55 c. understated by 55
b. overstated by 20 d. correctly stated
(Adapted)

136. WARY CAUTIOUS, Inc.'s first year financial statements have overstated accounts
payable by 400 and understated the accounts receivables by 225. Which of the
following is/are true?
I. Its profit is correctly stated if nominal accounts are unaffected.
II. Its profit is overstated by 175 if nominal accounts are affected.
III. Its working capital is overstated by 175.
IV. Its operating cash flows are overstated by 175.
a. II, III and IV b. I & III c. I & IV d. I, II & IV
(Adapted)

137. DESCRY DISCOVER Co.'s accountant has mistakenly understated depreciation by


125 and understated accounts payables by 65. DESCRY's tax rate is 40%. Which of the
following is/are true?
I. Assets are overstated by 125
II. Income is overstated by 125
III. Current liabilities are understated by 65
IV. Operating cash flow is understated by 125

147
a. III only b. I & III c. II, III & IV d. none of them
(Adapted)

138. A change in depreciation method during the year


a. affects cash flows from operating activities if the changed depreciation method
affects taxation.
b. affects cash flows from operating activities if the changed deprecation method
increased the depreciation expense for the period.
c. affects cash flows from operating activities only if the change is accounted for by
prospective application.
d. all of these.
e. does not affect cash flows from operating activities because depreciation is a non-
cash item.

139. Using the indirect method, which of the following may be added to profit in
determining the net cash flow from operating activities?
a. decrease in accounts payable
b. increase in accounts receivable
c. amortization of discount on investment in bonds
d. compensation expense under an equity-settled compensation plan

140. Which of the following is/are false?


I. Depreciation does not affect cash flows since it is a non-cash expense.
II. Depreciation never affects the preparation of statements of cash flows since it is an
allocation of past investing expense.
III. Under PAS 16, entities can use different methods of depreciation for financial
reporting and tax purposes every year.
a. III only b. I & II c. II & III d. I, II & III

Chapter 41 - Suggested answers to review theory questions


1. C 21. D 41. A 61. D 81. D 101. C 121. A
2. B 22. D 42. C 62. A 82. D 102. B 122. D
3. C 23. D 43. A 63. D 83. D 103. C 123. C
4. A 24. D 44. B 64. D 84. C 104. D 124. A
5. C 25. D 45. B 65. C 85. D 105. A 125. C
6. D 26. D 46. A 66. C 86. B 106. B 126. D
7. D 27. D 47. C 67. C 87. D 107. D 127. A
8. D 28. C 48. B 68. B 88. C 108. C 128. D
9. D 29. A 49. A 69. A 89. C 109. A 129. D
10. D 30. D 50. A 70. D 90. D 110. B 130. B
11. A 31. C 51. B 71. C 91. B 111. A 131. A
12. A 32. A 52. B 72. C 92. C 112. B 132. A
13. B 33. C 53. B 73. A 93. A 113. B 133. D
14. C 34. C 54. B 74. C 94. D 114. B 134. D
15. D 35. A 55. D 75. D 95. D 115. B 135. D
16. C 36. D 56. A 76. C 96. C 116. C 136. C
17. C 37. A 57. C 77. B 97. A 117. D 137. B
18. A 38. D 58. C 78. B 98. C 118. D 138. A
19. D 39. D 59. A 79. B 99. B 119. C 139. D
20. B 40. D 60. C 80. A 100. A 120. A 140. C

148
Chapter 42
Events after the Reporting Period

Chapter 42: Multiple choice – Computational (For classroom instruction purposes)


Identifying adjusting and non-adjusting events
1. TRIBULATION GREAT DISTRESS Co.’s current reporting period ends on December 31,
20x1. The following transactions occurred after the end of reporting period:
 On January 5, 20x2, TRIBULATION declared ₱8,000,000 dividends.
 On January 15, 20x2, TRIBULATION issued 1,000 shares with par value per share of
₱400 for ₱2,400 per share.
 On January 20, 20x2, TRIBULATION installed an oil rig. Current legislation requires that
the oil rig be uninstalled at the end of its useful life and the site where it was installed
be restored. TRIBULATION estimates the present value of the decommissioning and
restoration cost at ₱4,000,000.
 On February 1, 20x2, a building with a carrying amount as of December 31, 20x1 of
₱2,000,000 was totally razed by fire.
 On February 10, 20x2, TRIBULATION received notice of a litigation in relation to an
accident that happened on December 31, 20x1. TRIBULATION estimates a probable loss
of ₱800,000.
 On March 5, 20x2, TRIBULATION purchased a subsidiary for ₱40,000,000 in a business
combination accounted for using the acquisition method. Goodwill of ₱10,000,000 was
recognized on the business combination.

The financial statements were authorized for issue on March 1, 20x2.

What is the total amount of the adjusting events?


a. 6,800,000 b. 800,000 c. 4,800,000 d. 30,000,000

Accounting for adjusting events


2. UNCORK RELEASE Co.’s current reporting period ends on December 31, 20x1. The
following transactions occurred after the end of reporting period:
 On January 20, 20x2, a pending litigation was resolved requiring a settlement amount of
₱400,000. The 20x1 year-end financial statements included a provision for loss on
litigation of ₱480,000.
 Inventories costing ₱4,000,000 were recognized at their net realizable value of
₱3,600,000 in the 20x1 year-end financial statements. During January 20x2, the
inventories were sold for ₱3,520,000. Actual selling costs amounted to ₱120,000.
 The year-end accounts receivable include a ₱400,000 receivable from RELINQUISH, Inc.
No allowance for doubtful accounts was recognized on this receivable as of December
31, 20x1. On February 3, 20x2, RELINQUISH filed for bankruptcy. It was estimated that
the receivable will not be collected.
 The fair value of financial assets measured at fair value through profit or loss
significantly declined to ₱320,000 on February 28, 20x2. The financial assets are
recognized in the 20x1 year-end financial statements at ₱1,200,000 which is their fair
value as of December 31, 20x1.
 On March 5, 20x2, a case was resolved requiring a settlement amount of ₱800,000. The
20x1 year-end financial statements included a provision for loss on litigation of
₱600,000.

UNCORK Co.’s profit for the year ended December 31, 20x1 before consideration of the
above transactions is ₱8,800,000. The financial statements were authorized for issue on
March 1, 20x2.

149
How much is the adjusted profit?
a. 8,820,000 b. 9,020,000 c. 10,820,000 d. 8,280,000

The answers and solutions to the computational problems above


(Multiple choice – Computational (SET B) can be found in the
accompanying Teacher’s Manual.

Chapter 42: Theory of Accounts Reviewer


1. Which of the following statements is incorrect?
a. Events after the reporting period are those that occur between the end of reporting
period and the date the financial statements are authorized by an entity’s
management for issue.
b. Adjusting events are those that provide conditions that existed at the end of
reporting period. The financial statements are adjusted for these events.
c. Non-adjusting events are those that provide conditions that arose after the end of
reporting period. If relevant to users, these events are disclosed in the financial
statements.
d. The date the financial statements are authorized for issue is the date when an
entity’s management authorizes the financial statements for issue, if such
authorization is subject to further approval by other parties, the date the financial
statements are authorized for issue is the date the financial statements are
approved by other parties.

2. This refers to those events, favorable or unfavorable, that occur between the end of the
reporting period and the date that the financial statements are authorized for issue.
a. Events after the reporting period
b. Type I events
c. Adjusting events after the reporting period
d. Non-adjusting events after the reporting period

3. Under PAS 10, these refer to those events that provide evidence of conditions that
existed at the reporting period.
a. Events after the reporting period
b. Type I events
c. Adjusting events after the reporting period
d. Non-adjusting events after the reporting period

4. Under PAS 10, these refer to those that are indicative of conditions that arose after the
reporting period.
a. Events after the reporting period
b. Type I events
c. Adjusting events after the reporting period
d. Non-adjusting events after the reporting period

5. Which of the following statements is correct?


a. An entity should prepare its financial statements on a going concern basis even if
events after the reporting period indicate that the going concern assumption is not
appropriate.
b. When an entity’s management is required to submit its financial statements to its
shareholders for approval after the financial statements have been issued, for
purposes of determining the events after the reporting period, the financial
statements are deemed authorized for issue on the date when shareholders approve
the financial statements.
c. When an entity’s management is required to issue its financial statements to a
supervisory board (made up solely of non-executives) for approval, for purposes of
determining the events after the reporting period, the financial statements are

150
deemed authorized for issue when the supervisory board approves the financial
statements.
d. Events after the reporting period include all events up to the date when the financial
statements are authorized for issue, even if those events occur after the public
announcement of profit or of other selected financial information.

6. The management of an entity completes draft financial statements for the year to
December 31, 20x1 on February 28, 20x2. On March 18, 20X2, the board of directors
reviews the financial statements and authorizes them for issue. The entity announces
its profit and selected other financial information on March 19, 20x2. The financial
statements are made available to shareholders and others on April 1, 20x2. The
shareholders approve the financial statements at their annual meeting on May 15, 20x2
and the approved financial statements are then filed with a regulatory body on May 17,
20x2. For purposes of PAS 10 Events after the reporting period, the financial statements
are authorized for issue on
a. March 18, 20x2 d. May 15, 20x2
b. March 19, 20x2 e. May 17, 20x2
c. April 1, 20x2
(Adapted)

7. On March 18, 20x2, the management of an entity authorizes financial statements for
issue to its supervisory board. The supervisory board is made up solely of non-
executives and may include representatives of employees and other outside interests.
The supervisory board approves the financial statements on March 26, 20x2. The
financial statements are made available to shareholders and others on April 1, 20x2.
The shareholders approve the financial statements at their annual meeting on May 15,
20x2 and the financial statements are then filed with a regulatory body on May 17,
20x2. For purposes of PAS 10 Events After the Reporting Period, the financial statements
are authorized for issue on
a. March 18, 20x2 d. May 15, 20x2
b. March 19, 20x2 e. May 17, 20x2
c. April 1, 20x2
(Adapted)

8. The following are adjusting events after the reporting period.


I. The settlement after the reporting period of a court case that confirms that the entity
had a present obligation at the reporting period.
II. The receipt of information after the reporting period indicating that an asset was
impaired at the reporting period, or that the amount of a previously recognized
impairment loss for that asset needs to be adjusted
III. The determination after the reporting period of the cost of assets purchased, or the
proceeds from assets sold, before the reporting period.
IV. The determination after the reporting period of the amount of profit-sharing or bonus
payments, if the entity had a present legal or constructive obligation at the reporting
period to make such payments as a result of events before that date.
V. The determination after the reporting period that estimates made as of the reporting
period such as the expected useful lives of depreciable assets were inappropriate due
to sudden technological advances happening after the reporting period but before
their authorization for issue.
VI. The discovery of fraud or errors that show that the financial statements are incorrect.
a. I, II, III, and V c. I, III, IV, and V
b. I, II, III, IV, V, and VI d. I, II, III, IV, and VI

9. Which of the following is an adjusting event after the reporting period?


a. The bankruptcy of a customer that occurs after the reporting period due to a
casualty that happened after the reporting period but before the authorization of
the financial statements for issue.

151
b. The sale of inventories after the reporting period giving evidence that the net
realizable value assigned to the inventories at the reporting period were
inappropriate.
c. A decline in the value of held for trading securities was charged as loss as of the
reporting period. Subsequent to year-end but before the financial statements were
authorized for issue, the market price of the investment tripled.
d. The value of a financial asset through other comprehensive income was materially
overstated due to an unexpected shift in the stock market from bullish to bearish
after the reporting period.
e. An entity declares dividends to holders of equity instruments after the reporting
period.

10. After the reporting period but before the completion of financial statements, MOTLEY
MIXTURE Company is contemplating whether to present its financial statements as
going concern or as non-going concern. Which of the following situations would
preclude MOTLEY from issuing financial statements presented on a going concern
basis?
I. MOTLEY’s management determines after the reporting period that it intends to
liquidate the entity.
II. MOTLEY’s management determines after the reporting period that it will cease
trading.
III. MOTLEY’s management determines after the reporting period that it has no realistic
alternative but to cease trading and liquidate the entity.
IV. Deterioration in operating results and financial position after the reporting period.
a. III b. I, II, c. I, II, III d. I, II, III, IV

11. Which of the following statements is incorrect?


a. The process involved in authorizing the financial statements for issue will vary
depending upon the management structure, statutory requirements and procedures
followed in preparing and finalizing the financial statements.
b. An entity shall disclose the date when the financial statements were authorized for
issue and who gave that authorization. If the entity’s owners or others have the
power to amend the financial statements after issue, the entity shall disclose that
fact.
c. It is important for users to know when the financial statements were authorized for
issue, because the financial statements do not reflect events after this date.
d. If an entity receives information after the reporting period about conditions that
existed at the reporting period, it shall update disclosures that relate to those
conditions, in the light of the new information.
e. A disclosure regarding a contingent liability that existed at the reporting period
need not be updated when further information or evidence becomes available after
the reporting period.

12. Which of the following are examples of non-adjusting events after the reporting period
that would not result in disclosure?
I. A major business combination after the reporting period or disposing of a major
subsidiary
II. Announcing a plan to discontinue an operation;
III. Death of member of top management or a member of the board of directors
IV. Major purchases of assets, classification of assets as held for sale in accordance with
PFRS 5 Non-current Assets Held for Sale and Discontinued Operations, other disposals
of assets, or expropriation of major assets by government
V. The destruction of a major production plant by a fire after the reporting period
VI. Purchases or disposals of the entity’s own equity instruments by beneficial owners
VII. Announcing, or commencing the implementation of, a major restructuring
VIII. Major ordinary share transactions and potential ordinary share transactions after
the reporting period

152
IX. Abnormally large changes after the reporting period in asset prices or foreign
exchange rates
X. Changes in tax rates or tax laws enacted or announced after the reporting period
that have a significant effect on current and deferred tax assets and liabilities
XI. Entering into significant commitments or contingent liabilities, for example, by
issuing significant guarantees
XII. Commencing major litigation arising solely out of events that occurred after the
reporting period.
a. III b. VIII c. III, VI d. all of these should be disclosed

13. PAS 10 Events after the Financial Reporting Period defines the extent to which events
after the balance sheet date should be reflected in financial statements. Five such events
are listed below.
1. Merger with another company.
2. Insolvency of a customer.
3. Destruction of a major non-current asset.
4. Sale of inventory held at the balance sheet date for less than cost.
5. Discovery of fraud.

Which of the listed items are, according to PAS 10, normally to be classified as adjusting?
a. 1, 2 and 3 b. 2, 4 and 5 c. 1, 2 and 5 d. 1, 4 and 5
(Adapted)

14. VOCATION OCCUPATION Company decided to operate a new amusement park that will
cost ₱1 million to build in the year 20x1. Its financial year-end is December 31, 20x1.
VOCATION has applied for a letter of guarantee for ₱700,000. The letter of guarantee
was issued on March 31, 20x2. The audited financial statements have been authorized
to be issued on April 18, 20x2. The adjustment required to be made to the financial
statement for the year ended December 31, 20x1, should be
a. Booking a ₱700,000 long-term payable.
b. Disclosing ₱700,000 as a contingent liability in 2005 financial statement.
c. Increasing the contingency reserve by ₱700,000.
d. Do nothing.
(Adapted)

15. A new drug was introduced by OBNOXIOUS HARMFUL Inc. in the market on December
1, 20x1. OBNOXIOUS’ financial year ends on December 31, 20x1. It was the only
company that was permitted to manufacture this patented drug. The drug is used by
patients suffering from an irregular heartbeat. On March 31, 20x2, after the drug was
introduced, more than 1,000 patients died. After a series of investigations, authorities
discovered that when this drug was simultaneously used with another drug used to
regulate hypertension, the patient’s blood would clot and the patient suffered a stroke.
A lawsuit for ₱100,000,000 has been filed against OBNOXIOUS Inc. The financial
statements were authorized for issuance on April 30, 20x2. Which of the following
options is the appropriate accounting treatment for this post–balance sheet event
under PAS 10?
a. The entity should provide ₱100,000,000 because this is an “adjusting event” and the
financial statements were authorized to be issued after the accident.
b. The entity should disclose ₱100,000,000 as a contingent liability because it is an
“adjusting event.”
c. The entity should disclose ₱100,000,000 as a “contingent liability” because it is a
present obligation with an improbable outflow.
d. Assuming the probability of the lawsuit being decided against OBNOXIOUS Inc. is
remote, the entity should disclose it in the footnotes, because it is a nonadjusting
material event.
(Adapted)

153
16. At the balance sheet date, December 31, 2005, BELIE Inc. carried a receivable from TO
DISGUISE Corporation, a major customer, at ₱10 million. The “authorization date” of the
financial statements is on February 16, 2006. TO DISGUISE Corporation declared
bankruptcy on Valentine’s Day (February 14, 2006). BELIE Inc. will
a. Disclose the fact that TO DISGUISE Corporation has declared bankruptcy in the
notes.
b. Make a provision for this post–balance sheet event in its financial statements (as
opposed to disclosure in notes).
c. Ignore the event and wait for the outcome of the bankruptcy because the event took
place after the year-end.
d. Reverse the sale pertaining to this receivable in the comparatives for the prior
period and treat this as an “error” under PAS 8.
(Adapted)

17. TITTILLATE TO TICKLE Inc. built a new factory building during 20x1 at a cost of ₱20
million. At December 31, 20x1, the net book value of the building was ₱19 million.
Subsequent to year-end, on March 15, 20x2, the building was destroyed by fire and the
claim against the insurance company proved futile because the cause of the fire was
negligence on the part of the caretaker of the building. If the date of authorization of the
financial statements for the year ended December 31, 20x1, was March 31, 20x2,
TITTILLATE. should
a. Write off the net book value to its scrap value because the insurance claim would
not fetch any compensation.
b. Make a provision for one-half of the net book value of the building.
c. Make a provision for three-fourths of the net book value of the building based on
prudence.
d. Disclose this nonadjusting event in the footnotes.
(Adapted)

18. DERELICT VAGRANT BUM Inc. deals extensively with foreign entities, and its financial
statements reflect these foreign currency transactions. Subsequent to the balance sheet
date, and before the “date of authorization” of the issuance of the financial statements,
there were abnormal fluctuations in foreign currency rates. DERELICT Inc. should
a. Adjust the foreign exchange year-end balances to reflect the abnormal adverse
fluctuations in foreign exchange rates.
b. Adjust the foreign exchange year-end balances to reflect all the abnormal
fluctuations in foreign exchange rates (and not just adverse movements).
c. Disclose the post–balance sheet event in footnotes as a nonadjusting event.
d. Ignore the post–balance sheet event.
(Adapted)

19. Under PAS 10, the following are subsequent events that require disclosure in the
financial statements but do not result in adjustments:
I. sales of a bond or capital stock issue
II. purchase of a business
III. settlement of litigation when the event giving rise to the claim took place
subsequent to the reporting period
IV. loss of a plant or inventories as a result of fire or flood
V. bankruptcy of a customer that occurs after the reporting period that confirms that a
loss existed at the end of the reporting period on a trade receivable
a. I, II, III, IV, and V c. II, III, IV and V only
b. I, II, III, and IV only d. II, III and IV only

20. According to PAS 10, a subsequent event which usually requires adjustment of financial
statements is:
a. settlement of litigation when the event giving rise to the claim took place
subsequent to the balance sheet date

154
b. decline in market value of investment between the balance sheet date and date of
which financial statements are issued
c. loss on trade receivable which is confirmed by the bankruptcy of a customer
occurring after the reporting period
d. loss of inventories as a result of flood or fire

21. Subsequent events are those which occur between the balance sheet date and the date
on which the financial statements are issued. Which of the following subsequent events
requires adjustment (as opposed to mere disclosure) in the financial statements?
a. material decline in market value of inventory
b. loss of plant as a result of fire
c. material write-off of receivables resulting from customer’s major casualty after the
balance sheet date.
d. payment to BIR of disputed income tax assessment of previous years’ profit.

22. Which of the following subsequent events requires disclosure and adjustment of the
financial statements?
a. loss of plant due to fire
b. sales of a bond issue
c. loss on a receivable due to bankruptcy of a customer
d. purchase of a business

23. On January 12, 20x2, a fire at a production facility of DUB TO NAME Company damaged
a number of adjacent buildings (owned by other businesses). DUB’s insurance policy
does not cover damage to the property of others. Insurance companies for those other
businesses have billed DUB Company for the estimated cost of ₱2.4 million required to
restore the damaged buildings. DUB’s legal counsel believed that it is reasonably
possible that DUB will be held liable for damages but was uncertain as to the amount. In
its 20x1 financial statements authorized for issue on February 1, 20x2, DUB Company
should report
a. An accrued liability of ₱2,400,000
b. An accrued liability of ₱2,400,000 and necessary disclosure in the notes to financial
statements
c. Only a note disclosure is required
d. No information about the damaged buildings
(Adapted)

24. NOTION Co. guaranteed a loan of ₱200,000 granted to IDEA Co. by a bank. At the time
when the financial statements of NOTION are being finished, it is clear that IDEA is in
financial difficulties and it is probable that NOTION will meet the guarantee. In the
financial statements, NOTION should
a. Only disclose in the notes the amount of the guarantee
b. Recognize a provision for liability of ₱200,000
c. Not recognize and need not disclose the guarantee
d. Recognize a provision for liability of ₱200,000 and also disclose in the notes to
financial statements.
(Adapted)

25. On February 12, 20x2, a fire at a production facility of PRUDENT WISE Company
damaged a number of adjacent buildings (owned by other businesses). PRUDENT’s
insurance policy does not cover damage to the property of others. Insurance companies
for those other businesses have billed PRUDENT Company for the estimated cost of
₱2.4 million required to restore the damaged buildings. DUB’s legal counsel believed
that it is probable that DUB will be held liable for damages. In its 20x1 financial
statements authorized for issue on February 1, 20x2, PRUDENT Company should report
a. An accrued liability of ₱2,400,000

155
b. An accrued liability of ₱2,400,000 and necessary disclosure in the notes to financial
statements
c. Only a note disclosure is required
d. No information about the damaged buildings
(Adapted)

Chapter 42 - Suggested answers to theory of accounts questions


1. D 6. A 11. E 16. B 21. D
2. A 7. A 12. C 17. D 22. C
3. C 8. D 13. B 18. C 23. C
4. D 9. B 14. D 19. B 24. D
5. D 10. C 15. C 20. C 25. D

156
Chapter 43
Related Party Disclosures
Chapter 43: Multiple choice – Computational (For classroom instruction purposes)
Separate and Consolidated financial statements
The following relates to the transactions of GRIMACE FROWN Co. during 20x1:
Directors' and officers' remuneration 8,000,000
Post-employment benefits of officers 800,000
Fringe benefits in the form of housing assistance to
directors and officers 20,000,000
Share options granted to officers 1,200,000
Officers' expenses on travels, representation and
entertainment subject to liquidation and
reimbursement 400,000
Loans to directors and officers 12,000,000
Sales to related entities 40,000,000

1. How much is the amount of related party disclosures on GRIMACE’s separate financial
statements?
a. 30,000,000 b. 52,000,000 c. 82,000,000 d. 42,000,000

2. How much is the amount of related party disclosures on GRIMACE’s consolidated


financial statements?
a. 12,000,000 b. 30,000,000 c. 82,000,000 d. 42,000,000

The answers and solutions to the computational problems above


(Multiple choice – Computational (SET B) can be found in the
accompanying Teacher’s Manual.

Chapter 43: Theory of Accounts Reviewer


1. Which of the following statements is incorrect?
a. Related party transactions and outstanding balances with other entities in a
group are disclosed in an entity’s financial statements.
b. Intragroup related party transactions and outstanding balances are not
eliminated in the preparation of consolidated financial statements of the group.
c. Related party relationships are a normal feature of commerce and business.
d. A related party relationship could have an effect on the profit or loss and
financial position of an entity.
e. Knowledge of related party transactions, outstanding balances and relationships
may affect assessments of an entity’s operations by users of financial statements,
including assessments of the risks and opportunities facing the entity.

2. An entity’s ability to affect the financial and operating policies of an investee is through
the presence of
I. Control
II. Joint control
III. Significant influence
a. I only b. I or III c. Any of I, II, or III d. I, II and III

3. Which of the following statements is correct?


I. The profit or loss and financial position of an entity may be affected by a related
party relationship even if related party transactions do not occur.
II. The mere existence of the relationship may be sufficient to affect the transactions of
the entity with other parties.

157
a. True, true b. True, false c. False, false d. False, true

4. All of the parties enumerated are related to an entity , except


a. the entity is a subsidiary, an associate, or a venture in a joint venture.
b. the party is a member of the key management personnel of the entity or its parent.
c. the party is a close member of the family of an individual having control, significant
influence, or joint control over the entity or a member of the key management
personnel of the entity or its parent.
d. the party is a post-employment benefit plan for the benefit of employees of the
entity, or of any entity that is a related party of the entity.
e. two entities simply because they have a director or other member of key
management personnel in common

5. Which of the parties enumerated is related to an entity?


a. two entities simply because they have a director or key manager in common;
b. two venturers who share joint control over a joint venture
c. providers of finance, trade unions, public utilities, government departments and
agencies in the course of their normal dealings with an enterprise
d. a single customer, supplier, franchiser, distributor, or general agent with whom an
enterprise transacts a significant volume of business merely by virtue of the
resulting economic dependence.
e. a subsidiary or parent, an associate, or a venturer of the entity

6. According to PAS 24 Related Party Disclosures, which of the following is not a related
party of PEREGRINE WANDERING Company?
a. A shareholder of PEREGRINE Company owning 30% of the ordinary share capital
b. An entity providing banking facilities to PEREGRINE Company
c. An associate of PEREGRINE Company
d. Key management personnel of PEREGRINE Company
(ACCA)

7. According to PAS 24, this refers to a transfer of resources, services or obligations


between related parties, regardless of whether a price is charged.
a. Inter-company Transfer c. Departmental Transfer
b. Inter-company Transaction d. Related party transaction

8. Which of the following may fall within the close members of the family of an individual
(as provided in PAS 24) who may be expected to influence, or be influenced by, that
individual in their dealings with the entity?
a. the individual’s wife and their children
b. the individual’s domestic partner
c. the individual’s domestic partner’s children
d. the individual’s domestic partner’s dependents
e. all of the above

9. Raymunda is a director of and an owner of 25% interest in the NCPAR Company. She
also owns 65% of the PH Care, Inc. and is a director of the Cebu CPAR Corporation.
Raymunda's husband is the sole owner of the Beauty Parlor, Inc. Raymunda's daughter
holds 5% of the shares in the Feminine Products Company. The only involvement she
has in the company is to receive dividends. Which companies would be classified under
PAS 24 Related Party Disclosures as related parties of NCPAR?
I. PH Care, Inc.
II. Cebu CPAR
III. Beauty Parlor, Inc.
IV. Feminine Products Company
a. I and III b. I, II and III c. II and III d. all of these
(ACCA)

158
10.Which of the following statements is incorrect?
a. Control is the power to govern the financial and operating policies of an entity so as
to obtain benefits from its activities.
b. Relationships between parents and subsidiaries shall be disclosed only when there
have been transactions between those related parties.
c. Joint control is the contractually agreed sharing of control over an economic activity.
d. Key management personnel are those persons having authority and responsibility
for planning, directing and controlling the activities of the entity, directly or
indirectly, including any director (whether executive or otherwise) of that entity.
e. Significant influence is the power to participate in the financial and operating policy
decisions of an entity, but is not control over those policies. Significant influence
may be gained by share ownership, statute or agreement.

11.An entity shall disclose the name of the entity’s parent and, if different, the ultimate
controlling party. If neither the entity’s parent nor the ultimate controlling party
produces financial statements available for public use, the name of the next most senior
parent that does so shall also be disclosed. The next most senior parent is
a. the member of the group which has the greatest net assets, provided it is a listed
entity
b. the first parent in the group above the immediate parent that produces consolidated
financial statements available for public use
c. the first parent in the group above the immediate parent that produces consolidated
financial statements or separate financial statements available for public use,
provided that entity meets the required minimum capitalization required by
regulatory agencies such as SEC, BIR, BSP and the Office of the Insurance
Commission (OIC).
d. the member of the group which is a publicly-accountable entity.

12.Which of the following statements is correct?


I. To enable users of financial statements to form a view about the effects of related
party relationships on an entity, it is appropriate to disclose the related party
relationship when control exists, irrespective of whether there have been
transactions between the related parties.
II. If there have been transactions between related parties, an entity shall disclose the
nature of the related party relationship as well as information about the
transactions and outstanding balances necessary for an understanding of the
potential effect of the relationship on the financial statements.
III. Disclosures that related party transactions were made on terms equivalent to those
that prevail in arm’s length transactions are encouraged by GAAP.
IV. State-controlled entities are within the scope of PFRSs, i.e. those that are profit-
oriented are no longer exempted from disclosing transactions with other state-
controlled entities.
a. I, IV b. I, II, IV c. I, III, IV d. I, II, III, IV

13.According to PAS 24 Related Party Disclosures, which of the following fall within the
definition of an entity's related party?
I. Another entity in which the entity owns 5% of the voting rights
II. A post-employment benefit plan for the benefit of the employees of the entity's
parent
III. An executive director of the entity
IV. The partner of a key manager in a major supplier to the entity
a. I, II and III b. II and III c. III only d. II, III, and IV
(ACCA)

14.Which of the following payments by a company should be disclosed in the notes to the
financial statements as a related party transaction?

159
I. Royalties paid to a major shareholder as consideration for patents purchased from
the shareholder.
II. Key officers' salaries.
a. I only. b. II only. c. Both I and II. d. Neither I nor II.
(AICPA)

15.Which of the following is not a related party as envisaged by PAS 24?


a. A director of the entity.
b. The parent company of the entity.
c. A shareholder of the entity that holds 1% stake in the entity.
d. The son of the chief executive officer of the entity.
(Adapted)

16.PAS 24 requires disclosure of compensation of key management personnel. Which of


the following would not be considered “compensation” for this purpose?
a. Short-term benefits.
b. Share-based payments.
c. Termination benefits.
d. Reimbursement of out-of-pocket expenses.
(Adapted)

17.APATHETIC Company has a 70% subsidiary, INDIFFERENT, Inc. and is a venturer in


IMPARTIAL, a joint venture company. During the financial year to December 31, 20x1,
APATHETIC sold goods to both companies. Consolidated financial statements are
prepared combining the financial statements of APATHETIC and INDIFFERENT. Under
PAS 24 Related Party Disclosures, in the separate financial statements of APATHETIC for
20x1, disclosure is required of transactions with
a. neither INDIFFERENT nor IMPARTIAL
b. INDIFFERENT only
c. IMPARTIAL only
d. both INDIFFERENT and IMPARTIAL
(ACCA)

18.To enable financial statement users to form a view about the effects of the related party
transactions, PAS 24 requires certain disclosures to be made. Which of the following
disclosures is not a mandated disclosure under PAS 24?
a. Relationships between parents and subsidiaries irrespective of whether there have
been transactions between those related parties.
b. Names of all the “associates” that an entity has dealt with during the year.
c. Name of the entity’s parent and, if different, the ultimate controlling party.
d. If neither the entity’s parent nor its ultimate controlling entity produces financial
statements available for public use, then the name of the next most senior parent
that does so.
(Adapted)

19.Are the following statements in relation to related parties true or false, according to
PAS 24 Related Party Disclosures?
I. A party is related to another entity that it has jointly control.
II. A party is related to another entity that it controls.
a. False, False b. False, True c. True, False d. True, True
(ACCA)

20.If there have been related party transactions during the year, an entity needs to make,
at a minimum, certain disclosures. Which of the following is not a required minimum
disclosure under PAS 24?
a. The amount of the related party transactions.

160
b. The amount of the outstanding related party balances and their terms and
conditions along with details of guarantees given and received.
c. The amounts of similar transactions with unrelated (third) parties to establish that
comparable related party transactions have been entered at arm’s length.
d. Provisions for doubtful debts related to the amount of outstanding related party
balances and expense recognized during the year in respect of bad or doubtful debts
due from related parties.
(Adapted)

21.Are the following statements in relation to compensation true or false, according to PAS
24 Related Party Disclosures?
I. Compensation includes social security contributions paid by the entity.
II. Compensation includes post-employment benefits paid on behalf of a parent of the
entity in respect of the entity.
a. False, False b. False, True c. True, False d. True, True
(ACCA)

22.The minimum disclosures prescribed under PAS 24 are to be made separately for
certain categories of related parties. Which of the following is not among the list of
categories specified under the Standard for the purposes of separate disclosure?
a. Entities with joint control or significant influence over the entity.
b. The parent company of the entity.
c. An entity that has a common director with the entity.
d. Joint ventures in which the entity is a venturer.
(Adapted)

23.CUPIDITY GREED Company completed the following transactions in the year to


December 31, 20x1:
I. Sold a car for P9,250 to the uncle of CUPIDITY's finance director.
II. Sold goods to the value of P12,400 to AVARICE, a company owned by the daughter
of CUPIDITY's managing director. AVARICE has no other connection with CUPIDITY.

Which transactions, if any, require disclosure in the financial statements of CUPIDITY


under PAS 24 Related Party Disclosures?
a. Neither transaction c. Transaction (2) only
b. Transaction (1) only d. Both transactions
(ACCA)

24.STEAD Company has a wholly-owned subsidiary, AVAILABLE TO HELP Corporation.


During the year to June 30, 20x1, STEAD sold goods to AVAILABLE totaling ₱250,000.
AVAILABLE paid ₱135,000 of this debt before year-end and then encountered financial
difficulties. AVAILABLE is not expected to be able to pay the remainder of the balance
and therefore it has been provided against as uncollectible. Administration costs
incurred as a result of STEAD 's credit controllers chasing the debt by AVAILABLE have
been calculated as ₱600. Under the minimum disclosure requirements of PAS 24
Related Party Disclosures, which of the following are required to be disclosed in relation
to this arrangement?
I. The costs of the credit control department incurred in pursuing the debt
II. Details of any guarantees received in relation to the outstanding balance
III. The provision in relation to the debt being uncollectible
IV. Future plans regarding trading arrangements with this subsidiary
a. II and III c. II, III and IV
b. I, II and III d. all of these
(ACCA)

25.Which of the following statements is true?

161
I. Disclosures of material related party transactions that are eliminated in the
preparation of consolidated financial statements is required in those consolidated
combined financial statements.
II. An associate’s subsidiary and the investor that has significant influence over the
associate are related to each other.
a. True, False b. False, True c. True, True d. False, False

26.IMMANENT INHERENT Company carried out the following four transactions during the
year ended March 31, 20x1. Which of the following are related party transactions
according to PAS 24 Related Party Disclosures?
I. Transferred goods from inventory to a shareholder owning 40% of the company's
ordinary shares
II. Sold a company car to the wife of the managing director
III. Sold an asset to TRANSCENDENT Company, a sales agent
IV. Took out a ₱1 million bank loan
a. I and II b. I, II and III c. II and III d. all of these
(ACCA)

27.Which of the following related party transactions by a company should be disclosed in


the notes to the financial statements?
I. Payment of per diem expenses to members of the board of directors.
II. Consulting fees paid to a marketing research firm, one of whose partners is also a
director of the company.
a. I only. b. II only. c. Both I and II. d. Neither I nor II.
(AICPA)

28.HABILE Co. and SKILLFUL Co. are under the common management of ABLE Co. ABLE
can control the operating results of both HABILE and SKILLFUL. While HABILE had no
transactions with ABLE during the year, SKILLFUL sold merchandise to ABLE under the
same terms given to unrelated parties. In the notes to their respective financial
statements, should HABILE and SKILLFUL disclose their relationship with ABLE?
(Item #1) HABILE; (Item #2) SKILLFUL
a. Yes, Yes b. Yes, No c. No, Yes d. No, No
(AICPA)

29.VERSANT EXPERIENCED Co. has entered into a joint venture with an affiliate to secure
access to additional inventory. Under the joint venture agreement, VERSANT will
purchase the output of the venture at prices negotiated on an arms-length basis. Which
of the following is(are) required to be disclosed about the related party transaction?
I. The amount due to the affiliate at the balance sheet date.
II. The monetary amount of the purchases during the year.
a. I only b. II only c. Both I and II d. Neither I nor II.
(AICPA)

30.Consolidated financial statements shall include disclosures of material transactions


between related parties except
a. Nonmonetary exchanges between the reporting entity and its director.
b. Sales of inventory by a subsidiary to its parent.
c. Expense allowance for executives which exceed normal business practice.
d. A company’s agreement to act as surety for a loan to its chief executive officer.
(AICPA)

31.APATHETIC Company has a 70% subsidiary, INDIFFERENT, Inc. and is a venturer in


IMPARTIAL, a joint venture company. During the financial year to December 31, 20x1,
APATHETIC sold goods to both companies. Consolidated financial statements are
prepared combining the financial statements of APATHETIC and INDIFFERENT. The
investment in IMPARTIAL is accounted for under the equity method. Under PAS 24

162
Related Party Disclosures, in the combined financial statements of APATHETIC for 20x1,
disclosure is required of transactions with
a. neither INDIFFERENT nor IMPARTIAL c. IMPARTIAL only
b. INDIFFERENT only d. both INDIFFERENT and IMPARTIAL
(ACCA)

32.Which of the following statements are true?


I. Rex Corporation has a 25% interest in Darrell Company. Munda, Inc. is a subsidiary
of Darrell. Rex and Munda are related parties.
II. Rhad Corporation is a venturer of Andrix Company. Renante, Inc. is a subsidiary of
Andrix. Rhad and Renanted are related parties.
III. NCPAR Corporation is a venturer of Session Road Joint Venture and a 25% interest
investor of Pelizloy Fourth Floor Company. Session Road and Pelizloy Company are
related parties.
IV. Chowking Company and DBP Company are under the common control of St. Joseph
Corporation. Chowking and DBP are related parties
a. True, True, True, True c. True, False, False, True
b. False, True, False, True d. False, False, False, True

Chapter 43 - Suggested answers to theory of accounts questions


1. B 6. B 11. B 16. D 21. D 26. A 31. C
2. C 7. D 12. B 17. D 22. C 27. B 32. A
3. A 8. E 13. B 18. B 23. C 28. A
4. E 9. A 14. C 19. D 24. A 29. C
5. E 10. B 15. C 20. C 25. C 30. B

163
Chapter 44
Operating Segments

Chapter 44: Multiple choice – Computational (SET B) – (For classroom instruction


purposes)
Quantitative thresholds
1. DEMENTED INSANE Co. is preparing its year-end financial statements and has
identified the following operating segments:
Segments Revenues Profit (loss) Assets
A 4,000,000 800,000 56,000,000
B 4,800,000 560,000 72,000,000
C 1,080,000 (280,000) 48,000,000
D 960,000 (2,800,000) 4,000,000
E 1,160,000 200,000 5,600,000
Totals 12,000,000 (1,520,000) 185,600,000

What are the reportable segments?


a. A, B and D b. A, B, C and D c. A and B d. A, B, C, D and E

Management approach and Aggregation


2. EMBOSOM CHERISH Co. engages in five diversified operations namely, operations A, B,
C, D, and E. Information on these segments are shown below:
Segments Revenues Profit (loss) Assets
A 3,200 800 40,000
B 3,200 400 8,000
C 200 40 4,000
D 600 80 8,000
E 800 280 24,000
Totals 8,000 1,600 84,000

Additional information:
a. For internal reporting purposes, segments A and B are considered as one operating
segment.
b. Segment E is considered as an operating segment for internal decision making
purposes.
c. Segments C and D have similar economic characteristics and share a majority of the
aggregation criteria.

What are the reportable segments?


a. A, B, C, D and E
b. A, B and E
c. A and B as one segment and E
d. A and B as one segment, E, and C and D as one segment

Limit on external revenue


3. SORDID DIRTY Co. is preparing its year-end financial statements and has identified the
following operating segments:
Inter-
External segment Total
Segments revenues revenues revenues Profit Assets
A 4,800,000 2,400,000 7,200,000 2,800,000 48,000,000
B 1,600,000 400,000 2,000,000 1,600,000 28,000,000
C 1,000,000 - 1,000,000 400,000 4,000,000
D 800,000 - 800,000 320,000 3,200,000

164
E 600,000 - 600,000 280,000 2,800,000
F 400,000 - 400,000 200,000 2,000,000
Totals 9,200,000 2,800,000 12,000,000 5,600,000 88,000,000

Management believes that between segments C, D, E and F, segment C is most relevant to


external users of financial statements.

What are the reportable segments?


a. A and B b. A, B, C and D c. A, B and C d. A, B, C, D, E and F

Major customers
4. RUSTIC RURAL Co. has the following information on its operating segments.
Inter-
External segment Total
Segments revenues revenues revenues Profit Assets
A 4,800,000 2,400,000 7,200,000 2,800,000 48,000,000
B 1,600,000 400,000 2,000,000 1,600,000 28,000,000
C 1,000,000 - 1,000,000 400,000 4,000,000
D 800,000 - 800,000 320,000 3,200,000
E 600,000 - 600,000 280,000 2,800,000
F 400,000 - 400,000 200,000 2,000,000
Totals 9,200,000 2,800,000 12,000,000 5,600,000 88,000,000

RUSTIC Co. shall provide disclosure for major customers if revenues from transactions with
a single external customer amount to how much?
a. 920,000 b. 280,000 c. 1,200,000 d. 560,000

The answers and solutions to the computational problems above


(Multiple choice – Computational (SET B) can be found in the
accompanying Teacher’s Manual.

Chapter 44: Theory of Accounts Reviewer


1. Which of the following statements is incorrect?
a. A “management approach” is used in identifying operating segments.
b. A reportable operating segment is one which management uses in making decisions
about operating matters or results from aggregation of two or more segments and
qualify under any of the quantitative thresholds.
c. Even if an operating segment does not qualify in any of the quantitative thresholds,
such operating segment may still be reportable if management believes that
information about the segment would be useful to users of the financial statements.
d. Disclosures for major customer shall be provided if revenues from transactions with
a single external customer amount to 75% or more of the entity’s external revenues.

2. Operating segments that may be aggregated are those which exhibit similar economic
characteristics and are similar in the following, except
a. the nature of the products and services, their production processes, and distribution
methods
b. the type or class of customer for their products and services
c. their financial position, financial performance, and cash flows
d. regulatory environment

3. According to PFRS 8, the quantitative thresholds are


I. at least 10% of total revenues (external and internal),

165
II. at least 10% of the higher of total profits of segments reporting profits and total
losses of segments reporting losses, in absolute amount (i.e., disregarding negative
amounts.
III. at least 10% of total assets (inclusive of intersegment receivables).
a. I only b. II only c. III only d. I, II and III

4. Disclosures for major customer shall be provided if revenues from transactions with a
single external customer amount to
a. 10% or more of the entity’s external revenues.
b. 10% or more of the entity’s external and internal revenues.
c. 75% or more of the entity’s external revenues.
d. 75% or more of the entity’s external and internal revenues.

5. For segment reporting, interest revenue and interest expense


a. are reported separately for each reportable segment
b. may be presented at net amount if the chief operating decision maker relies
primarily on net interest revenue to assess the performance of the segment
c. are not reported
d. a or b

6. OBLITERATE TO ERASE Co. included interest expense in its determination of segment


profit, which OBLITERATE's chief financial officer considered in determining the
segment's operating budget. OBLITERATE is required to report the segment's financial
data under PFRS 8. Which of the following items should OBLITERATE disclose in
reporting segment data? (Item #1) Interest expense; (Item #2) Segment revenues
a. No, No b. No, Yes c. Yes, No d. Yes, Yes
(AICPA)

7. Which of the following statements is/(are) correct?


I. If an entity that is not required to apply PFRS 8 Operating Segments chooses to
disclose information about segments that does not comply with PFRS 8, it shall not
describe the information as segment information.
II. PFRS 8 does not require an entity to report information that is not prepared for
internal use if the necessary information is not available and the cost to develop it
would be excessive.
III. If a financial report contains both the consolidated financial statements of a parent
that is within the scope of PFRS 8 as well as the parent’s separate financial
statements, segment information is required only the consolidated financial
statements.
IV. PFRS 8 requires an entity to report interest revenue separately from interest
expense for each reportable segment unless a majority of the segment’s revenues
are from interest and the chief operating decision maker relies primarily on net
interest revenue to assess the performance of the segment and to make decisions
about resources to be allocated to the segment. PAS 14, the predecessor of PFRS 8,
did not require such disclosure.
V. Generally, an operating segment has a segment manager who is directly accountable
to and maintains regular contact with the chief operating decision maker to discuss
operating activities, financial results, forecasts, or plans for the segment.
VI. The term “segment manager” identifies a function which should be a segment
manager with a specific title.
a. VI only b. I and II c. all except VI d. all of the statements

8. PFRS 8 aims to help users of financial statements


a. Better understand enterprise performance
b. Better assess its prospects for future net cash flows
c. Make more informed judgments about the entity as a whole
d. all of the choices

166
9. PFRS 8 is required to be applied by
a. entities whose equity securities are traded in a public market and those entities who
are in the process of filing its financial statements with a securities commission or
other regulatory organization for the purpose of issuing any class of instruments in
a public market
b. entities whose debt and equity securities are traded in a public market.
c. entities whose debt and equity securities are traded in a public market and those
entities who are in the process of filing its financial statements with a securities
commission or other regulatory organization for the purpose of issuing any class of
instruments in a public market
d. all entities regardless of whether their securities are being traded or not

10. PFRS 8 Operating Segments is applied in


I. Separate or individual financial statements
II. Consolidated financial statements
a. I only b. I and II c. II only d. neither I nor II

11. Which of the following may not be considered as the chief operating decision maker of
an entity?
a. Chief Executive Officer (CEO) c. Chief Operating Officer (COO)
b. Executive Committee d. Shareholders

12. The following are required under PFRS 8 Operating Segments to disclose segment
information in its financial statements.
I. entities whose equity or debt securities are publicly traded
II. entities that are in the process of issuing equity or debt securities in public
securities markets
III. entities whose securities are not publicly traded or not in the process of issuing
securities to the public.
a. I and II b. I only c. II only d. None of these

13. A non-publicly listed entity may be required to comply with PFRS 8 Operating Segment
if
a. the entity is a subsidiary whose parent is a listed entity or in the process of issuing
securities to the public even in the entity’s individual (separate) financial
statements
b. the entity has a foreign operation
c. at least majority of its revenues comes from intercompany transactions
d. it discloses segment information in its general-purpose financial statements

14. In financial reporting for segments of a business enterprise, which of the following
should be taken into account in computing the amount of an industry segment's
identifiable assets?
(Item #1) Accumulated depreciation; (Item #2) Marketable securities valuation allowance
a. No, No b. No, Yes c. Yes, Yes d. Yes, No
(AICPA)

15. According to PFRS 8, how do firms identify reportable segments?


a. By geographic regions c. By industry classification
b. By product lines d. By designations used inside the firm

16. An entity shall report separately information about each operating segment that:
I. Management deems relevant to external users
II. Meets the quantitative thresholds
a. I b. I or II c. II d. neither I nor II

167
17. Operating segments may be aggregated if
a. they have similar economic characteristics
b. they have different economic characteristics
c. they have the same chief operating decision maker
d. the entity has a matrix organization

18. Which of the following tests may be used to determine if an operating segment of an
entity is a reportable segment under the provision of PFRS 8 regarding quantitative
thresholds?
a. Its revenue (both from external customers and internal segments) is equal to or
greater than 10 percent of total revenue (external and external).
b. The absolute value of its operating profit or loss is equal to or greater than 10
percent of the higher of the total of the operating profit for all segments that
reported profits and the total of the losses for all segments that reported losses.
c. The segment contains 10 percent or more of the combined assets of all operating
segments.
d. All of the above.

19. SUBJUGATE CONQUER Corporation sells 5 different types of products. The company is
divided for internal reporting purposes into 5 different divisions based on these 5
different product lines. The company should prepare the note disclosure for
disaggregated information based upon
a. the 5 types of products.
b. the 5 different divisions.
c. the materialty of each product line based on the revenue or operating profits
generated by each product line or the assets utilized by each product line.
d. the geographic areas in which the 5 products are sold.
(Adapted)

20. Nonreportable segments should


a. be aggregated and reported as “all other segments.”
b. be aggregated but neither reported nor disclosed
c. not reported but may be disclosed if included in the necessary reconciliation of
segment assets, liabilities, or profit or loss
d. not reported but may be disclosed whether or not included in the necessary
reconciliation of segment assets, liabilities, or profit or loss

21. Total external revenue reported by operating segments should


a. at least be 75 per cent of the entity’s revenue
b. not be more than 75% of the entity’s revenue
c. at least be 90% of the entity’s revenue
d. no limit set by PFRS 8

22. If the total external revenue reported by operating segments does not meet the limit
provided under PFRS 8, additional operating segments shall be identified as reportable
segments until the limit is met. Other operating segments may be included as
reportable
a. if they meet all of the quantitative thresholds
b. if they meet any of the quantitative thresholds
c. if they meet at least a majority of the quantitative thresholds
d. even if they do not meet any of the quantitative thresholds

23. Which of the following is not required under current standards for disaggregated
information relating to geographic area information?
a. Revenues from external customers from the home country of the firm and from all
foreign countries in total.

168
b. The total of long-lived assets located in the firm's home country and located in
foreign countries.
c. Operating profits from external customers from the home country of the firm and
from all foreign countries in total.
d. Revenues for any foreign country for which the revenues from that country are
material to the firm.
(AICPA)

24. Entity-wide disclosures include disclosures about


(Item #1) Geographic areas; (Item #2) Allocated costs
a. Yes, Yes b. Yes, No c. No, Yes d. No, No
(AICPA)

25. Entity-wide disclosures are required by publicly held companies with


(Item #1) Only one reportable segment; (Item #2) More than one reportable segment
a. Yes, Yes b. Yes, No c. No, Yes d. No, No
(AICPA)

26. An entity must disclose all of the following about each reportable segment if the
amounts are used by the chief operating decision maker, except
a. Depreciation expense c. Interest expense
b. Allocated expenses d. Income tax expense.
(AICPA)
27. In financial reporting for segments of a business, an enterprise shall disclose all of the
following except
a. Types of products and services from which each reportable segment derives its
revenues.
b. The title of the chief operating decision maker of each reportable segment.
c. Factors used to identify the enterprises reportable segments.
d. The basis of measurement of segment profit or loss and segment assets.
(AICPA)

28. In financial reporting for segments of a business enterprise, segment data may be
aggregated
a. Before performing the 10% tests if a majority of the aggregation criteria are met.
b. If the segments do not meet the 10% tests but meet all of the aggregation criteria.
c. Before performing the 10% tests if all of the aggregation criteria are met.
d. If any one of the aggregation criteria are met.
(AICPA)

29. Under PFRS 8, the method used to determine what information to report for operating
segments is referred to as the
a. Segment approach. c. Dramatic approach.
b. Friendly approach. d. Management approach.

30. FOLLY EVIL Co has two main classes of business: manufacturing of equipment and
servicing of equipment. These classes of business are divided into four divisions: heavy
equipment, light equipment, regular service contracts, and normal warranty. The
management receives segmental information based on these four divisions in its
quarterly internal reports. What should be the basis of FOLLY’s operating segment
reporting for purposes of PFRS 8?
a. The two main classes of business c. The risks and rewards
b. The four divisions d. The quantitative thresholds

31. DEMISE DEATH Corporation operates nationally. DEMISE has a single main product but
also produces different products from the sole production process. DEMISE’s internal
reporting is structured in to two: the main product and all other incidental products.

169
DEMISE proposed to disclose just one operating segment. Can the entity disclose just
one operating segment?
a. Yes, PFRS 8 allows an entity to disclose a single operating segment.
b. Yes, PFRS 8 allows an entity to disclose a single operating segment if no other
operating segments are identified using the “management approach” or no other
operating segments meet any of the quantitative thresholds under PFRS 8 or result
from aggregation.
c. No, PFRS 8 does not allow an entity to disclose just one segment. If there is only one
reportable segment then no segment information should be disclosed.
d. PFRS 8 is silent on this matter.

32. WISTFUL YEARNING Company established a new research and development division
during the year. The division will be financed internally as it will only incur costs but
not generate its own revenues. For internal reporting, this new division is considered a
“cost center.” WISTFUL has three other business segments: perishable goods, canned
goods and packaging. These segments will not receive any apparent benefits from the
new division. Will the new division be disclosed under PFRS 8 as a separate reportable
segment?
a. The new division should be separately reported.
b. The new division should be aggregated with the packaging division.
c. The new division should be aggregated with either or both the perishable goods and
canned goods segment but not the packaging segment.
d. The new division it should not be separately reported or combined with the other
segments but rather included as part of the “all other segments” category.

33. An entity is in the entertainment industry and organizes outdoor concerts in four
different areas of the world: Europe, North America, Australasia, and Japan. The entity
reports to the board of directors on the basis of each of the four regions. The
management accounts show the profitability for each of the four regions, with
allocations for that expenditure which is difficult to directly charge to a region. The
concerts are of two types: popular music and classical music. What is the appropriate
basis for segment reporting in this entity?
a. The segments should be reported by class of business, that is, popular and classical
music.
b. The segments should be reported by region, so Australasia and Japan would be
combined.
c. The segment information should be reported as North America and the rest of the
world.
d. Segment information should be reported for each of the four different regions.
(Adapted)

34. An entity has split its business segments on the basis of the law governing its different
types of business. Two business divisions that the entity has identified are insurance
and banking. Within the banking group, several different services are provided: retail
banking, merchant banking, and small business advisory service. The insurance entities
sell travel insurance, health insurance, and property insurance. The entity operates
throughout the world in several countries and continents. The operating results of each
type of service are regularly reviewed by the entity’s executive committee to make
decisions about resources to be allocated the type of service and assess their
performance. What basis should the entity report its segmental information?
a. On the basis of its business divisions.
b. By geographical location.
c. On the basis of the services it offers within those divisions.
d. The entity should just show one segment, entitled banking and insurance.
(Adapted)

170
35. An entity is engaged in the manufacturing industry and has recently purchased an 80%
holding in a small financial services group. This group does not meet any of the
threshold criteria for a reportable segment. Can the entity disclose the financial services
group as a separate business segment?
a. No, because it does not meet any of the PFRS criteria, it cannot be disclosed as a
separate segment.
b. Yes, even though it does not meet the PFRS criteria, an entity can disclose business
segments separately if they are a distinguishable component.
c. The entity can disclose only 80% of the results and net assets of the banking group.
d. Because of the disparity in types of business, the group should disclose its segmental
information on a geographical basis.
(Adapted)

36. An entity operates in the gas industry and has four different productive processes
within the production cycle. It is essentially a vertically integrated business. The entity
proposes to disclose segmental information regarding each of the four operations. Can
the entity disclose separately as business segments the four operations within the
production cycle?
a. No, it must show a single segment covering all the various operations.
b. PFRS 8 says that it is compulsory to show each different operation separately.
c. PFRS 8 encourages voluntary disclosure of the segments, and it is considered to be
good practice.
d. The entity should group together various operations and show exploration,
production, and chemicals as one segment and retailing as another segment.
(Adapted)

37. An entity manufactures suits, clothing, bed linen, and various cotton and manmade fiber
products. It has several segments, which are reported internally as
Segments Sales Profit Segment assets
Suits 40% 45% 50%
Shirts 30% 35% 33%
Bed linen 15% 10% 7%
Blinds 8% 6% 5%
Cloth 7% 4% 5%
100% 100% 100%

The table represents the percentages of sales, profit, and segment assets that are
attributable to the different segments. The entity wants to present bed linen and cloth as a
single segment but is wondering whether the information can be aggregated. How will the
segmental information be presented in the financial statements?
a. Bed linen and cloth, suits, and shirts, will all be shown as separate segments with
blinds in the other category.
b. All of the segments should be presented separately.
c. Suits, shirts, and bed linen will be separate segments with blinds and cloth shown as
a single segment.
d. Suits and cloth will be one segment with shirts, bed linen, and blinds shown as other
separate segments.
(Adapted)

38. State the correct sequence of identifying and reporting operating segments in
accordance with PFRS 8.
I. Non-reportable segments are combined and disclosed in an “all other segments”
category.
II. The segments are tested under the quantitative thresholds.
III. Operating segments are identified based on management’s internal reporting
system.

171
IV. Segments that have similar economic characteristics and are similar in a majority of
the respects enumerated in PFRS 8 are aggregated and their combined revenues,
profit or loss, and assets are tested under the quantitative thresholds.
V. Segments that have similar economic characteristics and are similar in all of the
respects enumerated in PFRS 8 are aggregated.
VI. The total external revenues of the identified reportable segments are tested under
the 75% limit on external revenues.
VII. Additional operating segments are identified as reportable segments until at least
75% of the entity’s external revenue is included in reportable segments.
a. III, V, IV, II, VI, VII, I c. II, III, V, IV, VI, VII, I
b. III, V, II, IV, VI, VII, I d. III, II, V, IV, VI, VII, I

39. Danggit, a company listed on a recognized stock exchange, reports operating results
from its Cebu activities to its chief operating decision maker. The segment information
for the year is:
Revenue ₱3,675,000
Profit 970,000
Assets 1,700,000
Number of employees 2,500

Danggit 's results for all of its segments in total are:


Revenue ₱39,250,000
Profit 9,600,000
Assets 17,500,000
Number of employees 18,500

According to PFRS 8 Operating Segments, which piece of information determines for


Danggit that the Cebu activities are a reportable segment?
a. Revenue b. Profit c. Assets d. Number of employees
(ACCA)

40. The equity of VAPID DULL Company is traded on a recognized stock exchange. VAPID
regularly reports the financial results of five different business units to its chief
operating decision maker. The relevant revenues for the year ended December 31, 20x1
for these five operations, as a percentage of total external and internal revenue, were as
follows:

Business operation % internal % external % total


1 3 35 38
2 10 14 24
3 15 5 20
4 0 9 9
5 0 9 9
28 72 100

In accordance with PFRS 8 Operating segments, the reportable segments of VAPID are
a. 1 and 2 only c. 1, 2, 3 and 4 only
b. 1, 2 and 3 only d. 1, 2, 3, 4 and 5
(ACCA)

41. Are the following statements true or false, according to PFRS 8 Operating Segments?
I. If an entity changes the way it is structured internally so that its reportable
segments change, the comparative information for earlier periods must be restated.
II. Disclosure is always required of the total assets of each reportable segment.
a. False, False b. False, True c. True, False d. True, True
(ACCA)

172
42. Are the following statements true or false, according to PFRS 8 Operating Segments?
I. The measurement of the profit or loss to be disclosed for each reportable segment is
defined in PFRS8.
II. The profit or loss disclosed for a segment should relate to the total assets attributed
to that segment.
a. False, False b. False, True c. True, False d. True, True
(ACCA)

43. Are the following statements true or false, according to PFRS 8 Operating Segments?
I. A major customer is defined as one providing revenue which amounts to 10% or
more of the combined revenue, internal and external, of all operating segments.
II. The identities of major customers need not be disclosed.
a. False, False b. False, True c. True, False d. True, True
(ACCA)

44. Which one of the following disclosures is not required under PFRS 8?
a. The total amount of revenues from a major external customer (with revenues from
that external customer exceeding 50% of the entity’s revenues).
b. The identity (say, the name) of a major customer that accounts for 20% of the
entity’s revenues.
c. Revenue from external customers attributed to the entity’s country of domicile and
attributed to all foreign countries in total from which the entity derives revenues
(assuming that necessary information is available and the cost to develop it is not
excessive).
d. Revenues from external customers for each product and service, or each group of
similar products and services, unless the necessary information is not available and
the cost to develop it would be excessive.
(Adapted)

45. PFRS 8 requires that an entity should provide reconciliations of segment information to
the entity’s financial information. One of the following reconciliations is not required by
PFRS 8. Which one is it?
a. The total of the reporting segments’ revenues to the entity’s revenues.
b. The total of the reportable segments’ measures of profit or loss to the entity’s profit
or loss before tax expense (tax income) and discontinued operations, and if the
entity allocates to reportable segments items such as tax expense (tax income), the
entity may reconcile the total of the segments’ measures of profit or loss to the
entity’s profit or loss after those items.
c. The total number of major customers of all segments to the total number of major
customers of the entity.
d. The total of the reportable segments’ assets to the entity’s assets.
(Adapted)

46. Which one of the following statements is not true in the context of PFRS 8?
a. The present PFRS on segmental reporting requires entities to report segmental
information using a “management approach” that allows the financial statement
user to review segmental information from the “eyes of the management.”
b. The “core principle” of PFRS 8 requires that an entity should disclose information to
enable users of its financial statements to evaluate the nature and financial effects of
the types of business activities in which it engages and the economic environments
in which it operates.
c. If an entity that is not required to apply PFRS 8 (such as an entity whose equity or
debt is not traded in a public market) but still chooses to disclose information about
segments in its financial statements, it shall not describe the information as segment
information.
d. The present PFRS on segmental reporting requires entities to report segmental
information using a “risks and rewards” approach.

173
(Adapted)

47. Not all operating segments would automatically qualify as reportable segments. PFRS 8
prescribes criteria for an operating segment to qualify as a reportable segment; these
are alternative quantitative thresholds. One of the quantitative thresholds listed below
is not a requirement of PFRS 8. Which one is it?
a. Its reported revenue, from both external customers and intersegment sales or
transfers, is 10% or more of the combined revenue, internal and external, of all
operating segments.
b. The absolute measure of its reported profit or loss is 10% or more of the greater, in
absolute amount, of (1) the combined reported profit of all operating segments that
did not report a loss and (2) the combined reported loss of all operating segments
that reported a loss.
c. Its assets are 10% or more of the combined assets of all operating segments.
d. Its assets are 20% or more of the combined assets of all operating segments.
(Adapted)

48. Which statement is not true with respect to a “chief operating decision maker” as
envisaged by PFRS 8?
a. The term “chief operating decision maker” identifies a function and not necessarily a
manager with a specific title.
b. In some cases the “chief operating decision maker” could be its chief operating
officer.
c. The Board of directors (Board), acting collectively, could qualify as the “chief
operating decision maker.”
d. The chief internal auditor who reports to (and takes directions from) the Board
usually plays a very important role in any organization and would generally qualify
as a “chief operating decision maker.”
(Adapted)

49. Regarding disclosure of major customers, which of the following are in accordance with
the provisions of PFRS 8 Operating Segments?
I. A single external customer is considered a major customer necessitating disclosure
if revenues from transactions with that single external customer amount to 10% or
more of an entity’s revenues.
II. The total amount of revenues from each major customer should be disclosed.
III. The identity of the segment or segments reporting the revenues from major
customers should be disclosed.
IV. The identity of a major customer should be disclosed.
V. The amount of revenues from major customers that each segment reports should be
disclosed.
VI. A group of entities known to a reporting entity to be under common control shall be
considered separate customers, and a government (national, state, provincial,
territorial, local or foreign) and entities known to the reporting entity to be under
the control of that government shall be considered separate customers.
a. I, II, III b. I, II, III, VI c. I, II, III, IV, V d. I, II, III, IV, V

50. Which of the following are included in the disclosures required by PFRS 8 Operating
Segments?
I. revenues from external customers
II. revenues from transactions with other operating segments of the same entity
III. interest revenue
IV. interest expense
V. income tax expense or benefit
a. I and II b. III, IV, and V c. I, II, III, and IV d. all of the choices

174
51. Which of the following are included in the disclosures required by PFRS 8 Operating
Segments?
I. Depreciation and amortization
II. Material items of income and expense disclosed in accordance with PAS 1
III. The entity’s interest in the profit or loss of associates and joint ventures accounted
for by the equity method
IV. Material non-cash items other than depreciation and amortization.
a. I and II b. III, IV, and V c. I, II, and III d. all of the choices

52. PFRS 8 requires the disclosure of information about major customers if revenues from
transactions with a single external customer amount to
a. 10 per cent or more of an entity’s total revenues from external and internal
customers
b. 10 per cent or more of an entity’s total revenues from external customers
c. more than 10 per cent of an entity’s total revenues from external and internal
customers
d. more than 10 per cent of an entity’s total revenues from external customers

53. An operating segment is a component of an entity:


I. that engages in business activities from which it may earn revenues and incur
expenses (including revenues and expenses relating to transactions with other
components of the same entity)
II. whose operating results are regularly reviewed by the entity’s chief operating
decision maker to make decisions about resources to be allocated to the segment
and assess its performance
III. for which discrete financial information is available
a. I only b. II only b. I and II d. all of these

54. Operating segments are identified using a “management approach,” this means that the
identification of operating segments is
a. based on internal reporting. c. based on quantitative computations
b. based on external reporting d. based on qualitative factors

55. An entity may combine information about operating segments that do not meet the
quantitative thresholds with information about other operating segments that do not
meet the quantitative thresholds to produce a reportable segment only if
a. the operating segments have similar economic characteristics and share a majority
of the aggregation criteria
b. the operating segments have similar economic characteristics and share in all of the
aggregation criteria
c. the operating segments have dissimilar economic characteristics and do not share a
majority of the aggregation criteria
d. an entity should not combine information about operating segments after
performing the quantitative thresholds.

56. An entity may combine information about operating segments before performing the
quantitative threshold tests if
a. the operating segments have similar economic characteristics and share a majority
of the aggregation criteria
b. the operating segments have similar economic characteristics and share in all of the
aggregation criteria
c. the operating segments have dissimilar economic characteristics and do not share a
majority of the aggregation criteria
d. an entity should not combine information about operating segments after
performing the quantitative thresholds.

175
57. After identifying operating segments based on internal reporting, management must
decide which of the segments should be reported separately.
a. If two or more of the segments have essentially the same business activities in
essentially the same economic environment, information for these individual
segments may be combined (aggregated).
b. If two or more of the segments have essentially dissimilar business activities in
essentially the same economic environment, information for these individual
segments may be combined (aggregated).
c. If two or more of the segments have essentially the same business activities in
essentially the same economic environment, information for these individual
segments shall be reported separately.
d. a or c

58. A retail chain may have 20 stores that individually meet the definition of an operating
segment but each store is essentially the same.
I. In this case management may desire to combine the 20 stores into one operating
segment.
II. In this case management may desire to combine the 20 stores into one operating
segment described as “all other segments”.
a. I only b. II only c. I and II d. Neither I nor II

59. After identifying reportable segments using “management approach,”


a. additional reportable segments are identified using the quantitative thresholds.
b. conformance with the limit on external revenues is tested.
c. additional reportable segments are identified based on management’s judgment.
d. the remaining segments are described as “all other segments.”

60. If an operating segment does not meet any of the quantitative threshold,
a. it may be combined with other operating segments not meeting the quantitative
threshold and the combined information is treated as reportable if the combined
results meet any of the quantitative thresholds.
b. it may be included in “all other segments”
c. it may nevertheless be reportable if management believes information on this
segment is relevant
d. any of these

61. Assume an entity identified its operating segments as segments A, B, C, D, and E with A
and B as the only reportable segments. The sum of the external revenues of A and B
constitutes less than 75% of the total external revenue of the entity. Which of the
following statements is incorrect?
a. Additional operating segments should be identified as reportable
b. If management judges that of the other remaining segments (C, D and E),
information on segment C will be important to users, then C will be included as a
reportable segment even though it was previously not assessed as reportable.
c. If management judges that C and E should be aggregated, then C and E will be
aggregated and included as additional reportable segment even if their combined
results do not meet any of the quantitative thresholds.
d. Any of these may be acceptable

62. According to PFRS 8, external revenue reported by reportable operating segments must
be at least
a. 75% of the total revenue of the entity including both internal and external revenues
b. 75% of the total external revenue of the entity
c. 10% of the total external revenue of the entity
d. a majority of the total revenue of the entity including both internal and external
revenues

176
63. Two or more operating segments may be aggregated into a single operating segment if
the segments have similar economic characteristics, and the segments are similar in
which of the following respects:
I. the nature of the products and services;
II. the nature of the production processes;
III. the type or class of customer for their products and services;
IV. the methods used to distribute their products or provide their services;
V. if applicable, the nature of the regulatory environment, for example, banking,
insurance or public utilities.
a. I, II, or III only
b. I, II, III, and IV only
c. any of I, II, III, IV, or V
d. all of these before 10% tests; majority of these after 10% tests.

64. According to PFRS 8, there may be a practical limit to the number of reportable
segments that an entity separately discloses beyond which segment information may
become too detailed. Although no precise limit has been determined, if the number of
segments that are reportable in accordance with the quantitative threshold reaches this
number, the entity should consider whether a practical limit has been reached.
a. above ten (10) c. below ten (10)
b. above three (3) d. below three (3)

65. The accounting system that permits management to break down segments of a business
into cost centers and to place accountability on those individuals responsible for the
incurrence of these costs
a. operations research accounting c. control accounting
b. responsibility accounting d. budgetary accounting

Chapter 44 - Suggested answers to theory of accounts questions


1. D 11. D 21. A 31. B 41. D 51. D 61. D
2. C 12. A 22. D 32. D 42. A 52. B 62. B
3. D 13. D 23. C 33. D 43. B 53. D 63. D
4. A 14. C 24. B 34. C 44. B 54. A 64. A
5. D 15. D 25. A 35. B 45. C 55. A 65. B
6. D 16. B 26. B 36. C 46. D 56. B
7. C 17. A 27. B 37. A 47. D 57. A
8. D 18. D 28. C 38. B 48. D 58. A
9. C 19. B 29. D 39. B 49. A 59. A
10. B 20. A 30. B 40. B 50. D 60. D

177
Chapter 45
Interim Financial Reporting

Chapter 45: Multiple choice – Computational (SET B) – (For classroom instruction


purposes)
Recognition and measurement in interim periods
1. QUIRK ACCIDENT Co. reports profit before tax of ₱1,500,000 in its 2 nd quarter interim
financial statements before consideration for the following:
a. Inventory with a carrying amount ₱200,000 has a net realizable value of ₱120,000. It is
expected that the decline in value will reverse in the 3rd quarter.
b. An investment property measured under the cost model has a carrying amount of
₱350,000 but its recoverable amount is ₱210,000.
c. An investment in FVPL measured at acquisition cost of ₱200,000 has a fair value of
₱250,000 as at the end of 2 nd the quarter. However, the increase in fair value is expected
to be only temporary.
d. No depreciation is recognized during the 2nd quarter. The annual straight-line
depreciation of items of PPE is ₱600,000.
e. ABC Co. has a policy of providing 12 days paid vacation leaves for its employees. The
vacation leaves are vesting and accumulating. Total paid vacation leaves eligibility of
employees for the full year is ₱440,000. However, only ₱100,000 worth of paid vacation
leaves have been availed of during the quarter.
f. It was discovered that depreciation in the previous year was overstated by ₱20,000.

How much is the adjusted profit before taxes for the current quarter?
a. 3,180,000 b. 1,590,000 c. 1,070,000 d. 1,090,000

Costs incurred unevenly


2. HIRSUTE HAIRY Co. is preparing its interim financial statements for the period ended
March 31, 20x1. The following relate to the transactions during the first quarter:
a. Total sales for the interim period was ₱4,000,000.
b. Cost of sales was ₱1,800,000.
c. HIRSUTE is liable for 5% commission on its sales to its sales representatives and
agents. No commission has yet been paid as of March 31, 20x1.
d. The allowance for doubtful accounts has a balance of ₱20,000 as of January 1, 20x1.
The required balance as of March 31, 20x1 is ₱60,000. There were no write-offs or
recoveries during the period.
e. A building with historical cost of ₱4,800,000 is being depreciated over 5 years using
straight line method.
f. HIRSUTE prepaid a one-year insurance on its assets for ₱160,000 on January 1,
20x1.
g. Property taxes for 20x1 amounting to ₱104,000 was paid in January.
h. Advertising costs of ₱200,000 were incurred in February on promotional activities
held on Valentine’s Day.
i. Year-end staff bonuses are expected to be around ₱368,000. Employees become
entitled to the bonuses as they provide services to HIRSUTE during the year.
j. HIRSUTE’s president is entitled to a 10% bonus on profit before bonus and taxes.
k. Loss on sale of a used equipment on March 2, 20x1 was ₱120,000.
l. HIRSUTE incurred ₱48,000 on unanticipated repairs on its factory equipment on
March 16, 20x1.
m. Due to the unexpected breakdown of the factory equipment on March 16, 20x1,
HIRSUTE has planned a major periodic overhaul of its other equipment to be held
annually starting on December 31, 20x1. The cost of the major planned periodic
overhaul is estimated at ₱192,000.

178
n. HIRSUTE leases one of its retail stores. Monthly rentals are ₱20,000, however, the
lease contracts provide for a contingent rent equal to 2% of the excess of sales over
₱3,600,000.
o. HIRSUTE’s budget for 20x1 included charitable contributions of ₱96,000 and
employee training costs of ₱52,000. None of those costs were incurred as of March
31, 20x1.
p. Other operating expenses incurred during the first quarter totaled ₱480,000.

How much is the profit (loss) for the first quarter ended March 31, 20x1?
a. 584,100 b. 635,400 c. 646,000 d. 581,400

Costs incurred unevenly


Use the following information for the next two questions:
PLUVIOUS RAINY Co.’s profits before tax for the 1 st and 2nd quarters of 20x1 were
₱3,520,000 and ₱3,680,000 before any necessary adjustments for the items listed below.
 Total unfavorable manufacturing cost variances amounted to ₱96,000 in the 1 st Quarter.
PLUVIOUS expects that the manufacturing cost variances will be absorbed by year-end.
There were no work-in-process inventories as of the end of the 1st and 2nd quarters.
 Newspaper advertisement costs of ₱360,000 were paid on April 1, 20x1. The
advertisement shall appear in the weekly newspaper publications over the remaining
months of the year.
 PLUVIOUS’ held for trading securities acquired on February 4, 20x1 for ₱800,000 had a
fair value of ₱400,000 on March 31, 20x1. PLUVIOUS had expected that the fair value
decline was only temporary. In fact, on June 30, 20x1, the recovery exceeded the
previous write-down in investment by ₱80,000.
 Research and development costs incurred during the 1 st and 2nd quarters totaled
₱40,000 and ₱48,000, respectively. In July 20x1, technical feasibility has been
established and, therefore, development costs of ₱20,000 and ₱28,000 expensed in the
1st and 2nd quarters would have qualified for capitalization.
 On January 20x1, PLUVIOUS recognized an account receivable denominated in US
dollars amounting to $4,000. The exchange rate on that date was ₱40:$1. On March 31,
20x1, the exchange rate was ₱30:$1. PLUVIOUS had expected that the change in the
exchange rate was only temporary. In fact, on June 30, 20x1, the exchange rate was
₱45:$1. The receivable is collectible on September 2, 20x1.
 A land with a carrying amount of ₱800,000 had a recoverable amount of ₱768,000 on
March 31, 20x1.

3. How much is the adjusted profit before tax for the 1st quarter?
a. 2,921,000 b. 2,982,000 c. 2,884,000 d. 2,912,000

4. How much is the adjusted profit before tax for the 2nd quarters?
a. 4,052,000 b. 4,205,000 c. 4,000,000 d. 4,025,000

Provisions – Changes in estimates


Use the following information for the next two questions:
Among the transactions of EFFACE WIPEOUT Co. for the first two quarters of 20x1 were
the following:
 EFFACE recognized a ₱400,000 write-down in its inventory during the first quarter.
EFFACE had expected that the write-down will reverse in the second quarter, and in
fact, in the second quarter, the recovery exceeded the previous write-down by ₱80,000.
 EFFACE provides warranty for its sales. In the first quarter, EFFACE estimated a 5%
warranty obligation on its first quarter sales of ₱4,000,000. In the second quarter, a
change in accounting estimate was made. It was estimated that the cost of warranty
should be 10% of total sales. The second quarter sales amounted to ₱4,800,000.
 EFFACE has been estimating its bad debt expense as 2% of credit sales. However, in the
second quarter, a change was made to the percentage of ending receivable. Under this
method, the required balance of the allowance for doubtful accounts as of June 30, 20x1

179
is computed at ₱120,000. The allowance has a balance of ₱20,000 at the beginning of
the year. Total write-offs during the first six months of 20x1 amounted to ₱48,000;
recoveries totaled ₱12,000. Credit sales for the 1st and 2nd quarters amounted to
₱4,000,000 and ₱8,000,000, respectively.

5. What is the net effect of the transactions listed above on profit or loss before tax in the
first quarter interim financial statements?
a. 680,000 b. (680,000) c. 280,000 d. (280,000)

6. What is the net effect of the transactions listed above on profit or loss before tax in the
second quarter interim financial statements?
a. (336,000) b. 336,000 c. (256,000) d. 256,000

Measurement of interim income tax expense


Use the following information for the next three questions:
BLASé BORED Co. expects to earn ₱400,000 pre-tax profit each quarter. BLASE has tax
rates of 20% on the first ₱800,000 of annual earnings and 30% on all additional earnings.
Actual earnings match expectations.

7. How much is the weighted average annual income tax rate?


a. 20% b. 25% c. 30% d. 33%

8. How much is the income tax expense recognized in the first quarter interim financial
statements?
a. 80,000 b. 100,000 c. 120,000 d. 132,000

9. How much is the income tax expense recognized in the third quarter interim financial
statements?
a. 80,000 b. 100,000 c. 120,000 d. 132,000

Measurement of interim income tax expense


Use the following information for the next three questions:
ASSIDUOUS DILIGENT Company expects to earn ₱40,000 pre-tax profit in each of the 1 st
and 2nd quarters of the year and ₱60,000 pre-tax profit in each of the last two quarters. The
tax rate as of the beginning of the year is 30%. However, following a newly enacted tax
legislation, the tax rate will be increased to 40% which shall take effect beginning on the
last quarter of the year. Actual earnings match expectations.

10. How much is the weighted average annual income tax rate?
a. 20% b. 25% c. 30% d. 33%

11. How much is the income tax expense recognized in the first quarter interim financial
statements?
a. 13,200 b. 16,500 c. 19,800 d. 15,400

12. How much is the income tax expense recognized in the third quarter interim financial
statements?
a. 13,200 b. 16,500 c. 19,800 d. 15,400

Income tax benefit


Use the following information for the next two questions:
AFFRAY ANSWER Company expects to incur losses of ₱60,000 in each of the first and
second quarters of the year but expects to earn pre-tax profits of ₱60,000 in each of the last
two quarters (thus having zero profit for the year). AFFRAY estimated a weighted average
income tax rate of 30%. Actual earnings match expectations.

180
13. How much is the income tax expense recognized in the first quarter interim financial
statements?
a. (18,000) b. 18,000 c. 4,500 d. 0

14. How much is the income tax expense recognized in the third quarter interim financial
statements?
a. (18,000) b. 18,000 c. 4,500 d. 0

Tax year and financial year do not coincide


Use the following information for the next two questions:
INDEMNIFY PAY Co.’s financial reporting year ends June 30 and reports quarterly. Its
taxable year ends December 31. For the financial year that begins July 1, 20x1 and ends
June 30, 20x2, INDEMNIFY earns ₱80,000 pre-tax profit each quarter. The estimated
average annual income tax rate is 30% in 20x1 and 40% in 20x2.

15. How much is the income tax expense recognized in the first quarter interim financial
statements?
a. 24,000 b. (24,000) c. 32,000 d. 28,000

16. How much is the income tax expense recognized in the third quarter interim financial
statements?
a. 24,000 b. (24,000) c. 32,000 d. 28,000

Net operating loss carryforward


Use the following information for the next two questions:
PARSIMONIOUS STINGY Co. reports quarterly and has an operating loss carryforward of
₱40,000 for income tax purposes at the start of the current financial year for which a
deferred tax asset has not been recognized. PARSIMONIOUS earns ₱40,000 in the first
quarter of the current year and expects to earn ₱40,000 in each of the three remaining
quarters. Excluding the carryforward, the estimated average annual income tax rate is
expected to be 40%.

17. How much is the income tax expense recognized in the first quarter interim financial
statements?
a. 9,000 b. 12,000 c. 16,000 d. 0

18. How much is the income tax expense recognized in the third quarter interim financial
statements?
a. 9,000 b. 12,000 c. 16,000 d. 0

Net operating loss carryforward


Use the following information for the next two questions:
STEPWISE GRADUAL Company reported a net operating loss carryforward of ₱80,000 in its
most recent annual financial statements. During the year, STEPWISE earned ₱40,000 in
each of the first two quarters and ₱60,000 in each of the last two quarters. Excluding the
carryforward, the estimated average annual income tax rate is 40%.

19. How much is the income tax expense recognized in the first quarter interim financial
statements?
a. 9,000 b. 9,600 c. 14,400 d. 12,000

20. How much is the income tax expense recognized in the third quarter interim financial
statements?
a. 9,000 b. 9,600 c. 14,400 d. 12,000

181
Restatement of previously reported interim financial reports
21. NONPAREIL UNEQUALLED Company reported profit after tax of ₱400,000 in its March
31, 20x2 interim financial statements. Additional information is shown below (amounts
are net of tax):
 A ₱40,000 cumulative-effect gain resulting from a change in inventory cost flow formula
was recognized in profit or loss during the 1st quarter.
 In March 20x2, a component of an entity was classified as held for sale. Of the total loss
on discontinued operation of ₱48,000, only ₱12,000 has been recognized in the 1 st
quarter. NONPAREIL intends to allocate the remaining ₱36,000 loss to the other the
quarters in 20x2.

How much is the restated profit after tax in the first quarter of 20x1?
a. 400,000 b. 360,000 c. 324,000 d. 364,000

The answers and solutions to the computational problems above


(Multiple choice – Computational (SET B) can be found in the
accompanying Teacher’s Manual.

Chapter 45: Theory of Accounts Reviewer


1. PAS 34 Interim Financial Reporting
a. requires listed entities to provide interim financial reports
b. requires listed entities and those in the process of enlisting their securities to
provide interim financial reports
c. requires listed entities and those in the process of enlisting their securities and non-
SMEs to provide interim financial reports
d. does not state which entities are required to provide interim financial reports.

2. PAS 34 Interim Financial Reporting


a. encourages publicly-listed entities to at least provide semi-annual interim financial
reports and publish them not later than 60 days after the end of the interim period.
b. encourages publicly-listed entities to at least provide quarterly interim financial
reports and publish them not later than 45 days after the end of the interim period.
c. requires publicly-listed entities to at least provide semi-annual interim financial
reports and publish them not later than 60 days after the end of the interim period.
d. requires publicly-listed entities to at least provide quarterly interim financial
reports and publish them not later than 45 days after the end of the interim period.

3. PAS 34 shall be applied by


I. entities which are required by the government or other entities to provide interim
financial reports
II. those who choose to provide interim financial reports.
a. I only b. I and II c. II only d. Neither I nor II

4. Which standard applies to the following? (Item #1) condensed financial statements;
(Item #2) complete set of financial statements?
a. PAS 34, PAS 1 c. PAS 1, PAS 34
b. PAS 24, PAS 1 d. PFRS for SMEs, PAS 1

5. If an entity opts to present complete set of financial statements during an interim


period, it shall apply
a. PAS 34 b. PAS 1 c. PFRS 1 d. PFRS 8

6. Which of the following statements is incorrect?


a. In the interest of relevance and timeliness, less information is normally provided in
a condensed set of interim financial statements.

182
b. Users of interim financial statements are assumed to have access to the most recent
annual financial report.
c. Only information that is significant to the understanding of changes in financial
position and performance of an entity since the end of the last annual reporting are
included in the interim financial report.
d. An entity shall not provide a complete set of financial statements in its interim
financial reporting. Only condensed financial statements shall be provided.

7. Which of the following statements is incorrect?


a. Estimates are used to a greater extent during interim periods as compared to annual
reporting.
b. The comparative interim statement of financial position is dated as of the end of the
immediately preceding annual financial year. The comparative financial statements
for the other financial statements are prepared on a year-to-date basis.
c. Statements of comprehensive income are prepared covering the interim period and
a year-to-date basis; comparative statements of comprehensive income are
provided for similar periods.
d. No additional information is encouraged by PAS 34 if the entity’s business is highly
seasonal.

8. Which of the following statements is incorrect?


a. The same accounting policies are followed during the interim period as those
followed in annual reporting, except for accounting policy changes made after the
date of the most recent annual financial statements that are to be reflected in the
next annual financial statements.
b. Costs that do not benefit other interim periods are recognized in full in the interim
period such costs were incurred.
c. Measurements for interim reporting purposes should be made on a year-to-date
basis, so that the frequency of the entity's reporting (annual, half-yearly, or
quarterly) does not affect the measurement of its annual results.
d. A change in accounting estimate during an interim period is accounted for
prospectively by restating financial statements reported in previous interim periods
but not the most recent annual financial statement.

9. Income tax expenses during interim periods are recognized based on


a. the best estimate of the weighted average income tax rate in effect during the
quarter.
b. the effective income tax rate during the current quarter.
c. the substantially enacted future income tax rate if different from the current income
tax rate
d. the best estimate of the weighted average annual income tax rate expected for the
full financial year.

10.A corporation issues quarterly interim financial statements and uses the lower of cost
or net realizable value to measure its inventory in its annual financial statements.
Which of the following statements is correct regarding how the corporation should
value its inventory in its interim financial statements?
a. Inventory losses should be recognized in the interim statements.
b. Inventory write-downs should be made only in the annual financial statements.
c. Only the cost method of valuation should be used.
d. Gains from valuations in previous interim periods should be fully recognized.
(AICPA)

11.Which of the following is not an apparent limitation or problem regarding interim


reporting?

183
a. Considerable judgment should be made in determining which information is
significant or insignificant to users which may or may not be included in a
condensed financial report.
b. As reporting periods are shortened, the effects of errors in estimation and allocation
are magnified, and randomly occurring events which might not be material in the
context of a full fiscal year could create major distortions in short interim period
summaries of reporting entity performance.
c. Seasonal fluctuations and temporary market conditions affects the reliability,
comparability, and predictive value of interim reports.
d. The conciseness and limited time of preparation undermine the usefulness of
interim financial reports making them useful only to internal users who have direct
access to additional information on the reporting entity.

12.If an estimate of an amount reported in an interim period is changed significantly


during the final interim period of the financial year but a separate financial report is not
published for that final interim period, which of the following statements is true?
a. the nature and amount of that change in estimate shall be disclosed in a note to the
annual financial statements for that financial year.
b. the nature and amount of that change in estimate need not be disclosed in a note to
the annual financial statements for that financial year.
c. a separate financial report should be published for that final interim period.
d. an estimate of an amount reported in an interim period should not be changed
significantly during the final interim period of the financial year

13.An interim financial report shall include, at a minimum, all of the following components,
except
a. If an entity publishes a complete set of financial statements in its interim financial
report, the form and content of those statements shall conform to the requirements
of PAS 1 for a complete set of financial statements.
b. If the financial statements are condensed, they should include, at a minimum, each
of the headings and sub-totals included in the most recent annual financial
statements and the explanatory notes required by PAS 34.
c. Additional line-items should be included if their omission would make the interim
financial information misleading. If the annual financial statements were
consolidated (group) statements, the interim statements should be group
statements as well.
d. In the statement that presents the components of profit or loss for an interim
period, an entity need not present basic and diluted earnings per share for that
period when the entity is within the scope of PAS 33 Earnings per Share.

14.Which of the following statements is incorrect regarding PAS 34?


a. A user of an entity’s interim financial report is assumed to have access to the most
recent annual financial report of that entity.
b. It is unnecessary for the notes to an interim financial report to provide relatively
insignificant updates to the information that was already reported in the notes in
the most recent annual report.
c. At an interim date, an explanation of events and transactions that are significant to
an understanding of the changes in financial position and performance of the entity
since the end of the last annual reporting period is more useful.
d. All disclosure requirements in PAS 1 should be provided when the entity prepares
condensed interim financial statements.

15.An entity shall include all of the following information, as a minimum, in the notes to its
interim financial statements, if material and if not disclosed elsewhere in the interim
financial report, except
a. a statement that the same accounting policies and methods of computation are
followed in the interim financial statements as compared with the most recent

184
annual financial statements or, if those policies or methods have been changed, a
description of the nature and effect of the change
b. explanatory comments about the seasonality or cyclicality of interim operations
c. the nature and amount of items affecting assets, liabilities, equity, net income, or
cash flows that are unusual because of their nature, size, or incidence
d. the nature and amount of changes in estimates of amounts reported in prior interim
periods of the current financial year or changes in estimates of amounts reported in
prior financial years, if those changes have a material effect in the current interim
period
e. segment information even if PFRS 8 Operating Segments does not require that entity
to disclose segment information in its annual financial

16.The following are examples of the kinds of disclosures required by PAS 34, except
a. a the write-down of inventories to net realizable value and the reversal of such a
write-down
b. recognition of a loss from the impairment of property, plant and equipment,
intangible assets, or other assets, and the reversal of such an impairment loss
c. acquisitions and disposals of items of property, plant and equipment
d. corrections of prior period errors
e. extraordinary items

17.In deciding how to recognize, classify, or disclose an item for interim financial reporting
purposes, materiality is to be assessed in relation to
a. the interim period financial data
b. forecasted annual data.
c. previous year’s risk assessment
d. shareholders’ interest

18.Timely and reliable interim financial reporting improves the ability of investors,
creditors, and others to understand an entity’s (choose the incorrect statement)
a. capacity to generate earnings
b. capacity to generate cash flows
c. financial condition and liquidity
d. financial position and accuracy of inventory on hand

19.Which of the following statements is incorrect?


I. PAS 34 mandates which entities should be required to publish interim financial
reports, how frequently, or how soon after the end of an interim period.
II. Unlike for publicly listed entities which are required by PAS 34 to present interim
financial statements, non-listed entities are just encouraged by the standard to
present interim financial statements.
III. The SEC and PSE require companies covered by the reportorial requirements of
Revised Securities Act to file quarterly interim financial reports within 45 days after
the end of each of the first three quarters.
IV. Publicly traded entities are encouraged under PAS 34 to provide interim financial
reports at least as of the end of the first half of their financial year; and to make their
interim financial reports available not later than 60 days after the end of the interim
period.
V. For interim reporting relevance is sometimes sacrificed for reliability.
a. V b. III and IV c. I, II, IV and V d. I, II and V

20.A financial reporting period shorter than a full financial year.


a. Quarter b. Semi-annual c. Interim period d. Fiscal period

21.It means a financial report containing either a complete set of financial statements or a
set of condensed financial statements for an interim period.
a. Quarterly Report c. Interim financial highlights

185
b. Interim financial report d. Interim management report

22.Which of the following statements is incorrect?


a. In the interest of timeliness and cost considerations and to avoid repetition of
information previously reported, an entity may be required to or may elect to
provide less information at interim dates as compared with its annual financial
statements.
b. The interim financial report is intended to provide an update on the latest complete
set of annual financial statements. Accordingly, it focuses on new activities, events,
and circumstances and does not duplicate information previously reported.
c. Basic and diluted earnings per share shall be presented on the face of an income
statement, under complete financial statements but not in condensed financial
statements, for an interim period.
d. It is unnecessary for the notes to an interim financial report to provide relatively
insignificant updates to the information that was already reported in the notes in
the most recent annual report.
e. At an interim date, an explanation of events and transactions that are significant to
an understanding of the changes in financial position and performance of the entity
since the last annual reporting date is more useful.

23.The following relate to the required periods interim financial statements (condensed or
complete) should be presented except
a. balance sheet for the current interim period, balance sheet as of the end of the
current interim period and a comparative balance sheet as of the end of the
immediately preceding financial year and comparable interim periods (current and
year-to-date) of the immediately preceding financial year;
b. income statements for the current interim period and cumulatively for the current
financial year to date, with comparative income statements for the comparable
interim periods (current and year-to-date) of the immediately preceding financial
year;
c. statement showing changes in equity cumulatively for the current financial year to
date, with a comparative statement for the comparable year-to-date period of the
immediately preceding financial year; and
d. cash flow statement cumulatively for the current financial year to date, with a
comparative statement for the comparable year-to-date period of the immediately
preceding financial year.

24.Which of the following statements is incorrect?


a. In making assessments of materiality, it shall be recognized that interim
measurements may rely on estimates to a greater extent than measurements of
annual financial data.
b. An item as material if its omission or misstatement could influence the economic
decisions of users of the financial statements.
c. The Standards contain quantified guidance as to materiality.
d. While judgment is always required in assessing materiality, PAS 34 bases the
recognition and disclosure decision on data for the interim period by itself for
reasons of understandability of the interim figures.
e. If an estimate of an amount reported in an interim period is changed significantly
during the final interim period of the financial year but a separate financial report is
not published for that final interim period, the nature and amount of that change in
estimate shall be disclosed in a note to the annual financial statements for that
financial year.

25.An entity shall apply the same accounting policies in its interim financial statements as
are applied in its annual financial statements, except
a. prior period errors discovered during the interim period

186
b. for accounting policy changes made after the date of the most recent annual
financial statements that are to be reflected in the next annual financial statements.
c. for accounting estimates that requires year-to-date measurements
d. in no case that an entity shall apply different accounting policies in its interim
financial statements as are applied in its annual financial statements

26.The frequency of an entity’s reporting (annual, half-yearly, or quarterly) shall not affect
the measurement of its annual results. To achieve that objective, measurements for
interim reporting purposes shall be made
a. on an interim basis
b. on a year-to-date basis
c. on an individual transaction basis
d. on a prospective basis except when retrospective application is warranted

27.Requiring that an entity apply the same accounting policies in its interim financial
statements as in its annual statements may seem to suggest that interim period
measurements are made as if each interim period stands alone as an independent
reporting period. However, by providing that the frequency of an entity’s reporting
shall not affect the measurement of its annual results, PAS 34 acknowledges that an
interim period is a part of a larger financial year. The latter view is also called
a. Integral View c. Independent View
b. Discrete View d. Mines View

28.The principles for recognizing and measuring losses from inventory write-downs,
restructurings, or impairments in an interim period are the same as those that an entity
would follow if it prepared only annual financial statements. However, if such items are
recognized and measured in one interim period and the estimate changes in a
subsequent interim period of that financial year,
a. the original estimate is changed in the subsequent interim period by restating the
prior interim financial statements
b. the original estimate is not changed in the subsequent interim period but the annual
financial statement is adjusted either by accrual of an additional amount of loss or
by reversal of the previously recognized amount
c. the original estimate is changed in the subsequent interim period either by accrual
of an additional amount of loss or by reversal of the previously recognized amount
d. ignored, since interim period measurements are made as if each interim period
stands alone as an independent reporting period

29.A cost that does not meet the definition of an asset at the end of an interim period is
a. not deferred on the balance sheet either to await future information as to whether it
has met the definition of an asset or to smooth earnings over interim periods within
a financial year
b. deferred on the balance sheet either to await future information as to whether it has
met the definition of an asset or to smooth earnings over interim periods within a
financial year
c. deferred on the balance sheet if it is material in order to smooth earnings over
interim periods within a financial year
d. not deferred on the balance sheet but disclosed to enable the entity to either await
for future information as to whether it has met the definition of an asset or to
smooth earnings over interim periods within a financial year
30.Income tax expense is recognized in each interim period based on
a. the current enacted rate
b. the current enacted rate and the substantially enacted rate during the interim
period
c. the best estimate of the weighted average annual income tax rate expected for the
full financial year
d. the substantially enacted rate during the interim period

187
31.Under the Conceptual Framework, recognition is the
a. process of incorporating in the notes to financial statements an item that meets the
definition of an element and satisfies the criteria for recognition
b. process of converting non-cash assets into claims for cash
c. process of incorporating in the balance sheet or income statement an item that
meets the definition of an element and satisfies the criteria for recognition
d. process of assigning monetary amounts to the effects of economic transactions

32.Which of the following statements is correct?


I. Revenues that are received seasonally, cyclically, or occasionally within a financial
year shall not be anticipated or deferred as of an interim date if anticipation or
deferral would not be appropriate at the end of the entity’s financial year.
II. Costs that are incurred unevenly during an entity’s financial year shall be
anticipated or deferred for interim reporting purposes if, and only if, it is also
appropriate to anticipate or defer that type of cost at the end of the financial year.
a. True, true b. True, false c. False, true d. False, false

33.A change in accounting policy, other than one for which the transition is specified by a
new PFRS, shall be reflected by:
I. restating the financial statements of prior interim periods of the current financial
year and the comparable interim periods of any prior financial years that will be
restated in the annual financial statements in accordance with PAS 8
II. when it is impracticable to determine the cumulative effect at the beginning of the
financial year of applying a new accounting policy to all prior periods, adjusting the
financial statements of prior interim periods of the current financial year, and
comparable interim periods of prior financial years to apply the new accounting
policy prospectively from the earliest date practicable
a. I only b. II only c. I or II d. I and II

Use the following information for the next four questions:


The entity’s financial year ends at December 31(calendar year). The entity will present the
following financial statements (condensed or complete) in its half-yearly interim financial
report as of June 30, 20x1:
34.Statement of financial position
a. At June 30, 20x1; December 31, 20x0
b. At June 30, 20x1; June 30, 20x0
c. At June 30, 20x1; June 30, 20x0; December 31, 20x0
d. At June 30, 20x1; June 30, 20x0

35.Statement of profit or loss and other comprehensive Income


a. 6 months ending June 30, 20x1; June 30, 20x0; Year ended December 31, 2000
b. 6 months ending June 30, 20x1; June 30, 20x0
c. 3 months ending June 30, 20x1; June 30, 20x0
d. 6 months ending June 30, 20x1; June 30, 20x0

36.Statement of changes in equity


a. 6 months ending June 30, 20 x1; June 30, 20x0
b. 6 months ending 30 June 20 x1 30 June 20x0
c. 3 months ending 30 June 20 x1 30 June 20x0
d. 6 months ending June 30, 20 x1; June 30, 20x0; Year ended December 31, 20x0

37.Statement of cash flows


a. 6 months ending June 30, 20x1; June 30, 20x0
b. 6 months ending 30 June 20x1; June 30, 20x0
c. 3 months ending 30 June 20x1; June 30, 20x0

188
d. 6 months ending June 30, 20x1; June 30, 20x0; Interim period ending June 30, 20x1;
Year ended December 31, 20x0

Use the following information for the next four questions:


The entity’s financial year ends December 31 (calendar year). The entity will present the
following financial statements (condensed or complete) in its quarterly interim financial
report as of June 30, 20x1:
38.Statement of financial position
a. At June 30, 20x1; December 31, 20x0
b. At March 31, 20x1; June 30, 20x1; June 30, 20x0
c. At March 31, 20x1; June 30, 20x1; June 30, 20x0; December 31, 20x0
d. At June 30, 20x1; June 30, 20x0; December 31, 20x0

39.Statement of profit or loss and other comprehensive income


a. 6 months ending June 30, 20x1; June 30, 20x0
b. 3 months ending June 30, 20x1; June 30, 20x0
6 months ending June 30, 20x1; June 30, 20x0
c. 3 months ending June 30, 20x1; June 30, 20x0
6 months ending June 30, 20x1; June 30, 20x0; Interim period ending June 30, 20x1;
Year ended December 31, 20x0
d. 6 months ending June 30, 20x1; June 30, 20x0
3 months ending June 30, 20x1; June 30, 20x0; Year ended December 31, 20x0

40.Statement of Changes in Equity


a. 6 months ending June 30, 20x1; June 30, 20x0
b. 3 months ending June 30, 20x1; June 30, 20x0
6 months ending June 30, 20x1; June 30, 20x0
c. 3 months ending June 30, 20x1; June 30, 20x0
6 months ending June 30, 20x1; June 30, 20x0; Interim period ending June 30, 20x1;
Year ended December 31, 20x0
d. 6 months ending June 30, 20x1; June 30, 20x0
3 months ending June 30, 20x1; June 30, 20x0; Year ended
December 31, 20x0

41.Statement of cash flows


a. 6 months ending June 30, 20x1; June 30, 20x0
b. 3 months ending June 30, 20x1; June 30, 20x0
6 months ending June 30, 20x1; June 30, 20x0
c. 3 months ending June 30, 20x1; June 30, 20x0
6 months ending June 30, 20x1; June 30, 20x0; Interim period ending June 30, 20x1;
Year ended December 31, 20x0
d. 6 months ending June 30, 20x1; June 30, 20x0
3 months ending June 30, 20x1; June 30, 20x0; Year ended December 31, 20x0

42.A bonus is anticipated for interim reporting purposes if, and only if,
I. the bonus is a legal obligation or past practice would make the bonus a constructive
obligation for which the entity has no realistic alternative but to make the payments
II. a reliable estimate of the obligation can be made.
a. True, true b. True, false c. False, true d. False, false

43.Under PAS 34, interim financial reports should be published


a. Once a year at any time in that year. c. On a quarterly basis.
b. Within a month of the half year end. d. Whenever the entity wishes.
(Adapted)

189
44. The IASB encourages publicly traded entities to provide interim financial reports
a. At least at the end of the half year and within 60 days of the end of the interim
period.
b. Within a month of the half-year-end.
c. On a quarterly basis.
d. Whenever the entity wishes.
(Adapted)

45. If an entity does not prepare interim financial reports, then


a. The year-end financial statements are deemed not to comply with PFRS.
b. The year-end financial statements’ compliance with PFRS is not affected.
c. The year-end financial statements will not be acceptable under local legislation.
d. Interim financial reports should be included in the year-end financial statements.
(Adapted)

46. Interim financial reports should include as a minimum


a. A complete set of financial statements complying with PAS 1.
b. A condensed set of financial statements and selected notes.
c. A balance sheet and income statement only.
d. A condensed balance sheet, income statement, and cash flow statement only.
(Adapted)

47. PAS 34 states a presumption that anyone reading interim financial reports will
a. Understand all Philippine Financial Reporting Standards.
b. Have access to the records of the entity.
c. Have access to the most recent annual report.
d. Not make decisions based on the report.
(Adapted)

48. An entity owns a number of farms that harvest produce seasonally. Approximately
80% of the entity’s sales are in the period August to October. Because the entity’s
business is seasonal, PAS 34 suggests
a. Additional notes be written in the interim reports about the seasonal nature of the
business.
b. Disclosure of financial information for the latest and comparative 12-month period
in addition to the interim report.
c. Additional disclosure in the accounting policy note.
d. No additional disclosure.
(Adapted)

49. An entity is preparing half-yearly financial information in line with PAS 34. The period
to be covered by the financial statements is the six months to June 30, 20x1. A new
PFRS has been published that is effective for periods beginning on or after January 1,
20x1. The entity must adopt the PFRS
a. In the financial statements for the year to December 31, 20x1, only.
b. In its interim financial statements to June 30, 20x1, only.
c. In its interim financial statements to June 30, 20x1, and its annual financial
statements to December 31, 20x1.
d. At its own discretion.
(Adapted)

50.Are the following statements in relation to an interim financial report true or false,
according to PAS 34 Interim Financial Reporting?
I. An interim financial report may consist of a complete set of financial statements.
II. An interim financial report may consist of a condensed set of financial statements.
a. False, False b. False, True c. True, False d. True, True

190
(ACCA)

51.Are the following statements with respect to interim reporting true or false, according
to PAS 34 Interim Financial Reporting?
I. It is necessary to count inventories in full at the end of each interim accounting
period.
II. The net realizable value of inventories is determined by reference to selling prices at
the interim date.
a. False, False b. False, True c. True, False d. True, True
(ACCA)
52.REPREHEND CRITICIZE Company is preparing interim financial statements for the six
months to June 30, 20x1 in accordance with the minimum requirements of PAS 34
Interim Financial Reporting. Its accounting year ends on December 31 each year. In the
interim financial statements for the six months to June 30, 20x1, a statement of financial
position at June 30, 20x1 and a statement of profit or loss and other comprehensive
income for the six months to June 30, 20x1 will be presented. In addition, which of the
following should be presented?
I. Statement of financial position at June 30, 20x1
II. Statement of financial position at December 31, 20x1
III. Statement of profit or loss and other comprehensive income for the half year to June
30, 20x1
IV. Statement of profit or loss and other comprehensive income for the half year to
December 31, 20x1
a. I, II, and III b. II and III c. I and II d. all of these
(ACCA)

53.The terms and conditions of employment with the EXPOSTULATE DISCUSS include
entitlement to share in the staff bonus system, under which 5% of the profits for the
year before charging the bonus are allocated to the bonus pool, provided the annual
profits exceed P50 million. The profits (before accrual of any bonus) for the first half of
20x1 amount to ₱40 million and the latest estimate of the profits (before accrual of any
bonus) for the year as a whole is ₱60 million. How much should be recognized in profit
or loss in respect of the staff bonus for the half year to June 30, 20x1, according to PAS
34 Interim financial reporting?
a. 60M x 5% x 6/12 c. 40M x 5%
b. (60M – 50M) x 5% x 6/12 d. Nil
(ACCA)

54.INEFFICACIOUS INEFFECTIVE Company's profit before tax for the six months to
September 30, 20x1 was ₱5 million. However, the business is seasonal and profit before
tax for the six months to March 31, 20x2 is almost certain to be ₱9 million. Profit before
tax equals taxable profit for this company. INEFFICACIOUS operates in a country where
income tax on companies is at a rate of 30% if annual profits are below ₱11 million and
a rate of 35% where annual profits exceed ₱11 million. These tax rates apply to the
entire profit for the year. Under PAS 34 Interim Financial Reporting, what should be the
income tax expense in INEFFICACIOUS' interim financial statements for the half year to
September 30, 20x1?
a. 5M x 35% c. 14M x 35% x 6/12
b. {[(11M x 30%) + (3M x 35%)] ÷ 14M} x 5M d. Indeterminable
(ACCA)

55.PUERILE CHILDISH SILLY Company is preparing its financial statements for the first
half of its financial year ending December 31, 20x1. One class of inventory has a cost per
unit of ₱5.00 and a net realizable value at June 30, 20x1 of ₱4.80 per unit. The business
is seasonal and the net realizable value at December 31, 20x1 is expected to be ₱5.50.
PUERILE's budget for the year scheduled a major refurbishment project for April to
June 20x1. For legal reasons the contract for the refurbishment was not signed until July

191
8, 20x1, on which date the work was started. Are the following statements true or false,
according to PAS 34 Interim Financial Reporting?
I. The inventory should be carried at its cost per unit of ₱5.00 at June 30, 20x1.
II. The cost of the major refurbishment project should be accrued at June 30, 20x1.
a. False, False b. False, True c. True, False d. True, True
(ACCA)

56.Which of the following is a unique reporting problem associated with the determination
of the results of operations for an interim period?
a. Advertising and similar costs expensed in one interim period may benefit other
interim periods in the same annual period.
b. Cost of goods sold for an interim period reflects only the amount of product cost
applicable to sales revenue recognized in the interim period.
c. Depreciation for an interim period represents an estimate.
d. An extraordinary loss occurring in the second quarter must be prorated over the
last three interim periods of the year.
(Adapted)

57.If a gain on reversal of impairment loss occurs in the second fiscal quarter. How should
the gain be accounted for?
a. Recognized in full in the second quarter.
b. Recognized equally over the second, third, and fourth quarters.
c. Recognized only in the annual financial statements.
d. Recognized equally in each quarter, by restating the first quarter.
(Adapted)

58.Which of the following statements is correct regarding the provisions of PAS 34?
a. An entity does not need to provide disclosures for operating segments in its interim
financial report even if the entity is required under PFRS 8 to provide such
disclosures in its annual financial statements.
b. An entity does not need to adjust its financial statements for adjusting events after
the reporting period as this requirement is applicable only to annual financial
statements.
c. Related party disclosures need not be provided in the interim financial report but
should be provided in the annual financial statements.
d. Disclosure of accounting policies need not be provided in the interim financial
report but should be provided in the annual financial statements.

59.LARCENY THEFT Co. incurred costs during the 2 nd quarter. The costs clearly benefit the
remainder of the year. How should LARCENY account for the costs incurred?
a. Recognize the costs as expense in full in the 2nd quarter.
b. Recognize ¼ of the costs as expense in the 2nd quarter.
c. Recognize 1/3 of the costs as expense in the 2nd quarter.
d. Initially recognize the costs as asset and expense only in the annual financial
statements.

60.For the purpose of external reporting, it is proper to use estimated gross profit rates to
determine ending inventory and cost of goods sold for
(Item #1) Interim financial reporting; (Item #1) Year-end financial reporting
a. Yes, Yes b. No, No c. Yes, No d. No, Yes

61.For interim financial reporting, a realized gain occurring in the second quarter should
be
a. Recognized ratably over the last three quarters.
b. Recognized ratably over all four quarters with the first quarter being restated.
c. Recognized in the second quarter.
d. Reported as note disclosure only in the second quarter.

192
62.Which of the following reporting practices is permissible for interim financial
reporting?
a. Use of the gross-profit method for interim inventory pricing.
b. Use of the direct-costing method for determining manufacturing inventories.
c. Deferral of unplanned variances under a standard cost system until year end.
d. Deferral of inventory write-downs until year end.

63.For interim financial reporting, a company's income tax provision for the second
quarter of 20x1 should be determined using the
a. Effective tax rate expected to be applicable for the full year of 20x1 as estimated at
the end of the first quarter of 20x1.
b. Best estimate of the weighted average annual income tax rate expected for the full
financial year of 20x1.
c. Effective tax rate expected to be applicable for the second quarter of 20x1.
d. Statutory tax rate for 20x1.

64.An inventory loss from a market price decline occurred in the first quarter, and the
decline was not expected to reverse during the fiscal year. However, in the third quarter
the inventory’s market price recovery exceeded the market decline that occurred in the
first quarter. For interim financial reporting, the peso amount of net inventory should
a. Decrease in the first quarter by the amount of the market price decline and increase
in the third quarter by the amount of the decrease in the first quarter.
b. Decrease in the first quarter by the amount of the market price decline and increase
in the third quarter by the amount of the market price recovery.
c. Decrease in the first quarter by the amount of the market price decline and not be
affected in the third quarter.
d. Not be affected in either the first quarter or the third quarter.
(Adapted)

65.The cost of inventories of INTERDICT FORBID Co. exceeds their net realizable value as
of the end of the second quarter. INTERDICT believes that market prices will return to
their previous levels by the end of the year. At the end of the year, the decline in the
value of the inventories had not reversed, but market prices did not decline further.
When should the loss be reported in INTERDICT’s interim income statements?
a. Equally over the 2nd, 3rd, and 4th quarters
b. In the 2nd and 4th quarters only
c. In the annual financial statements only
d. In the 2nd quarter only
(Adapted)

66.Conceptually, interim financial statements can be described as emphasizing


a. Timeliness over reliability. c. Relevance over comparability.
b. Reliability over relevance. d. Comparability over neutrality.
(Adapted)

67.In addition to the other information required for non-highly seasonal businesses, an
entity whose business is highly seasonal is encouraged under PAS 34 to present
a. financial information for the twelve months ending on the interim reporting date
and comparative information for the prior twelve-month period
b. financial information for the twelve months ending on the interim reporting date
and comparable information for the latest annual financial reporting period.
c. financial information for the twelve months ending on the interim reporting date
and no comparative information.
d. financial information for the thirteen months ending on the interim reporting date
and comparable information for the thirteen-month period of the immediately
preceding financial year

193
68.Which of the following statements is incorrect?
a. An entity may present complete set of financial statements in its interim financial
reporting.
b. The results for each interim period should be based on the accounting principles
and practices used by an entity in the preparation of its latest annual financial
statements unless a change in an accounting practice or policy has been adopted in
the current year.
c. PAS 34 requires that an entity apply the same criteria for recognizing and
measuring a provision at an interim date as it would at the end of its financial year.
The existence or non-existence of an obligation to transfer benefits is not a function
of the length of the reporting period. It is a question of fact.
d. For interim financial reporting, an inventory loss from a market decline in the
second quarter that is expected to be restored in the fiscal year should be
recognized as a loss proportionately in each of the first, second, third and fourth
quarters.

Chapter 45 - Suggested answers to review theory questions


1. D 11. D 21. B 31. C 41. A 51. B 61. C
2. A 12. A 22. C 32. A 42. A 52. B 62. A
3. B 13. D 23. A 33. C 43. D 53. C 63. B
4. A 14. D 24. C 34. A 44. A 54. A 64. A
5. B 15. E 25. B 35. D 45. B 55. A 65. D
6. D 16. E 26. B 36. A 46. B 56. A 66. A
7. D 17. A 27. A 37. A 47. C 57. A 67. A
8. D 18. D 28. C 38. A 48. B 58. D 68. D
9. D 19. D 29. A 39. B 49. C 59. C
10. A 20. C 30. C 40. A 50. D 60. C

194
Chapter 46
Development Stage Entities

Chapter 46: Theory of Accounts Reviewer


1. An entity is considered in the development stage if it is devoting substantially all of its
efforts to establishing a new business and
I. planned principal operations have not begun
II. if operations have begun, there has been no significant revenue therefrom.
a. I or II b. I only c. II only d. neither I nor II
(AICPA)

2. The basic financial statement(s) of a development stage entity to be presented should


include all of the following, except
a. A balance sheet showing any accumulated net losses with a descriptive caption such
as "deficit accumulated during the development stage" in the stockholders' equity
section.
b. A statement of profit or loss and other comprehensive income, showing amounts of
revenue and expenses for each period covered by the statement and cumulative
amounts from the enterprise's inception.
c. A statement of cash flows showing the investing and financing activities for each
period and cumulative amounts from the enterprise’s inception.
d. The financial statements should be identified as those of a development stage
enterprise and should include a description of the activities in which the enterprise
is engaged.
e. The first fiscal year in which an enterprise is no longer considered in the
development stage should not disclose its prior status as a development stage
enterprise.

3. A development stage entity


a. Issues a statement of profit or loss and other comprehensive income that shows
only cumulative amounts from the enterprise's inception.
b. Issues a statement of profit or loss and other comprehensive income that is the
same as an established operating enterprise, but does not show cumulative
amounts from the enterprise's inception as additional information.
c. Issues a statement of profit or loss and other comprehensive income that is the
same as an established operating enterprise, and shows cumulative amounts from
the enterprise's inception as additional information.
d. Does not issue a statement of profit or loss and other comprehensive income.
(AICPA)

4. Financial reporting by a development stage enterprise differs from financial reporting


for an established operating enterprise in regard to note disclosures
a. Only.
b. And expense recognition principles only.
c. And revenue recognition principles only.
d. And revenue and expense recognition principles.
(AICPA)

5. The primary distinction between the financial statements of a development stage


company and those of a company which has passed the development stage is that the
statements of the former:
a. can report losses properly as deferred charges
b. are more likely to reflect goodwill among the assets
c. are more likely to show continually increasing profits

195
d. will reflect specified cumulative amounts arising from the date of the company's
inception.
(AICPA)

6. A development stage enterprise should use the same generally accepted accounting
principles that apply to established operating enterprises for
(Item #1)Deferral of costs; (Item #2) Expensing of costs when incurred
a. Yes, Yes b. Yes, No c. No, No d. No, Yes
(AICPA)

7. Deficits accumulated during the development stage of a company should be


a. Reported as organization costs.
b. Reported as a part of stockholders' equity.
c. Capitalized and written off in the first year of principal operations.
d. Capitalized and amortized over a five-year period beginning when principal
operations commence.
(AICPA)

8. Start up and organization costs


a. are expensed in the year incurred.
b. are capitalized, but never amortized.
c. are capitalized and amortized, usually over the life of the corporation.
d. appear on the balance sheet as a current asset.
(Adapted)

9. Which of the following items usually includes professional fees, incorporation fees and
stock certificate costs?
a. Stock issuance costs c. Organization costs
b. Goodwill d. Copyrights
(Adapted)

10.Which of the following best describes a development stage company?


a. has commenced its planned operations, but is operating at a loss.
b. has not commenced its planned operations or has commenced operations but has
not yet produced any significant revenues.
c. is developing and expanding its planned operations, but its net income has been
very low.
d. is in the process of merging with another firm.
(Adapted)

11.Operating losses incurred during the start-up years of a new business should be
a. accounted for and reported like the operating losses of any other business.
b. written off directly against retained earnings.
c. capitalized as a deferred charge and amortized over five years.
d. capitalized as an intangible asset and amortized over a period not to exceed 20
years.
(AICPA)

12.Which of the following does not comprise organization costs of a company?


a. cost of printing stock certificates c. first year loss from operation
b. incorporation fees d. none of the above
(AICPA)

13.PAS 38 Intangible Assets requires start-up costs, including organization costs, to be


expensed immediately when incurred, unless another PFRS require otherwise. Which of
the following organization costs are expensed immediately?
a. direct transaction costs of issuing equity instruments

196
b. direct transaction costs of issuing debt securities measured at amortized cost
c. legal fees of acquiring land
d. costs of registering the business and filing fees paid to the SEC

14.According to PAS 32, direct costs of issuing shares of stocks are


a. recognized as expense
b. recognized directly in equity
c. capitalized and amortized
d. may be recognized as expense in some circumstances

15.According to PFRS 9, transaction costs of issuing debt securities measured at amortized


are
a. recognized immediately as expense
b. included in the initial measurement of the resulting financial liability
c. recognized as asset
d. ignored

16.According to PFRS 9, transaction costs of issuing debt securities measured at fair value
are
a. recognized immediately as expense
b. included in the initial measurement of the resulting financial liability
c. recognized as asset
d. ignored

17.According to PAS 16, costs directly attributable to bringing the asset to the location and
condition necessary for it to be capable of operating in the manner intended by
management are
a. recognized immediately as expense
b. included in the initial measurement of the resulting financial liability
c. recognized as asset
d. ignored

18.According to PFRS 6, costs incurred in the exploration for and evaluation of mineral
resources are
a. expensed immediately
b. initially recognized as asset and subsequently amortized
c. accounted for in accordance with the entity’s policy
d. recognized as liability

Chapter 46 - Suggested answers to review theory questions


1. A 6. A 11. A 16. A
2. E 7. B 12. C 17. C
3. C 8. A 13. D 18. C
4. A 9. C 14. B
5. D 10. B 15. B

197
Chapter 47
Cash Basis to Accrual Basis of Accounting

Chapter 47: Multiple choice – Computational (SET B) – (For classroom instruction


purposes)
Sales
Use the following information for the next two questions:
The following information was taken from the records of INCONDITE CRUDE Co.
Cash sales – gross 2,000,000
Trade accounts receivable – beg. 2,400,000
Trade accounts receivable – end 1,600,000
Trade notes receivable increased by 1,200,000
Collections on receivables 4,000,000
Sales returns and discounts (inclusive of ₱20,000 payments to customers) 80,000
Write-offs of accounts receivable 40,000
Recoveries of accounts receivable written-off (included in collections) 16,000
Trade notes receivable discounted (Note receivable was directly credited) 120,000

1. How much is the net sales under cash basis of accounting?


a. 4,604,000 b. 5,980,000 c. 6,524,000 d. 1,980,000

2. How much is the net sales under accrual basis of accounting?


a. 4,604,000 b. 5,980,000 c. 6,524,000 d. 1,980,000

Purchases and Cost of goods sold


Use the following information for the next four questions:
The following information was taken from the records of JUGATE PAIRED Co.
Cash purchases – gross 1,200,000
Trade accounts payable – beg. 2,000,000
Trade accounts payable – end 1,600,000
Trade notes payable decreased by 800,000
Cash payments on payables 4,000,000
Purchase returns and discounts (inclusive of P60,000 receipts from
suppliers) 80,000
Inventory increased by 400,000

3. How much is the net purchases under cash basis of accounting?


a. 4,740,000 b. 2,820,000 c. 3,940,000 d. 5,140,000

4. How much is the cost of goods sold under cash basis of accounting?
a. 3,960,000 b. 4,740,000 c. 3,960,000 d. 5,140,000

5. How much is the net purchases under accrual basis of accounting?


a. 3,960,000 b. 1,620,000 c. 3,940,000 d. 4,740,000

6. How much is the goods sold under accrual basis of accounting?


a. 2,820,000 b. 3,540,000 c. 3,960,000 d. 4,740,000

Other items of income


Use the following information for the next two questions:
INVETERATE HABITUAL Co. has the following information:
Accrued rent income – January 1 1,600,000
Accrued rent income – December 31 800,000

198
Unearned rent, January 1 1,200,000
Unearned rent, December 2,000,000
Rental payments received 4,000,000

7. How much is the rent income under cash basis of accounting?


a. 4,000,000 b. 2,400,000 c. 5,600,000 d. 4,800,000

8. How much is the rent income under accrual basis of accounting?


a. 4,000,000 b. 2,400,000 c. 5,600,000 d. 4,800,000

Other items of income


9. The following information was taken from the records of KATZENJAMMER DISTRESS
Co.
Accrued rent income – January 1 1,600,000
Accrued rent income – December 31 800,000
Unearned rent, January 1 1,200,000
Unearned rent, December 2,000,000
Rent income under accrual basis 2,400,000

How much is the rent income under cash basis?


a. 4,000,000 b. 2,400,000 c. 5,600,000 d. 4,800,000

Other items of expense


10.OBNUBIATE OBSCURE Co. has the following information:
Prepaid insurance – January 1 1,600,000
Prepaid insurance – December 31 800,000
Insurance payable decreased by 1,200,000
Insurance payments 4,000,000

How much is the rent expense under accrual basis?


a. 4,000,000 b. 3,400,000 c. 3,600,000 d. 4,800,000

Other items of expense


11.The following information was taken from the records of LEGERITY AGILITY Co.
Prepaid insurance – January 1 1,600,000
Prepaid insurance – December 31 800,000
Insurance payable decreased by 1,200,000
Rent expense under accrual basis 3,600,000

How much is the rent expense under cash basis?


a. 4,000,000 b. 2,400,000 c. 5,600,000 d. 4,800,000

Income tax
12.TOTEM REVERED SYMBOL Co. has the following information:
Payments made for income taxes 4,000,000
Income tax payable increased by 800,000
Deferred tax liability, January 1 1,200,000
Deferred tax liability, December 31 3,600,000
Deferred tax asset increased by 2,000,000

How much is the income tax expense to be presented in the statement of profit or loss and
other comprehensive income in accordance with PFRSs?
a. 9,200,000 b. 4,800,000 c. 400,000 d. 5,200,000

199
Comprehensive illustration
Use the following information for the next two questions:
On January 1, 20x1, APOTHEGM SHORT SAYING Co. started its operations with initial cash
investment of ₱400,000. APOTHEGM provided ₱1,200,000 of services in January and
received full payment in April. APOTHEGM incurred expenses of ₱480,000 in January
which were paid in March. During March, dividends of ₱200,000 were paid.

13.How much is the profit or loss for the first quarter under cash basis?
a. (480,000) b. 520,000 c. 720,000 d. (780,000)

14.How much is the profit or loss for the first quarter under accrual basis?
a. (480,000) b. 520,000 c. 720,000 d. (780,000)

Comprehensive illustration
15.TUSSLE WRETLE Co. reported profit of ₱800,000 in 20x1 under cash basis. The
following items are relevant in converting the cash basis profit into accrual basis.
Inventory, January 1 4,000,000
Inventory, December 31 2,400,000
Receivables, January 1 1,200,000
Receivables, December 31 3,600,000
Payables increased by 2,000,000

How much is the profit under accrual basis?


a. 2,000,000 b. (2,000,000) c. 5,200,000 d. (400,000)

Comprehensive illustration
16.INSIPID TASTELESS Co.’s accrual basis profit is computed as follows:
Sales 10,000,000
Cost of sales:
Inventory, Jan. 1 2,400,000
Net purchases 5,600,000
Cost of goods available for sale 8,000,000
Inventory, Dec. 31 (1,600,000) (6,400,000)
Gross profit 3,600,000
Other income 400,000
Operating expenses (2,800,000)
Profit for the year 1,200,000

Additional information:
 Operating expenses include depreciation of ₱280,000.
 Other income includes interest income of ₱320,000, ₱40,000 of which pertains to
amortization of discount on investment in bonds.
 Accounts receivable decreased by ₱400,000, prepaid expenses increased by ₱200,000;
accrued expenses increased by ₱80,000; and accounts payable decreased by ₱240,000;.

How much is INSIPID’s cash basis profit?


a. 2,280,000 b. 1,480,000 c. 1,840,000 d. 680,000

Comprehensive illustration
17.PITH IMPORTANCE Co. started its operations in 20x1. Its income statement prepared
under cash basis of accounting is provided below.
PITH IMPORTANCE Co.
Income statement
For the year ended December 31, 20x1
Revenue 10,000,000
Other income 80,000
Equipment (1,600,000)

200
Salaries expense (1,200,000)
Rent expense (720,000)
Utilities expense (320,000)
Insurance expense (160,000)
Commission expense (100,000)
Finance cost (120,000)
Profit before tax 5,860,000
Income tax expense (1,600,000)
Profit for the year 4,260,000

Additional information:
a. Amounts due from customers at year-end were ₱1,000,000. Of this amount, ₱80,000 is
doubtful of collection.
b. Interest income of ₱80,000 on a note receivable from a customer was recognized in
other income. However, an amortization of discount on the note receivable of ₱8,000
was not recorded.
c. The cost of equipment purchased to be used in business was expensed immediately.
The equipment has an estimated useful life of 10 years. PITH uses the straight line
method of depreciation.
d. Salaries of ₱120,000 incurred in December 20x1 were paid on January 4, 20x2.
e. PITH rents its office space for ₱48,000 a month, payable quarterly in advance. The
contract was signed on December 31, 20x0.
f. The bill for December’s utility costs of ₱40,000 was paid on January 9, 20x2.
g. A one-year insurance policy was obtained on July 1, 20x1. Premiums are paid annually
in advance.
h. Commissions of 1% of revenues are paid on the same day cash is received from
customers.
i. PITH borrowed ₱4,000,000 for one year on August 1, 20x1. Interest payments based on
an annual rate of 12% are made quarterly.
j. There are no unpaid income taxes as of year-end. However, deferred tax asset of
₱13,200 and deferred tax liability of ₱20,000 were not recognized.

How much is the profit for the year under accrual basis of accounting?
a. 6,569,000 b. 7,256,000 c. 7,486,000 d. 6,596,000

The answers and solutions to the computational problems above


(Multiple choice – Computational (SET B) can be found in the
accompanying Teacher’s Manual.

201
Chapter 48
Financial Reporting in Hyperinflationary Economies

Chapter 48: Multiple choice – Computational (SET B) – (For classroom instruction


purposes)
Inflation rate
1. Information on a country’s inflation rate is shown below:
Year CPI Change in CPI Annual inflation rate
  (a) (b) % = (b ÷ a) x 100%
Jan. 1, 20x1 400
Dec. 31, 20x1 520 120 30.00%
Dec. 31, 20x2 720 200 38.46%
Dec. 31, 20x3 880 160 22.22%

What is the cumulative inflation rate in 20x3 to be used in determining if there is


hyperinflation?
a. 100% b. 120% c. 133.33% d. 140%

Identifying monetary and non-monetary items


2. The information given below is taken from the records of COLLATION LIGHT MEAL
Company.
Cash 60 Trade accounts payable 160
Accounts receivable - net
of allowance 160 Income tax payable 40
Financial assets held for
trading 60 Utilities payable 80
Financial assets at FVOCI 80 Cash dividends payable 40
Financial assets at
amortized cost 240 Bonds payable 160
Inventories 240 Warranty obligation 80
Advances to suppliers 40 Deferred revenue 60
Prepaid expenses 20 Share capital 520
Property, plant, and
equipment 320 Share premium 80
Cash surrender value 80 Revaluation surplus 40
Investment in funds Cumulative translation
consisting of: differences 80
Shares of stocks 20 Non-controlling interest 20
Bonds measured at
amortized cost 120 Retained earnings 80
Total assets 1,440 Total liabilities and equity 1,440

How much is the net monetary items?


a. 180 c. 240 c. 320 d. 160

Restatement of financial statements – Historical to Constant peso


Use the following information for the next seven questions:
SOBRIQUET NICKNAME Co. operates in a hyperinflationary economy. Its unrestated
financial statements are provided below:
SOBRIQUET NICKNAME Co.
Statement of financial position
As of December 31, 20x2

202
  20x2 20x1
ASSETS
Cash 80,000 60,000
Accounts receivable 160,000 120,000
Allowance for doubtful accounts (40,000) (20,000)
Inventory (at cost) 200,000 160,000
Land (at cost) 400,000 400,000
Building (at cost) 2,000,000 2,000,000
Accumulated depreciation (800,000) (600,000)
Total assets 2,000,000 2,120,000
LIABILITIES AND EQUITY
Accounts payable 80,000 188,000
Loan payable 400,000 320,000
Total liabilities 480,000 508,000
Share capital 1,200,000 1,200,000
Retained earnings 320,000 412,000
Total equity 1,520,000 1,612,000
Total liabilities and equity 2,000,000 2,120,000

SOBRIQUET NICKNAME Co.


Statement of profit or loss and other comprehensive income
For the year ended December 31, 20x2
Sales 1,600,000
Cost of sales:
Inventory, January 1 160,000
Purchases 1,200,000
Total goods available for sale 1,360,000
(1,160,000
(200,000)
Inventory, December 31 )
Gross income 440,000
Depreciation expense (200,000)
Distribution costs (140,000)
Bad debts expense (20,000)
Finance cost (40,000)
Profit before tax 40,000
Income tax expense (12,000)
Profit for the year 28,000
Other comprehensive income -
Total comprehensive income for the year 28,000

Additional information:
 The land and building were acquired on April 1, 20x0.
 The share capital was issued on March 1, 20x0.
 Sales, purchases, and expenses (except interest expense) were incurred evenly during
the year.
 Interest expense was recognized and paid on December 31, 20x2.
 Dividends of ₱120,000 were declared and paid on December 31, 20x2.
 Selected values of general price indices (CPI) are shown below:
March 1, 20x0………………………………………………...100
April 1, 20x0…………………………………………………..100
Average for 20x1……………………………………………..110
December 31, 20x1……………………………….…………120
Average for 20x2……………………………………………..125
December 31, 20x2………………………………………….140
3. How much is the restated total assets in the 20x1 comparative statement of financial
position?

203
a. 2,910,303 b. 3,004,604 c. 3, 028,640 d. 2,910,340

4. How much is the restated total liabilities in the 20x1 comparative statement of financial
position?
a. 592,677 b. 508,000 c. 584,767 d. 592,667

5. How much is the restated total assets in the 20x2 statement of financial position?
a. 2,664,000 b. 2,894,00 c. 2784,000 d. 2,646,000

6. How much is the restated total liabilities in the 20x2 statement of financial position?
a. 520,000 b. 480,000 c. 460,000 d. 540,000

7. How much is the restated profit?


a. 13,636 b. (13,636) c. (13,726) d. 13,726

8. How much is the gain (loss) on net monetary position (purchasing power gain or loss)?
a. 28,420 b. (28,420) c. 28,240 d. (28,240)

9. How much is the restated retained earnings on December 31, 20x2?


a.637,636 b. 540,000 c. 637,663 d. 504,000

Gain or loss on net monetary position


10.PUSSILLANIMOUS COWARDLY Co. has the following information during 20x1:
Sales (all on account) made evenly throughout 20x1 1,600,000
Cash collections from customers on account made evenly
throughout 20x1 880,000
Purchases (all on account) made evenly throughout 20x1
throughout 20x1 800,000
Cash payments to suppliers on account made evenly
throughout 20x1 720,000
Land purchased for cash on February 1, 20x1 160,000
Equipment purchased on credit on March 1, 20x1 and
paid on April 1, 20x1 120,000
Proceeds on a loan taken on July 1, 20x1 400,000
Investment in held for trading securities sold on August 1, 20x1 240,000
Cash dividends declared on October 1, 20x1 and paid on
December 31, 20x1 200,000
Operating expenses paid evenly throughout 20x1 480,000
Share capital issued on December 31, 20x1 320,000
Partial payment on the loan on December 31, 20x1 200,000
Monetary assets:
January 1, 20x1 120,000
December 31, 20x1 800,000
Monetary liabilities:
January 1, 20x1 60,000
December 31, 20x1 340,000

The following are selected values of CPI-U for 20x1:


10
1-Jan 0 1-Apr 130 31-Dec 160
11
1-Feb 0 1-Aug 140 Average for the year 125
1- 12
Mar 0 1-Oct 150

How much is gain (loss) on net monetary position (purchasing power gain or loss)?
a. (33,826) b. 33,826 c. 37,866 d. (37,866)

204
Gain or loss on net monetary position
11.On January 1, 20x1, ZEPHYR GENTLE BREEZE Company had monetary assets of
₱8,000,000 and monetary liabilities of ₱4,000,000. During 20x1, ZEPHYR's monetary
inflows and outflows were relatively constant and equal so that it ended the year with
net monetary assets of ₱4,000,000. Assume that the Consumer Price Index was 200 on
January 1, 20x1, and 220 on December 31, 20x1. How much is gain (loss) on net
monetary position (purchasing power gain or loss)?
a. (400,000) b. 400,000 c. 4,000,000 d. (4,000,000)

Non-monetary items carried at other than cost


12.LAMBAST SCOLD SEVERLY Co. has previously been preparing financial statements
restated in accordance with PAS 29. As of December 31, 20x1, the following are among
the assets of LAMBAST Co.:
 Inventory from purchases made evenly throughout 20x1 with historical cost of ₱400,000
and net realizable value of ₱320,000 as of year-end.
 Held for trading securities acquired on January 1, 20x1 for ₱200,000. The fair value of
the securities as of year-end is ₱240,000.
 Land acquired on January 1, 20x0 for ₱4,000,000 was revalued to ₱4,800,000 on July 1,
20x1.

The following are the selected general price index numbers:


January 1, 20x0…………………100
January 1, 20x1…………………120
July 1, 20x1…………………….....125
December 31, 20x1…………...140

How much is the total restated amount for the items described above?
a. 560,000 b. 5,616,000 c. 5,376,000 d. 5,936,000

Equity – First time adoption


13.During 20x1, the economy of CHOCK COMPLETELY Co.’s functional currency became
hyperinflationary. CHOCK Co. is preparing its first financial statements prepared under
PAS 29. The following information before restatement was made available:
Cash 400 Accounts payable 1,200
Accounts receivables 1,400 Share capital 2,000
Prepaid expenses 200 Revaluation surplus 400
Land 4,000 Retained earnings 2,400
Total assets 6,000   Total liability and equity 6,000

Additional information:
 The prepaid expenses were recognized evenly during the year.
 The land was acquired on January 1, 20x0 for ₱3,600 but was revalued to ₱4,000 on July
1, 20x1. A revaluation surplus of ₱400 was recognized in equity.
 The share capital was issued on January 1, 20x0.
 The following are the selected general price index numbers:
January 1, 20x0…………………100
January 1, 20x1…………………120
July 1, 20x1……………………...125
December 31, 20x1……………..140
Average for 20x1………………..130

How much is the restated shareholders’ equity?


a. 5,295 b. 6,056 c. 5,856 d. 3,248

Approximation of average price index

205
14.DISCOMBOBULATE UPSET Co. reported sales of ₱4,000,000 in 20x1 which was made
evenly throughout the year. The CPI-U was 100 on January 1, 20x1 and 120 on
December 31, 201. How much are the sales adjusted to constant pesos?
a. 4,363,636 b. 4,800,000 c. 4,400,000 d. 4,333,333

Restatement of financial statements – Historical to Current cost


Use the following information for the next five questions:
After its first year of operations, ACRIMONY BITTERNESS Co. is preparing its financial
statements using current cost accounting. Its historical cost financial statements are
provided below:
ACRIMONY BITTERNESS Co.
Statement of financial position
As of December 31, 20x2
  Dec. 31 Jan. 1
ASSETS
Cash 180,000 80,000
Accounts receivable 160,000 -
Allowance for doubtful
(20,000) -
accounts
Inventory (at cost) 200,000 -
Land (at cost) 400,000 400,000
Building (at cost) 3,200,000 3,200,000
Accumulated depreciation (320,000) -
Total assets 3,800,000 3,680,000
LIABILITIES AND EQUITY
Accounts payable 80,000 -
Loan payable 480,000 480,000
Total liabilities 560,000 480,000
Share capital 3,200,000 3,200,000
Retained earnings 40,000 -
Total equity 3,240,000 3,200,000
Total liabilities and equity 3,800,000 3,680,000

ACRIMONY BITTERNESS Co.


Statement of profit or loss and other comprehensive income
For the year ended December 31, 20x2
Sales 1,960,000
Cost of sales:
Inventory, January 1 -
Purchases 1,360,000
Total goods available for sale 1,360,000
Inventory, December 31 (200,000) (1,160,000)
Gross income   800,000
Depreciation expense (320,000)
Distribution costs (140,000)
Bad debts expense (20,000)
Finance cost (40,000)
Profit before tax   280,000
Income tax expense (120,000)
Profit for the year   160,000
Other comprehensive income -
Total comprehensive income for the year 160,000

Additional information:
 Information on current costs is as follows:
Cost of goods sold at average current cost ₱1,400,000

206
Inventory, December 31, 20x2 300,000
Land, December 31, 20x2 600,000
Building, December 31, 20x2 4,800,000
 The building is being depreciated over 10 years using straight line method

15.How much is the restated current cost total assets in the December 31, 20x2 statement
of financial position?
a. 5,450,000 b. 4,540,00 c. 5,784,000 d. 5,540,000

16.How much is the restated current cost total liabilities in the December 31, 20x2
statement of financial position?
a. 520,000 b. 480,000 c. 560,000 d. 540,000

17.How much is the restated current cost profit?


a. 1,900,000 b. 1,930,636 c. 1,872,600 d. 1,970,000

18.How much is the gain (loss) on net monetary position (purchasing power gain or loss)?
a. (8,240) b. 8,240 c. 24,480 d. 0

19.How much is the restated current cost retained earnings on December 31, 20x2?
a. 1,900,636 b. 1,940,000 c. 1,780,000 d. 1,580,000

Restatement of financial statements – Current cost / Constant peso


Use the following information for the next five questions:
After its first year of operations, GRAVID PREGNANT Co.’s current cost financial
statements are provided below:

GRAVID PREGNANT Co.


Statement of financial position
As of December 31, 20x2
ASSETS
Cash 180,000
Accounts receivable 160,000
Allowance for doubtful accounts (20,000)
Inventory (at current cost) 300,000
Land (at current cost) 600,000
Building (at current cost) 4,800,000
Accumulated depreciation (at current cost) (480,000)
Total assets 5,540,000

LIABILITIES AND EQUITY


Accounts payable 80,000
Loan payable 480,000
Total liabilities 560,000
Share capital 3,200,000
Retained earnings (at current cost) 1,780,000
Total equity 4,980,000
Total liabilities and equity 5,540,000

GRAVID PREGNANT Co.


Statement of profit or loss and other comprehensive income
For the year ended December 31, 20x2
Sales 1,960,000
Cost of sales (at current cost) (1,400,000)
Gross income   560,000
Other income:

207
Realized holding gain 320,000
Unrealized holding gain 1,740,000
Total income   2,620,000
Operating expenses:
Depreciation expense (at current cost) (400,000)
Distribution costs (140,000)
Bad debts expense (20,000)
Finance cost (40,000)
Profit before tax   2,020,000
Income tax expense (120,000)
Profit for the year   1,900,000
Other comprehensive income -
Total comprehensive income for the
year 1,900,000

Additional information:
 The land and building were acquired on January 1, 20x2 for ₱400,000 and ₱3,200,000,
respectively.
 The building is being depreciated over 10 years using straight line method. The carrying
amount of the building as of December 31, 20x2 based on historical cost is ₱2,880,000.
 The share capital was issued on January 1, 20x2.
 The inventory on December 31, 20x2 has a historical cost of ₱200,000. There was no
inventory as of January 1, 20x2.
 Sales, purchases, and expenses (except interest expense) were incurred evenly during
the year.
 Purchases during the year amounted to ₱1,360,000 (at historical cost).
 Interest expense was recognized and paid on December 31, 20x2.
 Dividends of ₱120,000 were declared and paid on December 31, 20x2.
 Selected values of general price indices (CPI) are shown below:
January 1, 20x2……….……………………………………..100
Average for 20x2…………………………………………….120
December 31, 20x2…………………………………………140
 The net monetary liabilities as of January 1, 20x2 at historical cost is ₱400,000.

20.How much is the restated current cost / constant peso total assets in the December 31,
20x2 statement of financial position?
a. 5,664,000 b. 4,632,00 c. 5,784,000 d. 5,540,000

21.How much is the restated current cost / constant peso total liabilities in the December
31, 20x2 statement of financial position?
a. 520,000 b. 480,000 c. 560,000 d. 540,000

22.How much is the restated current cost / constant peso profit?


a. 1,020,000 b. 630,636 c. 620,000 d. 670,000

23.How much is the gain (loss) on net monetary position (purchasing power gain or loss)?
a. (119,200) b. 103,333 c. (103,333) d. 0

24.How much is the restated current cost / constant peso retained earnings on December
31, 20x2?
a. 500,636 b. 540,000 c. 500,000 d. 504,000

Current cost / Constant peso profit or loss


Use the following information for the next four questions:
EFFICACY EFFECTIVENESS Co. acquired inventory on January 1, 20x2 for ₱80,000. On
August 1, 20x2, when the current cost of the inventory is ₱128,000, three-fourths of the

208
inventory was sold for ₱240,000. At year-end, the current cost of the inventory on hand
was ₱32,000.

EFFICACY’s land acquired on January 1, 20x2 for ₱120,000 has a current cost of ₱160,000
on December 31, 20x2.
Operating expenses incurred evenly during the year amounted to ₱174,800.

On January 1, 20x2, EFFICACY had monetary assets of ₱800,000 and monetary liabilities of
₱400,000. During the year, EFFICACY's monetary inflows and outflows were relatively
constant and equal so that it ended the year with net monetary assets of ₱320,000.

Selected values of general price indices (CPI) are shown below:


January 1, 20x2……….……………………………………...200
August 1, 20x2………………………………………………..240
December 31, 20x2…………………………………………..260

25.How much is the profit (loss) under Historical (Nominal) cost basis?
a. 5,200 b. (5,200) c. 6,400 d. (6,400)

26.How much is the profit (loss) under Constant peso basis?


a. 96,300 b. (96,300) c. 111,600 d. (111,600)

27.How much is the profit (loss) under Current cost basis?


a. 57,200 b. (57,200) c. 36,000 d. (36,000)

28.How much is the profit (loss) under Current cost/ Constant peso basis?
a. 101,600 b. (101,600) c. 116,600 d. (116,600)

Depreciation expense
Use the following information for the next four questions:
SCURRILOUS VULGAR Co. acquired equipment on January 1, 20x1 for ₱240,000. The
equipment will be depreciated over 5 years under the straight line method. A specific price
index applicable to the equipment was 150 on January 1, 20x1 and 180 on December 31,
20x1. The CPI-U was 100 on January 1, 20x1 and 140 on December 31, 20x1. The average
CPI-U in 20x1 was 120.

29.How much is the depreciation expense under Historical cost (nominal cost) basis?
a. 48,000 b. 27,200 c. 76,200 d. 67,200

30.How much is the depreciation expense under Constant peso basis?


a. 76,200 b. 84,000 c. 67,200 d. 87,600

31.How much is the depreciation expense under Current cost basis?


a. 58,200 b. 52,400 c. 48,200 d. 52,800

32.How much is the depreciation expense under Current cost / Constant peso basis?
a. 61,600 b. 66,100 c. 68,200 d. 87,600

Cost of goods sold and Ending inventory


Use the following information for the next eight questions:
SENTENTIOUS MORALISTIC Co. has 10,000 units in inventory on January 1, 20x2 with per
unit cost of ₱20. These units were purchased on this date. Total purchases during the
period were 100,000 units at a unit cost of ₱24. Inventory on hand on December 31, 20x2
was 20,000 units. The current cost per unit of inventory was ₱32 on January 1, 20x2 and
₱40 on December 31, 20x2. The CPI-U was 100 on January 1, 20x2 and 120 on December
31, 20x2. SENTENTIOUS uses the average cost flow formula in accordance with PAS 2
Inventories.

209
33.How much is the cost of goods sold under Historical (Nominal) cost basis?
a. 2,472,800 b. 2,272,200 c. 2,127,200 d. 986,800

34.How much is the cost of goods sold under Constant peso basis?
a. 2,342,400 b. 2,515,780 c. 2,472,800 d. 2,686,800

35.How much is the cost of goods sold under Current cost basis?
a. 3,240,400 b. 3,127,780 c. 3,240,000 d. 3,800,000

36.How much is the cost of goods sold under Current cost/ Constant peso basis?
a. 3,515,780 b. 3,543,544 c. 3,686,800 d. 3,534,545

37.How much is the ending inventory under Historical (Nominal) cost basis?
a. 472,800 b. 472,200 c. 427,800 d. 486,800

38.How much is the ending inventory under Constant peso basis?


a. 342,400 b. 515,780 c. 472,800 d. 686,800

39.How much is the ending inventory under Current cost basis?


a. 840,000 b.727,700 c. 740,000 d. 800,000

40.How much is the ending inventory under Current cost/ Constant peso basis?
a. 815,780 b. 727,700 c. 860,800 d. 800,000

Restatement of deferred tax liability


41.As of December 31, 20x1, OVERBEARING PROUD Co. has a deferred tax liability of
₱24,000 arising from an equipment with carrying amount of ₱400,000 (before
restatement) and a tax base of ₱320,000. OVERBEARING’s income tax rate is 30%. The
CPI-U was 100 on January 1, 20x0 when the equipment was acquired and 140 on
December 31, 20x1. How much is the restated deferred tax liability?
a. 33,600 b. 72,000 c. 28,000 d. 72,737

The answers and solutions to the computational problems above


(Multiple choice – Computational (SET B) can be found in the
accompanying Teacher’s Manual.
Chapter 48: Theory of Accounts Reviewer
1. Entities whose functional currency is that of a hyperinflationary economy
I. disregard the concept of stable monetary unit assumption.
II. restate their financial statements to the measuring unit current at the end of the
reporting period, whether their financial statements are based on a historical cost
approach or a current cost approach,.
a. True, True b. True, False c. False, True d. False, False

2. Which of the following statements is incorrect?


a. PAS 29 does not establish an absolute rate at which hyperinflation is deemed to
arise.
b. PAS 29 establishes an absolute rate at which hyperinflation is deemed to arise,
which is an inflation rate approaching or exceeding 100%.
c. The determination of whether hyperinflation exists is a matter of judgment.
d. In making judgment of whether hyperinflation exists, PAS 29 provides some
indicators of hyperinflation.

3. General price level changes and the purchasing power of money have
a. an inverse relationship.
b. a direct relationship.

210
c. a or b under a hyperinflationary environment
d. no particular relationship

4. Which of the following statements is (are) true?


I. An increase in general price level means that the purchasing power of money has
decreased - a condition known as inflation.
II. An decrease in general price level means that the purchasing power of money has
increased - a condition known as deflation.
a. True, True b. True, False c. False, True d. False, False

5. It is an accounting method used to restate non-monetary items to the measuring unit


current at the end of the reporting period
a. Price level accounting
b. Constant purchasing power accounting
c. Constant peso accounting
d. All of these

6. Only non-monetary items, statement of financial position amounts not already


expressed in terms of the measuring unit current at the end of the reporting period, are
restated when using the constant peso accounting. Which of the following statements is
incorrect?
a. Monetary items are money held and items to be received or paid in fixed or
determinable amount of money. All other items are non-monetary except retained
earnings.
b. Non-monetary items already expressed in terms of the measuring unit current at
the end of the reporting period are not restated, e.g., those stated at NRV or fair
value as at the end of reporting period.
c. Revalued non-monetary items are restated from the date of the revaluation.
d. Non-monetary assets measured at fair value at some date other than the end of
reporting period need not be restated, according to PAS 29.

7. Net monetary items is


a. Monetary assets minus monetary liabilities
b. Monetary assets as of the start of the period
c. Nonmonetary assets minus nonmonetary liabilities
d. Monetary liabilities as of the end of the period

8. Which of the following statements is incorrect?


a. Only non-monetary items, statement of financial position amounts not already
expressed in terms of the measuring unit current at the end of the reporting period,
are restated when using the constant peso accounting.
b. After restatement, the carrying amounts of restated assets are compared to their
recoverable amounts. Any excess is recognized as impairment loss under PAS 36 or
as write-down under PAS 2, whichever standard is applicable.
c. At the beginning of the first period of application of PAS 29, any revaluation surplus
that arose in previous periods is eliminated. The balancing figure after restatement
is retained earnings.
d. Under constant peso accounting, some items in the statement of profit or loss and
other comprehensive income are restated.

9. The formula for restatement under the constant peso accounting is


a. (Current price index / Historical price index) x Historical cost
b. (Current price index / Average price index) x Historical cost
c. (Historical price index/ Current price index) x Historical cost
d. choice (a) except when the historical price index is indeterminable, then choice (b)

10. The gain or loss on net monetary position is computed as

211
a. Net monetary items, end, historical minus Net monetary items, end, restated
b. Net nonmonetary items, end, historical minus Net nonmonetary items, end, restated
c. Current cost of inventory sold minus historical cost of inventory sold
d. Depreciation on average current cost minus depreciation on historical cost

11. Which of the following statements is (are) correct?


I. Gain or loss on net monetary position (Purchasing power gain or loss) is recognized
under constant peso accounting.
II. Holding gains and losses are recognized under current cost accounting.
III. Both holding gains and losses and gain or loss on net monetary position are
recognized under current cost / constant peso accounting.
a. I only b. II only c. III only d. all of these

12. This is accomplished by applying appropriate indices to the historical cost of the assets.
a. Indexation b. Direct pricing c. Unit pricing d. Functional pricing

13. This relies on information provided by vendors and others having data about the selling
prices of replacement assets.
a. Indexation b. Direct pricing c. Unit pricing d. Functional pricing

14. When does a general purchasing power loss occur, and when is it recognized?
a. It occurs when holding net monetary assets during inflation and is recognized in
constant peso financial statements.
b. It occurs when holding net monetary liabilities during inflation and is recognized in
constant peso financial statements.
c. It occurs when holding net monetary assets during inflation and is recognized in
nominal peso financial statements and in constant peso financial statements.
d. It occurs when holding net monetary liabilities during inflation and is recognized in
nominal peso financial statements and in constant peso financial statements.
(AICPA)

15. In a hyperinflationary economy, monetary items


a. Are not restated because they are current assets or current liabilities.
b. Are restated by applying the general price index, normally the CPI
c. Are restated by applying the specific price index.
d. Are not restated because they are already expressed in terms of the measuring unit
current at the end of the reporting period.

16. When computing purchasing power gain or loss on net monetary items, which of the
following accounts is classified as nonmonetary?
a. Unamortized premium on bonds payable.
b. Accumulated depreciation of equipment.
c. Advances to unconsolidated subsidiaries.
d. Allowance for uncollectible accounts.
(AICPA)

The next two items are based on the following:


In a period of rising general price levels, RELENT TO SOFTEN Corp. discloses income on a
current cost basis.

17. Compared to historical cost gross income, which of the following conditions increases
RELENT’s current cost gross income?
a. Current cost of equipment is greater than historical cost.
b. Current cost of land is greater than historical cost.
c. Current cost of cost of goods sold is less than historical cost.
d. Ending net monetary assets are less than beginning net monetary assets.
(AICPA)

212
18. Which of the following contributes to RELENT’s purchasing power loss on net monetary
items?
a. Refundable deposits with suppliers.
b. Equity investment in unconsolidated subsidiaries.
c. Warranty obligations.
d. Wages payable.
(AICPA)

19. During a period of inflation, all of the following occur, except


a. Purchasing power decreases c. The value of the peso decreases
b. Price levels rises d. Purchasing power increases
(Adapted)

20. When purchasing power gains or losses are computed, how is each of the following
classified?
(Item #1) Patents; (Item #2) Unamortized premium on bonds payable
a. Nonmonetary; Monetary c. Monetary; Nonmonetary
b. Nonmonetary; Nonmonetary d. Monetary; Monetary
(AICPA)

21. During a period of inflation, the specific price of a parcel of land increased at a lower
rate than the consumer price index. The accounting method that would measure the
land at the highest amount is
a. Historical cost/nominal peso. c. Current cost/constant peso
b. Current cost/nominal peso. d. Historical cost/constant peso.
(AICPA)

22. LEVITY LIGHTNESS Co. prepares supplementary reports on income from continuing
operations on a current cost basis. How should LEVITY compute cost of goods sold on a
current cost basis?
a. Number of units sold times average current cost of units during the year.
b. Number of units sold times current cost of units at year end.
c. Number of units sold times current cost of units at the beginning of the year.
d. Beginning inventory at current cost plus cost of goods purchased less ending
inventory at current cost.
(AICPA)

23. Could current cost financial statements report holding gains for goods sold during the
period and holding gains on inventory at the end of the period?
(Item #1) Goods sold; (Item #2) Inventory
a. Yes, Yes b. Yes, No c. No, Yes d. No, No
(AICPA)

24. During a period of inflation in which a liability account balance remains constant, which
of the following occurs?
a. A purchasing power loss if the item is a nonmonetary liability.
b. A purchasing power gain if the item is a nonmonetary liability.
c. A purchasing power loss if the item is a monetary liability.
d. A purchasing power gain if the item is a monetary liability.
(AICPA)

25. SUSPIRE TO SIGH Co. adjusted its historical cost income statement by applying specific
price indexes to its depreciation expense and cost of goods sold. SUSPIRE's adjusted
income statement is prepared according to
a. Fair value accounting.
b. General purchasing power accounting.

213
c. Current cost/general purchasing power accounting.
d. Current cost accounting.
(AICPA)

26. During a period of inflation in which an asset account remains constant, which of the
following occurs?
a. A purchasing power gain, if the item is a monetary asset.
b. A purchasing power gain, if the item is a nonmonetary asset.
c. A purchasing power loss, if the item is a monetary asset.
d. A purchasing power loss, if the item is a nonmonetary asset.
(AICPA)

27. If the CPI-U value increased during the year, which of the following is correct?
a. An entity that reports increasing net monetary assets at the beginning and end of
the year recognizes gain on net monetary position when applying PAS 29.
b. An entity that reports increasing net monetary items at the beginning and end of the
year recognizes purchasing power gain when applying PAS 29.
c. An entity that reports increasing net monetary liabilities at the beginning and end of
the year recognizes gain on net monetary position when applying PAS 29.
d. Information is insufficient to form a valid conclusion.

28. If the CPI-U value increased during the year, which of the following is correct?
a. An entity that reports increasing net monetary assets at the beginning and end of
the year recognizes loss on net monetary position when applying PAS 29.
b. An entity that reports increasing net monetary items at the beginning and end of the
year recognizes purchasing power gain when applying PAS 29.
c. An entity that reports increasing net monetary liabilities at the beginning and end of
the year recognizes loss on net monetary position when applying PAS 29.
d. Information is insufficient to form a valid conclusion.

29. Under the stable monetary unit assumption:


a. all assets and liabilities are translated to pesos of constant purchasing power
b. inflation adjustments are incorporated directly into the financial statements
c. the peso is assumed to have constant purchasing power, regardless of the time
period
d. interest rates are increased for expected inflation

30. Most elements of financial statements are initially measured in


a. Constant pesos c. Fixed pesos
b. Nominal pesos d. Flexible pesos

31. Financial statements are most commonly prepared in accordance with an accounting
model based on
a. Recoverable historical cost and the nominal financial capital maintenance concept
b. Recoverable historical cost and the physical capital maintenance concept
c. Fair value and the nominal financial capital maintenance concept
d. Either recoverable historical cost and fair value and either nominal financial or
physical capital concept

32. Under the concept of financial capital maintenance where capital is defined in terms of
nominal monetary units, profit represents
a. the increase in nominal money capital over the period
b. the increase in that capital over the period
c. the increase in invested purchasing power over the period
d. the increase in invested purchasing power and net holding gains during the period

214
33. When the concept of financial capital maintenance is defined in terms of constant
purchasing power units, profit represents
a. the increase in nominal money capital over the period
b. the increase in that capital over the period
c. the increase in invested purchasing power over the period
d. the increase in invested purchasing power and net holding gains during the period

34. Under the concept of financial capital maintenance where capital is defined in terms of
nominal monetary units, increases in the prices of assets held over the period,
conventionally referred to as holding gains, are, conceptually
a. profits but they may not be recognized as such until the assets are disposed of in an
exchange transaction
b. profits and they may be recognized as such during the period they arise
c. not profits but they may be recognized as such over the period until the assets are
disposed of in an exchange transaction
d. not profits and they are not recognized as profits even if the assets are disposed of in
an exchange transaction

35. When the concept of financial capital maintenance is defined in terms of constant
purchasing power units and prices increase during the period
a. all of the increase in the prices of assets is considered as profits
b. only that part of the increase in the prices of assets that exceeds the increase in the
general level of prices is regarded as profit
c. that part of the increase in the prices of assets that exceeds the increase in the
general level and specific level of prices is regarded as profit, the rest of the increase
is not treated as profit
d. none of the increase is treated as profit

36. Financial statements are most commonly prepared in accordance with


a. an accounting model based on recoverable historical cost and the nominal financial
capital maintenance concept.
b. an accounting model based on fair value measurement and nominal financial capital
maintenance concept.
c. an accounting model based on recoverable historical cost and the constant
purchasing power financial capital maintenance concept.
d. an accounting model based on fair value measurement and constant purchasing
power financial capital maintenance concept.

37. PAS 29 Financial Reporting in Hyperinflationary Economies shall be applied to the


financial statements, including the consolidated financial statements, of any entity
whose functional currency is the currency of a hyperinflationary economy. Some
indicators of hyperinflation include
I. the general population prefers to keep its wealth in non-monetary assets or in a
relatively stable foreign currency. Amounts of local currency held are immediately
invested to maintain purchasing power;
II. the general population regards monetary amounts not in terms of the local currency
but in terms of a relatively stable foreign currency. Prices may be quoted in that
currency;
III. sales and purchases on credit take place at prices that compensate for the expected
loss of purchasing power during the credit period, even if the period is short;
IV. interest rates, wages and prices are linked to a price index;
V. the cumulative inflation rate over three years is approaching, or exceeds, 10%.
a. I, II, III b. I, II, III, V c. I, II, III, IV d. all of the above

38. Balance sheet amounts not already expressed in terms of the measuring unit current at
the balance sheet date are restated by applying a
a. general price index c. present value factor

215
b. specific price index d. complex formula

39. When restating the financial statements of an entity reporting under inflationary
economy, monetary items are not restated because they are already expressed in terms
of the monetary unit current at the reporting period. Monetary items are
a. financial assets
b. financial assets and financial liabilities
c. money held and items to be received or paid in money.
d. cash and cash equivalents

40. When restating the financial statements of an entity reporting under an inflationary
environment, the gain or loss on the net monetary position shall be
a. included in profit or loss and separately disclosed.
b. included in profit or loss but need not separately disclosed.
c. included in equity and in other comprehensive income.
d. no gain or loss should be recognized.

41. Which of the following statements is incorrect?


a. Constant peso accounting is the restatement of conventional or historical financial
statements in terms of the current purchasing power of the peso through the use of
index number - purchasing power or price level accounting.
b. The traditional concept of preparing financial statements based on historical cost is
known as nominal peso accounting.
c. A general price level change is increase or decrease in the overall level of prices of
goods or services throughout the economy. The relationship of the general price
level with the purchasing power of money is inverse relationship, meaning, if the
general price level increases, the purchasing power of money decreases.
d. Purchasing power means the goods and services that money can buy. If purchasing
power increases, there is inflation; if purchasing power decreases, there is deflation.
e. None, all of the following statements are correct.

42. Which of the following statements is incorrect?


a. A specific price change is increase or decrease in the price of a specific good or
service. Specific price change occurs primarily because of change in supply and
demand for a particular good or service.
b. Monetary items are units of currency held and assets and liabilities to be received or
paid in a fixed or determinable number of units of currency.
c. An essential feature of a monetary item is the right to receive or an obligation to
deliver a fixed or determinable amount of money.
d. All other items that are not monetary items are non-monetary items, with the
exception of retained earnings.
e. None, all of the above statements are correct.

43. Which of the following statements is incorrect?


a. Purchasing power gain on loss is computed under constant peso basis accounting
but not under current cost accounting.
b. Holding gain or loss is computed under current cost accounting but not under
constant peso basis accounting.
c. Purchasing power gain or loss and holding gain or loss are both computed under
constant peso accounting and current cost accounting.
d. Under constant peso basis accounting, depreciation expense is restated using the
general price index as of the current balance sheet date divided by the general price
index as of the beginning of the year.
e. Under current cost accounting, depreciation expense is based on average current
cost, in other words, depreciation expense is restated using the following
computation: historical cost plus average increase or less average decrease, divided
by the useful life.

216
44. Which of the following statements is incorrect?
a. Presentation of the information required by PAS 29 as a supplement to unrestated
financial statements is not permitted. Furthermore, separate presentation of the
financial statements before restatement is discouraged.
b. The impact of inflation is usually recognized in borrowing costs. It is not appropriate
both to restate the capital expenditure financed by borrowing and to capitalize that
part of the borrowing costs that compensates for the inflation during the same period.
This part of the borrowing costs is recognized as an expense in the period in which
the costs are incurred.
c. The financial statements of an entity whose functional currency is the currency of a
hyperinflationary economy, whether they are based on a historical cost approach or a
current cost approach, shall be stated in terms of the measuring unit current at the
balance sheet date. The corresponding figures for the previous period required by
PAS 1 Presentation of Financial Statements and any information in respect of earlier
periods shall also be stated in terms of the measuring unit current at the balance
sheet date.
d. The corresponding figures for the previous period required by PAS 1 Presentation of
Financial Statements and any information in respect of earlier periods need NOT be
stated in terms of the measuring unit current at the balance sheet date.

45. An entity has several subsidiaries that operate in a hyperinflationary economy which
uses the zloty as its local currency. Management wishes to show the financial
statements in U.S. dollars. Many of the operations of the entity are within countries that
are not hyperinflationary, and these subsidiaries use the euro as their functional
currency. What currency should the entity use to present its consolidated financial
statements?
a. U.S. dollars. c. The euro.
b. The zloty. d. The entity may use any currency.
(Adapted)

46. An entity has a subsidiary that operates in a hyperinflationary economy. The


subsidiary’s financial statements are measured in terms of the local currency, which is
the zloty. The subsidiary’s financial statements have been restated in accordance with
PAS 29. The parent is located in the United States and prepares the consolidated
financial statements in U.S. dollars. Which of the following accounting procedures is
correct in terms of the consolidation of the subsidiary’s financial statements?
a. The subsidiary’s financial statements should be prepared using the zloty and then
retranslated into U.S. dollars.
b. The subsidiary’s financial statements should be prepared using the zloty, then
restated according to PAS 29, and then retranslated into U.S. dollars at closing rates.
c. The subsidiary’s financial statements should be remeasured in U.S. dollars, then
restated according to PAS 29 and consolidated.
d. The subsidiary’s financial statements should be deconsolidated and not included in
the consolidated financial statements.
(Adapted)

47. An entity is trying to determine which liabilities are monetary and nonmonetary. Which
of the following liabilities is nonmonetary?
a. Deferred tax liability. c. Accrued expenses and other payables.
b. Current tax payable. d. Warranty obligation.

48. According to PAS 29 Financial Reporting in Hyperinflationary Economies, which of the


following would indicate that hyperinflation exists?
a. Non-monetary items increase in value, but monetary items do not
b. The cumulative inflation rate over three years is approaching, or exceeds, 10%
c. Inflation rates have exceeded interest rates in three successive years

217
d. The general population prefers to keep its wealth in non-monetary assets
(ACCA)

49. According to PAS 29 Financial Reporting in Hyperinflationary Economies, which of the


following would indicate that hyperinflation exists?
a. Sales on credit are at lower prices than cash sales
b. Inflation is approaching, or exceeds, 20% per year
c. Monetary items do not increase in value
d. People prefer to keep their wealth in non-monetary assets or a stable foreign
currency
(ACCA)

50. The gain or loss on the net monetary position in a hyperinflationary economy is
recognized in
a. Equity c. Profit or loss and separately disclosed
b. Retained earnings d. Other comprehensive income

51. An entity is reporting according to PAS 29 Financial Reporting in Hyperinflationary


Economies. Its monetary assets exceed its monetary liabilities. Are the following
statements true or false?
I. There will be a loss on the net monetary position.
II. Any gain or loss in the net monetary position of the company is recognized in other
comprehensive income.
a. False, False b. False, True c. True, False d. True, True
(ACCA)

52. According to PAS 29 Financial Reporting in Hyperinflationary Economies, which of the


following are monetary items?
I. Trade payables
II. Inventories
III. Administration costs paid in cash
IV. Loan repayable at face amount
a. I and III b. I, III and IV c. I and IV d. all of these
(ACCA)

53. Which of the following statements is incorrect?


a. Only nonmonetary items are restated from nominal pesos into constant pesos.
b. Gains or losses are recognized as a result of restating nonmonetary items are
presented immediately in profit or loss.
c. Monetary items are not restated because their sums are fixed.
d. Gains or losses are not recognized as a result of restating nonmonetary items.

54. Which of the following statements is incorrect?


a. Deferred tax assets and deferred tax liabilities are monetary items.
b. Gains or losses are recorded to reflect the increase or decrease in purchasing power
that results from holding monetary items during inflation.
c. The purchasing power gain/loss is the gain/loss from holding monetary items.
d. Deferred tax assets and deferred tax liabilities are nonmonetary items.

55. The restatement of historical peso financial statements to reflect general price level
changes results in presenting assets at
a. Lower of cost or market
b. Current appraisal values
c. Cost adjusted for purchasing power changes
d. Current replacement cost

218
56. An accounting approach whereby the financial statements are restated in terms of the
current purchasing power of the peso through the use of price index numbers:
a. monetary accounting c. inflation accounting
b. fund accounting d. stabilized accounting

57. A method of accounting based on measures of current cost or lower recoverable


amount without restatement into pesos of the same general purchasing power:
a. current cost/constant peso accounting
b. current cost/nominal peso accounting
c. historical cost/constant peso accounting
d. historical cost/nominal peso accounting

58. Restatements under constant-peso accounting involve use of the general price index. A
change in a price index from 120 to 150 during a period indicated that:
a. prices have increased 30% c. prices have increased 20%
b. prices have increased 25% d. purchasing power has declined 30%
(Adapted)

59. The following statements relate to constant peso accounting. Which statement is not
correct?
a. when preparing constant peso financial statements, accounts receivable is not
restated
b. constant peso accounting does not represent a departure from historical cost
c. purchasing power gains and losses are calculated for monetary assets only
d. holding monetary assets during a period of inflation causes purchasing power losses
(Adapted)

60. The following statements relate to current value accounting. Which statement is
correct?
a. the objective of current value accounting is to report the effects of general price
changes rather than specific price changes
b. the term “current value” may mean replacement cost, net realizable value, net
present value of expected future cash flows, or current cost
c. all items in a current value balance sheet are different in amount from what they
would be in a historical cost balance sheet
d. both purchasing power and holding gains and losses are recognized in current value
accounting
(Adapted)

61. The following statements relate to constant-peso accounting:


I. Constant-peso accounting is not synonymous with nominal peso accounting.
II. When general price-level balance sheets are prepared, they should be presented in
terms of the general purchasing power of the peso in the base year.
III. A purchasing power gain results when monetary assets are held during a period of
deflation.
IV. Financial statements adjusted for inflation by a general price level index are
prepared to provide data based on the historical cost of the assets and liabilities.
State whether the foregoing statements are true.
a. all of the statements are false c. only two statements are false
b. only one statement is false d. three statements are false
(RPCPA)

62. Financial statements prepared under the stable monetary unit assumption are
a. General price level financial statements
b. Historical cost financial statements
c. Current cost financial statements
d. Constant peso financial statements

219
63. Which of these statements is true during periods of inflation?
a. depreciation expense tends to be understated
b. depreciation expense tends to be overstated
c. the firm should change the method they use to account for depreciation
d. gross plant, plant and equipment are overstated
(RPCPA)

64. Current cost accounting focuses on changes in the specific costs of certain financial
statement items. The aim is to
a. to match changes in price levels against current revenue
b. to match historical costs against current revenues
c. to match current costs against current revenues
d. to present the value of the net amount of cash expected to be received from the sale
of an asset
(Adapted)
65. The accounting problem of recognizing change or fluctuation in the purchasing power
of a currency under a hyperinflationary economy may be taken care of by:
a. Price-level accounting c. Revaluation of fixed assets
b. Replacement cost theory d. Fair value accounting
(Adapted)

66. The financial statements of an entity whose functional currency is that of a


hyperinflationary economy shall be restated in terms of
a. Historical cost
b. Current cost
c. Fair value
d. Measuring unit current at the end of the reporting period

67. In a hyperinflationary economy, statement of financial position amounts not expressed


in the measuring unit current at the end of reporting period are restated by applying
a. General price index c. Combination of a and b
b. Specific price index d. Either a or b

68. Financial statements prepared under which of the following methods include
adjustments for both specific price changes and general price level changes?
a. Historical cost/ nominal peso c. Current cost/ constant peso
b. Current cost/ nominal peso d. Historical cost/ constant peso

69. A major objective of constant peso accounting is to maintain capital


a. In terms of a constant purchasing power.
b. In terms of operating capacity.
c. So that the company is as well off at the end of the year as at the beginning of the
year.
d. In terms of the ability to provide goods or services.
(Adapted)

70. Which of the following statements is incorrect?


a. Equipment is an asset that is considered nonmonetary in nature.
b. If a parent entity operates in a hyperinflationary economy but a subsidiary does not,
then the parent’s results should be restated for hyperinflation but the subsidiary’s
results need not be restated but should comply with PAS 21 The Effects of Changes
in Foreign Exchange Rates.
c. If a subsidiary is operating in a hyperinflationary economy and the parent entity is
not, then the parent entity would prepare financial statements using other PFRSs
and the subsidiary would use PAS 29.

220
d. All non-monetary items measured under the revaluation model need not be restated
when applying PAS 29.

71. Which of the following statements is incorrect?


a. Preferably, the financial statements of a foreign entity that is affected by very high
rates of inflation should be adjusted for the effects of changing prices before the
translation process is undertaken.
b. During a period of inflation, an account balance remains constant. With respect to
this particular account, a purchasing power loss shall be recognized if the account is
a monetary asset.
c. Constant peso accounting abandons the historical cost principle in its entirety.
d. It is appropriate to adjust financial statements for the effects of changing prices
before the translation process is undertaken where high rates of inflation exist.

72. Which of the following statements is incorrect?


a. Current value accounting abandons the historical cost principle.
b. There are various methods of determining the current value of set, restatement of
cost for changes in general price level is not one of them.
c. The historical approach to recording accounting data has been severely criticized in
periods of high inflation because often the income statement reports net income
when the economic value of the owners’ investment has declined.
d. Conventional, historical, or nominal cost basis accounting does not consider the
changes in the purchasing power of the peso.
(Adapted)

73. Which of the following most likely may indicate that hyperinflation exists?
a. Increasing volume of cash less transactions in the market.
b. Statistics show an increasing number of people who are getting hungry because of
endless corruption.
c. A bureau of investigation discovered that an indeterminable amount of counterfeit
currency is being circulated in the country.
d. Inflation rate of 26% per annum compounded annually.

74. Which of the following least likely favors the historical cost principle?
a. Assets measured at present value
b. Assets restated at the measuring unit current at the end of reporting period.
c. Assets measured at cost less accumulated depreciation.
d. Assets measured at current cost

75. The deferred tax assets and deferred tax liabilities of an entity whose functional
currency is that of a hyperinflationary economy are
a. restated by indexation
b. restated using the balance sheet liability method
c. restated by direct pricing
d. not restated

Chapter 48 - Suggested answers to theory of accounts questions


1. A 11. D 21. D 31. A 41. D 51. C 61. B 71. C
2. B 12. A 22. A 32. A 42. E 52. C 62. B 71. B
3. A 13. B 23. A 33. C 43. D 53. B 63. A 73. D
4. A 14. A 24. D 34. A 44. D 54. D 64. C 74. D
5. D 15. D 25. D 35. B 45. D 55. C 65. A 75. B
6. D 16. B 26. C 36. A 46. B 56. C 66. D
7. A 17. C 27. C 37. C 47. D 57. B 67. A
8. D 18. A 28. A 38. A 48. D 58. B 68. C
9. D 19. D 29. C 39. C 49. D 59. C 69. A
10. A 20. A 30. B 40. A 50. C 60. B 70. D

221
222
Chapter 49
PFRS for Small and Medium-sized Entities (SMEs)

Chapter 49: Multiple choice – Computational (For classroom instruction purposes)


Statement of changes in equity and Statement of income and retained earnings
1. An SME has the following information:

Retained earnings, Jan. 1 2,400


Transactions during the year:
Revenues 5,000
Dividend income 800
Operating and other
expenses 3,200
Dividends declared 350

Additional information:
 During the year, the SME changed the cost flow formula for its inventories from the
FIFO method to the weighted average method. Information on the effects of the change
is shown below:

Averag
FIFO e
Jan. 1 3,200 2,600
Dec.
31 5,300 6,400
 It was found out that depreciation in the previous period has been overstated by
₱1,200.

If the SME opts to present a statement of income and retained earnings, the bottom line in
the statement shows an amount equal to
a. 2,600 b. 2,850 c. 4,550 d. 4,050

Financial instruments
Use the following information for the next three questions:
On January 1, 20x0, an entity acquires a bond for ₱1,000, incurring transaction costs of ₱70.
Interest of ₱80 is receivable annually, in arrears, over the next five years starting December
31, 20x0. The bond has a mandatory redemption of ₱1,200 on December 31, 20x4.

2. How much is the initial carrying amount of the bond?


a. 1,070 b. 930 c. 1,000 d. 1,200

3. How much is the interest income in 20x0? (round-off interest rate to two decimal places)
a. 73 b. 84 c. 92 d. 87

4. How much is the carrying amount of the bond on December 31, 20x0?
a. 998 b. 1,093 c. 1,074 d. 1,104

Use the following information for the next three questions:


On January 1, 20x0, an entity issues a bond for ₱900, incurring transaction costs of ₱50.
Interest of ₱40 is payable annually, in arrears, over the next five years starting December
31, 20x0. The bond has a mandatory redemption of ₱1,100 on December 31, 20x4.

5. How much is the initial carrying amount of the bond?


a. 850 b. 950 c. 800 d. 900

223
6. How much is the interest expense in 20x0? (round-off interest rate to two decimal
places)
a. 94 b. 81 c. 87 d. 93

7. How much is the carrying amount of the bond on December 31, 20x0? (round-off
interest rate to two decimal places)
a. 981 b. 1,003 c. 934 d. 891

Use the following information for the next three questions:


Fact pattern
An SME sells a group of its accounts receivable with carrying amount of ₱1,000,000 and
fair value of ₱1,020,000 to a bank for ₱850,000. The SME continues to handle collections
from the debtors on behalf of the bank, including sending monthly statements, and the
bank pays the entity a market-rate fee for servicing the receivables. The entity is obliged to
remit promptly to the bank any and all amounts collected, but it has no obligation to the
bank for slow payment or non-payment by the debtors.

8. How much of the receivables are derecognized?


a. 1,000,000 b. 850,000 c. 1,020,000 d. 0

9. How much is gain or loss on derecognition?


a. 20,000 b. 170,000 c. 150,000 d. 0

10.Use the fact pattern from the preceding problem except that the SME has agreed to buy
back from the bank any receivables for which the debtor is in arrears as to principal or
interest for more than 120 days. The effect of the transaction is
a. derecognition of receivable
b. recognition of loss equal to ₱150,000
c. non-derecognition of receivable but a liability is recognized for the ₱850,000 cash
receipt.
d. a and b

Investment in associates
Use the following information for the next two questions:
On January 1, 20x1, an SME acquires 20% interest in the equity of another entity for
₱100,000. The interest acquired gives the SME significant influence over the investee.
During the period, the investee reports profit of ₱30,000 and declares dividends of
₱10,000. The fair value of the investment on December 31, 20x1 is ₱110,000. The entity
uses the PFRS for SMEs.

11.Which acceptable accounting model results to the highest carrying amount of the
investment as of December 31, 20x1?
a. Cost model c. Fair value model
b. Equity model d. None of these

12.Which acceptable accounting model results to the highest investment income


recognized in profit or loss in 20x1?
a. Cost model c. Fair value model
b. Equity model d. None of these

Leases
Use the following information for the next two questions:
SME Co. entered into a 5-year operating lease with Lessor, Inc. Annual lease is ₱360,000.
However, subsequent lease payments shall be increased taking into consideration the effect
of general inflation. The consensus forecast by local banks is that the general price level
index, as published by the government, will increase by an average of 10% annually over
the next five years.

224
13.How much is the rent expense recognized by SME Co. in the second year of the lease?
a. 439,567 b. 435,600 c. 396,000 d. 360,000

14.How much is the rent income recognized by Lessor Inc. in the third year of the lease?
a. 439,567 b. 479,106 c. 435,600 d. 479,160

Use the following information for the next four questions:


SME Co. entered into a 3-year operating lease with Lessor, Inc. Annual lease is ₱100,000.
The lease contract contains a provision for an annual increase in the rent equal to 5%. The
additional rentals are not related to general inflation.

15.How much is the balance of rent payable at the end of the first year of the lease?
a. 5,083 b. 5,938 c. 5,632 d. 0

16.How much is the balance of rent receivable at the end of the second year of the lease?
a. 5,094 b. 5,234 c. 5,167 d. 0

17.How much is the rent expense recognized by SME Co. in the second year of the lease?
a. 105,083 b. 105,000 c. 110,250 d. 150,000

18.How much is the rent income recognized by Lessor Inc. in the third year of the lease?
a. 105,083 b. 110,250 c. 115,763 d. 151,763

Provisions and contingencies


Use the following information for the next two questions:
An SME provides one-year warranty for the products it sells. During the period, the SME
made total sales of 5,000 units. It was estimated that 80% of the units sold may not have
any defects, 12% may have minor defects and 8% may have major defects. A major defect
requires repair costs of ₱100 while a minor defect requires repair costs of ₱20. Actual
repair costs incurred for defective units sold during the year amounted to ₱10,000.

19.How much warranty provision is recognized at year-end?


a. 42,000 b. 52,000 c. 58,000 d. 68,000

20.How much is the warranty expense for the period?


a. 42,000 b. 52,000 c. 68,000 d. 58,000

Revenue
Use the following information for the next two questions:
An SME sells goods in exchange for a ₱340,000 note receivable in two years’ time. The cash
sale price of the goods is ₱280,992. The gross profit rate based on cost on the transaction is
30%. Other operating expenses amounted to ₱50,000.

21.How much is the sale revenue recognized by the SME?


a. 340,000 b. 280,992 c. 64,844 d. 334,156

22.How much is the profit (loss) of the SME?


a. 14,844 b. (14,844) c. 46,988 d. 42,944

Impairment of assets
23.On December 31, 20x1, an SME tests for impairment an equipment which was acquired
8 years ago for ₱1,600,000. The equipment has an originally estimated useful life of 15
years and residual value of ₱100,000.

The fair value of the equipment on December 31, 20x1 is ₱700,000. Costs to sell are
estimated at ₱20,000. The SME makes the following cash flow projections for value in

225
use computation. The pre-tax discount rate is 12%. Cash flows are expected to occur at
the end of each year.
Year Net cash flows
20x
2 180,000
20x
3 167,400
20x
4 155,682
20x
5 144,784
20x
6 134,649
20x
7 125,224
20x
8 116,458

How much is the impairment loss?


a. 110,485 b. 112,746 c. 72,460 d. 65,250

Income tax
Use the following information for the next four questions:
An SME has a pretax profit of ₱18,000 in 20x1. The following were the transactions during
the period.
 A warranty provision of ₱3,000 has been recognized for goods sold during the period.
The amount recognized as a provision is not deductible for tax purposes until it is
actually paid or used.
 Interest receivable of ₱1,000 has been recognized on a note. The interest is taxable only
when actually collected.
 On January 1, 20x1, the SME acquired an equipment for ₱200,000. The asset is being
depreciated on a straight-line basis over 10 years. For tax purposes, the asset is
depreciated over 4 years on a straight-line basis (accelerated tax depreciation).
 Any tax losses are permitted to be carried forward as tax deduction in future periods.
However, the SME expects to use only 40% of any tax losses within the permitted carry
forward period.
 The currently enacted tax rate is 35% while the substantially enacted tax rate for 20x2
and future periods is 30%.
 The SME made quarterly income tax payments during the period for a total of ₱50,000.
 There are no temporary differences as at the beginning of the period.

24.How much is the current tax asset (liability) as of December 31, 20x1?
a. (4,000) b. 6,000 c. (40,000) d. 50,000

25.How much is the deferred tax asset as of December 31, 20x1?


a. 1,050 b. 2,450 c. 3,900 d. 2,100

26.How much is the deferred tax liability as of December 31, 20x1?


a. 9,300 b. 10,850 c. 8,700 d. 9,000

27.How much is the income tax expense as of December 31, 20x1?


a. 6,300 b. 3,600 c. 4,600 d. 0

Use the following information for the next three questions:


An SME reports pretax profit of ₱280,000. The tax rate is 30%. The pretax profit reflects
the following:
 Interest income of ₱30,000 that is subject to final tax.

226
 Entertainment expense of ₱25,000 that is not tax deductible.
 A ₱10,000 bad debt expense, of which ₱8,000 pertain to worthless receivables actually
written off and the remaining ₱2,000 pertain to estimated allowance for other
receivables. Only worthless receivables actually written off are tax deductible.
 Depreciation of ₱75,000. However, the tax deductible depreciation (accelerated) is
₱100,000.

28.How much is the current tax expense?


a. 82,500 b. 75,600 c. 83,600 d. 78,300

29.How much is the deferred tax expense (benefit)?


a. 23,000 b. (6,900) c. (23,000) d. 6,900

30.How much is the income tax expense?


a. 82,500 b. 75,600 c. 83,600 d. 78,300

The answers and solutions to the computational problems above


(Multiple choice – Computational (SET B) can be found in the
accompanying Teacher’s Manual.
Chapter 49: Theory of Accounts Reviewer (w/ problems included)
Overview
1. All of the following may not qualify as “small and medium-sized entity” (SME) except
a. banks c. investment house
b. insurance company d. cooperative

2. Which of the following most likely would not qualify as a “small and medium-sized
entity” (SME)?
a. A cooperative with total assets of ₱3M and liabilities of ₱2M.
b. A real estate company with total assets of ₱350M and liabilities of ₱250M.
c. A finance corporation with total assets of ₱2M and liabilities of ₱1M.
d. All of these entities qualify as SMEs.

3. Generally, non-financial liabilities are measured at


a. the present value of future cash flows on the obligation
b. the best estimate of the amount that would be required to settle the obligation at the
reporting date
c. the mid-point value of the obligation
d. fair value

4. Which of the following is incorrect regarding the application and compliance with the
PFRS for SMEs?
a. The application of the PFRS for SMEs, with additional disclosure when necessary, is
presumed to result in financial statements that achieve a fair presentation of the
financial position, financial performance and cash flows of SMEs.
b. The application of the PFRS for SME by an entity with public accountability does not
result in a fair presentation even when a local legislation permits entities with
public accountability to use the PFRS for SMEs.
c. An entity whose financial statements comply with the PFRS for SMEs shall make an
explicit and unreserved statement of such compliance in the notes and on the face of
each component of a complete set of financial statements as provided under the
PFRS for SME.
d. Financial statements shall not be described as complying with the PFRS for SMEs
unless they comply with all the requirements of the PFRS for SMEs.

227
5. According to the PFRS for SMEs, in assessing whether the going concern assumption is
appropriate, management takes into account all available information about the future,
which is at least, but is not limited to,
a. 12 months from the reporting date.
b. two years from the reporting date.
c. 3 months from the reporting date
d. it depends on professional judgment

6. There are a number of accounting standards and disclosures that may not provide
useful information to the users of SME financial statements. Which of the following
topics does the standard for SMEs not address?
a. Revenue c. Earnings per share
b. Provisions and contingencies d. Liabilities and equity
(Adapted)

7. The PRFRS for SMEs contains a section on transition that allows all of the exemptions in
PFRS 1 First-time Adoption of International Financial Reporting Standards. It also
contains certain exemptions for comparative information and the restatement of the
opening statement of financial position. What is the basis for the latter exemptions to
PFRS 1?
a. Impracticability b. Relevance c. Cost d. Materiality
(Adapted)

8. Which of the following approaches has the IASB taken in developing a standard for
SMEs?
a. The exemptions given to smaller entities are prescribed in the mainstream
accounting standards
b. The standard is a simplified self-contained set of accounting principles that are
based on full PFRS
c. GAAP for SMEs is to be developed on a national basis
d. The standard is an independently developed set of standards
(Adapted)

9. Where financial statements are prepared using the PFRS for SMEs, the basis of
presentation note and the auditor's report will refer to compliance with the PFRS for
SMEs. Which of the following statements suitably describes the nature of the
compliance with the standard?
a. The accounting practices used are a mix of full PFRS and the PFRS for SMEs
b. The accounting practices used are a mix of full PFRS and local GAAP
c. The SME has followed the PFRS for SMEs in its entirety
d. The accounting practices used are a mix of local GAAP and the PFRS for SMEs
(Adapted)

10.Which of the following is considered part of the objective of PFRS for SMEs?
a. To increase the audit fees of practitioners
b. To eliminate audit firms that does not comply with quality standards
c. To provide a basis on increasing the salaries of accountants
d. To provide a simplified, self-contained set of accounting principles that are
appropriate for smaller, non-listed companies and are based on full PFRSs

11.All of the following are valid reasons on adopting PFRS for SME by qualified entities,
except
a. cost benefit consideration
b. complexity of compliance with full PFRSs
c. PAS 101 Financial Reporting for Non-publicly Accountable Entities (NPAEs) which
previously provided temporary relief to NPAEs from adoption of certain items in full
PFRSs has already been superseded

228
d. automatic “unqualified” opinion on audited financial statements

12. The PFRS for SMEs became effective in the Philippines on


a. January 1, 2009 c. January 1, 2010
b. December 31, 2009 d. December 31, 2009

13.PFRS for SMEs applies to an entity with


I. total assets of between ₱3M and ₱350M(US$70,000 to $8,000,000)
II. total liabilities of between ₱3M and ₱250M (US$70,000 to $5,500,000).
 a. I only b. II only c. I or II d. none

14.In which of the following entities would the PFRS for SMEs apply?
I. Company A with total assets of ₱300M but with total liabilities of ₱1M only
II. Company B with total assets of ₱350M and total liabilities of ₱250M
III. Bank C with total assets of ₱350M and total liabilities of ₱250M
IV. Company D with total assets of ₱3M and total liabilities of ₱3M
  a. I and II b. I, II, IV c. II only d. all of the above

15.The ceiling of the threshold for total assets of an SME qualifier is


a. 400M b. 3M c. 350M d. 250M
 
16. The ceiling of the threshold for total liabilities of an SME qualifier is
a. 400M b. 3M c. 350M d. 250M

17.The floor of the threshold for total assets of an SME qualifier is


a. 2M b. 3M c. 350M d. 250M

18. The floor of the threshold for total liabilities of an SME qualifier is
a. 2M b. 3M c. 350M d. 250M

19.Entities below the threshold floor for SME qualification are called
a. Non-publicly accountable entities c. Micro entities
b. Small and medium entities d. Reportable entities

20.An entity may or may not qualify for the application of the PFRS for SMEs. If the entity
does not qualify because its total assets or total liabilities exceeds the threshold ceiling
for SME qualification, the entity
a. should use full PFRSs
b. may use another acceptable basis of accounting
c. may suffer monetary sanction from the SEC
d. a or b
 
21.An entity may or may not qualify for the application of the PFRS for SMEs. If the entity
does not qualify because its total assets or total liabilities is below the threshold floor
for SME qualification, the entity
a. may choose to use full PFRS
b. is permitted to use another acceptable basis of accounting
c. may choose to use the PFRS for SMEs
d. all of these

22.If an entity breaches the threshold floor or ceiling during an accounting period and the
change is considered “significant and continuing,” the entity should transition to the
applicable financial reporting framework
a. immediately and prospectively
b. immediately with restatement of prior period financial statements
c. after two years following the breach of threshold
d. in the next accounting period

229
23.In judging what constitutes a “significant and continuing” change that affects an SME’s
compliance with the threshold, which of the following statements is(are) true?
I. The determination of what is “significant and continuing” shall be based on
management’s judgment taking into consideration relevant qualitative and
quantitative factors.
II. As a general rule, 10% or more of total assets or total liabilities would be considered
significant.
a. I only b. II only c. I or II d. none

24.If an entity that uses a calendar year accounting initially adopts the PFRS for SMEs in
2x10, which of the following financial statements could possibly be used as reference in
determining the threshold for qualification as SME?
a. audited December 31, 2x10 financial statements
b. audited March 31, 2x10 financial statements
c. audited December 31, 2x09 financial statements
d. unaudited December 31, 2x09 financial statements

25.Which of the following may be used as “another acceptable basis of accounting” by a


micro-entity?
a. income tax basis accounting c. fraudulent accounting
b. cash basis accounting d. a or b

26.It is an accounting that follows the recognition principles applicable in the preparation
of income tax returns as provided under relevant tax laws rather than the provisions of
PFRSs.
a. income tax basis accounting c. sari-sari store accounting
b. cash basis accounting d. a or b

27.Which of the following statements is correct?


a. According to the PFRS for SMEs, SMEs are prohibited from disclosing earnings per
share and operating segment information.
b. According to the PFRS for SMEs, SMEs are prohibited from preparing interim
financial reports.
c. According to the PFRS for SMEs, SMEs may choose not to present a statement of
comprehensive income if there are no other comprehensive income in any of the
periods for which financial statements are presented, or they may present a
statement of comprehensive income in which the ‘bottom line’ is labeled “profit or
loss”.
d. According to the PFRS for SMEs, SMEs should not use titles for financial statements
other than those used in the PFRS for SMEs.

28.The PFRS for SMEs requires an entity to present its total comprehensive income for a
period in one or two financial statements. A change from the single-statement approach
to the two-statement approach, or vice versa, is
a. a change in accounting policy c. a correction of prior period error
b. a change in accounting estimate d. prohibited

29.Under the PFRS for SMEs, investments in equity instruments that are not publicly
traded and whose fair value cannot otherwise be measured reliably, and do not give the
entity significant influence, control, or joint control over the investee, shall be measured
at
a. cost b. amortized cost c. fair value d. cost less impairment

30.Under the PFRS for SMEs, investments in equity instruments that are not publicly
traded and do not give the entity significant influence, control, or joint control over the
investee, shall be measured at

230
a. cost less impairment
b. amortized cost
c. fair value unless fair value cannot be measured reliably, in which case , at cost less
impairment
d. fair value with changes in fair value recognized in other comprehensive income

31.An SME shall measure its investment in associate using


a. Fair value model b. Cost model c. Equity method d. any of these

32.If an SME does not choose to measure its investment in quoted equity instruments
classified as investment in associate using the equity method, the SME shall use
a. Fair value model b. Cost model c. Consolidation model d. a or b

33.LOONEY CRAZY FOOLISH Co. operates in a hyperinflationary environment. At the start


of the year, LOONEY had monetary assets of ₱100M and monetary liabilities of ₱150M.
At the end of the year, LOONEY had monetary assets of ₱200M and monetary liabilities
of ₱300. Which of the following statements is correct when LOONEY prepares its year-
end financial statements?
a. LOONEY will recognize a purchasing power loss in profit or loss.
b. LOONEY will recognize a purchasing power gain in profit or loss.
c. LOONEY will recognize a purchasing power gain in other comprehensive income.
d. None of these if LOONEY qualifies as an SME.

34.The PFRS for SMEs shall not be applied by an entity


a. engaged in extractive activities c. engaged in real estate development
b. organized as a credit cooperative d. regulated by the BSP

35.An entity can be a first-time adopter of the PFRS for SMEs


a. only once
b. if it has previously used the PFRS for SMEs but it has changed to full PFRSs then back
to PFRS for SMEs
c. if it has previously used the PFRS for SMEs but it has changed to Other
Comprehensive Basis of Accounting then back to PFRS for SMEs
d. any of these

36.HELLCAT WITCH Co. changed its financial reporting framework to the PFRS for SMEs in
20x2. This is the first time that HELLCAT will be using the PFRS for SMEs. HELLCAT uses
a calendar year as its financial reporting period. At the minimum, HELLCAT should
apply the provisions of the PFRS for SMEs as of
a. December 31, 20x2 c. January 1, 20x2
b. December 31, 20x1 d. January 1, 20x1

37.The accounting policies that an entity uses in its opening statement of financial position
under the PFRS for SMEs may differ from those that it used for the same date using its
previous financial reporting framework. The resulting adjustments arise from
transactions, other events or conditions before the date of transition to the PFRS for
SMEs. Therefore, an entity shall recognize those adjustments
a. in profit or loss
b. directly in retained earnings or, if appropriate, another category of equity
c. in other comprehensive income
d. a or b

38.When an entity that previously used the full PFRSs or “Other acceptable basis of
accounting” initially adopts the PFRS for SMEs, the entity is called
a. Fresh-timer c. Sweet 16
b. first-time adopter of the PFRS for SMEs d. any of these

231
39.Which of the following statements is incorrect?
a. The PFRS for SMEs permits SMEs to use the LIFO cost flow formula if this would
result to more reliable and more relevant financial information.
b. The PFRS for SMEs does not address presentation of segment information, earnings
per share, or interim financial reports by an SME. Such disclosures are normally for
entities with public accountability. An SME making such disclosures shall describe
the basis for preparing and presenting the information.
c. An entity may use titles for the financial statements other than those used in the
PFRS for SMEs as long as they are not misleading.
d. An entity using the PFRS for SMEs need not refer to the Conceptual Framework or to
the full PFRSs when developing its accounting policies.

40.If an entity qualifies as an SME and chooses to apply the PFRS for SME, the entity shal
a. apply the PFRS for SMEs in conjunction with the Conceptual Framework and the Full
PFRSs
b. apply the PFRS for SMEs but may refer to the Conceptual Framework and the Full
PFRSs in some cases stated in the PFRS for SMEs
c. apply the PFRS for SMEs in full without regard to either the Conceptual Framework
and the Full PFRSs (except for financial instruments which may be accounted for
under PAS 39.)
d. apply the PFRS for SMEs forever.

Section 1 Small and Medium-sized Entities


41.In which of the following situations can an entity that does not have public
accountability claim compliance with the IFRS for SMEs in its financial statements?
a. The entity prepares its financial statements in accordance with local tax
requirements that are substantially the same as the IFRS for SMEs.
b. The entity prepares its financial statements in accordance with local tax
requirements that are, except in name, word-for-word the same as the IFRS for
SMEs.
c. The entity prepares its financial statements in accordance with local tax
requirements that are, except in name, word-for-word the same as full IFRSs.
d. In both cases (b) and (c) above.
(IASCF)

42.In which of the following situations can an entity that does not have public
accountability claim compliance with the IFRS for SMEs in its financial statements?
a. The entity prepares its financial statements in accordance with local GAAP that is
substantially the same as the IFRS for SMEs.
b. The entity prepares its financial statements in accordance with local GAAP that is,
except in name, word-for-word the same as the IFRS for SMEs.
c. The entity prepares its financial statements in accordance with the IFRS for SMEs.
d. In both cases (b) and (c) above.
(IASCF)

43.Which of the following entities must not describe its financial statements as being in
compliance with the IFRS for SMEs even if it is required by law to prepare its financial
statements in accordance with the IFRS for SMEs?
a. the entity is a subsidiary whose parent uses full IFRSs.
b. the entity is an associate of an investor that uses full IFRSs.
c. the entity is a jointly controlled entity whose venturers (investors) use full IFRSs.
d. cases (a), (b) and (c) above.
e. none of the cases (a)–(c) above.
(IASCF)
44.Which of the following entities must not describe its financial statements as being in
compliance with the PFRS for SMEs even if it is required by law to prepare its financial
statements in accordance with the PFRS for SMEs?

232
a. an entity that holds assets in a fiduciary capacity for a broad group of outsiders as
its primary business (e.g., a bank).
b. an entity that operates two divisions in each of its retail outlets—a supermarket and
a bank (which holds assets in a fiduciary capacity for a broad group of outsiders)).
Both divisions are primary businesses of the entity.
c. an entity that operates primarily as supermarket chain. However, it also enters into
insurance contacts (as the insurer) with its customers. The entity’s short-term
insurance and life insurance operations are small relative to the size of its
supermarket operations and are operated from the entity’s supermarkets.
d. an entity holds assets in a fiduciary capacity for a broad group of outsiders for
reasons incidental to a primary business (e.g., a law firm that is legally required to
hold in trust advances from its clients for legal services to be rendered).
e. cases (a), (b) and (c) above.
f. cases (a)–(d) above.
(IASCF)

45.Which of the following entities must not describe its financial statements as being in
compliance with the PFRS for SMEs even if it is required by law to prepare its financial
statements in accordance with the PFRS for SMEs?
a. an entity whose shares are traded in a public market (e.g., a local securities
exchange).
b. an entity whose debt instruments (but not its shares) are traded in a public market
(e.g., a local securities exchange).
c. an entity that is in the process of issuing its shares for trading in a public market (eg
a local securities exchange).
d. an entity that is in the process of issuing its debt instruments (but not its shares) for
trading in a public market (e.g., a local securities exchange).
e. cases (a) and (b) above.
f. cases (a)–(d) above.
(IASCF)

Section 2 Concepts and Pervasive Principles


46.Which is not a primary objective of financial statements of an SME?
a. to provide information about the financial position, performance and cash flows of
the entity
b. to show the results of the stewardship of management
c. to provide information about economic resources, obligations, changes in them, and
cash flows of the entity.
d. to provide information about the assets, liabilities, equity, income, expenses, and
cash flows of the entity.

Section 3 Financial Statement Presentation


47.Fair presentation requires a faithful representation of the effect of transactions, other
events and conditions in accordance with the definitions and recognition criteria for
assets, liabilities, income and expenses set out in Section 2. Fair presentation, in
accordance with the PFRS for SMEs, is presumed to result from:
a. compliance with the PFRS for SMEs by an entity that has public accountability.
b. compliance with the PFRS for SMEs, with additional disclosures where necessary, by
an entity that has public accountability.
c. compliance with the PFRS for SMEs by an entity that does not have public
accountability.
d. compliance with the PFRS for SMEs, with additional disclosures where necessary, by
an entity that does not have public accountability.
(IASCF)

48.In which of the following situations can an entity that does not have public
accountability claim compliance with the IFRS for SMEs in its financial statements:

233
a. The entity prepares its financial statements in accordance with local GAAP that has
substantially converged with the IFRS for SMEs.
b. The entity prepares its financial statements in accordance with local GAAP that is,
except in name, word-for-word the same as the IFRS for SMEs.
c. The entity prepares its financial statements in accordance with the IFRS for SMEs.
d. In both cases (b) and (c) above.
(IASCF)

49.An entity that is not publicly accountable must make an explicit and unreserved
statement of compliance with the PFRS for SMEs:
a. if the entity complies with all the requirements of PFRS for SMEs.
b. if the entity complies with the vast majority of the requirements of PFRS for SMEs.
c. if the entity complies with the national GAAP based on PFRS for SMEs with some
specific differences.
d. if the entity complies with full PFRSs.
(IASCF)

50.Which of the following entities is not a going concern?


a. management intends to liquidate the entity.
b. management intends to cease the entity’s operations.
c. management has no realistic alternative but to cease the entity’s operations.
d. all of cases (a) to (c) above.
(IASCF)

51.When the classification of items in its financial statements is changed, the entity:
a. must not reclassify the comparative amounts.
b. can choose whether to reclassify the comparative amounts.
c. must reclassify the comparative amounts, unless it is impracticable to do so.
(IASCF)

52.Items of dissimilar nature or function:


a. must always be presented separately in financial statements.
b. must not be presented separately in financial statements (i.e., must be aggregated in
the financial statements).
c. must be presented separately in financial statements if those items are material.
(IASCF)

53.Materiality depends on:


a. the nature of the omission or misstatement.
b. the size of the omission or misstatement.
c. the size and nature of the omission or misstatement judged in the surrounding
circumstances.
(IASCF)

54.If the changes to the equity of an entity during the periods for which financial
statements are presented arise only from profit or loss, payment of dividends,
corrections of prior period errors, and changes in accounting policy:
a. the entity presents the income statement but not the statement of changes in equity.
b. the entity presents the statement of comprehensive income but not the statement of
changes in equity.
c. the entity presents the income statement and the statement of changes in equity.
d. the entity presents the statement of comprehensive income and the statement of
changes in equity.
e. the entity presents a single statement of income and retained earnings.
f. the entity may present the statement/s listed in either (c), (d) or (e).
(IASCF)

234
55.An entity must disclose comparative information for:
a. the previous comparable period for all amounts reported.
b. the previous comparable period for all amounts reported and for all narrative and
descriptive information.
c. the previous comparable period for all amounts reported, and for all narrative and
descriptive information when it is relevant to an understanding of the current
period’s financial statements.
d. the previous two comparable periods for all amounts reported.
(IASCF)

56.An entity shall present:


a. the statement of cash flows more prominently than the other statements.
b. the statement of financial position more prominently than the other statements.
c. the statement of comprehensive income more prominently than the other
statements.
d. each financial statement with equal prominence.
(IASCF)

Section 4 Statement of Financial Position


57.Section 4 Statement of Financial Position of the PFRS for SMEs:
a. prescribes information to be presented in a statement of financial position.
b. prescribes the sequence or format in which items are to be presented in the
statement of financial position.
c. does not permit the presentation of the additional line items, headings and subtotals
in the statement of financial position in addition to those set out in paragraph 4.2.
(IASCF)

58.In accordance with the PFRS for SMEs, an entity must present additional line items in a
statement of financial position when:
a. such presentation is relevant to an understanding of the entity’s financial position.
b. such presentation is a generally accepted practice in the sector in which the entity
operates.
c. such presentation is required by the tax authorities of the jurisdiction in which the
entity operates.
(IASCF)

59.In accordance with the PFRS for SMEs, in presenting a statement of financial position,
an entity:
a. must make the current/non-current presentation distinction.
b. must present assets and liabilities in order of liquidity.
c. must choose either the current/non-current or the liquidity presentation formats
(i.e., a ‘free’ choice of presentation format).
d. must make the current/non-current presentation distinction except when a
presentation based on liquidity provides information that is reliable and more
relevant.
(IASCF)

60.Assets to be sold, consumed or realized as part of the entity’s normal operating cycle
are:
a. current assets
b. non-current assets
c. classified as current or non-current in accordance with other criteria.
(IASCF)

61.When there is much variability in the duration of the entity’s normal operating cycle,
the operating cycle is measured at:
a. its mean value b. its median value c. twelve month d. three years

235
(IASCF)

62.Liabilities that an entity expects to settle in its normal operating cycle are:
a. classified as non-current liabilities
b. classified as current or non-current liabilities in accordance with other criteria
c. classified as current liabilities
(IASCF)

63.A dividend declared by the entity before its year-end and payable to its shareholders
three months after the end of the reporting period is classified as:
a. a non-current liability c. equity
b. a current liability d. a current asset
(IASCF)

64.An entity must present each of the line items listed in paragraph 4.2:
a. even if the amount recognized for the line item is nil
b. unless the amount recognized of the line item is nil
c. unless the line item is either immaterial or irrelevant
(IASCF)

65.In accordance with the PFRS for SMEs, the financial statement that presents an entity’s
assets, liabilities and equity at a point in time:
a. must be titled the statement of financial position
b. must be titled the balance sheet
c. could be titled the statement of financial position or the balance sheet
d. could be titled the statement of financial position, the balance sheet or any other
title that is not misleading
(IASCF)

66.A partnership that prepares its financial statements in accordance with the PFRS for
SMEs:
a. is required to disclose information equivalent to that required by paragraph 4.12(a)
showing changes during the period in each category of equity, and the rights,
preferences and restrictions attaching to each category of equity
b. is required to disclose information equivalent to that required by paragraph 4.12(a)
if the partners’ interests are classified as equity
c. is not required to report information about its equity
(IASCF)

Section 5 Statement of Comprehensive Income and Income Statement


67.In 20X8, after an entity’s 20X7 financial statements were approved for issue, the entity
discovered an error in the calculation of depreciation expense. The error occurred
during 20X6. The entity presents one year’s comparative figures. The effect of the
correction of the error in the entity’s 20X8 financial statements will be:
a. recognized in the entity’s profit or loss for the year ended 31 December 20X8.
b. recognized in the entity’s profit or loss for the year ended 31 December 20X7.
c. recognized outside of total comprehensive income, in the statement of changes in
equity as an adjustment to retained earnings at 1 January 20X7.
(IASCF)

68.Which of the following gains and losses are recognized in other comprehensive income
(i.e., in total comprehensive income outside of profit and loss)?
a. gains and losses from discontinued operations.
b. gains and losses arising on translating the financial statements of a foreign
operation.
c. gains on the revaluation of property, plant and equipment.
d. gains and losses that management considers extraordinary items.

236
(IASCF)

69.Which of the following gains and losses can an entity elect (an accounting policy choice)
to recognize in other comprehensive income (i.e., in total comprehensive income
outside of profit or loss)?
a. losses from discontinued operations.
b. gains and losses arising on translating the financial statements of a foreign
operation.
c. actuarial gains and losses of defined benefit plans.
d. gains and losses that management considers extraordinary items.
(IASCF)

70.Which of the following terms cannot be used to describe a line item in the statement of
comprehensive income?
a. revenue b. gross profit c. profit before tax d. extraordinary item
(IASCF)

71.Which of the following is a discontinued operation?


a. An entity has three machines located in one plant. All of the machines produce the
same product. The entity significantly scales down its operations by disposing of
one of the machines.
b. An entity has three machines located in one plant. Each machine produces a
completely different product and each machine is managed as a separate business
unit. The entity significantly scales down its operations by disposing of one of the
machines and in doing so discontinues manufacturing one of its three products.
c. An entity has three plants that all produce the same product. Each plant is located in
a separate continent and sells its output to customers local to the plant in which the
product is manufactured. The entity scales down its operations by disposing of one
of the plants.
d. Both (b) and (c) above.
e. Situations (a)–(c).
(IASCF)

72.Items of other comprehensive income are presented in the statement of comprehensive


income analyzed:
a. by nature.
b. by function.
b. either by nature or by function (an accounting policy choice).
c. both (a) and (b) above.
(IASCF)

73.Staff costs are:


a. administrative expenses.
b. distribution expenses.
c. cost of sales.
d. allocated to categories (a) - (c) above according to the function of the employee to
which the particular staff cost relates.
(IASCF)
74.An entity presents an analysis of expenses using a classification based on:
a. the nature of expenses.
b. the function of expenses.
c. either the nature of expenses or the function of expenses within the entity,
whichever provides information that is reliable and more relevant.
d. either the nature of expenses or the function of expenses within the entity,
whichever the entity would prefer to present.
(IASCF)

237
75.Separate line items in an analysis of expenses by nature include:
a. purchases of materials, transport costs, employee benefits, depreciation,
extraordinary items.
b. purchases of materials, distribution costs, administrative costs, employee benefits,
depreciation, taxes.
c. depreciation, purchases of materials, employee benefits and advertising costs.
d. cost of sales, administrative costs, transport costs, distribution costs etc.
(IASCF)

76.Separate line items in an analysis of expenses by function include:


a. purchases of materials, transport costs, employee benefits, depreciation,
extraordinary items.
b. purchases of materials, distribution costs, administrative costs, employee benefits,
depreciation, taxes.
c. depreciation, purchases of materials, employee benefits and advertising costs.
d. cost of sales, administrative expenses, distribution expenses etc.
(IASCF)

Section 6 Statement of Changes in Equity and Statement of Income and Retained


Earnings
77.The statement of changes in equity includes a reconciliation between:
a. the carrying amount of retained earnings at the beginning and the end of the period.
b. the carrying amount of total equity at the beginning and the end of the period.
c. the carrying amount of each component of equity at the beginning and the end of the
period separately disclosing changes resulting from: (i) profit or loss, (ii) each item
of comprehensive income, and (iii) the amounts of investments by, and dividends
and other distributions to, owners.
(IASCF)

78.In the statement of changes in equity the effects of the retrospective application of a
change in accounting policy is presented:
a. separately for each component of equity only.
b. in aggregate for total equity only.
c. in aggregate for total equity and separately for the total amounts attributable to
owners of the parent and to non-controlling interests.
(IASCF)
79.Which of the following are presented in the statement of changes in equity?
a. investments by owners (e.g., issues of shares).
b. distributions to owners (e.g., dividends).
c. changes in ownership interests in subsidiaries that do not result in a loss of control.
d. all of (a)–(c) above.
e. none of (a)–(c) above.
(IASCF)

80.An entity:
a. must chose to present either a statement of income and retained earnings or a
statement of comprehensive income and a statement of changes in equity (i.e., a free
accounting policy choice available to all entities that prepare their financial
statements in accordance with the PFRS for SMEs).
b. whose only changes to its equity in the periods for which financial statements are
presented arise from profit or loss, payment of dividends, corrections of prior
period errors, and changes in accounting policy is required to present a statement of
income and retained earnings in place of a statement of comprehensive income and
a statement of changes in equity.
c. whose only changes to its equity in the periods for which financial statements are
presented arise from profit or loss, payment of dividends, corrections of prior
period errors, and changes in accounting policy is permitted but not required to

238
present a statement of income and retained earnings in place of a statement of
comprehensive income and a statement of changes in equity.
d. that chooses to present a statement of income and retained earnings must also
present a statement of comprehensive income and a statement of changes in equity.
(IASCF)

81.The statement of income and retained earnings presents:


a. an entity’s profit or loss and retained earnings at the beginning and at the end of the
reporting period.
b. an entity’s profit or loss and, dividends declared and paid or payable in the period.
c. an entity’s profit or loss retained earnings at the beginning of the reporting period,
dividends declared and paid or payable in the period and retained earnings at the
end of the reporting period.
d. the items required by Section 5 Statement of Comprehensive Income and Income
Statement, retained earnings at the beginning of the reporting period, dividends
declared and paid or payable in the period, restatements of retained earnings for
corrections of prior period errors and changes in accounting policies and retained
earnings at the end of the reporting period.
(IASCF)

82.The existence of which of the following transactions in the current reporting period
would preclude an entity from electing to present a statement of income and retained
earnings?
a. The entity distributes land and buildings (classified as investment property) as a
dividend to its only shareholder.
b. The entity distributes land and buildings (classified as property, plant and
equipment) as a dividend to its only shareholder.
c. The entity distributes land and buildings (classified as inventory) as a dividend to its
only shareholder.
d. The entity acquired 100 of its own shares from one of its two shareholders.
(IASCF)

83.The existence of which of the following events in the current reporting period would
preclude an entity from electing to present a statement of income and retained
earnings?
a. The discovery of a material prior period error. Note: the error is corrected by
retrospective restatement.
b. The voluntary change of an accounting policy. Note: the new accounting policy is
applied retrospectively.
c. A change from presenting the analysis of expenses from a presentation by function
to a presentation by nature.
d. None of (a) -(c) above.
(IASCF)

84.Total comprehensive income for the period is presented in the statement of changes in
equity:
a. showing separately the total amount attributable to owners of the parent and to
non-controlling interests.
b. showing separately an analysis of expenses by function or by nature.
c. showing separately the items required by Section 5 Statement of Comprehensive
Income and Income Statement.
d. showing separately profit or loss and the total of other comprehensive income.
(IASCF)

85.In the statement of changes in equity the effects of the correction of a prior period error
are presented:
a. separately for each component of equity.

239
b. in aggregate for total equity.
c. in aggregate for total equity and separately for the total amounts attributable to
owners of the parent and the non-controlling interests.
(IASCF)

86.Since its formation a subsidiary was owned 75 per cent by the reporting entity (parent)
and 25 per cent by an independent third party. In the current reporting period when
the subsidiary’s equity was ₱100,000 (i.e., share capital of ₱1,000 and retained earnings
of ₱99,000) the parent acquired the remaining 25 per cent of the shares in its
subsidiary at their fair value of ₱60,000. How would the group present the parent’s
acquisition of 25 per cent of the equity of its subsidiary from the non-controlling
interest in its consolidated statement of changes in equity?
a. The entity would show a separate line item in which ₱25,000 would be shown as a
reduction in non-controlling interest.
b. The entity would show a separate line item in which ₱60,000 would be shown as a
reduction in non-controlling interest.
c. The entity would show a separate line item in which ₱25,000 would be shown as a
reduction in non-controlling interest and ₱35,000 would be shown as a reduction in
retained earnings.
(IASCF)

87.The facts are the same as in the immediately preceding question. However, in this
question, in the current reporting period the parent sold 25 per cent of the shares in its
subsidiary at their fair value of ₱60,000. The parent did not lose control of the
subsidiary. How would the group present the parent’s sale of 25 per cent of the equity
of its subsidiary to the non-controlling interest in its consolidated statement of changes
in equity?
a. The entity would show a separate line item in which ₱25,000 would be shown as an
increase in non-controlling interest.
b. The entity would show a separate line item in which ₱60,000 would be shown as an
increase in non-controlling interest.
c. The entity would show a separate line item in which ₱25,000 would be shown as an
increase in non-controlling interest and ₱35,000 would be shown as an increase in
retained earnings.
(IASCF)

Section 7 Statement of Cash Flows


88.When preparing a statement of cash flows, cash flows arising from transactions in
foreign currency during the year shall be translated to the entity’s functional currency
using
a. the exchange rate at the end of reporting period
b. the exchange rate at the date of the cash flow
c. the average exchange rate during the period
d. combination of a, b and c

Section 8 Notes to the Financial Statements


89.Notes to the financial statements:
a. contain only information required to be disclosed by the PFRS for SMEs that was not
presented in the statement of financial position, statement of comprehensive
income, statement of changes in equity or cash flow statement.
b. contain information required by Section 8 Notes to the Financial Statements without
reference to the other sections of the PFRS for SMEs.
c. contain the information required to be disclosed by the PFRS for SMEs that was not
presented in the statement of financial position, statement of comprehensive
income, statement of changes in equity or statement of cash flows and additional
information relevant to an understanding of the financial statements.
(IASCF)

240
90.The cross-reference between each line item in the financial statements and any related
information disclosed in the notes to the financial statements:
a. is voluntary. b. is mandatory. c. depends on the industry.
(IASCF)
91.The presentation of the notes to the financial statements in a systematic manner:
a. is voluntary. c. is mandatory, as far as is practicable
b. is mandatory.
(IASCF)

92.An entity normally presents the notes in the following order:


a. First, a statement that the financial statements have been prepared in compliance
with the PFRS for SMEs. Second, a summary of significant accounting policies
applied. Third, supporting information for items presented in the financial
statements, in the sequence in which each statement and each line item is
presented. Last, any other disclosures.
b. First, supporting information for items presented in the financial statements, in the
sequence in which each statement and each line item is presented. Second, a
statement that the financial statements have been prepared in compliance with the
PFRS for SMEs. Third, a summary of significant accounting policies applied. Last, any
other disclosures.
c. First, supporting information for items presented in the financial statements, in the
sequence in which each statement and each line item is presented. Second, a
summary of significant accounting policies applied. Third, a statement that the
financial statements have been prepared in compliance with the PFRS for SMEs.
Last, any other disclosures.
(IASCF)

93.An entity shall disclose in the summary of significant accounting policies:


a. the measurement basis (or bases) used in preparing the financial statements.
b. all the measurement bases specified in the PFRS for SMEs irrespective of whether
they were used by the entity in preparing its financial statements.
c. the measurement basis (or bases) used in preparing the financial statements and
the accounting policies used that are relevant to an understanding of the financial
statements.
d. all of the measurement bases and the accounting policy choices available to the
entity (i.e., specified in the PFRS for SMEs) irrespective of whether they were used
by the entity in preparing its financial statements.
(IASCF)

94.Disclosure of information about key sources of estimation uncertainty:


a. is voluntary. b. is mandatory.
(IASCF)

95.Disclosure of information about judgments, apart from those involving estimations, that
management has made in the process of applying the entity’s accounting policies and
that have the most significant effect on the amounts recognized in the financial
statements:
a. is voluntary. b. is mandatory.
(IASCF)

Section 9 Consolidated and Separate Financial Statements


96.According to the PFRS for SMEs a parent entity shall present
a. consolidated financial statements c. a and b
b. separate financial statements d. none

Section 10 Accounting Policies, Estimates and Errors

241
97.Prospective application of a change in accounting policy means:
a. applying the new accounting policy to transactions, other events and conditions
occurring after the date as at which the financial statements are authorized for
issue.
b. applying the new accounting policy to transactions, other events and conditions
occurring between the date as at which the policy is changed and the date when the
financial statements are authorized for issue.
c. applying the new accounting policy to transactions, other events and conditions
occurring after the date as at which the policy is changed.
d. applying a new accounting policy to transactions, other events and conditions as if
that policy had always been applied.
(IASCF)

98.Retrospective application of a change in accounting policy means:


a. applying a new accounting policy to transactions, other events and conditions,
identified before the date when the financial statements are authorized for issue, as
if that policy had always been applied.
b. applying the new accounting policy to transactions, other events and conditions
occurring between the date as at which the policy is changed and the date when the
financial statements are authorized for issue.
c. applying the new accounting policy to transactions, other events and conditions
occurring after the date as at which the policy is changed.
d. applying a new accounting policy to transactions, other events and conditions as if
that policy had always been applied.
(IASCF)

99.Which of the following statements is true?


a. The effect of a change in accounting estimate is recognized retrospectively.
b. To the extent practicable, an entity must correct a prior period error prospectively
in the first financial statements authorized for issue after its discovery.
c. When an entity discovers an error in its financial statements of a prior period, it
must immediately withdraw those financial statements and reissue them with the
error corrected.
d. To the extent practicable, an entity must correct a prior period error retrospectively
in the first financial statements authorized for issue after its discovery.
(IASCF)

100. On 25 March 20X4 the entity discovered that, as a result of a computational error,
depreciation expense for 20X3 is overstated by ₱29,000. The entity’s 31 December
20X3 financial statements were authorized for issue on 1 March 20X4. The entity must:
a. reissue its 31 December 20X3 financial statements with the correct depreciation
expense.
b. reduce depreciation for the year ended 31 December 20X4 by ₱29,000 (i.e.,
prospective allocation—a change in accounting estimate).
c. restate (correct) the depreciation expense reported for the year ended 31 December
20X3 in the comparative figures of its 20X4 financial statements (i.e., retrospective
restatement of a prior period error).
(IASCF)

101. The facts are the same as in the immediately preceding question. However, the
entity’s 31 December 20X3 financial statements were authorized for issue on 1 April
20X4. The entity must:
a. correct its 31 December 20X3 financial statements before issuing them.
b. reduce depreciation for the year ended 31 December 20X4 by ₱29,000 (i.e.,
prospective allocation—a change in accounting estimate).

242
c. restate (correct) the depreciation expense reported for the year ended 31 December
20X3 in the comparative figures of its 20X4 financial statements (i.e., retrospective
restatement of a prior period error).
(IASCF)

102. On 20 February 20X5, before an entity’s 31 December 20X4 financial statements


were authorized for issue, a court ordered the entity to pay ₱120,000 damages in full
and final settlement of a patent infringement lawsuit brought against the entity by one
of its competitors. The patent infringement occurred in 20X3. The amount of damages
awarded to the competitor was significantly higher than the ₱10,000–₱30,000 that the
entity had justifiably expected to pay throughout the duration of the case. The entity
will not contest the judgment. In its 31 December 20X3 annual financial statements the
entity reported its liability for the lawsuit at ₱20,000—this estimate was appropriately
made taking account of all available evidence at the time the financial statements were
authorized for issue. In its 31 December 20X4 financial statements the entity must:
a. restate the comparative information at 31 December 20X3 (i.e., retrospective
restatement of a prior period error).
b. measure the provision at 31 December 20X4 at ₱120,000 (comparative information
20X3: ₱20,000), i.e., account prospectively for the change in accounting estimate in
its 20X4 financial statements.
c. measure the provision at 31 December 20X4 at ₱20,000 (comparative information
20X3: ₱20,000) and record the effect of the higher than expected settlement in
profit or loss for the year ended 31 December 20X5 (i.e., account prospectively for
the change in accounting estimate in the period that the final settlement amount
was determined).
(IASCF)

103. On 20 February 20X5, before an entity’s 31 December 20X4 financial statements


were authorized for issue, a court ordered the entity to pay ₱120,000 damages in full
and final settlement of a patent infringement lawsuit brought against the entity by one
of its competitors. The patent infringement occurred in 20X3. The amount of damages
awarded to the competitor was consistent with similar cases settled in that jurisdiction
since 20X2. In its 31 December 20X3 annual financial statements the entity reported its
liability for the lawsuit at ₱20,000. The entity deliberately understated the amount
presented, because it did not want to make public its true estimate, believing that this
would be detrimental to its defense. In its 31 December 20X4 financial statements the
entity must:
a. restate the comparative information at 31 December 20X3 (ie retrospective
restatement of a prior period error).
b. measure the provision at 31 December 20X4 at ₱120,000 (comparative information
20X3: ₱20,000), i.e., account prospectively for the change in accounting estimate in
its 20X4 financial statements.
c. measure the provision at 31 December 20X4 at ₱20,000 (comparative information
20X3: ₱20,000) and record the effect of the higher than expected settlement in
profit or loss for the year ended 31 December 20X5 (ie account prospectively for the
change in accounting estimate in the period that the final settlement amount was
determined).
(IASCF)

104. In 20X5, in accordance with the entity’s newly formulated equity remuneration
scheme, the entity issued options to acquire 100 of its own shares to each of its 6,000
employees. This is the only share-based payment transaction into which the entity has
ever entered. In its 31 December 20X5 annual financial statements, Entity A accounted
for the equity-settled share-based payment transaction in accordance with Section 26
Share-based Payment. Must the entity account for a change in accounting policy in its
20X5 financial statements?
a. Yes b. No

243
(IASCF)

105. Which of the following statements is true?


a. Financial statements of subsequent periods need not repeat the disclosures
required for a change in accounting policy and the correction of a prior period error.
b. Financial statements of subsequent periods must repeat the disclosures required for
a change in accounting policy and the correction of a prior period error.
c. Financial statements of subsequent periods must repeat the disclosures required for
a change in accounting policy and the correction of a prior period error unless it is
impracticable to identify the period to which they relate.
(IASCF)

106. Which of the following is not a change in accounting policy?


a. In the current reporting period an entity changed the basis on which it measures a
building that is an investment property from the fair value model to the cost model
because fair value can no longer be measured reliably without undue cost or effort
on an ongoing basis.
b. An entity measures its only investment property at fair value. In the current
reporting period, the entity acquired a second investment property which it
measures using the cost model because the fair value of the second investment
property cannot be measured reliably without undue cost or effort on an ongoing
basis. It continues to account for the first investment property using the fair value
model.
c. In the current reporting period the entity changed the method on which it calculates
depreciation of buildings, classified as property, plant and equipment, from the
reducing balance method to the straight-line method.
d. All of (a)–(c) above.
(IASCF)

Section 11 Basic Financial Instruments


107. Under the PFRS for SMEs an entity can choose to apply the provisions of both
Section 11 and Section 12 in full, or alternatively the entity may apply:
a. full PFRSs for financial instruments (i.e., the recognition and measurement
provisions of PAS 39 Financial Instruments: Recognition and Measurement, and the
presentation and disclosure requirements of PAS 32 Financial Instruments:
Presentation and PFRS 7 Financial Instruments: Disclosures).
b. the recognition and measurement provisions of Section 11 and Section 12 and the
disclosure requirements of PFRS 7 Financial Instruments: Disclosures.
c. the recognition and measurement provisions of PAS 39 Financial Instruments:
Recognition and Measurement and the disclosure requirements of Section 11 and
Section 12.
d. the recognition and measurement provisions of PAS 39 Financial Instruments:
Recognition and Measurement and the disclosure requirements of PFRS 7 Financial
Instruments: Disclosures.
(IASCF)

108. Which of the following items in an entity’s statement of financial position is a


financial asset or financial liability within the scope of Section 11?
a. a liability for an amount due to a supplier for a past receipt of goods.
b. an asset for a prepayment made to a supplier for the rent of a machine for two
months.
c. a liability for a fine for the late payment of income tax by the entity.
d. all of the above.
(IASCF)

109. Which of the following financial assets is not in the scope of Section 11?
a. cash.

244
b. trade receivables.
c. a 5 per cent holding in the non-puttable ordinary shares of another entity (investee).
d. a 30 per cent holding in the non-puttable ordinary shares of another entity
(investee) where the investee is classified as an associate of the entity.
(IASCF)

110. Which of the following financial instruments are not within the scope of Section 11?
a. investments in non-convertible, non-puttable preference shares.
b. financial instruments that meet the definition of an entity’s own equity.
c. a fixed-interest fixed-term loan from a bank.
d. an interest-free three-year loan from a parent entity.
(IASCF)

111. An entity buys 100 non-puttable ordinary shares in a listed company on the market
for cash of ₱20 per share. The entity also incurred broker’s fees of ₱100. At what
amount should the entity measure the investment in shares on initial recognition?
a. ₱1,900. b. ₱2,000. c. ₱2,100.

112. A bank provides an entity with a five-year loan for ₱10,000 with fixed interest
payable annually in arrears at a rate of 6 per cent of the principal amount. Six per cent is
considered to be the market rate for a similar five-year loan with interest payable
annually in arrears. The bank charges the entity a fee of ₱50 for paperwork. At what
amount should the entity measure the loan on initial recognition?
a. ₱9,384. b. ₱9,484. c. ₱9,950. d. ₱10,000. e. ₱10,050.
(IASCF)

113. At the end of each reporting period investments in non-convertible preference


shares and non-puttable ordinary or preferences shares should be measured as follows:
a. all such investments shall be measured at fair value with changes in fair value
recognized in profit or loss.
b. all such investments shall be measured at amortized cost using the effective interest
method.
c. all such investments shall be measured at cost less impairment.
d. if the shares are publicly traded, the investment should be measured at fair value
with changes in fair value recognized in profit or loss. All other such investments
must be measured at cost less impairment.
e. if the shares are publicly traded or their fair value can otherwise be measured
reliably, the investment should be measured at fair value with changes in fair value
recognized in profit or loss. All other such investments must be measured at cost
less impairment.
(IASCF)

114. On 1 January 20X1 an entity provides an employee with a four-year interest-free


loan of ₱1,000. The market rate of interest for a loan to this individual is 8 per cent per
year. At what amount is the loan measured on initial recognition and how much interest
income is recognized for the year ended 31 December 20X1?
a. on 1 January 20X1 the loan is measured at ₱735.03. Interest income for the year
ended 31 December 20X1 is ₱0 in 20X1.
b. on 1 January 20X1 the loan is measured at ₱735.03. Interest income for the year
ended 31 December 20X1 is ₱58.80 in 20X1.
c. on 1 January 20X1 the loan is measured at ₱1,000. Interest income for the year
ended 31 December 20X1 is ₱0 in 20X1.
d. on 1 January 20X1 the loan is measured at ₱1,000. Interest income for the year
ended 31 December 20X1 is ₱80 in 20X1.
(IASCF)

245
115. When assessing financial assets held at amortized cost or cost for impairment an
entity must assess which of the following assets individually?
a. only financial assets that are individually significant.
b. only equity instruments that are individually significant.
c. only equity instruments.
d. all financial assets except equity instruments.
e. all equity instruments and other financial assets that are individually significant.
(IASCF)

116. An entity sells a group of its accounts receivable to a bank at less than their ‘face
amount’. The entity continues to handle collections from the debtors on behalf of the
bank, and the bank pays the entity a market-rate fee for servicing the receivables. The
entity is obliged to remit promptly to the bank any and all amounts collected, but it has
no obligation to the bank for slow payment or non-payment by the debtors. What is the
correct accounting treatment for this transaction?
a. The entity should remove the receivables from its statement of financial position
(i.e., derecognize them), and show no liability in respect of the proceeds received
from the bank.
b. The entity should continue to recognize the receivables in its statement of financial
position and show a liability in respect of the proceeds received from the bank.
c. The entity should continue to recognize the receivables in its statement of financial
position and show no liability in respect of the proceeds received from the bank.
d. The entity should remove the receivables from its statement of financial position
(i.e., derecognize them), and show a liability in respect of the proceeds received
from the bank.
(IASCF)

Section 12 Other Financial Instruments Issues


117. Which of the following is not considered as basic financial instrument and is,
therefore, outside the scope of Section 11 of the PFRS for SMEs and included within the
scope of Section 12?
a. Asset-backed securities, such as collateralized mortgage obligations, repurchase
agreements and securitized packages of receivables.
b. Options, rights, warrants, futures contracts, forward contracts and interest rate
swaps that can be settled in cash or by exchanging another financial instrument.
c. Financial instruments that qualify and are designated as hedging instruments
d. Investments in non-convertible preference shares and non-puttable ordinary shares
or preference shares.

Section 13 Inventories
118. Inventories are defined as:
a. assets held for sale in the ordinary course of business, in the process of production
for such sale, or in the form of materials or supplies to be consumed in the
production process or in the rendering of services.
b. assets held for sale, in the process of production, or in the form of materials or
supplies to be consumed in the production process.
c. tangible assets held for sale in the ordinary course of business, in the process of
production, or in the form of materials or supplies to be consumed in the production
process or in the rendering of services.
(IASCF)

119. Inventories must be measured at:


a. cost.
b. the lower of cost and estimated selling price less costs to complete and sell.
c. the lower of cost and fair value less costs to complete and sell.
(IASCF)

246
120. The cost of inventory is the sum of:
a. costs of purchase and costs of conversion.
b. direct costs, indirect costs and other costs (allocated production overheads).
c. costs of purchase, costs of conversion (e.g., allocated production of overheads) and
other costs incurred in bringing the inventory to their present location and
condition.
(IASCF)

121. The cost of inventories does not include:


a. salaries of factory staff.
b. storage costs necessary in the production process before a further production stage
and selling costs.
c. abnormal amounts of wasted materials and selling costs.
(IASCF)

122. An entity must assign the cost of inventories by:


a. using the LIFO cost formula.
b. using specific identification of individual costs for inventories that are not ordinarily
interchangeable and, for inventories that are ordinarily interchangeable, the first in
first out (FIFO) method or the weighted average cost formula.
c. specific identification of individual costs for inventories that are ordinarily
interchangeable, and, for inventories that are not ordinarily interchangeable, the
first in-first out (FIFO) method or the weighted average cost formula.
(IASCF)

123. Consumable stores (i.e., supplies to be consumed in the production process) are:
a. inventories. c. investment property
b. property, plant and equipment d. intangible assets
(IASCF)

124. On 1 January 20X1 an entity acquired goods for sale in the ordinary course of
business for ₱100,000, including ₱5,000 refundable purchase taxes. The supplier
usually sells goods on 30 days’ interest-free credit. However, as a special promotion, the
purchase agreement for these goods provided for payment to be made in full on 31
December 20X1. In acquiring the goods transport charges of ₱2,000 were incurred:
these were due on 1 January 20X1. An appropriate discount rate is 10 per cent per year.
The entity shall measure the cost of inventories at:
a. ₱102,000 b. ₱97,000 c. ₱88,364 d. ₱107,000
(IASCF)

125. On 1 January 20X1 an entity acquired 100 units of goods for sale in the ordinary
course of business for ₱100,000. On 1 March 20X1 20 further units were acquired for
₱20,400. On 1 August 20X1 30 units were sold for ₱33,000. The entity assigns the cost
of inventories by using the first-in, first-out (FIFO) formula. On 31 December 20X1, the
entity must measure the carrying amount of the 90 units of goods at:
a. ₱100,000 b. ₱90,000 c. ₱90,400 d. ₱91,800
(IASCF)

126. A retailer of perishable produce seeks to avoid obsolescence by arranging its


produce in such a way that customers are most likely to purchase the oldest inventory
first. The cost formula that is most appropriate for the entity is:
a. FIFO b. LIFO c. weighted average cost d. specific identification
(IASCF)

127. A property developer must classify properties that it holds for sale in the ordinary
course of business as:
a. inventory c. financial asset

247
b. property, plant and equipment d. investment property
(IASCF)

Section 14 Investments in Associates


128. An associate is:
a. an entity over which the investor has significant influence.
b. an entity over which the investor has joint control.
c. an entity over which the investor has significant influence or joint control and that is
not a subsidiary.
d. an entity over which the investor has significant influence and that is neither a
subsidiary nor an interest in a joint venture.
(IASCF)

129. Significant influence is:


a. the power to participate in the financial and operating policy decisions of the
investee but is not control or joint control over those policies.
b. active participation in the financial and operating policy decisions of the investee
but is not control or joint control over those policies.
c. the power to govern the financial and operating policies of an entity so as to obtain
benefits from its activities.
d. the contractually agreed sharing of control over an economic activity.
(IASCF)

130. An entity must account for its investments in associates after initial recognition
using:
a. either the cost model or the fair value model (using the same accounting policy for
all investments in associates).
b. either the cost model or the fair value model (model can be elected on an
investment-by-investment basis).
c. either the cost model, the equity method or the fair value model (using the same
accounting policy for all investments in associates).
d. either the cost model, the equity method or the fair value model (model can be
elected on an investment-by-investment basis).
(IASCF)

131. Investments in associates must be tested for impairment in accordance with Section
27 Impairment of Assets, if the entity uses:
a. the cost model, equity method or fair value model.
b. the cost model or the equity method.
c. the cost model or the fair value model.
d. the equity method or the fair value model.
(IASCF)

132. Entity A owns 30 per cent of the ordinary shares that carry voting rights at a general
meeting of shareholders of entity C. In the absence of evidence to the contrary, entity A:
a. has significant influence over entity C.
b. has significant influence over entity C, provided that it does not have joint control
over entity C.
c. has significant influence over entity C, provided that it does not have control over
entity C.
d. has significant influence over entity C, provided that it does not have control or joint
control over entity C.
(IASCF)

133. Which, if any, of the scenarios below would not lead to the presumption that an
entity exerts significant influence over another entity?
a. holding directly 20 per cent or more of the voting power of the investee.

248
b. holding indirectly, through a subsidiary, 20 per cent or more of the voting power of
the investee.
c. holding indirectly, through a joint venture, 20 per cent or more of the voting power
of the investee.
d. holding directly 10 per cent of voting power of the investee and holding indirectly,
through a subsidiary, 10 per cent of the voting power of the investee.
(IASCF)

134. On 31 December 20X1 entity A acquired 30 per cent of the ordinary shares that
carry voting rights of entity B for ₱100,000. Entity A incurred transaction costs of
₱1,000 in acquiring these shares. Entity A has significant influence over entity B. Entity
A uses the cost model to account for its investments in associates. In January 20X2
entity B declared and paid a dividend of ₱20,000 out of profits earned in 20X1. No
further dividends were paid in 20X2, 20X3 or 20X4. A published price quotation does
not exist for entity B. At 31 December 20X1, 20X2 and 20X3, in accordance with Section
27 Impairment of Assets, management assessed the fair values of its investment in
entity B as ₱102,000, ₱110,000 and ₱90,000 respectively. Costs to sell are estimated at
₱4,000 throughout. Entity A measures its investment in entity B on 31 December 20X1,
20X2 and 20X3 respectively at:
a. ₱100,000, ₱100,000, ₱100,000. d. ₱98,000, ₱101,000, ₱86,000.
b. ₱95,000, ₱95,000, ₱86,000. e. ₱102,000, ₱110,000, ₱90,000.
c. ₱98,000, ₱106,000, ₱86,000. f. ₱101,000, ₱101,000, ₱101,000.
(IASCF)

135. The facts are the same as in the immediately preceding question. However, in this
example, a published price quotation exists for entity B. Entity A measures its
investment in entity B on 31 December 20X1, 20X2 and 20X3 respectively at:
a. ₱100,000, ₱100,000, ₱100,000. d. ₱98,000, ₱101,000, ₱86,000.
b. ₱95,000, ₱95,000, ₱86,000. e. ₱102,000, ₱110,000, ₱90,000.
c. ₱98,000, ₱106,000, ₱86,000. f. ₱101,000, ₱101,000, ₱101,000.
(IASCF)

136. Which of the following indicators could provide evidence to support the existence of
significant influence by an investor?
a. representation on the board of directors or equivalent governing body of the
investee.
b. participation in policy-making processes, including participation in decisions about
dividends or other distributions.
c. material transactions between the investor and the investee.
d. interchange of managerial personnel.
e. provision of essential technical information.
f. (a) and (b) above.
g. all of the above.
(IASCF)

137. Which of the following statements is false?


a. Significant influence can be gained or lost without a change in absolute or relative
ownership levels.
b. In determining whether an entity has significant influence over another entity, the
existence and effect of potential voting rights that it holds that are currently
exercisable or convertible are considered.
c. An entity considers the existence and effect of potential voting rights held by other
parties that are currently exercisable or convertible when determining whether it
has significant influence over another entity.
d. In determining whether an entity has significant influence over another entity, only
present ownership interests are considered. The possible exercise or conversion of
potential voting rights is not considered.

249
(IASCF)

Section 15 Investments in Joint Ventures


138. A joint venture is:
a. an entity whose equity is owned in equal shares (i.e., 20 per cent each) by five
investors.
b. an entity whose equity is owned in equal shares (i.e, 25 per cent each) by four
investors.
c. a contractual arrangement whereby two or more parties undertake an economic
activity.
d. a contractual arrangement whereby two or more parties undertake an economic
activity that is subject to joint control.
(IASCF)

139. Two entities enter into a contractual arrangement to exercise joint control of a
property, each taking a share of the rents received and bearing a share of the expenses.
The entities are the registered joint owners of the property. The two entities have:
a. a jointly controlled asset. c. the cost model or the fair value model.
b. a jointly controlled operation.
(IASCF)

140. An jointly controlled entity is:


a. an entity over which the investor has significant influence.
b. an entity over which the investor has joint control.
c. an entity over which the investor has significant influence or joint control and that is
not a subsidiary.
d. an entity over which the investor has significant influence and that is neither a
subsidiary nor an interest in a joint venture.
(IASCF)

141. Joint control is:


a. the power to participate in the financial and operating policy decisions of the
investee but is not control or joint control over those policies.
b. active participation in the financial and operating policy decisions of the investee
but is not control or joint control over those policies.
c. the power to govern the financial and operating policies of an entity so as to obtain
benefits from its activities.
d. the contractually agreed sharing of control over an economic activity.
(IASCF)

142. An entity must account for its investments in jointly controlled entities after initial
recognition using:
a. either the cost model or the fair value model (using the same accounting policy for
all investments in jointly controlled entities).
b. either the cost model or the fair value model (model can be elected on an
investment-by-investment basis).
c. either the cost model, the equity method or the fair value model (using the same
accounting policy for all investments in jointly controlled entities).
d. either the cost model, the equity method or the fair value model (model can be
elected on an investment-by-investment basis).
(IASCF)

143. Investments in jointly controlled entities must be tested for impairment in


accordance with Section 27 Impairment of Assets, if the entity uses:
a. the cost model, equity method or fair value model.
b. the cost model or the equity method.
c. the cost model or the fair value model.

250
d. the equity method or the fair value model.
(IASCF)

144. On 31 December 20X1 entity A acquired 30 per cent of the ordinary shares that
carry voting rights of entity Z for ₱100,000. In acquiring those shares entity A incurred
transaction costs of ₱1,000. Entity A has entered into a contractual arrangement with
another party (entity C) that owns 25 per cent of the ordinary shares of entity Z,
whereby entities A and C jointly control entity Z. Entity A uses the cost model to account
for its investments in jointly controlled entities. A published price quotation does not
exist for entity Z. In January 20X2 entity Z declared and paid a dividend of ₱20,000 out
of profits earned in 20X1. No further dividends were paid in 20X2, 20X3 or 20X4. At 31
December 20X1, 20X2 and 20X3, in accordance with Section 27 Impairment of Assets,
management assessed the fair values of its investment in entity Z as ₱102,000,
₱110,000 and ₱90,000 respectively. Costs to sell are estimated at ₱4,000 throughout.
Entity A measures its investment in entity Z on 31 December 20X1, 20X2 and 20X3
respectively at:
a. ₱100,000, ₱100,000, ₱100,000. d. ₱98,000, ₱101,000, ₱86,000.
b. ₱95,000, ₱95,000, ₱86,000. e. ₱102,000, ₱110,000, ₱90,000.
c. ₱98,000, ₱106,000, ₱86,000. f. ₱101,000, ₱101,000, ₱101,000.
(IASCF)

145. The facts are the same as in the immediately preceding question. However, in this
example, a published price quotation exists for entity Z. Entity A measures its
investment in entity Z on 31 December 20X1, 20X2 and 20X3 respectively at:
a. ₱100,000, ₱100,000, ₱100,000. d. ₱98,000, ₱101,000, ₱86,000.
b. ₱95,000, ₱95,000, ₱86,000. e. ₱102,000, ₱110,000, ₱90,000.
c. ₱98,000, ₱106,000, ₱86,000. f. ₱101,000, ₱101,000, ₱101,000.
(IASCF)

146. An investor in a joint venture that does not have joint control accounts for that
investment:
a. in accordance with Section 11 Basic Financial Instruments.
b. in accordance with Section 14 Investments in Associates.
c. in accordance with Section 11 Basic Financial Instruments or, if it has significant
influence in the joint venture, in accordance with Section 14 Investments in
Associates.
d. in accordance with Section 9 Consolidated and Separate Financial Statements.
(IASCF)

147. Which of the following statements is false?


a. Joint control over an entity can be gained or lost without a change in absolute or
relative ownership levels.
b. In determining whether two or more entities jointly control another entity, the
existence and effect of potential voting rights that they hold that are currently
exercisable or convertible are considered.
c. An entity considers the existence and effect of potential voting rights held by other
parties that are currently exercisable or convertible when determining whether it
together with its co-venturers jointly controls another entity.
d. In determining whether an entity and its co-venturers jointly control another entity,
only present ownership interests are considered. The possible exercise or
conversions of potential voting rights are not considered.
(IASCF)

Section 16 Investment Property


148. Investment property is defined as:
a. property (land or a building, or part of a building, or both) held for sale in the
ordinary course of business.

251
b. property (land or a building, or part of a building, or both) held to earn rentals.
c. property (land or a building, or part of a building, or both) held for capital
appreciation.
d. property (land or a building, or part of a building, or both) held to earn rentals or for
capital appreciation or both.
(IASCF)

149. A property interest that is held by a lessee under an operating lease may be
classified and accounted for as investment property if, and only if,
a. the property would otherwise meet the definition of an investment property and the
lessee can measure the fair value of the property interest without undue cost or
effort on an ongoing basis. Furthermore, the entity accounts for all its qualifying
operating leasehold property interests as investment property.
b. the property would otherwise meet the definition of an investment property and the
lessee can measure the fair value of the property interest without undue cost or
effort on an ongoing basis (irrespective of whether other qualifying operating
leasehold property interests are accounted for as investment property (i.e., the
election is available to the entity on a property-by-property basis)).
c. the property would otherwise meet the definition of an investment property and the
lessee accounts for all of its investment property (and qualifying operating
leasehold property interests) at fair value with the change in fair value recognized in
profit or loss.
d. the property would otherwise meet the definition of an investment property and the
lessee accounts for all of its investment property (and qualifying operating
leasehold property interests) using a cost-amortization-impairment model set out in
Section 17 Property, Plant and Equipment.
(IASCF)

150. An entity operates a bed and breakfast from a building it owns. The entity also
provides its guests with other services including housekeeping, satellite television and
broadband internet access. The daily room rental is inclusive of these services.
Furthermore, upon request, the entity conducts tours of the surrounding area for its
guests. Tour services are charged for separately. The entity should account for the
building as:
a. inventory b. investment property c. property, plant and equipment
(IASCF)

151. An entity must measure its investment property after initial recognition:
a. either at fair value or using the cost-depreciation-impairment model (same
accounting policy for all investment property).
b. either at fair value or using the cost-depreciation-impairment model (elected item
by item).
c. at fair value.
d. at fair value, for those properties that fair value can be measured reliably without
undue cost or effort on an ongoing basis, with all other investment property
accounted for using the cost-depreciation-impairment model in Section 17.
(IASCF)

152. Investment property whose fair value cannot be measured reliably without undue
cost or effort on an ongoing basis is accounted for after initial recognition:
a. as inventory in accordance with Section13.
b. as property, plant and equipment in accordance with Section 17.
c. as a financial asset in accordance with Section 11.
d. as an intangible asset with a finite useful life in accordance with Section 18.
(IASCF)

252
153. A building is owned by a subsidiary (lessor) to earn rentals under an operating lease
from its parent (lessee). The parent manufactures its products in the rented building.
The fair value of the building can be measured reliably without undue cost or effort on
an ongoing basis. The building is:
a. accounted for as an item of property, plant and equipment by the subsidiary and an
investment property by the group.
b. accounted for as an investment property by the subsidiary and as an item of
property, plant and equipment by the group.
c. accounted for as an investment property by both the subsidiary and the group.
d. accounted for as an item of property, plant and equipment by both the subsidiary
and the group.
(IASCF)
154. On 1 January 20X1 an entity acquired a building for ₱95,000, including ₱5,000 non-
refundable purchase taxes. The purchase agreement provided for payment to be made
in full on 31 December 20X1. Legal fees of ₱2,000 were incurred in acquiring the
building and paid on 1 January 20X1. The building is held to earn lease rentals and for
capital appreciation. An appropriate discount rate is 10 per cent per year. The entity
shall measure the initial cost of the building at:
a. ₱88,364 b. ₱97,000 c. ₱102,000 d. ₱107,000
(IASCF)

155. On 1 January 20X1 an entity acquired an investment property (building) in a remote


location for ₱100,000. After initial recognition, the entity measures the investment
property using the cost-depreciation-impairment model, because its fair value cannot
be measured reliably without undue cost or effort on an ongoing basis. At 31 December
20X1 management:
 assessed the building’s useful life at 50 years from the date of acquisition
 presumed the residual value of the building to be nil (given that the fair value
cannot be determined reliably)
 assessed that the entity will consume the building’s future economic benefits evenly
over 50 years from the date of acquisition
 declined an unsolicited offer to purchase the building for ₱130,000. This is a ‘one-
off’ offer that is unlikely to be repeated in the foreseeable future.
The entity should measure the carrying amount of the building on 31 December 20X1 at:
a. ₱98,000 b. ₱100,000 c. ₱130,000 d. ₱127,400
(IASCF)

156. On 31 December 20X2 the entity reassessed the remaining useful life of the
investment property described in the preceding question as 73 years. The revised
assessment is supported by new information that became available in late 20X2. The
entity should measure the carrying amount of the building on 31 December 20X2 at:
a. ₱130,000 b. ₱96,676 c. ₱126,533 d. ₱97,333
(IASCF)

157. On 1 January 20X1 an entity acquired a tract of land for an undetermined purpose.
On 1 January 20X4 the entity started constructing a building on the land for use as its
administrative headquarters. On 1 January 20X5 the entity’s administrative staff moved
into that building. Three years later (on 1 January 20X8) the entity’s administrative staff
moved into newly acquired premises. The old building was immediately rented to an
independent third party under an operating lease. On 31 December 20X9 the entity
accepted an unsolicited offer from the tenant to purchase the building from the entity
with immediate effect. The fair value of the property (land and related buildings) can be
measured reliably without undue cost or effort on an ongoing basis. The entity shall
account for the tract of land and the related building as:
a. investment property from 1 January 20X1 to 31 December 20X9.
b. investment property during 20X1–20X3 and 20X8–20X9 and as property, plant and
equipment during 20X4–20X7.

253
c. investment property during 20X1–20X3 and as property, plant and equipment
during 20X4–20X10.
d. property, plant and equipment during 20X1–20X7 and as investment property
during 20X8–20X9.
(IASCF)
Section 17 Property, Plant and Equipment
158. Property, plant and equipment are defined as:
a. tangible assets held for sale in the ordinary course of business.
b. tangible assets held to earn rentals or for capital appreciation or both.
c. tangible assets held for use in the production or supply of goods or services, for
rental to others, or for administrative purposes, and expected to be used during
more than one reporting period.
(IASCF)

159. An entity must measure its property, plant and equipment after initial recognition
at:
a. cost.
b. cost less any accumulated depreciation less any accumulated impairment losses.
c. cost less any accumulated depreciation less any accumulated impairment losses
plus the cost of day-to-day servicing.
d. cost plus the cost of day-to-day servicing.
(IASCF)

160. An entity operates an executive aviation service. The entity’s only item of property,
plant and equipment is an aircraft that it acquired for ₱10,400,000. The cost of the
aircraft is attributed to its significant parts as follows: the jet engine (60%), body (20%)
and aviation equipment (10%) and furniture and fittings (10%). A condition of
operating an aircraft is that it is inspected by the aviation authorities every three years.
An inspection costs ₱400,000. The jet had been inspected at the manufacturer’s expense
before delivery to the entity. Aviation regulations require the jet engine to be replaced
when it has flown 2,000,000 air miles. Management intends fitting a new engine to the
aircraft when it requires replacement so that the aircraft can be used for approximately
10 years, at which time it intends to scrap the aircraft. Management does not expect to
replace the body of the aircraft or the aviation equipment. However, management
assesses the useful life of the furniture and fittings as five years at which time they will
be scrapped and replaced. What is the cost of each of the significant parts of the aircraft
that the entity must depreciate separately:
a. ₱6,240,000 jet engine, ₱2,080,000 body, ₱1,040,000 aviation equipment and
₱1,040,000 furniture and fittings.
b. ₱10,400,000 jet aircraft.
c. ₱6,000,000 jet engine, ₱3,000,000 body and equipment, ₱1,000,000 furniture and
fittings and ₱400,000 aviation inspection.
(IASCF)

161. Facts are the same as in the immediately preceding question. What depreciation
methods are most appropriate for the entity to apply to compute depreciation for the
significant parts of the aircraft:
a. straight-line method for all parts of the aircraft.
b. units of production method, based on air miles flown, for the jet engines and the
straight-line method for all other parts of the aircraft.
c. units of production method, based on air miles flown, for all parts of the aircraft.
d. diminishing balance method for all parts of the aircraft.
(IASCF)

162. On 1 January 20X1 an entity acquired a building for ₱100,000. At 31 December 20X1
management:
 assessed the building’s useful life as 40 years from the date of acquisition

254
 assessed the building’s residual value as ₱20,000
 assessed the entity will consume the building’s future economic benefits evenly over
40 years from the date of acquisition
 assessed the fair value of the building at ₱130,000.
The building is occupied by the entity’s sales staff. The entity should measure the carrying
amount of the building on 31 December 20X1 at:
a. ₱100,000 b. ₱98,000 c. ₱130,000 d. ₱127,250
(IASCF)

163. On 31 December 20X2 the entity reassessed the property described in the preceding
question as follows:
 the building’s useful life as 60 years from the date of acquisition
 the building’s residual value as ₱10,000
 the entity will consume the building’s future economic benefits evenly over 60 years
from the date of acquisition
 the fair value of the building at ₱160,000.
The entity should measure the carrying amount of the building on 31 December 20X2 at:
a. ₱96,508 b. ₱96,000 c. ₱160,000 d. ₱125,263
(IASCF)

164. On 1 January 20X1 an entity acquired a tract of land for an undetermined purpose.
On 1 January 20X4 the entity began to construct a building on the land for use as its
administrative headquarters. On 1 January 20X8 the entity’s administrative staff moved
out of the building and into newly acquired premises. The building was immediately
rented to an independent third party under an operating lease. On 31 December 20X9
the entity accepted an unsolicited offer from the tenant to purchase the building from
the entity with immediate effect. The fair value of the building can be determined
reliably without undue cost or effort on an ongoing basis. The entity should account for
the building as:
a. investment property from the date of acquisition (1 January 20X1) to the date of
disposal (31 December 20X9).
b. investment property during 20X1–20X3 and as property, plant and equipment
during 20X4–20X9.
c. investment property during 20X1–20X3 and 20X8–20X9 and as property, plant and
equipment during 20X4–20X7.
d. property, plant and equipment during 20X1–20X7 and as investment property
during 20X8–20X9.
(IASCF)

Section 18 Intangible Assets other than Goodwill


165. An intangible asset (other than goodwill) is:
a. an identifiable asset without physical substance.
b. a non-monetary asset without physical substance.
c. an identifiable non-monetary asset without physical substance.
(IASCF)

166. An intangible asset is identifiable when:


a. it is separable (i.e., capable of being separated or divided from the entity and sold,
transferred, licensed, rented or exchanged, either individually or together with a
related contract, asset or liability).
b. it arises from contractual or other legal rights, regardless of whether those rights
are transferable or separable from the entity or from other rights and obligations.
c. either (a) or (b) applies.
d. none of the above.
(IASCF)

167. An entity must measure intangible assets after initial recognition:

255
a. at fair value.
b. at fair value or at cost less any accumulated amortization and any accumulated
impairment losses (same accounting policy for all items in the same class of
intangible asset).
c. at fair value or at cost less any accumulated amortization and any accumulated
impairment losses (elected on an item-by-item basis).
d. at cost less any accumulated amortization and any accumulated impairment losses.
(IASCF)

168. The cost of an intangible asset at initial recognition is measured at its fair value
when:
a. it is internally generated. d. all of (a)–(c) above.
b. it is acquired as part of a business combination. e. both (b) and (c) above.
c. it is acquired by way of a government grant. f. none of the above.
(IASCF)

169. An entity acquired a trademark for a leading consumer product. The trademark has
a remaining legal life of five years but is renewable every ten years at little cost. The
acquiring entity intends to renew the trademark continuously and evidence supports its
ability to do so. An analysis of (i) product life cycle studies, (ii) market, competitive and
environmental trends, and (iii) brand extension opportunities provides evidence that
the trademarked product will generate net cash inflows for the acquiring entity for an
indefinite period. The useful life of the intangible asset is:
a. five years—the initial period of the contractual rights.
b. presumed to be 10 years—if the entity is unable to make a reliable estimate of its
finite useful life.
c. 15 years—the initial period of the contractual rights plus a renewal period.
d. five years—the period of the contractual rights, but with no amortization charges, as
it is expected to generate cash flows for a indefinite period.
(IASCF)

170. On 31 December 20X2 entity A sold a brand name to entity B for ₱250,000. Entity A
estimates that it cost ₱100,000 to develop the brand name during 20X1. Entity B
estimates that it spent ₱50,000 in maintaining and developing the brand name in 20X3.
On 31 December 20X3 entity C gained control over entity B in a business combination,
when the fair value of the brand was estimated at ₱400,000. For the purpose of this
example, ignore amortization. The brand name must be recognized:
a. on 31 December 20X1 by entity A at ₱100,000; on 31 December 20X2 by entity B at
₱250,000; on 31 December 20X3 by entity C (in its consolidated financial
statements) at ₱400,000.
b. on 31 December 20X1 by entity A at nil; on 31 December 20X2 by entity B at
₱300,000; on 31 December 20X3 by entity C (in its consolidated financial
statements) at ₱400,000.
c. on 31 December 20X1 by entity A at nil; on 31 December 20X2 by entity B at
₱250,000; on 31 December 20X3 by entity C (in its consolidated financial
statements) at ₱400,000.
(IASCF)

171. On 1 January 20X1 an entity acquired a taxi license for ₱95,000, including ₱5,000
non-refundable purchase taxes. The purchase agreement provided that payment must
be made in full on 31 December 20X1. Legal fees of ₱2,000 were incurred in acquiring
the taxi license and paid on 1 January 20X1. An appropriate discount rate is 10 per cent
per year. On 1 January 20X1 the entity shall measure the initial cost of the taxi license
at:
a. ₱102,000 b. ₱97,000 c. ₱88,364 d. ₱107,000.
(IASCF)

256
172. On 1 January 20X1 an entity acquired a patent for ₱100,000. At 31 December 20X1
management:
 assessed the patent’s useful life at 20 years from the date of acquisition
 assessed that the entity will consume the patent’s future economic benefits evenly
over 20 years from the date of acquisition
 assessed the fair value of the patent at ₱130,000.
The entity shall measure the carrying amount of the patent on 31 December 20X1 at:
a. ₱100,000 b. ₱95,000 c. ₱130,000 d ₱123,500
(IASCF)

173. On 31 December 20X5 the entity reassessed the patent described in the preceding
question as follows:
 the patent’s useful life at 14 years from the date of acquisition
 the entity will consume the patent’s future economic benefits evenly over 14 years
from the date of acquisition
 the recoverable amount (fair value less costs to sell) of the patent at ₱70,000.
The entity shall measure the carrying amount of the patent on 31 December 20X5 at:
a. ₱72,000 b. ₱100,000 c. ₱64,286 d. ₱70,000
(IASCF)

174. On 1 January 20X1 an entity (a publisher) acquired a competitor’s publishing title


for ₱30,000. On 1 January 20X4 the entity commenced publishing using the new title.
On 31 December 20X8 the entity decided to sell the publishing title and took actions
that make its sale within 12 months highly probable. The publishing title was sold on 31
March 20X9. The entity shall account for the publishing title as:
a. an intangible asset (other than goodwill) from the date of acquisition (1 January
20X1) to 31 December 20X8 and as an inventory (asset held for sale in the ordinary
course of business) from 31 December 20X8 to the date of disposal (31 March
20X9).
b. an item of inventory from the date of acquisition (1 January 20X1) to the date of
disposal (31 March 20X9).
c. an intangible asset (other than goodwill) from the date of acquisition (1 January
20X1) to the date of disposal (31 March 20X9).
d. none of the above.
(IASCF)

Section 19 Business Combinations and Goodwill


175. According to the PFRS for SMEs, applying the purchase method involves all of the
following steps, except
a. Identifying an acquiree.
b. Identifying an acquirer.
c. Measuring the cost of the business combination.
d. Allocating, at the acquisition date, the cost of the business combination to the assets
acquired and liabilities and provisions for contingent liabilities assumed.

Section 20 Leases
176. Which of the following arrangements is accounted for in accordance with the
requirements of Section 20?
a. licensing agreements for such items as motion picture films, video recordings, plays,
manuscripts, patents and copyrights.
b. agreements that transfer the right to use assets even though substantial services by
the lessor may be called for in connection with the operation or maintenance of such
assets.
c. leases to explore for or use minerals, oil, natural gas and similar non-regenerative
resources.

257
d. leases that could result in a loss to the lessor or the lessee as a result of contractual
terms that are unrelated to changes in the price of the leased asset, changes in
foreign exchange rates, or a default by one of the counterparties.
(IASCF)

177. An entity entered, as lessee, into a five-day non-cancellable lease of a motor vehicle
that has an economic life of five years and nil residual value. Lease payments are ₱180
per day. At the end of the lease term the lessee returns the motor vehicle to the lessor.
The lease is:
a. accounted for as a finance lease in accordance with Section 20.
b. accounted for as an operating lease in accordance with Section 20.
c. not accounted for in accordance with Section 20.
(IASCF)

178. An entity entered, as lessee, into a two-year non-cancellable lease over a motor
vehicle that has an economic life of five years and nil residual value. At the inception of
the lease, the present value of the minimum lease payments approximates the fair value
of the motor vehicle. Ownership of the motor vehicle passes to the lessee at the end of
the lease term. The lease is:
a. accounted for as a finance lease in accordance with Section 20.
b. accounted for as an operating lease in accordance with Section 20.
c. not accounted for in accordance with Section 20.
(IASCF)

179. A lease could be classified as:


a. a finance lease by the lessor and the lessee.
b. an operating lease by the lessor and the lessee.
c. a finance lease by the lessor and an operating lease by the lessee.
d. all of (a)–(c) above.
e. none of (a)–(c) above.
(IASCF)

180. A lease transfers ownership of the leased asset from the lessor at the end of the
lease term for a variable payment equal to the asset’s then fair value.
a. The lessee must classify the lease as a finance lease.
b. The lessee must classify the lease as an operating lease.
c. The classification of the lease depends on other facts and circumstances.
(IASCF)

181. Depreciation of a leased machine is:


a. recognized by the lessee where the lessor and the lessee have classified the lease as
a finance lease.
b. recognized by the lessor where the lessor and the lessee have classified the lease as
an operating lease.
c. recognized by neither the lessor nor the lessee when the lease is classified as a
finance lease by the lessor and an operating lease by the lessee.
d. all of the above.
(IASCF)

182. A lessee paid ₱2,000 to a broker for arranging a finance lease. The lessee must
account for the broker’s fee:
a. as an expense in the period in which the fee was incurred (i.e., probably at the
inception of the lease).
b. include the fee in the cost of the leased asset.
c. defer recognition of the expense and recognize the fee in profit or loss on the
straight-line method over the lease term.
(IASCF)

258
183. An entity enters as lessee into a two-year lease in respect of a machine that has a fair
value of ₱16,000 and an economic life of four years with nil scrap value. Rent of ₱8,500
per year is payable yearly in advance. The lessee holds an option to acquire the machine
for ₱1. The option is exercisable at the end of the lease term, when the fair value of the
machine (estimated at the inception of the lease) is expected to be ₱6,000. At the
commencement of the lease term, the lessor would:
a. derecognize the machine and recognize a lease receivable of ₱16,000.
b. continue to recognize the carrying amount of the machine subject to the lease as an
item of property, plant and equipment.
(IASCF)

184. On 1 January 20X0 an entity entered, as lessee, into a ten-year non-cancellable


operating lease of a building. In accordance with the lease no amount is payable for the
first five years of the lease. The lease payments for years ending 31 December 20X5–
20X9 are ₱5,000 per year, payable in arrears (i.e., payable on 31 December each year).
In determining profit or loss for the year ended 31 December 20X0, the lessee would
recognize:
a. ₱2,500 lease expense. b. nil. c. ₱5,000.
(IASCF)

185. On 1 January 20X1 an entity entered into an arrangement to sell a machine to a bank
and lease it back for a period of three years. At that time, the main facts about the
machine and the lease were:
 selling price = ₱200,000
 carrying amount = ₱70,000
 fair value = ₱200,000
 remaining economic life = three years
 residual value = nil
 lease payments = ₱77,606 per year (payable in arrears on 31 December each year).
 interest rate implicit in the lease = 8 per cent per year.

In accounting for the arrangement, the seller-lessee would recognize in the determination
of its profit or loss for the year ended 31 December 20X1:
a. income of ₱130,000 (gain on sale of machine) and expense of ₱77,606 (lease
expense).
b. expenses of ₱23,333 (depreciation) and ₱16,000 (finance cost) (and no income).
c. income of ₱43,333 (amortized deferred gain on sale of machine) and expenses of
₱23,333 (depreciation) and ₱16,000 (finance cost).
d. income of ₱43,333 (amortized deferred gain on sale of machine) and expenses of
₱66,667 (depreciation) and ₱16,000 (finance cost).
(IASCF)

186. The facts are the same as in the immediately preceding question. However, in this
question, the remaining economic life of the machine is 30 years and the lease rent is
₱23,000 per year of the three-year lease term. In accounting for the arrangement, the
seller-lessee would recognize in the determination of its profit or loss for the year
ended 31 December 20X1:
a. income of ₱130,000 (gain on sale of machine) and expense of ₱23,000 (lease
expense).
b. expenses of ₱23,333 (depreciation) and ₱16,000 (finance cost) (and no income).
c. income of ₱43,333 (amortized deferred gain on sale of machine) and expenses of
₱2,333 (depreciation) and ₱16,000 (finance cost).
d. income of ₱43,333 (amortized deferred gain on sale of machine) and expenses of
₱66,667 (depreciation) and ₱16,000 (finance cost).
(IASCF)

259
Section 21 Provisions and Contingencies
187. A provision is:
a. a liability of uncertain timing or amount.
b. a possible obligation as a result of past events that is of uncertain timing or amount.
c. an adjustment to the carrying amount of assets (e.g., attributable to impairment or
uncollectibility).
(IASCF)

188. An entity recognizes a provision only when:


a. the entity has a present obligation as a result of a past event.
b. it is probable (i.e., more likely than not) that the entity will be required to transfer
economic benefits in settlement.
c. the amount of the obligation can be estimated reliably.
d. all of the above apply.
e. only (a) and (b) above apply.
(IASCF)

189. An entity measures a provision at the best estimate of the amount required to settle
the obligation at the reporting date. When the provision involves a large population of
items, the estimate of the amount:
a. reflects the weighting of all possible outcomes by their associated probabilities.
b. is determined as the individual most likely outcome.
c. may be the individual most likely outcome. However, the entity should also consider
the other possible outcomes.
(IASCF)

190. An entity measures a provision at the best estimate of the amount required to settle
the obligation at the reporting date. When the provision arises from a single obligation,
the estimate of the amount:
a. reflects the weighting of all possible outcomes by their associated probabilities.
b. is determined as the individual most likely outcome.
c. the individual most likely outcome adjusted to take account of the effect of other
possible outcomes.
(IASCF)

191. A manufacturer gives warranties at the time of sale to purchasers of its product.
Under the terms of the contract for sale the manufacturer undertakes to make good, by
repair or replacement, manufacturing defects that become apparent within one year
from the date of sale. On the basis of experience, it is probable (i.e., more likely than
not) that there will be some claims under the warranties. Sales of ₱10 million were
made evenly throughout 20X1. At 31 December 20X1 the expenditures for warranty
repairs and replacements for the product sold in 20X1 are expected to be made 50 per
cent in 20X1 and 50 per cent in 20X2. Assume for simplicity that all the 20X2 outflows
of economic benefits related to the warranty repairs and replacements take place on 30
June 20X2. Experience indicates that 95 per cent of products sold require no warranty
repairs; 3 per cent of products sold require minor repairs costing 10 per cent of the sale
price; and 2 per cent of products sold require major repairs or replacement costing 90
per cent of sale price. The entity has no reason to believe future warranty claims will be
different from its experience. At 31 December 20X1 the appropriate discount factor for
cash flows expected to occur on 30 June 20X2 is 0.95238. Furthermore, an appropriate
risk adjustment factor to reflect the uncertainties in the cash flow estimates is an
increment of 6 per cent to the probability-weighted expected cash flows. At 31
December 20X1 the entity recognizes a warranty provision measured at:
a. ₱0. b. ₱210,000. c. ₱222,600. d. ₱113,300. e. ₱106,000.
(IASCF)

260
192. An entity is the defendant in a patent infringement lawsuit. The entity’s lawyers
believe there is a 30 per cent chance that the court will dismiss the case and the entity
will incur no outflow of economic benefits. However, if the court rules in favor of the
claimant, the lawyers believe that there is a 20 per cent chance that the entity will be
required to pay damages of ₱200,000 (the amount sought by the claimant) and an 80
per cent chance that the entity will be required to pay damages of ₱100,000 (the
amount that was recently awarded by the same judge in a similar case). Other outcomes
are unlikely. The court is expected to rule in late December 20X2. There is no indication
that the claimant will settle out of court. A 7 per cent risk adjustment factor to the
probability-weighted expected cash flows is considered appropriate to reflect the
uncertainties in the cash flow estimates. An appropriate discount rate is 10 per cent per
year. At 31 December 20X1 the entity recognizes a provision for the lawsuit measured
at:
a. ₱0. b. ₱100,000. c. ₱89,880. d. ₱81,709.
(IASCF)

193. The facts are the same as in the immediately preceding question. However, in this
question, because of extremely rare circumstances disclosure of some of the
information about the case required by paragraphs 21.14–21.16 can be expected to
prejudice seriously the position of the entity in the dispute over the alleged breach of
patent. At 31 December 20X1 the entity would:
a. not recognize a provision and disclose the general nature of the dispute, together
with the fact that, and reason why, the information has not been disclosed.
b. recognize a provision measured at the amount determined in the preceding
question and disclose the general nature of the dispute, together with the fact that,
and reason why, the information has not been disclosed.
c. recognize a provision measured at the amount determined in the preceding
question and disclose the information required by paragraphs 21.14–21.16.
(IASCF)

194. The facts are the same as in the preceding question. However, in this question, the
entity’s lawyers believe there is a 60 per cent chance that the court will dismiss the case
and the entity will incur no outflow. At 31 December 20X1, the entity:
a. recognizes a provision measured at ₱100,000.
b. recognizes a provision measured at ₱48,000.
c. recognizes a provision measured at ₱46,691.
d. discloses a contingent liability (and does not recognize a provision in its statement
of financial position).
(IASCF)

195. On 20 February 20X5, before an entity’s 31 December 20X4 financial statements


were authorized for issue, a court ordered the entity to pay ₱120,000 damages in full
and final settlement of a patent infringement lawsuit brought against the entity by one
of its competitors. The patent infringement occurred in 20X3. The amount of damages
awarded to the competitor was significantly higher than the ₱10,000–₱30,000 that the
entity had justifiably expected to pay throughout the duration of the case. The entity
will not contest the judgment. In its 31 December 20X3 annual financial statements the
entity reported its liability for the lawsuit at ₱20,000—this estimate was appropriately
made taking account of all available evidence at the time the financial statements were
authorized for issue. In its 31 December 20X4 financial statements the entity:
a. restates the comparative information as at 31 December 20X3 (i.e., retrospective
restatement of a prior period error).
b. measures the provision as at 31 December 20X4 at ₱120,000 (comparative
information 20X3: ₱20,000) (i.e., it is a change in accounting estimate in its 20X4
financial statements).
c. measures the provision as at 31 December 20X4 at ₱20,000 (comparative
information 20X3: ₱20,000) and record the effect of the higher than expected

261
settlement in profit or loss for the year ended 31 December 20X5 (i.e., account
prospectively for the change in accounting estimate in the period that the final
settlement amount was determined).
(IASCF)

196. At 31 December 20X1 an entity is pursuing a claim against an insurance company


through legal processes. The court is expected to rule in late December 20X2. At the
reporting date (31 December 20X1) the outcome of the case is uncertain. The entity’s
lawyers believe there is a 70 per cent chance that the entity will win the case.
Furthermore, they believe that there is a 20 per cent chance that the entity will be
awarded ₱200,000 (the amount sought by the entity) and an 80 per cent chance that
the entity will be awarded ₱100,000 (the amount that was recently awarded by the
same judge in a similar case). Other outcomes are unlikely. A 7 per cent risk adjustment
factor to the probability-weighted expected cash flows is considered appropriate to
reflect the uncertainties in the cash flow estimates. An appropriate discount rate is 10
per cent per year. At 31 December 20X1 the entity:
a. recognizes an asset measured at ₱100,000.
b. recognizes an asset measured at ₱84,000.
c. recognizes a contingent asset measured at ₱81,709.
d. discloses a contingent asset (and does not recognize an asset in its statement of
financial position).
(IASCF)

Section 22 Liabilities and Equity


197. According to the PFRS for SMEs, puttable instrument that is classified as equity in a
subsidiary’s financial statements is
a. classified as a liability in the consolidated group financial statements.
b. also classified as equity in the consolidated group financial statements.
c. classified as an asset in the consolidated group financial statements.
d. none of these

Section 23 Revenue
198. Section 23 is applied in accounting for revenue arising from the sale of goods, the
rendering of services, construction contracts in which the entity is the contractor and
the use by others of entity assets yielding interest, royalties or dividends. However,
Section 23 does not apply to revenue arising from:
a. lease agreements.
b. changes in the fair value of financial assets and financial liabilities or their disposal.
c. the initial recognition and changes in the fair value of biological assets related to
agricultural activity.
d. all of the above.
(IASCF)

199. An entity sold a good with a list price (advertised price) of ₱1,000 to a customer on
normal credit terms (i.e., 30 days interest-free credit). Ten days after the sale the
customer paid the entity ₱690 in full and final settlement of a debt that arose from the
sale of the goods. ₱50 of the amount received from the customer is sales tax collected by
the entity on behalf of the national government. The difference between the list price
and the settlement amount are as follows: ₱1,000 list price less ₱200 trade discount less
₱100 volume rebate less ₱10 prompt settlement discount. At what amount should the
entity measure revenue from the sale of the good:
a. ₱640 b. ₱1,000 c. ₱700 d. ₱690
(IASCF)

200. An entity (the seller) bills a customer for goods that are yet to be delivered to the
customer. Delivery is delayed in accordance with an instruction from the customer. The
seller recognizes revenue when the customer takes title, provided:

262
a. it is probable that delivery will be made.
b. the item is on hand, identified and ready for delivery to the customer at the time the
sale is recognized.
c. the customer specifically acknowledges the deferred delivery instructions.
d. the usual payment terms apply.
e. all of the above.
(IASCF)
201. An entity must not:
a. recognize revenue from the sale of goods if it retains significant risks and rewards of
ownership of goods sold.
b. recognize revenue from the rendering of services using the percentage of
completion method if it cannot estimate the outcome of the transaction reliably.
c. recognize revenue from a construction contract using the percentage of completion
method if it cannot estimate the outcome of a contract reliably.
d. recognize revenue from any of (a) to (c) above.
(IASCF)

202. The percentage of completion method is applied to recognize revenue from:


a. the rendering of services and construction contracts.
b. the rendering of services only when the outcome of the revenue transaction can be
estimated reliably.
c. construction contracts only when the outcome of the contract can be estimated
reliably.
d. both (b) and (c) above.
(IASCF)

203. In a promotion, a car dealer undertakes to service and maintain cars sold in the
promotion period free of charge for two years from the date of sale. Furthermore,
promotion sales are made on two-year interest-free credit. The car dealer enters into a
sale which has the following separately identified elements to which the entity must
apply the recognition criteria separately:
a. the sale of goods.
b. the sale of goods and the rendering of maintenance services.
c. the sale of goods, the rendering of services and a financing element (interest)
related to the deferred payment for the sale.
(IASCF)

204. On 1 January 20X1 an entity incurred ₱2,000 selling costs to sell a good for ₱95,000.
The sale agreement provided that the customer pay the ₱95,000 selling price in full on
31 December 20X1. The prevailing rate for one-year credit granted to trade customers
in the industry is 10% per year. This is the more clearly determinable way of
determining the imputed rate of interest in accordance with paragraph 23.5. The entity
must measure revenue from the sale of the good at:
a. ₱95,000 b. ₱86,364 c. ₱97,000 d. ₱93,000
(IASCF)

205. A construction contractor builds a home under a contract with a fixed price of
₱1,000,000. The contractor incurred contract costs of ₱10,000, ₱890,000 and ₱200,000
in 20X1, 20X2 and 20X3 respectively. At the end of 20X1 the outcome of the transaction
cannot be estimated reliably however it is probable that the costs incurred in 20X1 will
be recoverable. At the end of 20X2 the contractor can estimate the outcome of the
contract reliably and estimates costs to complete the contract at ₱200,000. The contract
was completed in 20X3. The contractor determines the stage of completion of the
construction contract by reference to the proportion that costs incurred for work
performed to date bear to the estimated total costs. In 20X2 the contractor must:
a. recognize contract revenue of ₱818,182 and contract costs of ₱900,000.
b. recognize contract revenue of ₱808,182 and contract costs of ₱890,000.

263
c. recognize contract revenue of ₱808,182 and contract costs of ₱908,182.
d. recognize contract revenue of ₱808,182 and contract costs of ₱900,000.
(IASCF)

206. A construction contractor builds a home under a contract with a fixed price of
₱1,000,000. The contractor incurred contract costs of ₱200,000, ₱400,000 and
₱100,000 in 20X1, 20X2 and 20X3 respectively. At the end of 20X1 the contractor
estimated (with sufficient reliability) the future costs to complete the contract as
₱400,000. At the end of 20X2 the contractor estimated (with sufficient reliability) the
future costs to complete the contract as ₱150,000. The contract was completed in 20X3.
The contractor determines the stage of completion of the construction contract by
reference to the proportion that costs incurred for work performed to date bear to the
estimated total costs. The contractor must recognize contract revenue at:
a. ₱333,333 in 20X1, ₱466,667 in 20X2 and ₱200,000 in 20X3.
b. ₱1,000,000 in 20X1, ₱0 in both 20X2 and 20X3.
c. ₱0 in both 20X1 and 20X2 and ₱1,000,000 in 20X3.
d. ₱333,333 in 20X1, ₱333,333 in 20X2 and ₱333,333 in 20X3.
(IASCF)

207. Consider the information in the immediately preceding question. However, in this
example, contract costs incurred at the end of 20X2 included ₱50,000 prepaid wages.
The contractor must recognize contract revenue at:
a. ₱333,333 in 20X1, ₱466,667 in 20X2 and ₱200,000 in 20X3.
b. ₱333,333 in 20X1, ₱400,000 in 20X2 and ₱266,667 in 20X3.
c. ₱0 in 20X1 and 20X2 and ₱1,000,000 in 20X3.
d. ₱333,333 in 20X1, ₱333,333 in 20X2 and ₱333,333 in 20X3.
(IASCF)
Section 24 Government Grants
208. A government grant is:
a. assistance from the government in the form of a transfer of resources to an entity in
return for past or future compliance with specified conditions relating to the
operating activities of the entity.
b. unconditional assistance from the government in the form of a transfer of resources
to an entity.
c. any type of assistance from the government to the entity from which the entity has
benefited directly.
(IASCF)

209. Government grants exclude which of the following forms of government assistance?
a. those forms of government assistance that cannot reasonably have a value placed
upon them.
b. transactions with government that cannot be distinguished from the normal trading
transactions of the entity.
c. both (a) and (b).
d. neither (a) nor (b).
(IASCF)

210. An entity shall measure all government grants:


a. at the amount of cash or cash equivalents received.
b. at the amount of cash or cash equivalents received or receivable.
c. at the fair value of the asset received or receivable.
d. any of the above.
(IASCF)

211. An entity must recognize a government grant that does not impose specified future
performance conditions on that entity (the recipient):
a. in income when the grant proceeds are receivable.

264
b. in income over the periods necessary to match it with the related costs for which it
is intended to compensate, on a systematic basis.
c. by applying either (a) or (b) above depending upon the accounting policy adopted
by the entity.
d. none of the above.
(IASCF)

212. An entity must recognize a government grant that imposes specified future
performance conditions upon that entity:
a. in income when the grant proceeds are receivable.
b. in income over the periods necessary to match it with the related costs for which it
is intended to compensate, on a systematic basis.
c. in income only when the performance conditions are met.
d. none of the above.
(IASCF)

213. An entity must recognize government grants received before the income recognition
criteria are satisfied:
a. in income when the grant proceeds are received. c. as a liability.
b. in equity – deferred income. d. none of the above.
(IASCF)

214. On 1 January 20X1 an entity acquired a transferable nine-year taxi license by way of
government grant when the fair value of the license was ₱90,000. The license is given,
free of charge, to the entity on the basis of the entity’s performance and there are no
future performance conditions attached to the grant. The entity shall account for the
government grant as follows:
a. recognize ₱90,000 in income on 1 January 20X1.
b. recognize ₱90,000 in income evenly over the nine-year period of the license, i.e.,
₱10,000 per year.
c. credit ₱90,000 directly to retained earnings on 1 January 20X1.
(IASCF)

215. On 1 January 20X1 an entity acquired, free of charge, a non-transferable nine-year


taxi license by way of government grant when the fair value of the taxi license was
₱90,000. In accordance with the terms of the license the entity must operate at least 10
taxis in a deprived neighborhood of the capital city during that nine-year period. Failure
to do so will result in the taxi license being revoked immediately. The entity shall
account for the government grant as follows:
a. recognize ₱90,000 in income on 1 January 20X1.
b. recognize ₱90,000 in income evenly over the nine-year period of the license (i.e.,
₱10,000 per year when the performance condition of the government grant is met).
c. credit ₱90,000 directly to retained earnings on 1 January 20X1.
d. recognize ₱90,000 in income on 31 December 20X9.
(IASCF)

216. On 1 January 20X1 an entity acquired, free of charge, a herd of 100 cattle by way of
government grant when the fair value of the herd was ₱1,000,000. On average the
remaining life of the cattle is expected to be 10 years. The grant does not impose future
performance conditions on the entity. The entity shall account for the government grant
as follows:
a. recognize ₱1,000,000 in income on 1 January 20X1.
b. recognize ₱1,000,000 in income evenly over the 10-year expected remaining life of
the cattle (i.e., ₱100,000 per year).
c. credit ₱1,000,000 directly to retained earnings on 1 January 20X1.
d. none of the above.
(IASCF)

265
217. In 20X1 the management of a private entity attended one of the world’s biggest
trade fairs for that entity’s industry to promote and demonstrate its latest products. In
order to promote overseas trade for that particular industry, the national government
provided free support to the management which involved helping them to design its
display, secure a space at the event and make travel and logistical arrangements. The
government also provided free advice on what the management should say to those
attending the fair and the type of promotional literature it should give them. The
government assistance cannot reasonably have a value placed on it. The entity must:
a. determine the fair value of the government assistance and recognize income equal
to that fair value for the year ended 31 December 20X1.
b. disclose the fact that the entity has benefited directly from marketing support from
the national government at the trade fair (i.e., indicate the nature of government
assistance received in its 20X1 financial statements).
c. neither recognize nor disclose the government assistance received during the year.
d. recognize in equity the fair value of the assistance received directly during 20X1.
(IASCF)

Section 25 Borrowing Costs


218. Borrowing costs are:
a. interest and other costs that an entity incurs in connection with the borrowing of
funds.
b. interest expense calculated using the effective interest method as described in
Section 11 Basic Financial Instruments only.
c. finance charges in respect of finance leases recognized in accordance with Section
20 Leases only.
d. none of the above.
(IASCF)

219. Borrowing costs do not include:


a. interest incurred on bank overdrafts.
b. incremental administrative fees incurred in connection with raising loans.
c. finance charges in respect of finance leases recognized in accordance with Section
20 Leases.
d. dividends declared to equity holders.
(IASCF)

220. An entity must:


a. recognize all borrowing costs as an expense in profit or loss in the period in which
they are incurred.
b. recognize all borrowing costs as an expense in profit or loss in the period in which
they are incurred, except to the extent that borrowing costs that are directly
attributable to the acquisition, construction or production of an asset that
necessarily takes a substantial period of time to get ready for its intended use or sale
are capitalized as part of the cost of that asset.
c. choose either (a) or (b) above as its accounting policy for borrowing costs and apply
the chosen policy consistently to all of its borrowing costs.
(IASCF)

Section 26 Share-based Payment


221. The goods or services received or acquired in a share-based payment transaction
are recognized as
a. assets b. expenses c. income d. a or b

Section 27 Impairment of Assets


222. An impairment loss occurs when
a. the carrying amount of an asset exceeds its recoverable amount.

266
b. the recoverable amount of an asset exceeds its carrying amount.
c. a or b
d. none of these

Section 28 Employee Benefits


223. Employee benefits are all forms of consideration given by an entity in exchange for
service rendered by employees, including directors and management. Section 28
applies to the accounting for four types of employee benefits. It does not apply to the
accounting for:
a. short-term employee benefits. d. termination benefits.
b. post-employment benefits. e. share-based payments.
c. other long-term employee benefits.
(IASCF)

Use the following information to answer the next five questions:


An entity’s employees are each entitled to 20 days of paid holiday leave per calendar year.
Unused holiday leave cannot be carried forward and does not vest. The entity’s annual
reporting date is 31 December.
224. The holiday leave is:
a. a short-term employee benefit. c. an other long-term employee benefit.
b. a post-employment benefit. d. a termination benefit.
(IASCF)

225. Consider the previous information. However, in this question, unused holiday leave
is paid on 31 December of each year (i.e., it vests at the end of each calendar year but
does not accumulate). The holiday leave is:
a. a short-term employee benefit. c. other long-term employee benefit.
b. a post-employment benefit. d. a termination benefit.
(IASCF)

226. Consider the previous information. However, in this question, unused holiday leave
may be carried forward for one calendar year (i.e., it accumulates but does not vest).
The holiday leave is:
a. a short-term employee benefit. c. an other long-term employee benefit.
b. a post-employment benefit. d. a termination benefit.
(IASCF)

227. Consider the previous information. However, in this question, unused holiday leave
may be carried forward for two calendar years (i.e., it accumulates but does not vest).
The holiday leave is:
a. a short-term employee benefit. c. an other long-term employee benefit.
b. a post-employment benefit. d. a termination benefit.
(IASCF)

228. Consider the previous information. However, in this question, unused holiday leave
may be carried forward until the employee leaves the employment of the entity, at
which time the entity will pay the employee for all unused holiday leave (i.e., it
accumulates and vests). The holiday leave is:
a. a short-term employee benefit. c. an other long-term employee benefit.
b. a post-employment benefit. d. a termination benefit.
(IASCF)

229. An entity made a public announcement of its commitment to a voluntary


redundancy plan. The entity has an obligation to pay employees that choose voluntary
redundancy a lump sum equal to twice their gross annual salary. The obligation to pay
employees that choose voluntary redundancy is:
a. a short-term employee benefit. c. an other long-term employee benefit.

267
b. a post-employment benefit. d. a termination benefit.
(IASCF)

230. An entity reimburses 50 per cent of past employees’ post-employment medical costs
if the employee provides 25 years of service, or more, to the entity. The obligation to
pay 50 per cent of qualifying past employees’ post-employment medical costs is:
a. a short-term employee benefit. d. an other long-term employee benefit.
b. a defined benefit post-employment benefit. e. a termination benefit.
c. a defined contribution post-employment benefit.
(IASCF)

231. A profit-sharing plan requires an entity to pay a specified proportion of its


cumulative profit for a five-year period to employees who serve throughout the five-
year period. The profit-sharing plan is:
a. a short-term employee benefit. c. an other long-term employee benefit.
b. a post-employment benefit. d. a termination benefit.
(IASCF)

232. A profit-sharing plan requires an entity to pay a specified proportion of its


cumulative profit for the year to employees who served the entity throughout the year.
The profit-sharing plan is:
a. a short-term employee benefit. c. an other long-term employee benefit.
b. a post-employment benefit. d. a termination benefit.
(IASCF)

233. Which of the following best describes the simplified method of calculating a defined
benefit obligation permitted by paragraph 28.19?
a. it measures the pension obligation on the basis of the plan formula applied to years
of service to date and existing salary levels.
b. it measures the pension obligation on the basis of the plan formula applied to years
of service to date and future salary levels.
c. it estimates the total benefit at retirement and then computes the level cost that will
be sufficient, together with interest expected to accumulate at the assumed rate, to
provide the total benefits at retirement.
d. it measures the pension obligation and pension cost on the basis of the shortest
possible period for funding to maximize the tax deduction.
(IASCF)

234. An entity that uses the simplified method of calculating a defined benefit obligation
is not permitted:
a. to ignore estimated future salary increases (i.e., assume current salaries continue
until current employees are expected to begin receiving post-employment benefits).
b. to ignore future service of current employees (i.e., assume closure of the plan for
existing as well as any new employees).
c. to ignore possible in-service mortality of current employees between the reporting
date and the date employees are expected to begin receiving post-employment
benefits (i.e., assume all current employees will receive the post-employment
benefits).
d. to ignore possible mortality after service (i.e., life expectancy).
(IASCF)

Section 29 Income Tax


235. Income tax consists of:
a. domestic taxes that are based on taxable profits.
b. foreign taxes that are based on taxable profits.
c. taxes that are payable by a subsidiary, associate or joint venture on distributions to
the reporting entity.

268
d. all of the above.
(IASCF)

236. An entity determines its taxable profit for the year ended 30 April 20X8 to be
₱200,000. The tax rate for 20X8 is 40 per cent. Which of the following journal entries is
appropriate to record the current tax for the year?
a. Debit Current tax asset ₱80,000; Credit Current tax income ₱80,000.
b. Debit Current tax asset ₱200,000; Credit Current tax income ₱200,000.
c. Debit Current tax expense ₱80,000; Credit Current tax liability ₱80,000.
d. Debit Current tax expense ₱200,000; Credit Current tax liability ₱200,000.
(IASCF)

237. (i) to (iv) list the first four steps in accounting for deferred tax (the first four steps of
the deferred tax methodology). What is their correct order?
i. Compute any temporary differences, unused tax losses and unused tax credits.
ii. Determine the tax basis at the reporting date of all those assets and liabilities, and of
other items that have a tax basis.
iii. Recognise deferred tax assets and liabilities arising from the temporary differences,
unused tax losses and unused tax credits.
iv. Identify which assets and liabilities are expected to affect taxable profit if they were
recovered or settled for their carrying amount.
a. (i), (ii), (iii), (iv). c. (ii), (iv), (i), (iii).
b. (iv), (ii), (i), (iii). d. (ii), (i), (iii), (iv).
(IASCF)

238. The tax basis is the measurement, under applicable substantively enacted tax law, of
an asset, liability or equity instrument. If the recovery of the asset through sale will
increase taxable profit, the asset’s tax basis is equal to which of the following?
a. The amount that would have been deductible in arriving at taxable profit if the
carrying amount of the asset had been recovered through use at the end of the
reporting period.
b. The amount that would have been deductible in arriving at taxable profit if the
carrying amount of the asset had been recovered through sale at the end of the
reporting period.
c. The amount that would have been deductible in arriving at taxable profit if the
carrying amount of the asset had been recovered through the expected manner of
recovery (i.e., either use or sale, or a combination of use or sale) at the end of the
reporting period.
d. Nil
(IASCF)

239. On 31 December 20X1, an entity has an asset of ₱4,000 for interest receivable that
will be taxed when the cash is received in 20X2. Tax is payable at 20 per cent on the
first ₱500,000 of taxable profit earned and 30 per cent on any remainder (i.e., excess
above ₱500,000). In 20X1 the entity earned taxable profit of ₱450,000. In 20X2 the
entity expects to earn taxable profit of ₱550,000. What amount should the entity
recognize for the deferred tax liability relating to the interest receivable?
a. ₱1,200. b. ₱1,000. c. ₱940. d. ₱836. e. ₱800.
(IASCF)

240. Which of the following items should not be recognized by an entity?


a. A deferred tax asset or liability for temporary differences associated with
unremitted earnings from foreign subsidiaries, branches, associates and joint
ventures to the extent that the investment is essentially permanent in duration,
unless it is apparent that the temporary difference will reverse in the foreseeable
future.

269
b. A deferred tax liability for a temporary difference associated with the initial
recognition of goodwill.
c. Both (a) and (b).
(IASCF)

241. In 20X1 an entity reports taxable profit of ₱50,000 to the tax authority, which will
be taxed at the corporate income tax rate in the jurisdiction of 30 per cent. The capital
gains rate is nil and so capital gains are excluded from taxable profit. The management
of the entity considers the effect of uncertainty over the amounts reported in the tax
return to be immaterial except in relation to the treatment of the profit on the sale of a
particular asset. In relation to that asset, management have determined that there is an
80 per cent probability that the gain of ₱5,000 is a capital gain and therefore will not be
taxed, and that there is a 20 per cent probability that the gain of ₱5,000 is not a capital
gain and therefore will be taxed at 30 per cent. How should the entity measure its
current tax liability based on the taxable profit?
a. ₱16,500. b. ₱16,200. c. ₱15,300. d. ₱15,000.
(IASCF)

242. What is the correct treatment regarding discounting of income tax assets and
liabilities?
a. Current tax assets and liabilities are discounted. Deferred tax assets and liabilities
are not discounted.
b. Current tax assets and liabilities are not discounted. Deferred tax assets and
liabilities are discounted.
c. Current and deferred tax assets and liabilities are discounted.
d. Current and deferred tax assets and liabilities are not discounted.
(IASCF)

243. An entity operates in a jurisdiction where income taxes are payable at a lower rate
on undistributed profits (20 per cent) with an additional amount (10 per cent) being
payable when profits are distributed (i.e., the tax rate on distributed profits is 30 per
cent). On 31 December 20X1 the entity expects to propose dividends in March 20X2 of
approximately ₱20,000 for the year ended 20X1. The financial statements will be
authorized for issue in April 20X2. Taxable profit for 20X1 is ₱100,000. The entity has
temporary differences that are expected to increase taxable profit in the future for the
year 20X1 of ₱30,000. The entity was formed on 1 January 20X1. On 31 December 20X1
the entity should recognize the following:
a. A current tax liability (and expense) of ₱20,000 and a deferred tax liability (and
expense) of ₱6,000.
b. A current tax liability (and expense) of ₱20,000 and a deferred tax liability (and
expense) of ₱9,000.
c. A current tax liability (and expense) of ₱22,000 and a deferred tax liability (and
expense) of ₱6,000.
d. A current tax liability (and expense) of ₱25,000 and a deferred tax liability (and
expense) of ₱7,500.
e. A current tax liability (and expense) of ₱30,000 and a deferred tax liability (and
expense) of ₱9,000.
(IASCF)

Section 30 Foreign Currency Translation


244. It is the currency of the primary economic environment in which the entity
operates.
a. Presentation currency c. Inflationary currency
b. Functionality currency d. Functional currency

Section 31 Hyperinflation

270
245. When an entity that operates under a hyperinflationary environment restates its
financial statements, the comparative information for the previous period and any
information presented in respect of earlier periods
a. shall also be stated in terms of the measuring unit current at the reporting date.
b. shall also be stated in terms of the measuring unit current at the previous reporting
date.
c. shall also be stated in terms of the measuring unit current at the reporting date for
non-monetary units only.
d. shall also be stated in terms of the measuring unit current at the reporting date for
monetary units only.

Section 32 Events after the End of the Reporting Period


246. Events after the end of the reporting period are defined as:
a. events, favorable and unfavorable, that occur between the end of the reporting
period and the date of the entity’s next annual financial statements.
b. events, favorable and unfavorable, that occur between the end of the reporting
period and the date of the entity’s next interim (or annual) financial statements.
c. events, favorable and unfavorable, that occur between the end of the reporting
period and the date when the financial statements are authorized for issue.
(IASCF)

247. Section 32 of the PFRS for SMEs has specific requirements for:
a. events after the end of the reporting period that provide evidence of conditions that
existed at the end of the reporting period.
b. events after the end of the reporting period that are indicative of conditions that
arose after the end of the reporting period.
c. events after the end of the reporting period that are indicative of conditions that
arose after the end of the reporting period or that provide evidence of conditions
that existed at the end of the reporting period.
(IASCF)

248. Adjusting events are those that:


a. provide evidence of conditions that existed at the end of the reporting period.
b. are indicative of conditions that arose after the end of the reporting period.
c. are favorable or unfavorable, and indicative of conditions that arose after the end of
the reporting period.
(IASCF)

249. When after the end of the reporting period an event occurs that is indicative of
conditions that arose after the end of the reporting period:
a. the entity discloses the nature and effect of the event in the financial statements.
b. the entity adjusts the related amounts recognized in the financial statements.
c. both of the above statements are true.
(IASCF)

250. On 15 March 20X1 the entity authorized for issue its annual financial statements for
the year ended 31 December 20X0. On 10 March 20X1 the entity’s factory and several
items of equipment were damaged in an earthquake. The event (quake damage):
a. is an adjusting event after the end of the 31 December 20X0 reporting period.
b. is a non-adjusting event after the end of the 31 December 20X0 reporting period.
c. is neither an adjusting event after the end of the 31 December 20X0 reporting
period nor a non-adjusting event after the end of the 31 December 20X0 reporting
period.
(IASCF)

271
251. Which of the following is a non-adjusting event after the end of the reporting period
that an entity should disclose in its financial statements for 20X5? In each case, the
financial statements for 20X5 have not yet been authorized for issue.
a. An entity has a portfolio of shares with quoted market prices. These are measured at
fair value through profit or loss in accordance with Section 11 of the PFRS for SMEs.
After the end of the reporting period, there was a substantial decline in the stock
market. The fair value of the entity’s portfolio of shares declined significantly.
b. At 31 December 20X5 one individual owned 100 per cent of the entity’s outstanding
shares. In February 20X6 that individual sold 80 per cent of her holding to another
party.
c. All of the above.
(IASCF)

252. On 25 March 20X4 the entity discovered that, as a result of a computational error,
depreciation expense for the year ended 31 December 20X3 is overstated by ₱29,000.
The entity’s 31 December 20X3 financial statements were authorized for issue on 1
April 20X4. The entity must:
a. correct its 31 December 20X3 financial statements before issuing them.
b. reduce depreciation for the year ended 31 December 20X4 by ₱29,000 (i.e.,
prospective allocation—a change in accounting estimate).
c. restate (correct) the depreciation expense reported for the year ended 31 December
20X3 in the comparative figures of its 20X4 financial statements (i.e., retrospective
restatement of a prior period error).
(IASCF)

253. The information in the preceding information applies. However, in this question, the
entity’s 31 December 20X3 financial statements were authorized for issue on 1 March
20X4. The entity must:
a. reissue its 31 December 20X3 financial statements with the correct depreciation
expense.
b. reduce depreciation for the year ended 31 December 20X4 by ₱29,000 (i.e.,
prospective allocation—a change in accounting estimate).
c. restate (correct) the depreciation expense reported for the year ended 31 December
20X3 in the comparative figures of its 20X4 financial statements (i.e., retrospective
restatement of a prior period error).
(IASCF)

254. On 20 January 20X5, before an entity’s 31 December 20X4 financial statements were
authorized for issue, a court ordered the entity to pay ₱120,000 damages in full and
final settlement of a patent infringement lawsuit brought against the entity by one of its
competitors. The patent infringement occurred during 20X3. The amount of damages
awarded to the competitor was significantly higher than the ₱10,000–₱30,000 that the
entity had justifiably expected to pay. However, the entity will not contest the judgment.
In its 31 December 20X3 annual financial statements the entity reported its liability for
the lawsuit at ₱20,000 - this estimate was made in good faith and taking account of all
available evidence. In its 31 December 20X4 financial statements the entity must:
a. restate the comparative information as at 31 December 20X3 (i.e., retrospective
restatement of a prior period error).
b. measure the provision as at 31 December 20X4 at ₱120,000 with comparative
information for 20X3: ₱20,000 (i.e., account prospectively for the change in
accounting estimate in its 20X4 financial statements).
c. measure the provision as at 31 December 20X4 at ₱20,000 (comparative
information 20X3: ₱20,000) and record the effect of the higher than expected
settlement in profit or loss for the year ended 31 December 20X5 (i.e., account
prospectively for the change in accounting estimate in the period that the final
settlement amount was determined).
(IASCF)

272
255. On 20 January 20X5, before an entity’s 31 December 20X4 financial statements were
authorized for issue, a court ordered that entity to pay ₱120,000 damages in full and
final settlement of a patent infringement lawsuit brought against the entity by one of its
competitors. The patent infringement occurred in 20X3. The amount of damages
awarded to the competitor was consistent with similar cases settled in that jurisdiction
since 20X2. In its 31 December 20X3 annual financial statements the entity reported its
liability for the lawsuit at ₱20,000. At the time of approving its 20X3 financial
statements the entity deliberately understated the amount presented, because it did not
want to make public its true estimate, believing that this would be detrimental to the
entity’s defense. In its 31 December 20X4 financial statements the entity must:
a. restate the comparative information as at 31 December 20X3 (i.e., retrospective
restatement of a prior period error).
b. measure the provision as at 31 December 20X4 at ₱120,000 (comparative
information 20X3: ₱20,000), (i.e., account prospectively for the change in accounting
estimate in its 20X4 financial statements).
c. measure the provision as at 31 December 20X4 at ₱20,000 (comparative
information 20X3: ₱20,000) and record the effect of the higher than expected
settlement in profit or loss for the year ended 31 December 20X5 (i.e., account
prospectively for the change in accounting estimate in the period that the final
settlement amount was determined).
(IASCF)

256. On 15 March 20X1 the entity authorized for issue its annual financial statements for
the year ended 31 December 20X0. On 10 March 20X1 the entity’s factory and several
items of equipment were damaged in an earthquake. As a result of the uninsured
earthquake damage, the management of the entity determines that the entity will be
unable to continue trading and hence the entity cannot be regarded as a going concern.
The entity must:
a. prepare its 31 December 20X0 financial statements on a going concern basis,
including disclosing a non-adjusting event after the end of the 31 December 20X0
reporting period (i.e., disclose the nature and effect of the earthquake in the notes to
its 31 December 20X0 annual financial statements).
b. not prepare its 31 December 20X0 financial statements on a going concern basis.
Any financial statements the entity prepares must disclose that they are not on a
going concern basis, together with the basis on which the financial statements are
prepared and the reason why the entity is not regarded as a going concern.
c. prepare its 31 December 20X0 financial statements on a going concern basis,
including accounting for an adjusting event after the end of the 31 December 20X0
reporting period (i.e., a recognize the impairment loss in its 31 December 20X0
annual financial statements).
(IASCF)

273
Section 33 Related Party Disclosures
257. Relationships between a parent and its subsidiaries shall be disclosed
a. only when there have been related party transactions.
b. irrespective of whether there have been related party transactions.
c. even when control is lost
d. any of these

Section 34 Specialized Activities


258. An entity applying the PFRS for SMEs that is engaged in agricultural activity shall
measure each class of its biological assets using
a. Fair value model
b. Cost model
c. (a) if fair value is readily determinable without undue cost or effort, otherwise (b)
d. none of these

Section 35 Transition to the PFRS for SMEs


259. Which of the following entities is a first-time adopter of the PFRS for SMEs in the
current period?
a. An entity that has decided to use the PFRS for SMEs with effect from some future
date.
b. An entity that presents its first annual financial statements using the PFRS for SMEs
for the current period, except that the entity’s accounting policy for research and
development costs is to capitalize all costs as a separate intangible asset.
c. An entity that presents its first annual financial statements that conform to the PFRS
for SMEs for the current period, except that the entity does not make an explicit
statement of compliance with the PFRS for SMEs.
d. An entity that presents its first annual financial statements that conform to the PFRS
for SMEs for the current period. Its previous accounting framework was full PFRSs.
e. An entity that presents its first annual financial statements that conform to the PFRS
for SMEs for the current period. Its previous accounting framework was local GAAP.
f. Both (d) and (e).
(IASCF)

260. Which of the following entities is a first-time adopter of the PFRS for SMEs in its 31
December 20X4 annual financial statements?
a. Entity A presented financial statements for the years ended 31 December 20X1 and
20X4 in compliance with the PFRS for SMEs. For the years ended 31 December 20X2
and 20X3 Entity A prepared only financial statements in compliance with full PFRSs.
b. Entity B’s financial statements for the year ended 31 December 20X4 are its first
financial statements that conform to local GAAP, which is consistent with the PFRS
for SMEs in all respects except in name. The entity made an explicit and unreserved
statement of compliance with the local GAAP (not the PFRS for SMEs).
c. Entity C’s financial statements for the year ended 31 December 20X4 are its first
financial statements that conform to local GAAP, which is consistent with the PFRS
for SMEs in all respects except in name. The entity made an explicit and unreserved
statement of compliance with both the local GAAP and the PFRS for SMEs.
d. Entity D has not presented financial statements for previous years –it is not
required to do so. In 20X4 the entity voluntarily adopted the PFRS for SMEs and
presented financial statements that conform to that standard (including an explicit
and unreserved statement of compliance with the PFRS for SMEs).
e. Both entities C and D.
f. Both entities B and C.
(IASCF)

261. The date of transition to the PFRS for SMEs is:

274
a. The beginning of the latest period for which the entity presents full comparative
information in accordance with the PFRS for SMEs in its first financial statements
that conform to the PFRS for SMEs.
b. The beginning of the earliest period for which the entity presents partial
comparative information in accordance with the PFRS for SMEs in its first financial
statements that conform to the PFRS for SMEs.
c. The beginning of the earliest period for which the entity presents full comparative
information in accordance with the PFRS for SMEs in its first financial statements
that conform to the PFRS for SMEs.
d. The beginning of the earliest period for which the entity presents full comparative
information in accordance with the PFRS for SMEs in its latest financial statements
that conform to the PFRS for SMEs.
(IASCF)

262. An entity that had never presented financial statements decided to adopt the PFRS
for SMEs in 20X8. The entity’s financial statements for the year ended 31 December
20X8 conformed to the PFRS for SMEs (including an explicit and unreserved statement
of compliance with the PFRS for SMEs in the notes). Full comparative information is
provided for one year. What is the entity’s date of transition to the PFRS for SMEs?
a. 1 January 20X5 c. 1 January 20X7
b. 1 January 20X6 d. 1 January 20X8.
(IASCF)

263. The facts are the same as in the immediately preceding question. However, in this
example, full comparative information is provided for two years. What is the entity’s
date of transition to the PFRS for SMEs?
a. 1 January 20X5 c. 1 January 20X7
b. 1 January 20X6 d. 1 January 20X8.
(IASCF)

264. An entity acquired a machine on 1 January 20X1 for ₱100,000. From 20X1 to 20X3,
in accordance with its previous financial reporting framework the entity depreciated
the machine using the straight-line method over 10 years to a nil residual value.
However, on 31 December 20X4, under its previous financial reporting framework, the
entity revalued the machine to its fair value of ₱90,000. Consequently, the entity
measured the machine at ₱75,000 (i.e., ₱90,000 gross less ₱15,000 accumulated
depreciation) in statement of financial position at 31 December 20X5. In 20X6 the
entity decided to adopt the PFRS for SMEs. At 1 January 20X6, when the fair value of the
machine is ₱80,000 management estimated the following in accordance with the PFRS
for SMEs:
 the remaining useful life of the machine to be 5 years;
 the residual value of the machine to be zero; and
 the straight-line method of depreciation to be most appropriate.

The entity’s first financial statements that will conform to the PFRS for SMEs will be for the
year ended 31 December 20X7. In its opening statement of financial position at 1 January
20X6 the entity could measure the machine at:
a. ₱50,000 (i.e., as if the machine had always been accounted for in accordance with
b. Section 17 Property, Plant and Equipment—₱100,000 historic cost less ₱50,000
accumulated depreciation).
c. ₱75,000 (i.e., using the revaluation made in accordance with the previous financial
reporting framework—₱90,000 historic cost less ₱15,000 accumulated
depreciation).
d. ₱80,000 (i.e., fair value on the date of transition to the PFRS for SMEs).
e. Any of (a) to (c) above.
(IASCF)

275
265. On 1 January 20X1 an entity acquired a business for ₱100,000, when the fair value
of the identifiable acquired assets was ₱90,000 (the business acquired had no liabilities
and no contingent liabilities). In accordance with the previous financial reporting
framework, management accounted for the ₱10,000 goodwill as an expense
immediately (i.e., in the group’s consolidated statement of comprehensive income for
the year ended 31 December 20X1). This is the only business combination that the
entity entered into. At 1 January 20X6 the entity estimated the fair value of the goodwill
in respect of the acquired business is ₱8,000. In 20X6 the entity decided to adopt the
PFRS for SMEs. Its first financial statements that will conform to the PFRS for SMEs will
be for the year ended 31 December 20X7. If the entity had applied the PFRS for SMEs at
the time of the business combination (1 January 20X1) it would have allocated a useful
life of 10 years to the goodwill from 1 January 20X1. Assume that in accordance with
the PFRS for SMEs no impairment of that goodwill would have been required between 1
January 20X1 and 31 December 20X5). In its opening consolidated statement of
financial position at 1 January 20X6 the group would measure the goodwill at:
a. ₱5,000 (i.e., as if the goodwill had always been accounted for in accordance with
Section 19 Business Combinations and Goodwill—₱10,000 historic cost less ₱5,000
accumulated amortization).
b. Nil (i.e., no restatement of the goodwill. Goodwill expensed immediately in
accordance with the previous financial reporting framework).
c. ₱8,000 (i.e., estimated fair value of the goodwill on the date of transition to the PFRS
for SMEs).
d. Either (a) or (b) above.
e. Any of (a) to (c) above.
(IASCF)

266. The facts are the same as in the immediately preceding question. However, in this
example there are two business combinations (the entity acquired businesses on 1
January 20X1 and on 1 January 20X4). Management has decided that in preparing the
group’s opening statement of financial position at 1 January 20X6 they will choose the
exemption in paragraph 35.10 (ie not to apply Section 19 Business Combinations and
Goodwill to the business acquired on 1 January 20X4). Goodwill of ₱5,000 arose on the
acquisition of the second business (ie the business acquired on 1 January 20X4). At 1
January 20X6 the entity estimated the fair value of the goodwill in respect of the second
acquired business is ₱4,500. In its opening consolidated statement of financial position
at 1 January 20X6 the group must measure the goodwill for the first business
combination (i.e., acquired on 1 January 20X1) at:
a. ₱5,000 (i.e., as if the goodwill had always been accounted for in accordance with
b. Section 19 Business Combinations and Goodwill—₱10,000 historic cost less ₱5,000
accumulated amortization).
c. nil (i.e., no restatement of the goodwill recognized as an expense immediately in
accordance with the previous financial reporting framework).
d. ₱8,000 (i.e., estimated fair value on the date of transition to the PFRS for SMEs).
e. Either (a) or (b) above.
f. Any of (a) to (c) above.
(IASCF)

267. The facts are the same as in the immediately preceding question. However, in this
example, management have decided that in preparing the group’s opening statement of
financial position at 1 January 20X6 they will not apply Section 19 Business
Combinations and Goodwill to the accounting for the acquisition of the first business
(i.e., the business acquired on 1 January 20X1). In its opening consolidated statement of
financial position at 1 January 20X6 the group could measure the goodwill for business
acquired on 1 January 20X4 at:
a. ₱4,000 (i.e., as if the goodwill had always been accounted for in accordance with
Section 19 Business Combinations and Goodwill—₱5,000 historic cost less ₱1,000
accumulated amortization).

276
b. Nil (i.e., expenses immediately in accordance with the previous financial reporting
framework).
c. ₱4,500 (i.e., fair value on the date of transition to the PFRS for SMEs).
d. Either (a) or (b) above.
e. Any of (a) to (c) above.
(IASCF)

268. An entity concludes that it is impracticable to restate the opening statement of


financial position at the date of transition to the PFRS for SMEs for one or more of the
adjustments required by paragraph 35.7. Which of the following statements is correct?
a. The entity must apply paragraphs 35.7–35.10 for such adjustments in the earliest
period for which it is practicable to do so.
b. The entity must identify the data presented for prior periods that are not
comparable with data for the period in which it prepares its first financial
statements that conform to the PFRS for SMEs.
c. Both (a) and (b) are correct.
d. The entity cannot make an explicit and unreserved statement of compliance with
the PFRS for SMEs.
e. The entity is not considered a first-time adopter of the PFRS for SMEs.
f. Both (d) and (e) are correct.
(IASCF)

269. An entity’s first financial statements that conform to the PFRS for SMEs are
presented for the year ended 31 December 20X4. Those financial statements include
only one year of comparative information (i.e., 20X3). The entity’s financial statements
for the year ended 31 December 20X3 were presented in accordance with local GAAP.
The entity is required to explain how the transition from the previous financial
reporting framework to the PFRS for SMEs affected its reported financial position,
financial performance and cash flows. To comply with this requirement, an entity’s first
financial statements that conform to the PFRS for SMEs must present a number of
reconciliations. Which one of the following four reconciliations is not required to be
disclosed?
a. A reconciliation of its profit or loss in accordance with its previous financial
reporting framework for 20X3 to its profit or loss in accordance with the PFRS for
SMEs for 20X3.
b. A reconciliation of its profit or loss in accordance with its previous financial
reporting framework for 20X4 to its profit or loss in accordance with the PFRS for
SMEs for 20X4.
c. A reconciliation of its equity under its previous financial reporting framework to its
equity in accordance with the PFRS for SMEs at 1 January 20X3.
d. A reconciliation of its equity under its previous financial reporting framework to its
equity in accordance with the PFRS for SMEs at 31 December 20X3.
(IASCF)

277
Chapter 49 - Suggested answers to theory of accounts questions
1. D 41. B 81. D 121. C 161. B 201. D 241. C*
2. C 42. D 82. D 122. B 162. B 202. D 242. D*
3. B 43. E 83. D 123. A 163. A 203. C 243. A
4. C 44. E 84. A 124. C* 164. C 204. B 244. D
5. A 45. F 85. A 125. C 165. C 205. C 245. A
6. C 46. B 86. C 126. A 166. C 206. A 246. C
7. A 47. D 87. C 127. A 167. D 207. B 247. C
8. B 48. D 88. B 128. D 168. E 208. A 248. A
9. C 49. A 89. C 129. A 169. B 209. C 249. A
10. D 50. D 90. B 130. C 170. C 210. C 250. B
11. D 51. C 91. C 131. B 171. C* 211. A 251. C
12. C 52. C 92. A 132. D 172. B 212. C 252. A
13. C 53. C 93. C 133. C 173. D 213. C 253. C
14. B 54. F 94. B 134. D* 174. C 214. A 254. B
15. C 55. C 95. B 135. E* 175. A 215. B 255. A
16. D 56. D 96. A 136. G 176. B 216. A 256. B
17. B 57. A 97. C 137. D 177. B 217. B 257. B
18. B 58. A 98. D 138. D 178. A 218. A 258. C
19. C 59. D 99. D 139. A 179. D 219. D 259. F
20. A 60. A 100. C 140. B 180. C 220. A 260. E
21. D 61. C 101. A 141. D 181. D 221. D 261. C
22. D 62. C 102. B 142. C 182. B 222. A 262. C
23. A 63. B 103. A 143. B 183. A 223. E 263. B
24. C 64. C 104. B 144. D* 184. A 224. A 264. D
25. D 65. D 105. A 145. E 185. D 225. A 265. D
26. A 66. A 106. D 146. C 186. A 226. A 266. B
27. C 67. C 107. C 147. D 187. A 227. C 267. D
28. A 68. B 108. A 148. D 188. D 228. C 268. C
29. D 69. C 109. D 149. B 189. A 229. D 269. B
30. C 70. D 110. B 150. C 190. C 230. B
31. D 71. D 111. B 151. D 191. E* 231. C
32. A 72. A 112. C 152. B 192. D* 232. A
33. B 73. D 113. E 153. B 193. B 233. A
34. D 74. C 114. B 154. A* 194. D 234. D
35. A 75. C 115. E 155. A 195. B 235. D
36. D 76. D 116. A 156. B 196. D 236. C
37. B 77. C 117. D 157. B 197. A 237. B
38. B 78. A 118. A 158. C 198. D 238. B
39. A 79. D 119. B 159. B 199. A 239. D*
40. C 80. C 120. C 160. C 200. E 240. C

*Solutions to selected problems:


Question 124.
₱95,000 (₱100,000 excluding refundable tax of ₱5,000) multiplied by PV of ₱1 @10%, n=1
(or simply divide by 1.1) plus ₱2,000 transport costs = ₱88,364.

Question 134.
20x1: Fair value less cost to sell (102K – 4K) = ₱98,000 lower than cost of ₱101K (cost of
100K + transaction cost of 1K).
20x2: Cost of ₱101,000 = previous carrying amount of 98K + 3K reversal of impairment
loss.
20x3: Fair value less cost to sell (90K – 4K) = ₱86,000 lower than previous carrying amount
of ₱101K.

Question 135.

278
₱102,000, ₱110,000, and ₱90,000 published price quotations without deduction for costs
to sell.

Question 144. See solutions in Question 134.

Question 154.
₱95,000 including non-refundable tax multiplied by PV of ₱1 @10%, n=1 (or simply divide
by 1.1) plus ₱2,000 legal fees = ₱88,364.

Question 171. See solution in Question 154.

Question 191.
10M x 3% x 10% 30,000
10M x 2% x 90% 180,000
Total 210,000
Multiply by: Discount rate (given) 0.95238
Total 200,000
Multiply by: Risk adjustment (100% + 6%) 106%
Total 212,000
Multiply by: Amount to be settled in 20x2 50%
Warranty provision – Dec. 31, 20x1 106,000

Question 192.
200K x 20% 40,000
100K x 80% 80,000
Total 120,000
Multiply by: PV of ₱1 @10%, n=1 0.90909
Total 109,090
Multiply by: Risk adjustment (100% + 7%) 107%
Total 116,727
Multiply by: Probability of settlement (100% - 30%) 70%
Provision for lawsuit – Dec. 31, 20x1 81,709

Question 239.
Tax on first ₱500,000 of profit (500K x 20%) 100,000
Tax on excess profit over ₱500,000 (50K x 30%) 15,000
Tax on expected profit of ₱550K in 20x2 115,000
Divide by: Expected profit in 20x2 550,000
Average rate expected to apply on reversal date 20.91%
Multiply by: Temporary difference 4,000
Deferred tax liability 836.36

Question 241.
Tax on taxable profit excluding capital gain (50,000 x 30%) 15,000
Tax on capital gain (5,000 x 80% x 0%) 0
Tax on non-capital gain (5,000 x 20% x 30%) 300
Current tax expense 15,300

Question 242.
Current tax expense = Taxable profit of ₱100,000 x Tax rate on taxable profit of 20% =
₱20,000
Deferred tax liability = Future increase in taxable profit ₱30,000 x Tax rate on taxable profit
of 20% = ₱6,000

279

You might also like